Schwartz's Principles of Surgery ABSITE and Board Review, 11th Edition [11 ed.] 9781260469769, 126046976X, 9781260469752, 1260469751

The premier exam-prep guide for surgery―with 800+ board-style practice questions with detailed answers and rationales B

593 165 32MB

English Pages 432 [427] Year 2022

Report DMCA / Copyright

DOWNLOAD FILE

Polecaj historie

Schwartz's Principles of Surgery ABSITE and Board Review, 11th Edition [11 ed.]
 9781260469769, 126046976X, 9781260469752, 1260469751

Table of contents :
Cover
Title Page
Copyright Page
Contents
Contributors
Preface
Acknowledgments
PART I Basic Considerations
1 Leadership
2 Systemic Response to Injury and Metabolic Support
3 Fluid and Electrolyte Management of the Surgical Patient
4 Hemostasis, Surgical Bleeding, and Transfusion
5 Shock
6 Surgical Infection
7 Trauma
8 Burns
9 Wound Healing
10 Oncology
11 Transplantation
12 Patient Safety
13 Physiologic Monitoring of the Surgical Patient
14 Minimally Invasive Surgery
15 Molecular and Genomic Surgery
PART II Specific Consideration
16 The Skin and Subcutaneous Tissue
17 Breast
18 Head and Neck
19 Chest Wall, Lung, Mediastinum, and Pleura
20 Congenital Heart Disease
21 Acquired Heart Disease
22 Thoracic Aneurysms and Aortic Dissection
23 Arterial Disease
24 Venous and Lymphatic Disease
25 The Esophagus and Diaphragmatic Hernia
26 Stomach
27 The Surgical Management of Obesity
28 Small Intestine
29 Colon, Rectum, and Anus
30 The Appendix
31 Liver
32 The Gallbladder and Extrahepatic Biliary System
33 Pancreas
34 The Spleen
35 Abdominal Wall, Omentum, Mesentery, and Retroperitoneum
36 Soft Tissue Sarcomas
37 Inguinal Hernias
38 Thyroid, Parathyroid, and Adrenal
39 Pediatric Surgery
40 Urology
41 Gynecology
42 Neurosurgery
43 Orthopedic Surgery
44 Surgery of the Hand and Wrist
45 Plastic and Reconstructive Surgery
46 Anesthesia for the Surgical Patient
47 Surgical Considerations in the Elderly
48 Ethics, Palliative Care, and Care at the End of Life
49 Global Surgery
50 Optimizing Perioperative Care: Enhanced Recovery and Chinese Medicine
51 Understanding, Evaluating, and Using Evidence for Surgical Practice
52 Ambulatory Surgery
53 Skills and Simulation
54 Web-Based Education and Implications of Social Media
Index

Citation preview

SCHWARTZ’S

PRINCIPLES OF SURGERY ABSITE and Board Review

Brunicardi_FM_p00i-xii.indd 1

08/07/22 11:25 AM

NOTICE Medicine is an ever-changing science. As new research and clinical experience broaden our knowledge, changes in treatment and drug therapy are required. The authors and the publisher of this work have checked with sources believed to be reliable in their efforts to provide information that is complete and generally in accord with the standards accepted at the time of publication. However, in view of the possibility of human error or changes in medical sciences, neither the authors nor the publisher nor any other party who has been involved in the preparation or publication of this work warrants that the information contained herein is in every respect accurate or complete, and they disclaim all responsibility for any errors or omissions or for the results obtained from use of the information contained in this work. Readers are encouraged to confirm the information contained herein with other sources. For example and in particular, readers are advised to check the product information sheet included in the package of each drug they plan to administer to be certain that the information contained in this work is accurate and that changes have not been made in the recommended dose or in the contraindications for administration. This recommendation is of particular importance in connection with new or infrequently used drugs.

Brunicardi_FM_p00i-xii.indd 2

08/07/22 11:25 AM

SCHWARTZ’S

PRINCIPLES OF SURGERY ABSITE and Board Review 11th Edition Edited by F. Charles Brunicardi, MD, FACS

Professor of Surgery, Department of Surgery Senior Vice President Dean, College of Medicine SUNY Downstate Health and Sciences University Brooklyn, New York

Associate Editors Dana K. Andersen, MD, FACS

Senior Scientific Advisor Division of Digestive Diseases and Nutrition National Institute of Diabetes and Digestive and Kidney Disease National Institutes of Health Bethesda, Maryland

Timothy R. Billiar, MD, FACS

George Vance Foster Professor and Chair Department of Surgery Distinguished Professor of Surgery Executive Vice President and Chief Scientific Officer for UPMC Associate Senior Vice Chancellor for Clinical Academics University of Pittsburgh Health Sciences Associate Medical Director, UPMC International and Commercial Services Division University of Pittsburgh School of Medicine University of Pittsburgh Pittsburgh, Pennsylvania

Karen J. Brasel, MD, MPH, FACS

Professor of Surgery Division of Trauma, Critical Care and Acute Care Surgery, School of Medicine Assistant Dean for Graduate Medical Education Office of the Dean, School of Medicine Residency Program Director, Surgery, School of Medicine Oregon Health and Science University Portland, Oregon

Lisa Dresner, MD, FACS

Program Director and Vice Chair for Education Associate Professor of Surgery Department of Surgery College of Medicine, SUNY Downstate Health and Sciences University Brooklyn, New York

John G. Hunter, MD, FACS

Professor of Surgery, School of Medicine Executive Vice President, OHSU Chief Executive Officer, OHSU Health System Oregon Health and Science University Portland, Oregon

Lillian S. Kao, MD, MS, FACS

Jack H. Mayfield, MD, Chair in Surgery Vice-Chair for Quality of Care Professor and Chief, Division of Acute Care Surgery Department of Surgery McGovern Medical School at UTHealth Houston Houston, Texas

Jeffrey B. Matthews, MD, FACS Dallas B. Phemister Professor Chairman, Department of Surgery Pritzker School of Medicine University of Chicago Chicago, Illinois

Raphael E. Pollock, MD, PhD, FACS

Director, Ohio State University Comprehensive Cancer Center Kathleen Wellenreiter Klotz Chair in Cancer Research Arthur G. James Cancer Hospital and Richard J. Solove Research Institute Professor, Division of Surgical Oncology, Department of Surgery Ohio State University College of Medicine Columbus, Ohio

New York Chicago San Francisco Athens London Madrid Mexico City Milan New Delhi Singapore Sydney Toronto

Brunicardi_FM_p00i-xii.indd 3

08/07/22 11:25 AM

Copyright © 2023, 2016, 2011, 2007 by McGraw Hill LLC. All rights reserved. Except as permitted under the United States Copyright Act of 1976, no part of this publication may be reproduced or distributed in any form or by any means, or stored in a database or retrieval system, without the prior written permission of the publisher. ISBN: 978-1-26-046976-9 MHID: 1-26-046976-X The material in this eBook also appears in the print version of this title: ISBN: 978-1-26-046975-2, MHID: 1-26-046975-1. eBook conversion by codeMantra Version 1.0 All trademarks are trademarks of their respective owners. Rather than put a trademark symbol after every occurrence of a trademarked name, we use names in an editorial fashion only, and to the benefit of the trademark owner, with no intention of infringement of the trademark. Where such designations appear in this book, they have been printed with initial caps. McGraw Hill eBooks are available at special quantity discounts to use as premiums and sales promotions or for use in corporate training programs. To contact a representative, please visit the Contact Us page at www.mhprofessional.com. TERMS OF USE This is a copyrighted work and McGraw-Hill Education and its licensors reserve all rights in and to the work. Use of this work is subject to these terms. Except as permitted under the Copyright Act of 1976 and the right to store and retrieve one copy of the work, you may not decompile, disassemble, reverse engineer, reproduce, modify, create derivative works based upon, transmit, distribute, disseminate, sell, publish or sublicense the work or any part of it without McGraw-Hill Education’s prior consent. You may use the work for your own noncommercial and personal use; any other use of the work is strictly prohibited. Your right to use the work may be terminated if you fail to comply with these terms. THE WORK IS PROVIDED “AS IS.” McGRAW-HILL EDUCATION AND ITS LICENSORS MAKE NO GUARANTEES OR WARRANTIES AS TO THE ACCURACY, ADEQUACY OR COMPLETENESS OF OR RESULTS TO BE OBTAINED FROM USING THE WORK, INCLUDING ANY INFORMATION THAT CAN BE ACCESSED THROUGH THE WORK VIA HYPERLINK OR OTHERWISE, AND EXPRESSLY DISCLAIM ANY WARRANTY, EXPRESS OR IMPLIED, INCLUDING BUT NOT LIMITED TO IMPLIED WARRANTIES OF MERCHANTABILITY OR FITNESS FOR A PARTICULAR PURPOSE. McGraw-Hill Education and its licensors do not warrant or guarantee that the functions contained in the work will meet your requirements or that its operation will be uninterrupted or error free. Neither McGraw-Hill Education nor its licensors shall be liable to you or anyone else for any inaccuracy, error or omission, regardless of cause, in the work or for any damages resulting therefrom. McGraw-Hill Education has no responsibility for the content of any information accessed through the work. Under no circumstances shall McGraw-Hill Education and/ or its licensors be liable for any indirect, incidental, special, punitive, consequential or similar damages that result from the use of or inability to use the work, even if any of them has been advised of the possibility of such damages. This limitation of liability shall apply to any claim or cause whatsoever whether such claim or cause arises in contract, tort or otherwise.

CONTENTS

Contributors

vii

PART II

Preface

ix

Specific Consideration

Acknowledgments

xi

PART I Basic Considerations

1

1 Leadership

3

2 Systemic Response to Injury and Metabolic Support

7

3 Fluid and Electrolyte Management of the Surgical Patient

13

4 Hemostasis, Surgical Bleeding, and Transfusion

21

5 Shock

29

6 Surgical Infection

37

7 Trauma

49

8 Burns

65

9 Wound Healing

73

10 Oncology

83

11 Transplantation

87

12 Patient Safety

97

13 Physiologic Monitoring of the Surgical Patient

105

14 Minimally Invasive Surgery

115

15 Molecular and Genomic Surgery

119

127

16 The Skin and Subcutaneous Tissue

129

17 Breast

137

18 Head and Neck

143

19 Chest Wall, Lung, Mediastinum, and Pleura

145

20 Congenital Heart Disease

157

21 Acquired Heart Disease

171

22 Thoracic Aneurysms and Aortic Dissection

181

23 Arterial Disease

189

24 Venous and Lymphatic Disease

197

25 The Esophagus and Diaphragmatic Hernia

207

26 Stomach

223

27 The Surgical Management of Obesity

233

28 Small Intestine

235

29 Colon, Rectum, and Anus

241

30 The Appendix

259

31 Liver

263

32 The Gallbladder and Extrahepatic Biliary System

279

33 Pancreas

289

34 The Spleen

297

35 Abdominal Wall, Omentum, Mesentery, and Retroperitoneum

307

v

Brunicardi_FM_p00i-xii.indd 5

08/07/22 11:25 AM

vi CONTENTS

36 Soft Tissue Sarcomas

311

37 Inguinal Hernias

313

38 Thyroid, Parathyroid, and Adrenal

323

39 Pediatric Surgery

329

40 Urology

337

41 Gynecology

341

42 Neurosurgery

345

43 Orthopedic Surgery

351

44 Surgery of the Hand and Wrist

355

45 Plastic and Reconstructive Surgery

361

46 Anesthesia for the Surgical Patient

363

47 Surgical Considerations in the Elderly

365

Brunicardi_FM_p00i-xii.indd 6

48 Ethics, Palliative Care, and Care at the End of Life

369

49 Global Surgery

373

50 Optimizing Perioperative Care: Enhanced Recovery and Chinese Medicine

377

51 Understanding, Evaluating, and Using Evidence for Surgical Practice

381

52 Ambulatory Surgery

385

53 Skills and Simulation

387

54 Web-Based Education and Implications of Social Media

389

Index

391

08/07/22 11:25 AM

CONTRIBUTORS

Dana K. Andersen, MD, FACS Senior Scientific Advisor Division of Digestive Diseases and Nutrition National Institute of Diabetes and Digestive and Kidney Disease National Institutes of Health Bethesda, Maryland Shay Behrens, MD General Surgery Resident Department of Surgery Oregon Health and Science University Portland, Oregon Andrew J. Benjamin, MD Assistant Professor of Surgery Pritzker School of Medicine University of Chicago Chicago, Illinois Timothy R. Billiar, MD, FACS George Vance Foster Professor and Chair Department of Surgery Distinguished Professor of Surgery Executive Vice President and Chief Scientific Officer for UPMC Associate Senior Vice Chancellor for Clinical Academics University of Pittsburgh Health Sciences Associate Medical Director UPMC International and Commercial Services Division University of Pittsburgh School of Medicine University of Pittsburgh Pittsburgh, Pennsylvania Jillian Bonaroti, MD Department of Surgery University of Pittsburgh Medical Center Pittsburgh, Pennsylvania Karen J. Brasel, MD, MPH, FACS Professor of Surgery Division of Trauma, Critical Care and Acute Care Surgery School of Medicine Assistant Dean for Graduate Medical Education Office of the Dean School of Medicine Residency Program Director, Surgery School of Medicine Oregon Health and Science University Portland, Oregon F. Charles Brunicardi, MD, FACS Professor of Surgery, Department of Surgery Senior Vice President Dean, College of Medicine SUNY Downstate Health and Sciences University Brooklyn, New York

Brunicardi_FM_p00i-xii.indd 7

Daniel T. Dempsey, MD, FACS Professor of Surgery, Perelman School of Medicine University of Pennsylvania Philadelphia, Pennsylvania Ankit Dhiman, MD Research Fellow, Department of Surgery Pritzker School of Medicine University of Chicago Chicago, Illinois Lisa Dresner, MD, FACS Program Director and Vice Chair for Education Associate Professor of Surgery Department of Surgery College of Medicine SUNY Downstate Health and Sciences University Brooklyn, New York Jodi-Ann Edwards, MD Resident Department of Surgery College of Medicine SUNY Downstate Health and Sciences University Brooklyn, New York Erfan Faridmoayer, MD Resident Department of Surgery College of Medicine SUNY Downstate Health and Sciences University Brooklyn, New York Sam Guyer, MD General Surgery Resident Department of Surgery Oregon Health and Science University Portland, Oregon Patrick Harbour, MD Chief Resident in Plastic Surgery Department of Plastic Surgery Georgetown University School of Medicine Washington, DC Ahmed Hozain, MD Chief Resident Department of Surgery College of Medicine SUNY Downstate Health and Sciences University Brooklyn, New York

vii

08/07/22 11:25 AM

viii CONTRIBUTORS

John G. Hunter, MD, FACS Professor of Surgery School of Medicine Executive Vice President, OHSU Chief Executive Officer, OHSU Health System Oregon Health and Science University Portland, Oregon Lillian S. Kao, MD, MS, FACS Jack H. Mayfield, MD, Chair in Surgery Vice-Chair for Quality of Care Professor and Chief, Division of Acute Care Surgery Department of Surgery McGovern Medical School at UTHealth Houston Houston, Texas Marina Affi Koprowski, MD General Surgery Resident Department of Surgery Oregon Health and Science University Portland, Oregon Jeffrey B. Matthews, MD, FACS Dallas B. Phemister Professor Chairman, Department of Surgery Pritzker School of Medicine University of Chicago Chicago, Illinois Munier Nazzal, MD, MBA, FRCS, FACS, DFSVS Professor, Department of Surgery Vice Chair, Research and Education Chief, Division of Vascular, Endovascular and Wound Surgery University of Toledo College of Medicine Toledo, Ohio Junaid Nizamuddin, MD Assistant Professor of Anesthesia and Critical Care Pritzker School of Medicine University of Chicago Chicago, Illinois Michael O’Connor, MD Professor of Anesthesia and Critical Care Pritzker School of Medicine University of Chicago Chicago, Illinois Raphael E. Pollock, MD, PhD, FACS Director, Ohio State University Comprehensive Cancer Center Kathleen Wellenreiter Klotz Chair in Cancer Research Arthur G. James Cancer Hospital and Richard J. Solove Research Institute Professor, Division of Surgical Oncology, Department of Surgery Ohio State University College of Medicine Columbus, Ohio Ryan Randle, MD General Surgery Resident Department of Surgery Oregon Health and Science University Portland, Oregon

Brunicardi_FM_p00i-xii.indd 8

Katherine Reitz, MD, FACS Department of Surgery University of Pittsburgh Medical Center Pittsburgh, Pennsylvania Robert E. Roses, MD, FACS Associate Professor of Surgery Perelman School of Medicine University of Pennsylvania Philadelphia, Pennsylvania Ashley Sweet, MD General Surgery Resident Department of Surgery Oregon Health and Science University Portland, Oregon Alexander Schwartzman, MD, MBA, FACS Vice Chair and Clinical Professor of Surgery Department of Surgery College of Medicine SUNY Downstate Health Sciences University Brooklyn, New York David H. Song, MD, FACS Professor and Chairman Department of Plastic Surgery Georgetown University School of Medicine Washington, DC Joseph Tobias, MD General Surgery Resident Department of Surgery Oregon Health and Science University Portland, Oregon Samuel Torres Landa Fernandez, MD General Surgery Resident Department of Surgery Oregon Health and Science University Portland, Oregon Kiran K. Turaga MD, MPH, FACS Professor of Surgery Pritzker School of Medicine University of Chicago Chicago, Illinois David Warner, MD General Surgery Resident Department of Surgery Oregon Health and Science University Portland, Oregon Lye-Yeng Wong, MD General Surgery Resident Department of Surgery Oregon Health and Science University Portland, Oregon Sarah Wonn, MD General Surgery Resident Department of Surgery Oregon Health and Science University Portland, Oregon

08/07/22 11:25 AM

PREFACE

This 11th edition of Schwartz’s Principles of Surgery: ABSITE and Board Review marks a new milestone of excellence in surgical education for the betterment of craft, quality of care, and the edification of surgical students and colleagues alike. With 870 questions spanning the 54 updated chapters of this edition, including five new chapters, Optimizing Perioperative Care: Enhanced Recovery and Chinese Medicine, Understanding, Evaluating and Using Evidence for Surgical Practice, Ambulatory Surgery, Skills and Simulation, and Web-Based Education and Implications of Social Media, this is the comprehensive companion text for reviewing and assessing the information compiled in the main textbook and for preparation for the American Board of Surgery In-Training Examination (ABSITE) and Board Review.

Contributors of the primary book have updated the questions for each chapter since the last edition in an effort to continue to provide a high level of review on the most up-to-date information and techniques currently taught and employed in the operating theater. We have maintained the proven format of providing the answer-bearing portion of the text immediately following the question and answer as an efficient method for reinforcement and recall. The user may read the question followed by the answer as a form of review, or by covering the right-hand column of the page, the user can complete the questions in a more authentic test format and uncover the answers for review/scoring.

ix

Brunicardi_FM_p00i-xii.indd 9

08/07/22 11:25 AM

This page intentionally left blank

Brunicardi_FM_p00i-xii.indd 10

08/07/22 11:25 AM

ACKNOWLEDGMENTS

To Jason Malley, Peter Boyle, Catherine Saggese, and all at McGraw Hill, we are thankful for the continued belief in and support for this book.

We wish to thank Katie Elsbury for her dedication to the organization and editing of this book. F. Charles Brunicardi, MD, FACS

xi

Brunicardi_FM_p00i-xii.indd 11

08/07/22 11:25 AM

This page intentionally left blank

Brunicardi_FM_p00i-xii.indd 12

08/07/22 11:25 AM

PART I

Basic Considerations

Brunicardi_Ch01_p001-006.indd 1

30/06/22 10:36 AM

Brunicardi_Ch01_p001-006.indd 2

30/06/22 10:36 AM

CHAPTER

1

Leadership

1. The fundamental principles of leadership are: A. Vision and willingness B. Command and control C. Time management and mentoring D. Coaching, pacesetting, and democratic

Answer: A Vision and willingness are the two fundamental principles of leadership. Command and control is a colloquial term for the previously predominant style of leadership in surgery based on fear and intimidation. Time management and mentorship are key leadership skills. Coaching, pacesetting, and democratic are leadership styles. (See Schwartz 11th ed., p. 4.)

2. A senior resident and attending are performing a laparoscopic cholecystectomy. The resident initially performs the majority of the case following instruction from the attending. However, the resident informs the attending that he feels uncertain whether he has truly obtained the critical view of safety. With the attending’s assistance, the case was successfully completed. After the case, the resident then asks the attending for a debrief of the case. Which fundamental principle of leadership is being demonstrated by the resident? A. Conflict resolution B. Vision C. Effective communication D. Willingness to lead

Answer: C Leadership is a complex concept. Surgeons should strive to adopt leadership qualities that provide the best outcomes for their patients based on the following fundamental principles: vision, willingness, time management, conflict resolution, recruitment, and culture (see Table 1-1). (See Schwartz 11th ed., p. 5.) TABLE 1-1

The fundamental principles of leadership

Leadership Skill

Description and Application in the Field of Medicine

Vision

The act of establishing tangible goals of care for patients on both a daily basis as well as for long-term purposes.

Effective communication

Establishing an open, respectful, and nonjudgmental forum for communication among different members of the health care team and with the patient.

Willingness to lead

Taking on full responsibility for the care of patients and remaining ethical, professional, and committed despite the especially challenging rigors of joining the field of surgery.

Willingness to learn

A commitment to lifelong learning of the latest scientific, medical, and surgical updates to deliver optimized patient care.

Conflict resolution

The art of resolving conflicts in a peaceful and ethical manner in team settings.

3

Brunicardi_Ch01_p001-006.indd 3

30/06/22 10:36 AM

4 CHAPTER 1 Leadership

3. Effective communication is a key component of leadership, given that miscommunication is a leading cause of medical errors. Which of the following statements is FALSE regarding communication? A. To Err Is Human, a publication by the US Institute of Medicine, identified medical errors as the eighth leading cause of death in the United States, causing 100,000 deaths annually. B. Effective communication that ensures all team members understand daily goals of care for an ICU patient can significantly decrease their length of stay in the ICU. C. Communication errors are often simply due to negligence and failure to transmit information. D. Information transfer and communication errors cause delays in patient care and can cause serious adverse events. E. Improved communication in the OR among cardiac surgery patients is associated with decreased adverse outcomes.

Answer: C Communication errors are often caused by miscommunication due to hierarchical differences, concerns with upward influence, conflicting roles and role ambiguity, and interpersonal conflict. (See Schwartz 11th ed., p. 8.)

4. Which of the following statements is FALSE regarding modern conflict resolution techniques? A. Based upon objectivity and willingness to listen. B. Should seek a solution that benefits all involved and that is based upon core values of the organization. C. Traditional command and control based on fear and intimidation can lead to sanctions and lawsuits. D. Conflict resolution is more successful when both sides can admit they share some fault.

Answer: D Modern conflict resolution techniques are based upon objectivity, willingness to listen, and pursuit of principle-based solutions. For example, an effective style of conflict resolution is the utilization of the “abundance mentality” model, which attempts to achieve a solution that benefits all involved and is based upon core values of the organization, as opposed to the utilization of the traditional fault-finding model, which identifies sides as right or wrong. Application of the abundance mentality in surgery elevates the conflict above the affected parties and focuses on the higher unifying goal of improved patient care. Morbidity and mortality (M&M) conferences are managed in this style and have the purpose of practice improvement and improving overall quality of care within the system, as opposed to placing guilt or blame on the surgeon or surgical trainees for the complication being reviewed. The traditional style of the command-and-control technique based on fear and intimidation is no longer welcome in any health care system and can lead to sanctions, lawsuits, and removal of hospital privileges or position of leadership. (See Schwartz 11th ed., p. 9.)

5. Daniel Goleman of the Harvard Business Review described six key leadership styles. Which of the following statements is FALSE regarding leadership styles? A. The coercive style of leadership is antiquated and is no longer effective in surgery. B. Democratic leadership is useful for building team consensus and minimizing conflict but may frustrate team members if there is no clear, unifying vision. C. The pacesetter leads by example and sets high standards for his team but typically takes over the tasks of something falling behind instead of building them up. D. The authoritative leader is often the most effective and focuses on directing the team toward a common vision allowing team members room for innovation and experimentation, and supporting their efforts.

Answer: A The coercive leader demands immediate compliance. This style reflects the command-and-control style that has historically dominated surgery. Excessive coercive leadership erodes team members’ sense of responsibility, motivation, sense of participation in a shared vision, and ultimately, performance. The phrase “Do what I tell you!” brings to mind the coercive leader. However, it is effective in times of crisis to deliver clear, concise instruction. This style should be used sparingly and is best suited for emergencies. (See Schwartz 11th ed., p. 12.)

Brunicardi_Ch01_p001-006.indd 4

30/06/22 10:36 AM

5

7. Which of the following statements about how leaders in surgery effectively address disparities in social determinants of health and recruitment of women and underrepresented minority residents and faculty are TRUE? A. Diversity training obstructs leadership and strategic human resource management due to excess use of resources. B. Diversity programs improve recruitment of women and underrepresented minority residents and faculty. C. Diversity helps address disparities in social determinants of health. D. B and C.

Answer: D The past quarter century has seen a steady increase in diversity within the field of surgery. Women, as of 2015, represent 38% of surgical trainees and 10% of academic professors currently, but have doubled their representation in the past 20 years. Some fields, such as head and neck surgery and plastic surgery, have studied their own subspecialty groups with similar findings. African Americans comprise both 6% of medical school graduates, 6% of surgical trainees, and 2% to 4% of professors of surgery nationwide. Hispanics represent 5% of graduating medical students, 9% of general surgery trainees, and 4% to 5% of persons at all levels of academic surgery. Physician diversity is crucial and may help to address disparities in social determinants of health. (See Schwartz 11th ed., p. 12.)

8. The development and demonstration of effective leadership skills are essential during surgical training. Which of the following is considered the gold standard for the training and assessment of clinical and nontechnical skills with high reliability and validity? A. Command-and-control leadership B. Simulation training C. Coworker observation reporting system D. Objective structured clinical examination (OSCE)

Answer: D The past decade has seen a demonstrable increase in our knowledge of how to develop leadership skills, particularly through simulation, as well as leadership evaluation through OSCE and other tools. Multiple groups have assessed multidisciplinary teams, typically composed of nurses, anesthesia groups, and surgeons for the leadership-associated nontechnical skills of communication, teamwork, and situational awareness. Through increasingly validated instruments and assessment tools, these nontechnical skills have been found to be trainable. The OSCE has been established as the gold standard for the training and assessment of a wide range of clinical and nontechnical skills with high reliability and validity. (See Schwartz 11th ed., p. 14.)

9. Mentoring should be formally included in resident training programs. It provides wisdom, guidance, and insight essential for the successful development of a surgical leader. Which of the following statements is FALSE about mentees and mentors? A. The benefits of mentorship are unidirectional. B. Mentees have enhanced research productivity. C. Mentees have increased probability of obtaining grants/funding. D. Mentees have increased probability of obtaining desired academic/practice positions.

Answer: A In academic medicine, evidence-based studies have shown benefits to the mentees that include enhanced research productivity, higher likelihood of obtaining research grants, and greater success in obtaining desired positions in practice or at academic institutions. Mentoring provides benefits to the mentors themselves, including refinement of their own personal leadership skills and a strong sense of satisfaction and accomplishment. (See Schwartz 11th ed., p. 18.)

Brunicardi_Ch01_p001-006.indd 5

Leadership

Answer: B Studies indicate that the bottleneck in diversity occurs at the level of the medical school application pool, which in turn is caused by educational deficiencies at the primary, secondary, and collegiate levels. As an attempted solution, the University of Michigan developed a “pipeline” program that pairs gradeschool and high-school students with physicians for experiential learning and the development of mentoring, presentation skills, and networking. It is important for departments of surgery to develop a diversity program for recruitment of residents and faculty. Multi-institutional blinded studies indicate that the implementation of formal leadership and diversity training improves diversity leadership and strategic human resource management. (See Schwartz 11th ed., p. 12.)

CHAPTER 1

6. Which type of program couples grade/high-school students with physicians to provide experiential learning and development of mentoring, presentation skills, and networking? A. Community outreach B. “Pipeline” C. Physician teacher D. Bottleneck

30/06/22 10:36 AM

This page intentionally left blank

Brunicardi_Ch01_p001-006.indd 6

30/06/22 10:36 AM

CHAPTER

2

Systemic Response to Injury and Metabolic Support 1. C-reactive protein (CRP): A. Is secreted in a circadian rhythm with higher levels in the morning. B. Increases after eating a large meal. C. Is a soluble pattern recognition molecule of the pentraxin family. D. Is synthesized in lung and kidney in response to IL-6.

Answer: C Pentraxins are a group of soluble pattern recognition molecules that act to activate complement, agglutination and neutralization, and opsonization. CRP belongs to the pentraxin group of molecules. CRP is low in normal circumstances, but is synthesized in the liver in response to IL-6 with serum levels increasing 1000-fold. In this way, CRP is a marker of acute inflammation. CRP levels do not vary with circadian rhythms or with caloric intake. (See Schwartz 11th ed., Ch. 2, p. 34.)

2. Which of the following is TRUE regarding the inflammatory response following traumatic injury? A. There is an acute proinflammatory response caused by stimulation of the adaptive immune system. B. There is an anti-inflammatory response that leads to a return to homeostasis accompanied suppression of the innate immune system. C. An acute proinflammatory response secondary to activate of innate immunity, an anti-inflammatory response, and suppression of adaptive immunity occur currently following injury. D. Systemic inflammation following trauma is related to the immune response to microbes.

Answer: C Traumatic injury rapidly induces multiple responses from the immune system. There is (1) an acute proinflammatory response that is triggered by activation of the innate immune system via DAMPs, (2) an anti-inflammatory response that likely serves to modulate the pro-inflammatory response and return the host toward homeostasis, and (3) suppression of cellular-mediated adaptive immunity. Recent studies indicate that these three responses occur concurrently immediately following injury. (See Schwartz 11th ed., Ch. 2, p. 28.)

3. Which of the following is TRUE regarding damageassociated molecular pattern (DAMP) molecules? A. DAMPs function early in the immune response to trauma by activating adaptive immunity. B. Pathogen-associated molecular patterns (PAMPs) and DAMPs have different signaling characteristics and do not bind the same types of receptors. C. DAMPs are ligands for a group of receptors broadly term pattern recognition receptors (PRR), which includes toll-like receptors (TLRs). D. DAMPs signal only to immune cells.

Answer: C DAMPs are endogenous molecules that are produced during tissue damage or cellular stress. These molecules interact with both immune and nonimmune cells and function to activate the adaptive immune response early following injury. DAMPs are similar to PAMPs in both types of receptors they bind and their role in signaling during the immune response. Both DAMPs and PAMPs signal via PRRs, which includes receptors such as TLRs, RAGE, calcium-dependent receptors (CLRs), and nucleotide-binding domain, leucine-rich repeat-containing proteins (NLRs). (See Schwartz 11th ed., Ch. 2, p. 28.)

7

Brunicardi_Ch02_p007-012.indd 7

30/06/22 10:36 AM

8 CHAPTER 2 Systemic Response to Injury and Metabolic Support

4. High-mobility group protein B1 (HMGB1): A. Is one of the best-characterized damage-associated molecular pattern (DAMP) molecules, detectable in the circulation within 30 minutes of trauma. B. Is a protein secreted by immune-competent cells stimulated by pathogen-associated molecular patterns (PAMPs) or inflammatory cytokines. C. Is also secreted by endothelial cells, platelets, and also as a part of cell death. D. All of the above.

Answer: D The best-characterized DAMP in the context of the injuryassociated inflammatory response is high-mobility group protein B1 (HMGB1), which is rapidly released into the circulation within 30 minutes following trauma. Subsequent studies have proven, however, that HMGB1 is actively secreted from immune-competent cells stimulated by PAMPs (eg, endotoxin) or by inflammatory cytokines (eg, tumor necrosis factor and interleukin-1). Stressed nonimmune cells, such as endothelial cells, and platelet also actively secrete HMGB1. Finally, passive release of HMGB1 can occur following cell death, whether it is programmed or uncontrolled (necrosis). The diverse proinflammatory biological responses that result from HMGB1 signaling include: (1) the release of cytokines and chemokines from macrophage/monocytes and dendritic cells; (2) neutrophil activation and chemotaxis; (3) alterations in epithelial barrier function, including increased permeability; and (4) increased procoagulant activity on platelet surfaces; among others. (See Schwartz 11th ed., Ch. 2, p. 28.)

5. The most abundant amino acid in the human body is: A. Carnitine B. Arginine C. Glutamine D. Methionine

Answer: C Glutamine is the most abundant amino acid in the human body, comprising nearly two-thirds of the free intracellular amino acid pool. (See Schwartz 11th ed., Ch. 2, p. 69.)

6. What is the role of mitochondrial damage-associated molecular patterns (DAMPs) in the injury-mediated inflammatory response? A. Mitochondrial DNA (mtDNA) induces production of HMGB1. B. mtDNA and peptides from damaged mitochondria activate the macrophage inflammasome and the stimulator of interferon gene pathway (STING). C. mtDNA and peptides modulate the anti-inflammatory response that suppresses the adaptive immune system. D. mtDNA is directly toxic to the liver and lung in high amounts.

Answer: B Cell-free mtDNA has been shown to be significantly higher in the circulation of trauma patients than in normal controls. Mitochondrial proteins and/or DNA can act as DAMPs by triggering an inflammatory response to necrosis and cellular stress. Specifically, the release of mtDNA and formyl peptides from damaged or dysfunctional mitochondria has been implicated in activation of the macrophage inflammasome, a cytosolic signaling complex that responds to cellular stress. mtDNA also activates the STING. With stress or tissue injury, mtDNA and peptides are released from damaged mitochondria where they can contribute to a sterile inflammatory response. From an evolutionary perspective, given that eukaryotic mitochondria derive from bacterial origin, it would make sense that they retain bacterial features capable of eliciting a strong response that is typically associated with a pathogen trigger. (See Schwartz 11th ed., Ch. 2, p. 30.)

7. Which of the following statements is FALSE regarding the hypothalamic-pituitary-adrenal (HPA) axis and injury-associated stress? A. The HPA axis is initiated by the hypothalamusproducing corticotropin-releasing hormone (CRH) released from the paraventricular nucleus in response to inflammatory cytokines. B. CRH acts on the anterior pituitary to stimulate adrenocorticotropin hormone (ACTH) secretion into systemic circulation. C. CRH simulates the zona fasciculata of the adrenal gland to synthesize and secrete glucocorticoids. D. Insufficient cortisol in response to critical illness can lead to tachycardia, hypotension, weakness, hypoglycemia, hyponatremia, and hyperkalemia.

Answer: C The HPA axis is one of the main mechanism by which the brain responds to injury. CRH is released from the paraventricular nucleus in response to inflammatory cytokines. CRH acts on the anterior pituitary to stimulate the secretion of ACTH into the systemic circulation. ACTH acts on the zona fasciculata of the adrenal glands to synthesize and secrete glucocorticoids. Cortisol is the major glucocorticoid in humans and is essential for survival during significant physiologic stress. The resulting increase in cortisol levels following trauma has several important anti-inflammatory actions. Adrenal insufficiency represents a clinical syndrome highlighted largely by inadequate amounts of circulating cortisol and aldosterone. Classically, adrenal insufficiency is described in patients with atrophic adrenal glands caused by exogenous steroid

Brunicardi_Ch02_p007-012.indd 8

30/06/22 10:36 AM

9

Answer: A Hepatic-failure enteral nutrition formulas have protein content made up of ~50% branched-chain amino acids. The increase of branched-chain amino acids and reduction of aromatic amino acid levels may reverse encephalopathy in patients with hepatic failure; however, no clear benefit has been demonstrated in clinical trials. Lower fluid volume and concentrations of potassium, phosphorous, and magnesium are characteristic of renal-failure formulas. Predigested nutrients and minimal fat content are characteristic of elemental formulas, which are frequently used in patients with malabsorption, gut impairment, or pancreatitis. (See Schwartz 11th ed., Ch. 2, p. 71.)

9. Which of the following statements is TRUE regarding autophagy? A. It is an energy-dependent, controlled mechanism for removing senescent of dysfunctional cells, with no resulting activation of the inflammatory response. B. It is a mechanism by which cells dispose of damaged organelles and debris under normal circumstances, but can be used to provide additional nutrients during stress. C. It is NOT stimulated by cytokines, and does not play a role in cytokine signaling. D. It has been shown to contribute to lymphopenia in sepsis.

Answer: B Autophagy is a process whereby cells, under normal conditions, dispose of damaged organelles or debris. During conditions of stress, such as hypoxia or low cellular energy, autophagy is activated to provide additional nutrients. Increased levels of autophagy occur in activated immune cells and autophagy plays a role in the immune response. It is stimulated by Th1 cytokines and toll-like receptor (TLR) signaling in macrophages, and it is inhibited by Th2 cytokines. In turn, autophagy also regulates cytokine signaling by sequestering and degrading pro-IL-1B and inflammasome components. Answers A and D describe apoptosis. (See Schwartz 11th ed., Ch. 2, p. 40.)

10. Which of the following cytokines is produced primarily by CD4+ T cells and promotes T lymphocyte activity and proliferation? A. IL-10 B. IL-8 C. TNF-α D. IL-2

Answer: D IL-2 plays a critical role in the immune response. It is produced primarily by CD4+ T cells, but to a lesser degree by CD8+ T cells, NK T, mast, and activated dendritic cells. IL-2 promotes the cytolytic activity of CD8+ T and natural killer cells, and also promotes naïve CD4+ T cell differentiation into Th1 and Th2 phenotypes. In addition, IL-2 is essential for development and maintenance of T regulatory cells. (See Schwartz 11th ed., Ch. 2, p. 45.)

11. Which of the following statements best describes the role of IL-10 in the immune response to injury or stress? A. Proinflammatory mediator that is rapidly mobilized and is one of the first cytokines released following trauma, mediates coagulation, cell migration, and macrophage phagocytosis. B. Anti-inflammatory mediator that is secreted from a variety of immune cells and acts to inhibit secretion of proinflammatory cytokines, such as TNF and IL-1. C. Proinflammatory mediator that is secreted from a variety of immune cells, promotes B cell maturation, induces Th17 development and inhibits Treg differentiation, promotes angiogenesis, and increases vascular permeability. D. Proinflammatory mediator that is secreted primarily by macrophages and monocytes, and acts as a chemoattractant for neutrophils, basophils, eosinophils, and lymphocytes.

Answer: B IL-10 plays an important role in the immune response in that it acts in an anti-inflammatory role in order to reestablish immune homeostasis. It regulates the duration and magnitude of the inflammatory response. IL-10 is secreted by both myeloid and lymphoid immune cells, and its synthesis is upregulated during stress and inflammation. IL-10 inhibits the secretion of proinflammatory cytokines IL-1 and TNF through downregulation of NF-kB signaling. Answer A describes TNF-α, Answer C describes IL-6, and Answer D describes IL-8. (See Schwartz 11th ed., Ch. 2, p. 46.)

Brunicardi_Ch02_p007-012.indd 9

Systemic Response to Injury and Metabolic Support

8. Enteral nutritional formulas used to treat hepatic failure are characterized by: A. Lower fluid volume. B. Lower concentrations of potassium, phosphorous, and magnesium. C. Predigested nutrients and minimal fat content. D. ~50% branched-chain amino acids (eg, leucine, isoleucine, valine).

CHAPTER 2

administration who undergo a stressor, such as surgery. These patients subsequently manifest signs and symptoms, such as tachycardia, hypotension, weakness, nausea, vomiting, and fever. (See Schwartz 11th ed., Ch. 2, p. 35.)

30/06/22 10:36 AM

10 CHAPTER 2 Systemic Response to Injury and Metabolic Support

12. Which of the following statements is TRUE regarding transcriptional events following trauma? A. Over 80% of cellular functions and pathways in leukocytes demonstrate a shift in gene expression following blunt trauma. B. Changes in gene expression take 3 to 5 days to develop. C. Changes in gene expression occur rapidly and are transient, present for only 1 to 2 days. D. Patterns of gene expression varied widely when blunt trauma, burn injury, and endotoxemia were compared.

Answer: A Recent studies examining the transcriptomic response to blunt trauma in circulating leukocytes from human patients demonstrated a significant shift in the leukocyte transcriptomic with >80% of cellular functions and pathways demonstrating alterations. These pathways included those involved in innate and adaptive immunity, and compensatory anti-inflammatory mechanisms. Transcriptomic alterations occurred rapidly (4–12 hours) and persisted for days to weeks. Different injury patterns (blunt, burn, endotoxemia) demonstrated similar patterns of gene expression. (See Schwartz 11th ed., Ch. 2, p. 52.)

13. Which of the following statements about monocytes and macrophages is FALSE? A. Monocytes and macrophages are the primary effector cells to respond to danger signals through phagocytosis, release of inflammatory mediators, and recruitment of additional immune cells. B. Macrophages undergo M1 or M2 activation, with M1 macrophages expressing high levels of proinflammatory cytokines. C. Monocytes and tissue macrophages are derived from a common bone marrow progenitor. D. An imbalance of M1/M2 macrophage populations in the lung contributes to acute lung injury following hemorrhagic shock.

Answer: C Monocytes and macrophages are mononuclear cells that are characterized by their ability to phagocytize damage-associated molecular patterns (DAMPs) and pathogen-associated molecular patterns (PAMPs) and release inflammatory mediators to initiate the immune response to injury and inflammation by recruiting additional immune cells to the site of injury. Monocytes are derived from bone marrow progenitors, while tissue macrophages are derived from embryonic precursors. In response to external cues (PAMPs, DAMPs, activated lymphocytes), macrophages undergo M1 or M2 activation. M1 macrophages are characterized by high levels of expression of proinflammatory cytokines and are able to recruit additional macrophages to the site of injury. M2 macrophages are thought to promote a return to immune homeostasis through expression of arginase-1 and IL-10, and are additionally involved in wound repair. Recent data has shown that an imbalance between these two phenotypes in the lung contributes to acute lung injury after hemorrhagic shock due to a significant decrease in the M2 population and a delayed increase in the M1 population. (See Schwartz 11th ed., Ch. 2, p. 54.)

14. Platelet-specific TLR4 activation results in binding and activation of what cell type, with subsequent extraceullar trap formation? A. Macrophage B. CD8+ T cell C. Mast cell D. Neutrophil

Answer: D The role of the platelet in the immune response has been recently better defined. Platelets express TLRs and are able to detect pathogen-associated molecular patterns (PAMPs) and damage-associated molecular patterns (DAMPs). Plateletspecific TLR activation leads to platelets-binding and -activating neutrophils. Neutrophils then extrude their DNA to form neutrophil extracellular traps (NETs). This facilitates trapping of bacteria, but can also lead to endothelial cell damage. (See Schwartz 11th ed., Ch. 2, p. 56.)

15. Which of the following statements correctly pairs the type of CD4+ Th cell with its description? A. Th1: generally anti-inflammatory, attenuates killing of intracellular pathogens B. Th2: generally proinflammatory, involved in cellular immunity, promotes recognition and killing of intracellular pathogens C. Th17: strongly anti-inflammatory, characterized by IL-10 expression D. Th1: generally proinflammatory, promote recognition and killing of intracellular pathogens

Answer: D CD4+ helper T cells are divided into three subsets, Th1, Th2, and Th17, each with distinct roles in the immune response and a unique set of effector cytokines. Th1 cells are generally proinflammatory in nature and are involved in cellular immunity. These cells secrete IL-2, IL-3, IL-6, and TNF-α and promote recognition and killing of intracellular pathogens. Th2 cells are generally anti-inflammatory and play a role in humoral immunity by regulating antibody production. These cells secrete IL-3, IL-4, IL-5, and IL-10. Th17 cells have both pro- and anti-inflammatory implications and are thought to have two distinct phenotypes. A pathogenic Th17 phenotype

Brunicardi_Ch02_p007-012.indd 10

30/06/22 10:36 AM

11

Answer: D Metabolism in critically ill patients following injury or infection varies from that of unstressed fasting due to the effects of the immunologic and neuroendocrine responses. In the initial phases, there is an overall hypometabolic state with a subsequent rapid increase in energy expenditure at ~2 weeks. This increase in expenditure is mediated by sympathetic nervous system activation and catecholamine release. The magnitude of energy expenditure over time is proportional to the degree of insult, and thermal injuries and severe infections have the highest energy demands. Lipid metabolism is critically important in these disease states, as it is the predominant energy source and also impacts the structural integrity of cell membranes and the immune response. (See Schwartz 11th ed., Ch. 2, p. 62.)

17. What is an appropriate number of kcal and grams of protein per day for a 70-kg patient during a state of severe stress? A. 2100 kcal, 70 g B. 4200 kcal, 175 g C. 3360 kcal, 140 g D. 1925 kcal, 70 g

Answer: C Table 2-1 demonstrates caloric adjustments above basal energy expenditure (BEE) for various hypermetabolic conditions. The appropriate calculation for a patient in severe stress is to use a target of 30–35 kcal/kg/day with a 1.6x adjustment, and 2 g/kg/day of protein. (See Schwartz 11th ed., Ch. 2, Table 2-9, p. 68.)

TABLE 2-1

Caloric adjustments above basal energy expenditure (BEE) in hypermetabolic conditions

Condition

kcal/kg per day

Adjustment above BEE

Grams of Protein/kg per day

Nonprotein Calories: Nitrogen

Normal/moderate malnutrition

25–30

1.1

1.0

150:1

Mild stress

25–30

1.2

1.2

150:1

Moderate stress

30

1.4

1.5

120:1

Severe stress

30–35

1.6

2.0

90–120:1

Burns

35–40

2.0

2.5

90–100:1

18. Which of the following is NOT a reason that enteral nutrition is preferred to parenteral nutrition when possible? A. Lower cost of enteral feeding B. Lower risk of vascular access and infectious complications C. Fewer complications associated with gut disuse D. Improved immune response homeostasis

Brunicardi_Ch02_p007-012.indd 11

Systemic Response to Injury and Metabolic Support

16. Which of the following statements characterizes metabolism after injury and infection? A. An initial hypometabolic state following by a dramatic increase in energy expenditure at ~2 weeks. B. Adipose stores within the body are the predominant energy source during critical illness. C. Increased energy expenditure mediated by sympathetic activation and catecholamine release. D. All of the above.

CHAPTER 2

is characterized by increased IL-17 production, whereas a regulatory phenotype demonstrates increased IL-10 expression. (See Schwartz 11th ed., Ch. 2, p. 55.)

Answer: D Enteral nutrition (EN) is preferred to parenteral nutrition (PN) when feasible because of the lower cost of EN, lower risk of vascular access and infectious complications, and lower risk of gastrointestinal tract disuse complications. These complications include diminished soluble IgA and cytokine production, bacterial overgrowth, altered mucosal barrier function, and immune defenses. There is no evidence of improved immune homeostasis with the use of EN. Several studies have been performed, and a recent meta-analysis demonstrated a significant reduction in infectiously complications and length of ICU stay in critically ill patients receiving EN over PN. The positive impact of EN over PN may be more pronounced in malnourished patients. Further, there is evidence that initiating EN early (within 48 hours) at 80% of the estimated total energy goals provides benefit. (See Schwartz 11th ed., Ch. 2, p. 68.)

30/06/22 10:36 AM

This page intentionally left blank

Brunicardi_Ch02_p007-012.indd 12

30/06/22 10:36 AM

CHAPTER

3

Fluid and Electrolyte Management of the Surgical Patient 1. Metabolic acidosis with a normal anion gap (AG) occurs with: A. Diabetic acidosis. B. Renal failure. C. Severe diarrhea. D. Starvation.

Answer: C Metabolic acidosis with a normal AG results from either acid administration (HCl or NH4+) or a loss of bicarbonate from gastrointestinal (GI) losses, such as diarrhea, fistulas (enteric, pancreatic, or biliary), ureterosigmoidostomy, or from renal loss. The bicarbonate loss is accompanied by a gain of chloride, thus the AG remains unchanged. (See Schwartz 11th ed., Ch. 3, p. 91.)

2. All of the following are possible causes of postoperative hyponatremia EXCEPT: A. Excess infusion of normal saline intraoperatively. B. Administration of antipsychotic medication. C. Transient decrease in antidiuretic hormone (ADH) secretion. D. Excess oral water intake.

Answer: C Hyponatremia is caused by excess free water (dilution) or decreased sodium (depletion). Thus, excessive intake of free water (oral or IV) can lead to hyponatremia. Also, medications can cause water retention and subsequent hyponatremia, especially in older patients. Primary renal disease, diuretic use, and secretion of ADH are common causes of sodium depletion. ADH can be released transiently postoperatively, or less frequently, in syndrome of inappropriate ADH secretion. Lastly, pseudohyponatremia can be seen on laboratory testing when high serum glucose, lipid, or protein levels compromise sodium measurements. (See Schwartz 11th ed., Ch. 3, p. 86.)

3. Which of the following is an early sign of hyperkalemia? A. Peaked T waves B. Peaked P waves C. Peaked (shortened) QRS complex D. Peaked U waves

Answer: A Symptoms of hyperkalemia are primarily gastrointestinal (GI), neuromuscular, and cardiovascular. GI symptoms include nausea, vomiting, intestinal colic, and diarrhea; neuromuscular symptoms range from weakness to ascending paralysis to respiratory failure; while cardiovascular manifestations range from electrocardiogram (ECG) changes to cardiac arrhythmias and arrest. ECG changes that may be seen with hyperkalemia include: Peaked T waves (early change) Flattened P wave Prolonged PR interval (first-degree block) Widened QRS complex Sine wave formation Ventricular fibrillation (See Schwartz 11th ed., Ch. 3, p. 89.)

13

Brunicardi_Ch03_p013-020.indd 13

30/06/22 10:36 AM

14 CHAPTER 3 Fluid and Electrolyte Management of the Surgical Patient

4. Hypocalcemia may cause which of the following? A. Congestive heart failure B. Atrial fibrillation C. Pancreatitis D. Hypoparathyroidism

Answer: A Mild hypocalcemia can present with muscle cramping or digital/perioral paresthesias. Severe hypocalcemia leads to decreased cardiac contractility and heart failure. Electrocardiogram (ECG) changes of hypocalcemia include prolonged QT interval, T-wave inversion, heart block, and ventricular fibrillation. Hypoparathyroidism and severe pancreatitis are potential causes of hypocalcemia. (See Schwartz 11th ed., Ch. 3, p. 90.)

5. The next most appropriate test to order in a patient with a pH of 7.1, Pco2 of 40, sodium of 132, potassium of 4.2, and chloride of 105 is: A. Serum bicarbonate B. Serum magnesium C. Serum ethanol D. Serum salicylate

Answer: A Metabolic acidosis results from an increased intake of acids, an increased generation of acids, or an increased loss of bicarbonate. In evaluating a patient with a low serum bicarbonate level and metabolic acidosis, first measure the anion gap (AG), an index of unmeasured anions. AG = [Na] – [Cl + HCO3] Metabolic acidosis with an increased AG occurs from either exogenous acid ingestion (ethylene glycol, salicylate, or methanol) or endogenous acid production of β-hydroxybutyrate and acetoacetate in ketoacidosis, lactate in lactic acidosis, or organic acids in renal insufficiency. (See Schwartz 11th ed., Ch. 3, p. 92.)

6. Which of the following statement is FALSE regarding hypertonic saline? A. It is an arteriolar vasodilator and may increase bleeding. B. It should be avoided in closed head injury. C. It should not be used for initial resuscitation. D. It increases cerebral perfusion.

Answer: B Hypertonic saline (7.5%) has been used as a treatment modality in patients with closed head injuries. It has been shown to increase cerebral perfusion and decrease intracranial pressure, thus decreasing brain edema. However, there also have been concerns of increased bleeding because hypertonic saline is an arteriolar vasodilator. (See Schwartz 11th ed., Ch. 3, p. 94.)

7. Normal saline is: A. 135 mEq NaCl/L. B. 145 mEq NaCl/L. C. 148 mEq NaCl/L. D. 154 mEq NaCl/L.

Answer: D Sodium chloride is mildly hypertonic, containing 154 mEq of sodium that is balanced by 154 mEq of chloride. The high chloride concentration imposes a significant chloride load upon the kidneys and may lead to a hyperchloremic metabolic acidosis. It is an ideal solution, however, for correcting volume deficits associated with hyponatremia, hypochloremia, and metabolic alkalosis. (See Schwartz 11th ed., Ch. 3, p. 93.)

8. Fluid resuscitation using albumin: A. Is associated with coagulopathy. B. Is available as 1% or 5% solutions. C. Can lead to pulmonary edema. D. Decreases factor XIII.

Answer: C Albumin is available as 5% (osmolality of 300 mOsm/L) or 25% (osmolality of 1500 mOsm/L). Due to increased intravascular oncotic pressure, fluid is drawn into the intravascular space, leading to pulmonary edema when albumin is used for resuscitation for hypovolemic shock. Hydroxyethyl starch solutions are associated with postoperative bleeding in cardiac and neurosurgery patients. (See Schwartz 11th ed., Ch. 3, p. 94.)

9. Water constitutes what percentage of total body weight? A. 30%–40% B. 40%–50% C. 50%–60% D. 60%–70%

Answer: C Water constitutes approximately 50% to 60% of total body weight. The relationship between total body weight and total body water (TBW) is relatively constant for an individual and is primarily a reflection of body fat. Lean tissues, such as

Brunicardi_Ch03_p013-020.indd 14

30/06/22 10:36 AM

15

Answer: C Osmotic pressure is measured in units of osmoles (osm) or milliosmoles (mOsm) that refer to the actual number of osmotically active particles. For example, 1 millimole (mmol) of sodium chloride contributes to 2 mOsm (one from sodium and one from chloride). The principal determinants of osmolality are the concentrations of sodium, glucose, and urea (blood urea nitrogen [BUN]): Calculated serum osmolality = 2 sodium + glucose/18 + BUN/2.8 (See Schwartz 11th ed., Ch. 3, p. 84.)

11. What is the actual potassium of a patient with pH of 7.8 and serum potassium of 2.2? A. 2.2 B. 2.8 C. 3.2 D. 3.4

Answer: D The change in potassium associated with alkalosis can be calculated by the following formula:

12. The free water deficit of a 70-kg man with serum sodium of 154 is: A. 0.1 L B. 0.7 L C. 1 L D. 3.5 L

Answer: D This is the formula used to estimate the amount of water required to correct hypernatremia

13. A patient with serum calcium of 6.8 and albumin of 1.2 has a corrected calcium of: A. 7.7. B. 8.0. C. 8.6. D. 9.0.

Answer: D When measuring total serum calcium levels, the albumin concentration must be taken into consideration. Adjust total serum calcium down by 0.8 mg/dL for every 1 g/dL decrease in albumin. (See Schwartz 11th ed., Ch. 3, p. 90.)

14. All of the following treatments for hyperkalemia reduce serum potassium EXCEPT: A. Bicarbonate. B. Kayexalate. C. Glucose infusion with insulin. D. Calcium.

Answer: D When electrocardiogram (ECG) changes are present, calcium chloride or calcium gluconate (5–10 mL of 10% solution) should be administered immediately to counteract the myocardial effects of hyperkalemia. Calcium infusion should be used cautiously in patients receiving digitalis, because digitalis toxicity may be precipitated. Glucose and bicarbonate shift potassium intracellularly. Kayexalate is a cation

Brunicardi_Ch03_p013-020.indd 15

Fluid and Electrolyte Management of the Surgical Patient

10. If a patient’s serum glucose increases by 180 mg/dL, what is the increase in serum osmolality, assuming all other laboratory values remain constant? A. Does not change B. 8 C. 10 D. 12

CHAPTER 3

muscle and solid organs, have higher water content than fat and bone. As a result, young, lean men have a higher proportion of body weight as water than elderly or obese individuals. An average young adult male will have 60% of his total body weight as TBW, while an average young adult female’s will be 50%. The lower percentage of TBW in women correlates with a higher percentage of adipose tissue and lower percentage of muscle mass in most. Estimates of TBW should be adjusted down approximately 10% to 20% in obese individuals and up by 10% in malnourished individuals. The highest percentage of TBW is found in newborns, with approximately 80% of their total body weight composed of water. This decreases to about 65% by 1 year and thereafter remains fairly constant. (See Schwartz 11th ed., Ch. 3, p. 83.)

Potassium decreases by 0.3 mEq/L for every 0.1 increase in pH above normal (See Schwartz 11th ed., Ch. 3, p. 90.)

serum sodium − 140 × TBW 140 Estimate TBW (total body water) as 50% of lean body mass in men and 40% in women. (See Schwartz 11th ed., Ch. 3, p. 94.) Water deficit L =

30/06/22 10:36 AM

16 CHAPTER 3

exchange resin that binds potassium, either given enterally or as an enema. (See Schwartz 11th ed., Ch. 3, p. 95.)

Fluid and Electrolyte Management of the Surgical Patient

15. An alcoholic patient with serum albumin of 3.9, K of 3.1, Mg of 2.4, Ca of 7.8, and PO4 of 3.2 receives three boluses of IV potassium and has serum potassium of 3.3. You should: A. Continue to bolus potassium until the serum level is >3.6. B. Give MgSO4 intravenously. C. Check the ionized calcium. D. Check the blood urea nitrogen (BUN) and creatinine.

Answer: B Magnesium depletion is a common problem in hospitalized patients, particularly in the ICU. The kidney is primarily responsible for magnesium homeostasis through regulation by calcium/magnesium receptors on renal tubular cells that sense serum magnesium levels. Hypomagnesemia results from a variety of etiologies ranging from poor intake (starvation, alcoholism, prolonged use of IV fluids, and total parenteral nutrition with inadequate supplementation of magnesium), increased renal excretion (alcohol, most diuretics, and amphotericin B), gastrointestinal (GI) losses (diarrhea), malabsorption, acute pancreatitis, diabetic ketoacidosis, and primary aldosteronism. Hypomagnesemia is important not only for its direct effects on the nervous system, but also because it can produce hypocalcemia and lead to persistent hypokalemia. When hypokalemia or hypocalcemia coexist with hypomagnesemia, magnesium should be aggressively replaced to assist in restoring potassium or calcium homeostasis. (See Schwartz 11th ed., Ch. 3, p. 91.)

16. Calculate the daily maintenance fluids needed for a 60-kg woman. A. 2060 B. 2300 C. 2360 D. 2400

Answer: B A 60-kg woman would receive a total of 2300 mL of fluid daily: 1000 mL for the first 10 kg of body weight (10 kg × 100 mL/kg/day), 500 mL for the next 10 kg (10 kg × 50 mL/ kg/day), and 800 mL for the last 40 kg (40 kg × 20 mL/kg/ day). (See Schwartz 11th ed., Ch. 3, p. 96.)

17. A patient who has spasms in the hand when a blood pressure cuff is blown up most likely has: A. Hypercalcemia B. Hypocalcemia C. Hypermagnesemia D. Hypomagnesemia

Answer: B Asymptomatic hypocalcemia may occur with hypoproteinemia (normal ionized calcium), but symptoms can develop with alkalosis (decreased ionized calcium). In general, symptoms do not occur until the ionized fraction falls below 2.5 mg/dL, and are neuromuscular and cardiac in origin, including paresthesias of the face and extremities, muscle cramps, carpopedal spasm, stridor, tetany, and seizures. Patients will demonstrate hyperreflexia and positive Chvostek sign (spasm resulting from tapping over the facial nerve) and Trousseau sign (spasm resulting from pressure applied to the nerves and vessels of the upper extremity, as when obtaining a blood pressure). Decreased cardiac contractility and heart failure can also accompany hypocalcemia. (See Schwartz 11th ed., Ch. 3, p. 90.)

18. The actual anion gap (AG) of a chronic alcoholic with Na 133, K 4, Cl–101, HCO3– 22, albumin of 2.5 mg/dL is: A. 6. B. 10. C. 14. D. 15.

Answer: D The normal AG is 100 mg/dL B. Hyperkalemia C. Severe acidosis D. Uremic pericarditis E. Uremic encephalopathy

Answer: A Hyperkalemia, severe acidosis, uremic encephalopathy, and uremic pericarditis are all indications of life-threatening problems, and urgent correction is mandatory. Elevation of BUN is commonly seen as well, but is not itself an indication for dialysis. (See Schwartz 11th ed., Ch. 3, p. 99.)

27. An elderly diabetic patient who has acute cholecystitis is found to have a serum sodium level of 122 mEq/L and a blood glucose of 600 mg/dL. After correcting the glucose concentration to 100 mg/dL with insulin, the serum sodium concentration would: A. Decrease significantly unless the patient also received 3% saline. B. Decrease transiently but return to approximately 122 mEq/L without specific therapy. C. Remain essentially unchanged. D. Increase to the normal range without specific therapy.

Answer: D A rise in the extracellular fluid concentration of a substance that does not diffuse passively across cell membranes (eg, glucose or urea) causes an increase in effective osmotic pressure, a transfer of water from cells, and dilutional hyponatremia. For each 100 mg/dL rise in blood glucose above normal, the serum sodium level falls approximately to 3 mEq/L. Alternatively, the serum sodium level would increase by about 15 mEq/L if the blood glucose level falls from 600 to 100 mg/dL. (See Schwartz 11th ed., Ch. 3, p. 88.)

28. Excessive administration of normal saline for fluid resuscitation can lead to what metabolic derangement? A. Metabolic alkalosis B. Metabolic acidosis C. Respiratory alkalosis D. Respiratory acidosis

Answer: B Sodium chloride is mildly hypertonic, containing 154 mEq of sodium that is balanced by 154 mEq of chloride. The high chloride concentration imposes a significant chloride load on the kidneys and may lead to a hyperchloremic metabolic acidosis. Sodium chloride is an ideal solution, however, for correcting volume deficits associated with hyponatremia, hypochloremia, and metabolic alkalosis. (See Schwartz 11th ed., Ch. 3, p. 93.)

29. The first step in the management of acute hypercalcemia should be: A. Correction of deficit of extracellular fluid volume. B. Hemodialysis. C. Administration of furosemide. D. Administration of mithramycin.

Answer: A Patients with acute hypercalcemia usually have either acute hyperparathyroidism or metastatic breast carcinoma with multiple bony metastases. These patients develop severe headaches, bone pain, thirst, emesis, and polyuria. Unless treatment is instituted promptly, the symptoms may be rapidly fatal. Immediate correction of the associated deficit of extracellular fluid volume is the most important step in treatment. When effective, this results in the lowering of the serum calcium level by dilution. Once extracellular fluid volume has been replaced, furosemide is effective treatment. Hemodialysis may also be employed, but its effect is less rapid. Mithramycin is very useful in controlling metastatic bone disease, but its effect is slow, and it cannot be depended upon when the patient has acute hypercalcemia. (See Schwartz 11th ed., Ch. 3, p. 95.)

Brunicardi_Ch03_p013-020.indd 18

30/06/22 10:36 AM

19

31. Three days after surgery for gastric carcinoma, a 50-yearold alcoholic male exhibits delirium, muscle tremors, and hyperactive tendon reflexes. Magnesium deficiency is suspected. All of the following statements regarding this situation are TRUE EXCEPT: A. A decision to administer magnesium should be based on the serum magnesium level. B. Adequate cellular replacement of magnesium will require 1 to 3 weeks. C. A concomitant calcium deficiency should be suspected. D. Calcium is a specific antagonist of the myocardial effects of magnesium.

Answer: A Magnesium deficiency should be suspected in any malnourished patient who exhibits disturbed neuromuscular or cerebral activity in the postoperative period. Laboratory confirmation often is not reliable, and the syndrome may exist in the presence of a normal serum magnesium level. Hypocalcemia often coexists, particularly in patients who have clinical signs of tetany. Intravenous magnesium can be administered safely to a well-hydrated patient for initial treatment of a severe deficit, but concomitant electrocardiographic monitoring is essential. The electrocardiographic changes associated with acute hypermagnesemia resemble those of hyperkalemia, and calcium chloride or gluconate should be readily available to counteract any adverse myocardial effects of excess magnesium ions. Partial or complete relief of symptoms may follow the initial infusion of magnesium, although continued replacement for a period of 1 to 3 weeks is necessary to replenish cellular stores. (See Schwartz 11th ed., Ch. 3, pp. 19, 95.)

32. Refeeding syndrome can be associated with all of the following EXCEPT: A. Respiratory failure. B. Hyperkalemia. C. Confusion. D. Cardiac arrhythmias.

Answer: B With refeeding, a shift in metabolism from fat to carbohydrate substrate stimulates insulin release, which results in the cellular uptake of electrolytes, particularly phosphate, magnesium, potassium, and calcium. However, severe hyperglycemia may result from blunted basal insulin secretion. (See Schwartz 11th ed., Ch. 3, p. 98.)

Brunicardi_Ch03_p013-020.indd 19

Fluid and Electrolyte Management of the Surgical Patient

Answer: C In patients suffering from hemorrhagic shock, the presence of a metabolic acidosis early in the postresuscitative period is indicative of tissue hypoxia due to persistent inadequate tissue perfusion. Attempts to correct this problem by administering an alkalizing agent will not solve the basic problem. However, proper volume replacement by means of a balanced salt solution such as Lactated Ringer solution will restore perfusion and correct the metabolic acidosis by ending anaerobic metabolism. (See Schwartz 11th ed., Ch. 3, p. 92.)

CHAPTER 3

30. A victim of a motor vehicle accident arrives in hemorrhagic shock. His arterial blood gases are pH, 7.25; Po2, 95 mm Hg; Pco2, 25 mm Hg; HCO3–, 15 mEq/L. The patient’s metabolic acidosis would be treated best with: A. Ampule of sodium bicarbonate. B. Sodium bicarbonate infusion. C. Lactated Ringer solution. D. Hyperventilation.

30/06/22 10:36 AM

This page intentionally left blank

Brunicardi_Ch03_p013-020.indd 20

30/06/22 10:36 AM

CHAPTER

4

Hemostasis, Surgical Bleeding, and Transfusion

1. Which of the following is NOT one of the four major physiological events of hemostasis? A. Vasodilation B. Platelet aggregation C. Clot Formation D. Fibrinolysis

Answer: A Hemostasis is a complex process, which limits blood loss from an injured vessel. The four physiologic events of the hemostatic process include vasoconstriction, platelet aggregation, clot formation, and fibrinolysis. These processes are the result of two different cascades, the extrinsic pathway and intrinsic pathway. Both cause the conversion of prothrombin to thrombin to fibrin, which are essential for clot formation. (Schwartz 11th ed., p. 103.)

2. Which of the following is required for platelet adherence to injured endothelium? A. Thromboxane A2 B. Glycoprotein (GP) IIb/IIIa C. Adenosine diphosphate (ADP) D. Von Willebrand factor (vWF)

Answer: D Platelets do not normally adhere to each other or to the vessel wall but can form a plug that aids in cessation of bleeding when vascular disruption occurs. Injury to the intimal layer in the vascular wall exposes subendothelial collagen to which platelets adhere. This process requires vWF, a protein in the subendothelium that is lacking in patients with von Willebrand’s disease. vWF binds to glycoprotein (GP) I/IX/V on the platelet membrane. Following adhesion, platelets initiate a release reaction that recruits other platelets from the circulating blood to seal the disrupted vessel. Up to this point, this process is known as primary hemostasis. Platelet aggregation is reversible and is not associated with secretion. Additionally, heparin does not interfere with this reaction, and thus, hemostasis can occur in the heparinized patient. ADP and serotonin are the principal mediators in platelet aggregation. (Schwartz 11th ed., p. 103.)

3. Which of the following clotting factors is the first factor common to both intrinsic and extrinsic pathways? A. Factor I (fibrinogen) B. Factor IX (Christmas factor) C. Factor X (Stewart-Prower factor) D. Factor XI (plasma thromboplasma antecedent)

Answer: C The intrinsic pathway begins with the activation of factor XII that subsequently activates factors XI, IX, and VIII. In this pathway, each of the primary factors is “intrinsic” to the circulating plasma, whereby no surface is required to initiate the process. In the extrinsic pathway, tissue factor (TF) is released or exposed on the surface of the endothelium, binding to circulating factor VII, facilitating its activation to VIIa. Each of these pathways continues on to a common sequence that begins with the activation of factor X to Xa (in the presence of VIIIa). Subsequently, Xa (with the help of factor Va) converts factor II (prothrombin) to thrombin and then factor I (fibrinogen) to fibrin. Clot formation occurs after fibrin monomers are cross-linked to polymers with the assistance of factor XIII. (Schwartz 11th ed., p. 105.)

21

Brunicardi_Ch04_p021-028.indd 21

30/06/22 10:37 AM

22 CHAPTER 4 Hemostasis, Surgical Bleeding, and Transfusion

4. Which of the following factors are NOT involved in preventing clot propagation? A. Thrombomodulin B. Tissue plasminogen activator (tPA) release by the endothelium C. Nitric oxide release from the endothelium D. APC complexes with protein S

Answer: C Feedback inhibition on the coagulation cascade deactivates the enzyme complexes leading to thrombin formation. Thrombomodulin (TM) presented by the endothelium serves as a “thrombin sink” by forming a complex with thrombin, rendering it no longer available to cleave fibrinogen. This then activates protein C (APC) and reduces further thrombin generation by inhibiting factors V and VIII. Second, tissue plasminogen activator (tPA) is released from the endothelium following injury, cleaving plasminogen to initiate fibrinolysis. APC then consumes plasminogen activator inhibitor-1 (PAI-1), leading to increased tPA activity and fibrinolysis. Building on the anticoagulant response to inhibit thrombin formation, tissue factor pathway inhibitor (TFPI) is released, blocking the TF-VIIa complex and reducing the production of factors Xa and IXa. Antithrombin III (AT-III) then neutralizes all of the procoagulant serine proteases and also inhibits the TF-VIIa complex. The most potent mechanism of thrombin inhibition involves the APC system. APC forms a complex with its cofactor, protein S, on a phospholipid surface. This complex then cleaves factors Va and VIIIa so that they are no longer able to participate in the formation of TF-VIIa or prothrombinase complexes. (Schwartz 11th ed., p. 106.)

5. Which of the following congenital factor deficiency often presents in the first few days of life? A. Factor VII B. Factor IX C. Factor XI D. Factor XII

Answer: D Congenital factor XIII (FXIII) deficiency, originally recognized by Duckert in 1960, is a rare autosomal recessive disease usually associated with a severe bleeding diathesis. The maleto-female ratio is 1:1. Although acquired FXIII deficiency has been described in association with hepatic failure, inflammatory bowel disease, and myeloid leukemia, the only significant association with bleeding in children is the inherited deficiency. Bleeding is typically delayed because clots form normally but are susceptible to fibrinolysis. Umbilical stump bleeding is characteristic, and there is a high risk of intracranial bleeding. Spontaneous abortion is usual in women with factor XIII deficiency unless they receive replacement therapy. Replacement can be accomplished with FFP, cryoprecipitate, or a factor XIII concentrate. Levels of 1% to 2% are usually adequate for hemostasis. (Schwartz 11th ed., p. 107.)

6. Which of the following is NOT a cause of thrombocytopenia? A. Immune thrombocytopenia purpura (ITP) B. Hemolytic uremia syndrome (HUS) C. B12 deficiency D. Clopidogrel administration

Answer: D Decrease in number of circulating platelets can be the result of (1) failure of production, (2) shortened survival, and (3) sequestration. Failure of production occurs in bone marrow disorders, including leukemias, myelodysplastic syndromes, severe B12 deficiency, folate deficiency, chemotherapy, radiation treatment, alcohol intoxication, and viral syndromes. Shortened platelet survival is seen in idiopathic thrombocytopenia purpura, heparin-induced thrombocytopenia (HIT), thrombotic thrombocytopenia purpura (TTP), and in hemolytic uremia syndrome (HUS). Sequestration occurs with trapping of platelets in enlarged spleen secondary to portal hypertension, sarcoid, lymphoma, or Gaucher’s disease. Clopidogrel does not cause a decrease in platelet number, but irreversibly inhibits platelet function. (Schwartz 11th ed., p. 108.)

Brunicardi_Ch04_p021-028.indd 22

30/06/22 10:37 AM

23

8. Which of the following is NOT an acquired platelet hemostatic defect? A. Massive blood transfusion following Trauma B. Acute renal failure C. Disseminated intravascular coagulation (DIC) D. Polycythemia vera

Answer: C Impaired platelet function often accompanies thrombocytopenia, but may also occur in the presence of a normal platelet count. The importance of this is obvious when one considers that 80% of overall clot strength is related to platelet function. The life span of platelets ranges from 7 to 10 days, placing them at increased risk for impairment by medical disorders and prescription and over-the-counter medications. Impairment of ADP-stimulated aggregation occurs with massive transfusion of blood products. Uremia may be associated with increased bleeding time and impaired aggregation. Defective aggregation and platelet dysfunction are also seen in patients with severe trauma, thrombocythemia, polycythemia vera, and myelofibrosis. DIC is an acquired syndrome characterized by systemic activation of coagulation pathways that result in excessive thrombin generation and diffuse formation of microthrombi. (Schwartz 11th ed., p. 110.)

9. What drug irreversibly inhibits platelet function by irreversible acetylation of platelet prostaglandin synthase? A. Aspirin B. Clopidogrel C. Dipyridamole D. Glycoprotein IIB/IIIA inhibitors

Answer: A Drugs that interfere with platelet function include aspirin, clopidogrel, prasugrel, dipyridamole, and GP IIb/IIIa inhibitors. Aspirin, clopidogrel, and prasugrel all irreversibly inhibit platelet function. Clopidogrel and prasugrel do so through selective irreversible inhibition of ADP-induced platelet aggregation. Aspirin works through irreversible acetylation of platelet prostaglandin synthase. (Schwartz 11th ed., p. 110.)

10. Which is TRUE about trauma-induced coagulopathy (TIC)? A. The acute coagulopathy of trauma is mechanistically similar to disseminated intravascular coagulation (DIC). B. Coagulopathy can develop in trauma patients following acidosis, hypothermia, and dilution of coagulation factors though coagulation is normal on admission. C. TIC is caused by shock and tissue injury. D. Acute coagulopathy of trauma is mainly a dilutional coagulopathy.

Answer: C Traditional teaching regarding trauma-related coagulopathy attributed its development to acidosis, hypothermia, and dilution of coagulation factors. Recent data, however, have shown that over one-third of severely injured patients have laboratorybased evidence of coagulopathy at the time of admission, a phenotype called trauma- TIC. TIC is independent of traditional (iatrogenic) causes of posttraumatic coagulopathy, such as hemodilution, is precipitated by tissue injury and/or hemorrhagic shock, and is associated with significantly higher risk of mortality, especially in the first 24 hours after injury. Furthermore, TIC is a separate and distinct process from disseminated intravascular coagulopathy with its own specific components of hemostatic failure. (Schwartz 11th ed., p. 111.)

Brunicardi_Ch04_p021-028.indd 23

Hemostasis, Surgical Bleeding, and Transfusion

Answer: B Primary immune thrombocytopenia is also known as idiopathic thrombocytopenic purpura (ITP). In children, it is usually acute at the onset, short-lived, and typically follows a viral illness. In contrast, ITP in adults is gradual in onset, chronic in nature, and has no identifiable cause. Because the circulating platelets in ITP are young and functional, bleeding is less for a given platelet count than when there is failure of platelet production. The pathophysiology of ITP is believed to involve both impaired platelet production and T cell–mediated platelet destruction. Heparin-induced thrombocytopenia (HIT) is a form of drug-induced immune thrombocytopenia. It is an immunologic event during which antibodies against platelet factor 4 formed during exposure to heparin affect platelet activation and endothelial function with resultant thrombocytopenia and intravascular thrombosis. TTP is a disorder of platelet activation and production of platelet thrombi. (Schwartz 11th ed., p. 108.)

CHAPTER 4

7. Primary immune thrombocytopenia (ITP): A. Occurs more often in children with adults, but has a similar clinical course. B. Includes HIT as a subtype of drug-induced ITP. C. is also known as thrombotic thrombocytopenic purpura (TTP). D. is a disease of impaired platelet production of unknown etiology.

30/06/22 10:37 AM

24 CHAPTER 4 Hemostasis, Surgical Bleeding, and Transfusion

11. Warfarin use is often associated with an increased morbidity and mortality in acutely injured and emergency surgery patients, with rapid reversal, these complications can be reduced: Which is NOT TRUE about rapid reversal of warfarin effect? A. Vitamin K should be given to sustain the effects of plasma and prothrombin complex concentrate (PCC). B. PCC is superior to plasma. C. PCC more rapidly corrects INR but is associated with excess thromboembolic events. D. Four-factor PCC’s have more reliable correction of INR compared to three-factor PCCs.

Answer: B Although warfarin use is often associated with a significant increase in morbidity and mortality in acutely injured and emergency surgery patients, with rapid reversal, these complications can be reduced. There are several reversal options that include vitamin K administration, plasma, cryoprecipitate, recombinant factor VIIa, and factor concentrates. The 2012 CHEST guidelines for the Management of Anticoagulant Therapy, Antithrombotic Therapy, and Prevention of Thrombosis recommends patients with major life-threatening bleeding due to warfarin receive reversal with vitamin K and a rapid reversal agent such as plasma or PCC. Vitamin K is given to sustain the effects of the plasma or PCC due to their short half-lives. In major bleeds, vitamin K 10 mg given as a slow IV infusion is utilized for more rapid onset compared to the oral form. Studies have shown that PCC is superior to plasma for speed of reversal and has decreased risk of fluid overload, but it is equivalent in adverse and thromboembolic events and costlier. PCC is available in two forms: three-factor PCC (factors II, IX, and X) and four-factor PCC (factors II, VII, IX, and X). Four-factor PCCs have been shown to have a more reliable correction of INR compared to three-factor PCCs. (Schwartz 11th ed., p. 112.)

12. What is the best laboratory test for determine degree of anticoagulation with direct oral anticoagulants such as dabigatran and rivaroxaban? A. Prothrombin time/International normalized ratio (PT/INR) B. Partial thromboplastin time (PTT) C. Bleeding time D. None of the above

Answer: D Direct oral anticoagulants (DOACs) include direct thrombin inhibitors and factor Xa inhibitors and have no readily available method of detection of the degree of anticoagulation. More concerning is the difficulty in the reversal of these new anticoagulants. Recently, idarucizumab, a humanized monoclonal antibody fragment that binds dabigatran, has been approved for use for reversal of the thrombin inhibitor, dabigatran, and dabigatran-related coagulopathy. (Schwartz 11th ed., p. 113.)

13. A 45-year-old man is now postoperative day 6 after colon resection for perforated diverticulitis, and is noted to have new bruising at venipuncture sites. Platelet count is measured at 45,000 platelets per milliliter. You suspect this is due to Heparin-induced thrombocytopenia (HIT). Which of the following is TRUE? A. HIT is due to an antibody against platelet factor 4 (PF4). B. This can only occur with full dose unfractionated heparin. C. HIT is avoided by using fractionated heparins only. D. Anticoagulation with oral warfarin should be initiated immediately.

Answer: A HIT is a drug-induced immune thrombocytopenia. Antibodies against platelet factor 4 (PL4) cause destruction of platelets. Platelet count falls on 5 to 7 days after starting heparin therapy, but sooner, 1 to 2 days after reexposure. HIT generally occurs after treatment with full-dose unfractionated heparins, but can occur with prophylactic doses or with low molecular weight (fractionated) heparins. The clinical diagnosis of HIT is confirmed by a positive anti-platelet factor 4. In addition to thrombocytopenia, HIT is associated with arterial and venous thrombosis. Heparins are discontinued, and a direct thrombin inhibitor, such as lepirudin, argatroban, or danaparoid is prescribed. Warfarin therapy should be started only after anticoagulation it can initially induce a hypercoagulable state (Schwartz 11th ed., p. 109.)

14. Which findings are not consistent with thrombotic thrombocytopenic purpura (TTP)? A. Splenomegaly B. Fever C. Schistocytes on peripheral blood smear D. Platelet activation

Answer: D In TTP inhibition of a metalloproteinase enzyme, ADAM S13 allows for unrestrained growth of microthrombi. Von Willebrand factor (VWF) is secreted as large molecules. Normally, ADAM S13 cleaves large vWF molecules which limits thrombi growth and prevents microvascular thrombosis. In TTP, the microvascular thrombosis leads to tissue

Brunicardi_Ch04_p021-028.indd 24

30/06/22 10:37 AM

25

Answer: C Under normal conditions, homeostasis of the coagulation system is maintained by complex interactions between the endothelium, platelets, and coagulation factors. In patients undergoing CPB, contact with circuit tubing and membranes results in abnormal platelet and clotting factor activation, as well as activation of inflammatory cascades, that ultimately results in excessive fibrinolysis and a combination of both quantitative and qualitative platelet defects. Platelets undergo reversible alterations in morphology and their ability to aggregate, which causes sequestration in the filter, partially degranulated platelets, and platelet fragments. This multifactorial coagulopathy is compounded by the effects of shear stress in the system, induced hypothermia, hemodilution, and anticoagulation. (Schwartz 11th ed., p. 113.)

16. Which of the following facts about transfusion and crossmatching is FALSE? A. Universal donor type O-negative red blood cells and type AB plasma may be transfused to all recipients. B. Platelets also require crossmatching. C. The administration of Rh-positive red blood cells is acceptable if Rh-negative red blood cells blood is not available. D. Crossmatched whole blood may be ideal therapy for resuscitation of trauma patients.

Answer: C Platelets do not require crossmatching. In emergency situations, universal donor type O-negative red blood cells and type AB plasma may be transfused to all recipients. Due to a shortage of type AB plasma, low anti-B titer type A plasma has become widely adopted for emergency (uncrossmatched) transfusion. In the United States, 85% of individuals are type A or type O, making type A plasma compatible with the vast majority of potential recipients. Uncrossmatched plasma is routinely transfused as part of platelet transfusions, with major transfusion reactions reported rarely, and type AB plasma currently carries a higher risk of TRALI compared to other plasma types. Many centers have transitioned to low-titer type A plasma for emergency transfusions, with no increase in adverse events. O negative and type-specific red blood cells are equally safe for emergency transfusion. In patients known to have clinically significant cold agglutinins, blood should be administered through a blood warmer. If these antibodies are present in high titer, hypothermia is contraindicated. Whole blood as an ideal therapy for acute traumatic hemorrhagic shock has increased in the last several years with multiple reports of successful use in military and civilian trauma patients. However, there is still limited access in most civilian centers. (Schwartz 11th ed., p. 115.)

17. Following recent abdominal surgery, your patient is admitted to the ICU with septic shock. Below what level of hemoglobin would a blood transfusion be indicated? A. 20 mph, lack of restraint use, and lateral impact. Lowenergy trauma, such as being struck with a club or falling from a bicycle, usually does not result in widely distributed injuries. However, potentially lethal injuries of internal organs can occur because the net energy transfer to any given location may be substantial. (See Schwartz 11th ed., Ch. 7, p. 196.)

26. When performing diagnostic peritoneal lavage (DPL) to rule out diaphragm injury, the appropriate laboratory cutoff values to use are: A. RBC >7500/mL, WBC >350/mL, Amylase >10 IU/L, AP >1.5 IU/L, Bilirubin >0.01 mg/dL B. RBC >15000/mL, WBC >750/mL, Amylase >15 IU/L, AP >2.5 IU/L, Bilirubin >0.05 mg/dL C. RBC >10000/mL, WBC >500/mL, Amylase >19 IU/L, AP >2.0 IU/L, Bilirubin >0.01 mg/dL D. RBC >12500/mL, WBC >1000/mL, Amylase >12 IU/L, AP >1.5 IU/L, Bilirubin >0.10 mg/dL

Answer: C Penetrating thoracoabdominal wounds may cause occult injury to the diaphragm. Patients with gunshot or stab wounds to the left lower chest should be evaluated with diagnostic laparoscopy or DPL to exclude diaphragmatic injury. In general, penetrating right diaphragm injury is ignored unless there is a major underlying liver injury with a risk of biliopleural fistula. Diagnostic laparoscopy may be preferred in patients with a positive chest radiograph (hemothorax or pneumothorax) or in those who would not tolerate a DPL. For patients undergoing DPL evaluation, laboratory value cutoffs to rule out diaphragm injury are different from traditional values formerly used for abdominal stab wounds (Table 7-3). An RBC count of >10,000/μL is considered a positive finding and an indication for abdominal evaluation; patients with a DPL RBC count between 1000/μL and 10,000/μL should

Brunicardi_Ch07_p049-064.indd 59

Trauma

Answer: D Major air leak occurs from tracheobronchial injuries. Type I injuries are those occurring within 2 cm of the carina. These may not be associated with a pneumothorax due to the envelopment in the mediastinal pleura. Type II injuries are more distal injuries within the tracheobronchial tree and hence manifest with a pneumothorax. Bronchoscopy confirms the extent of the injury and its location, and directs management. (See Schwartz 11th ed., Ch. 7, p. 187.)

CHAPTER 7

23. Which of the following describes a type I tracheobronchial injury? A. Occurring within 3 cm of the mainstem bronchus B. Likely to cause a pneumothorax C. A distal bronchial injury D. Occurring within 2 cm of the carina

04/07/22 1:26 PM

60 CHAPTER 7

undergo laparoscopy or thoracoscopy. (See Schwartz 11th ed., Ch. 7, p. 202.) TABLE 7-3

Trauma 27. Focused assessment with sonography in trauma (FAST) examination is sensitive for detecting what volume of intraperitoneal fluid? A. >100 cc B. >250 cc C. >500 cc D. >1000 cc

Hemodynamically stable

No

Peritonitis?

Abdominal Trauma

Thoracoabdominal Stab Wounds

Red blood cell count

>100,000/mL

>10,000/mL

White blood cell count

>500/mL

>500/mL

Amylase level

>19 IU/L

>19 IU/L

Alkaline phosphatase level

>2 IU/L

>2 IU/L

Bilirubin level

>0.01 mg/dL

>0.01 mg/dL

Answer: B Blunt abdominal trauma is now evaluated initially by FAST examination, and this has supplanted diagnostic peritoneal lavage (DPL) (Fig. 7-3). FAST is not 100% sensitive, however, so diagnostic peritoneal aspiration is warranted in hemodynamically unstable patients without a defined source of blood loss to rule out abdominal hemorrhage. FAST is used to identify free intraperitoneal fluid in Morrison’s pouch, the left upper quadrant, and the pelvis. Although this method is sensitive for detecting intraperitoneal fluid of >250 mL, it does not reliably determine the source of hemorrhage nor grade solid organ injuries. Patients with fluid on FAST examination, considered a “positive FAST,” who do not have immediate indications for laparotomy (hemodynamically stable, no evidence of peritonitis) undergo CT scanning to quantify their injuries. (See Schwartz 11th ed., Ch. 7, p. 202.)

FAST +

No

No

Yes Yes

No

FAST +

Yes Equivocal

Laparotomy

No

Criteria for “positive” finding on diagnostic peritoneal lavage

Candidate for nonoperative management or patient with cirrhosis

Indications for CT: -Altered mental status -Confounding injury -Gross hematuria -Pelvic fracture -Abdominal tenderness -Unexplained Hct 2.0 g/dL B. Alkaline phosphatase >250 IU/L C. Transfusion of 4U RBC in 6 hours, or 6U over 24 hours D. Persistent right upper quadrant pain

Answer: C Nonoperative management of solid organ injuries is pursued in hemodynamically stable patients who do not have overt peritonitis or other indications for laparotomy. Patients with >grade II injuries should be admitted to the surgical intensive care unit (SICU) with frequent hemodynamic monitoring, determination of hemoglobin, and abdominal examination. The only absolute contraindication to nonoperative management is hemodynamic instability from intraperitoneal hemorrhage. Factors such as high injury grade, large hemoperitoneum, contrast extravasation, or pseudoaneurysms may predict complications or failure of nonoperative management. Angioembolization and endoscopic retrograde cholangiopancreatography (ERCP) are useful adjuncts that can improve the success rate of nonoperative management. The indication for angiography to control hepatic hemorrhage is transfusion of 4 units of RBCs in 6 hours or 6 units of RBCs in 24 hours attributable to the liver. (See Schwartz 11th ed., Ch. 7, p. 225.)

Brunicardi_Ch07_p049-064.indd 61

Trauma

29. A left medial visceral rotation is appropriate for exposing injuries to all of the following EXCEPT: A. Inferior vena cava (IVC). B. Celiac axis. C. Proximal superior mesenteric artery (SMA). D. Left renal artery.

CHAPTER 7

the left subclavian artery is injured outside the thoracic outlet, vascular control can be obtained via the sternotomy and definitive repair done through the supraclavicular incision. Emergent median sternotomy is optimal for anterior stab wounds to the heart. Typically, these patients have pericardial tamponade and may undergo placement of a pericardial drain before a semiurgent median sternotomy is performed. Patients in extremis, however, should undergo anterolateral thoracotomy. (See Schwartz 11th ed., Ch. 7, p. 210.)

04/07/22 1:26 PM

62 CHAPTER 7

Answer: D The most commonly missed gastric injury is the posterior wound of a through and through penetrating injury. Injuries also can be overlooked if the wound is located within the mesentery of the lesser curvature or high in the fundus. To delineate a questionable injury, the stomach can be digitally occluded at the pylorus while methylene blue-colored saline is instilled via a nasogastric (NG) tube. Alternatively, air can be introduced via the NG tube with the abdomen filled with saline. Little controversy exists regarding the repair of injuries to the stomach or small bowel because of a rich blood supply. Gastric wounds can be oversewn with a running single-layer suture line or closed with a stapler. If a single-layer closure is chosen, full-thickness bites should be taken to ensure hemostasis from the well-vascularized gastric wall. Partial gastrectomy may be required for destructive injuries, with resections of the distal antrum or pylorus reconstructed using a Billroth procedure. Patients with injuries that damage both Latarjet nerves or vagi should undergo a drainage procedure. (See Schwartz 11th ed., Ch. 7, p. 228.)

32. During the initial phases of shock resuscitation, what is an appropriate goal hemoglobin level? A. 8.0 g/dL B. 10 g/dL C. 7 g/dL D. 12 g/dL

Answer: B The period of acute resuscitation, typically lasting for the first 12 to 24 hours after injury, combines several key principles: optimizing tissue perfusion, ensuring normothermia, and restoring coagulation status. There are a multitude of management algorithms aimed at accomplishing these goals, the majority of which involve goal-directed resuscitation with initial volume loading to attain adequate preload, followed by judicious use of inotropic agents or vasopressors. Although the optimal hemoglobin level remains debated, during shock resuscitation a hemoglobin level of >10 g/dL is generally accepted to optimize hemostasis and ensure adequate oxygen delivery. After the first 24 hours of resuscitation, a more judicious transfusion trigger of a hemoglobin level of 70 years can achieve satisfactory outcomes. (See Schwartz 11th ed., p. 382.)

22. Which of the following patients with hepatic failure benefit from liver transplantation? A. Model for End-Stage Liver Disease (MELD) > 18 B. All patients with MELD < 18 C. MELD 15–18 if they have significant morbidity from cirrhosis D. A and C only

Answer: D The MELD was originally developed to assess risk for transjugular intrahepatic portosystemic shunt (TIPS) placement. Later analysis revealed it to be an excellent model to predict survival among patients with cirrhosis, especially those on the waiting list for a liver transplant. In 2002, liver graft allocation was restructured to be based on the MELD score. Although the historic indication for a liver transplant is decompensated cirrhosis, a landmark analysis comparing waiting list mortality with posttransplant mortality established that a minimum MELD score of 18 is necessary to have a survival benefit posttransplant. A MELD score between 15 and 18 does not confer a survival advantage, but a transplant may be justified if the patient has significant morbidity from cirrhosis. (See Schwartz 11th ed., p. 381.)

Brunicardi_Ch11_p087-096.indd 95

Transplantation

19. The most common diagnosis leading to heart transplant is: A. COPD. B. Congenital heart disease. C. Ischemic dilated cardiomyopathy. D. Idiopathic dilated cardiomyopathy.

CHAPTER 11

immunosuppressive protocol consisting of low-dose tacrolimus, sirolimus, and IL-2 receptor antibody induction. Those results were replicated at other experienced transplant centers, but the rates of long-term (>5 year) insulin independence remained poor, well below those of whole-pancreas transplants. Still, despite the low rates of long-term insulin independence, most islet recipients were C-peptide positive and retained hypoglycemia awareness, indicating residual islet function and benefit. In fact, at 9 years posttransplant, 15% remained insulin-independent, and 73% had hypoglycemia awareness and corrected hemoglobin Aic levels. (See Schwartz 11th ed., p. 378.)

04/07/22 1:31 PM

This page intentionally left blank

Brunicardi_Ch11_p087-096.indd 96

04/07/22 1:31 PM

CHAPTER

12

Patient Safety

1. High reliability organization theory suggests that: A. Eliminating problematic individuals improves organizational reliability. B. Strict reporting hierarchy and rules are key to success at decreasing error rates. C. Low error rates are not possible in large organizations. D. Highly reliable organizations have friendly, cooperative, resilient staffs where creativity and open relationships are encouraged.

Answer: D High reliability organization theory suggests that proper oversight of people, processes, and technology can handle complex and hazardous activities and keep error rates acceptably low. Studies of multiple high reliability organizations show that they share the following common characteristics: • People are supportive of one another. • People trust one another. • People have friendly, open relationships emphasizing credibility and attentiveness. • The work environment is resilient and emphasizes creativity and goal achievement, providing strong feelings of credibility and personal trust. Developing these characteristics is an important step toward achieving a low error rate in any organization. (See Schwartz 11th ed., p. 399.)

2. Causes of death in the United States from the highest to the lowest: A. Heart Disease – Cancer – COPD – Medical Errors B. Cancer – Heart Disease – Medical Errors – Suicide C. Cancer – COPD – Heart Disease – Motor Vehicles D. Heart Disease – Cancer – Medical Errors – COPD

Answer: D The most commonly cited report on the incidence of deaths due to medical error, the 1999 Institute of Medicine (IOM) report, describes an incidence of 44,000 to 98,000 deaths annually. However, this estimate by the IOM was not based on primary research conducted by the IOM; rather, it was based on two older studies conducted in 1984 and 1992. Both studies were small and limited. In 2013, after compiling more recent evidence from multiple sources, James estimated an incidence range of 210,000 to 400,000 deaths a year associated with medical errors among hospital patients. Any point estimate in this range would rank the problem of dying from “medical care gone wrong” as the third leading cause of death in the United States. In caring for patients and considering the risks of tests and procedures done for borderline indications, it is important to consider the magnitude of the problem of patients dying from the care they receive rather than from the disease or injury that brought them to care. (See Schwartz 11th ed., p. 398.)

97

Brunicardi_Ch12_p097-104.indd 97

05/07/22 3:32 PM

98 CHAPTER 12

Answer: C The Donabedian model of measuring quality identifies three main types of improvements: changes to organizational structure, changes in organizational processes, and changes in outcomes. Structure refers to the physical and organizational tools, equipment, and policies that improve safety. Structural measures ask, “Do the right tools, equipment, and policies exist?” Process is the application of these tools, equipment, and policies/procedures to patients (good practices and evidencebased medicine). Process measures ask, “Are the right tools, policies, and equipment being used?” Outcome is the result on patients. Outcome measures ask, “How often are patients harmed?” In this model, structure (how care is organized) plus process (what we do) influences patient outcomes (the results achieved). (See Schwartz 11th ed., p. 399.)

4. Surgical Care Improvement Project (SCIP) Measures include: A. Process of care performance measures and outcome measures. B. The training of surgeons and staff. C. How surgeons document in operative reports. D. Operating room turnover.

Answer: A SCIP has identified three broad areas within surgery where potential complications have a high incidence and cost and there is a significant opportunity for prevention: surgical site infections (SSIs), venous thromboembolism, and adverse cardiac events. The SCIP measures aim to reduce the incidence of these events during the perioperative period by advocating the use of proven process and outcome measures. These process and outcome measures are detailed in Table 12-1. (See Schwartz 11th ed., p. 407.)

Patient Safety

3. The Donabedian model of measuring quality identifies all of the following as main types of improvements EXCEPT: A. Changes to structure B. Changes to process C. Changes to culture D. Changes to outcomes

TABLE 12-1

The Surgical Care Improvement Project measures

Process of care performance measures Infection • Prophylactic antibiotic received within 1 h before surgical incision • Prophylactic antibiotic selection for surgical patients • Prophylactic antibiotics discontinued within 24 h after surgery end time (48 h for cardiac patients) • Cardiac surgery patients with controlled 6 a.m. postoperative serum glucose • Surgery patients with appropriate hair removal • Colorectal surgery patients with immediate postoperative normothermia Venous thromboembolism • Surgery patients with recommended venous thromboembolism prophylaxis ordered • Surgery patients who received appropriate venous thromboembolism prophylaxis within 24 h before surgery to 24 h after surgery Cardiac events • Surgery patients on a β-blocker prior to arrival who received a β-blocker during the perioperative period Proposed outcome measures Infection • Postoperative wound infection diagnosed during index hospitalization Venous thromboembolism • Intra- or postoperative pulmonary embolism diagnosed during index hospitalization and within 30 d of surgery • Intra- or postoperative deep vein thrombosis diagnosed during index hospitalization and within 30 d of surgery Cardiac events • Intra- or postoperative acute myocardial infarction diagnosed during index hospitalization and within 30 d of surgery Global measures • Mortality within 30 d of surgery • Readmission within 30 d of surgery Data from The Joint Commission, 2012.

Brunicardi_Ch12_p097-104.indd 98

05/07/22 3:32 PM

99

6. The root cause of the majority of wrong-site surgeries results from: A. Communication errors. B. Emergency surgery. C. Multiple procedures. D. Multiple surgeons.

Answer: A The risk of performing wrong-site surgery increases when there are multiple surgeons involved in the same operation or multiple procedures are performed on the same patient, especially if the procedures are scheduled or performed on different areas of the body. Time pressure, emergency surgery, abnormal patient anatomy, and morbid obesity are also thought to be risk factors. Communication errors are the root cause in more than 70% of the wrong-site surgeries reported to The Joint Commission. Other risk factors include receiving an incomplete preoperative assessment; having inadequate procedures in place to verify the correct surgical site; or having an organizational culture that lacks teamwork or reveres the surgeon as someone whose judgment should never be questioned. (See Schwartz 11th ed., p. 411.)

7. Retained surgical items: A. Occur in approximately 1:1500 surgeries in the United States. B. Occur more frequently in elective procedures. C. Are less likely to occur when multiple surgeons take part in an operation. D. Are most frequently surgical needles.

Answer: A A retained surgical item refers to any surgical item found to be inside a patient after he or she has left the operating room (OR), thus requiring a second operation to remove the item. Estimates of retained foreign bodies in surgical procedures range from one case per 8000 to 18,000 operations, corresponding to one case or more each year for a typical large hospital or approximately 1500 cases per year in the United States. This estimate is based on an analysis of malpractice claims and is likely to underestimate the true incidence. The risk of having a retained surgical item increases during emergency surgery, when there are unplanned changes in procedure (due to new diagnoses encountered in the OR), and in patients with higher body mass index (Table 12-2). (See Schwartz 11th ed., p. 410.) TABLE 12-2

Patient Safety

Answer: D The National Surgical Quality Improvement Program (NSQIP) is a measurement program that allows hospitals to sample their rates of postoperative events and compare them to similar hospitals. Created by the Veterans Health Administration (VA) in 1991, NSQIP has been credited with measuring and improving morbidity and mortality outcomes at the VA, reducing 30-day mortality rate after major surgery by 31%, and 30-day postoperative morbidity by 45% in its first decade. Beta testing at 18 non-VA sites from 2001 to 2004 demonstrated the feasibility and utility of the program in the private sector. The program was subsequently expanded to the private sector in 2004. (See Schwartz 11th ed., p. 407.)

CHAPTER 12

5. National Surgical Quality Improvement Program: A. Is essentially the same as Surgical Care Improvement Project (SCIP). B. Collects data on individual surgeon outcomes. C. Was created by the Institute of Medicine. D. Allows hospitals to compare their rates of postoperative events and compare them to similar hospitals.

Risk factors for retained surgical sponges

• Emergency surgery • Unplanned changes in procedure • Patient with higher body mass index • Multiple surgeons involved in same operation • Multiple procedures performed on same patient • Involvement of multiple operating room nurses/staff members • Case duration covers multiple nursing “shifts”

Brunicardi_Ch12_p097-104.indd 99

05/07/22 3:32 PM

100 CHAPTER 12 Patient Safety

8. Regarding complications of Central lines: A. Pneumothorax occurs in 10%. B. Pneumothorax is eliminated using ultrasound when placing the line. C. Central line infections are associated with significant mortality and increased hospital costs. D. Frequent line changes are recommended to decrease infection risk.

Answer: C Pneumothorax occurrence rates from both subclavian and internal jugular vein approaches are 1% to 6%. Prevention requires proper positioning of the patient and correct insertion technique. A postprocedure chest X-ray is recommended to confirm the presence or absence of a pneumothorax, regardless of whether a pneumothorax is suspected. Recent reports have questioned whether a chest X-ray is required when the line is placed and confirmed under ultrasound guidance. Pneumothorax rates are higher among inexperienced providers and underweight patients but occur with experienced operators as well. The Centers for Disease Control and Prevention (CDC) reports mortality rates of 12% to 25% when a central venous line infection becomes systemic, with a cost of approximately $25,000 per episode. The CDC does not recommend routine central line changes, but when the clinical suspicion of infection is high, the site of venous access must be changed. (See Schwartz 11th ed., p. 416.)

9. Laryngoscopic findings after a superior laryngeal nerve injury include: A. Ipsilateral vocal cord in a paramedian position. B. Ipsilateral vocal cord in a middling position. C. Asymmetry of the glottic opening. D. Normal examination.

Answer: C Superior laryngeal nerve injury is less debilitating, as the common symptom is loss of projection of the voice. The glottis aperture is asymmetrical on direct laryngoscopy, and management is limited to clinical observation. (See Schwartz 11th ed., p. 419.)

10. Ventilator-associated pneumonia (VAP) in ventilated ICU patients reaches a 70% probability at: A. 5 days. B. 15 days. C. 30 days. D. 45 days.

Answer: C Pneumonia is the second most common nosocomial infection and is the most common infection in ventilated patients. VAP occurs in 15% to 40% of ventilated ICU patients, with a probability rate of 5% per day, up to 70% at 30 days. The 30-day mortality rate of nosocomial pneumonia can be as high as 40% and depends on the microorganisms involved and the timeliness of initiating appropriate antimicrobials Protocol-driven approaches for prevention and treatment of VAP are recognized as beneficial in managing these difficult infectious complications. (See Schwartz 11th ed., p. 419.)

11. Included in the definition of acute respiratory distress syndrome (ARDS) is: A. Required increased positive end-expiratory pressure (PEEP). B. PaO2:FiO2 < 200 regardless of PEEP. C. Cardiac failure. D. Respiratory failure despite normal chest X-ray.

Answer: B The Berlin definition of ARDS developed by the AmericanEuropean Consensus Conference of 2012 not only simplifies the definition of ARDS but also eliminates the term Acute Lung Injury (ALI) from critical care vernacular. ARDS is now classified by partial pressure of oxygen in arterial blood (PaO2)/fraction of inspired oxygen (Fio2) ratios as mild (300–201 mmHg), moderate (200–101 mmHg), and severe (25

Grade IV IAH

Physiologic Monitoring of the Surgical Patient

TABLE 13–1

CHAPTER 13

into the peritoneal cavity could measure IAP to substantiate the diagnosis. In practice, transurethral bladder pressure measurement reflects IAP and is most often used to confirm the presence of ACS. After instilling 50 to 100 mL of sterile saline into the bladder via a Foley catheter, the tubing is connected to a transducing system to measure bladder pressure in the supine position at end-expiration. Intra-abdominal hypertension is defined as an IAP ≥ 12 mm Hg recorded on three standard measurements conducted 4 to 6 hours apart and is separated into several grades. The diagnosis of ACS is the presence of an IAP ≥ 20 mm Hg recorded by three measurements 1 to 6 hours apart, along with new onset of organ dysfunction (Table 13-1). Less commonly, gastric or inferior vena cava pressures can be monitored with appropriate catheters to detect elevated intra-abdominal pressures. (See Schwartz 11th ed., p. 447.)

In the presence of new-onset organ dysfunction: >20

ACS

Data from Kirkpatrick AW, Roberts DJ, De Waele J, et al: Intraabdominal hypertension and the abdominal compartment syndrome: updated consensus definitions and clinical practice guidelines from the World Society of the Abdominal Compartment Syndrome, Intensive Care Med. 2013;39(7):1190-1206.

Brunicardi_Ch13_p105-114.indd 113

04/07/22 1:36 PM

This page intentionally left blank

Brunicardi_Ch13_p105-114.indd 114

04/07/22 1:36 PM

CHAPTER

14

Minimally Invasive Surgery

1. The most common arrhythmia seen during laparoscopy is: A. Atrial fibrillation. B. Sinus tachycardia. C. Premature ventricular contractions. D. Sinus bradycardia.

Answer: D The pressure effects of the pneumoperitoneum on cardiovascular physiology also have been studied. In the hypovolemic individual, excessive pressure on the inferior vena cava and a reverse Trendelenburg position with loss of lower extremity muscle tone may cause decreased venous return and decreased cardiac output. This is not seen in the normovolemic patient. The most common arrhythmia created by laparoscopy is bradycardia. A rapid stretch of the peritoneal membrane often causes a vagovagal response with bradycardia and, occasionally, hypotension. The appropriate management of this event is desufflation of the abdomen, administration of vagolytic agents (eg, atropine), and adequate volume replacement. (See Schwartz 11th ed., p. 455.)

2. A 23-year-old man presents to the emergency department with a 17-hour history of abdominal pain that started in the periumbilical region, now located in the RLQ, associated with fever, nausea, WBC 14,000, and imaging consistent with acute appendicitis. You consent the patient for a laparoscopic appendectomy. During initial insufflation of the abdomen the heart rate goes down to 36 and the patient becomes hypotensive. What is the best next step in the management of this patient? A. Give 1L of crystalloids B. Immediate IV atropine C. Desufflation of the abdomen D. Evaluate for hemorrhage

Answer: C The pressure effects of the pneumoperitoneum on cardiovascular physiology also have been studied. In the hypovolemic individual, excessive pressure on the inferior vena cava and a reverse Trendelenburg position with loss of lower extremity muscle tone may cause decreased venous return and decreased cardiac output. This is not seen in the normovolemic patient. The most common arrhythmia created by laparoscopy is bradycardia. A rapid stretch of the peritoneal membrane often causes a vasovagal response with bradycardia and, occasionally, hypotension. The appropriate management of this event is desufflation of the abdomen, administration of vagolytic agents (eg, atropine), and adequate volume replacement. (See Schwartz 11th ed., p. 455.)

3. A patient undergoing laparoscopic colon resection is noted to have decreased urine output during the last hour of the case. A bolus is given at the end of the case. One hour later, there is still low urine output. The appropriate treatment is: A. Repeat bolus. B. Intravenous (IV) furosemide. C. Check urine electrolytes. D. Observe.

Answer: D Although the effects of the pneumoperitoneum on renal blood flow are immediately reversible, the hormonally mediated changes such as elevated antidiuretic hormone levels decrease urine output for up to 1 hour after the procedure has ended. Intraoperative oliguria is common during laparoscopy, but the urine output is not a reflection of intravascular volume status; IV fluid administration during an uncomplicated laparoscopic procedure should not be linked to urine output.

115

Brunicardi_Ch14_p115-118.indd 115

04/07/22 1:39 PM

116 CHAPTER 14

Because insensible fluid losses through the open abdomen are eliminated with laparoscopy, the need for supplemental fluid during a laparoscopic surgical procedure should only keep up with venous pooling in the lower limbs, third-space losses into the bowel, and blood loss, which is generally less than occurs with an equivalent open operation. (See Schwartz 11th ed., p. 456.)

Minimally Invasive Surgery

4. While performing a laparoscopic Nissen fundoplication, during the transhiatal dissection the mediastinal pleura is compromised and a CO2 pneumothorax develops. What is the initial preferred management of the pneumothorax? A. Needle thoracostomy over the second intercostal space, mid-clavicular line. B. Enlargement of the defect and placement of an 18-French red rubber catheter across the defect. C. Abort the procedure and emergent tube thoracostomy with a 28-French chest tube. D. No intervention is needed. Continue with the planed procedure.

Answer: B When a pneumothorax occurs with laparoscopic Nissen fundoplication or Heller myotomy, it is preferable to place an 18-French red rubber catheter with multiple side holes cut out of the distal end across the defect. At the end of the procedure, the distal end of the tube is pulled out a 10-mm port site (as the port is removed), and the pneumothorax is evacuated to a primitive water seal using a bowl of sterile water or saline. During laparoscopic esophagectomy, it is preferable to leave a standard chest tube, as residual intra-abdominal fluid will tend to be siphoned through the defect postoperatively if the tube is removed at the end of the case. (See Schwartz 11th ed., p. 456.)

5. During pneumoperitoneum, the increased intra-abdominal pressure directly and indirectly results in: A. Increased renal blood flow. B. Decreased urine output. C. Decreased levels of renin. D. Decreased sodium retention.

Answer: B Increased intra-abdominal pressure decreases renal blood flow, glomerular filtration rate, and urine output. These effects may be mediated by direct pressure on the kidney and the renal vein. The secondary effect of decreased renal blood flow is to increase plasma renin release, thereby increasing sodium retention. (See Schwartz 11th ed., p. 456.)

6. Which type of radiofrequency electrosurgery mode has the highest risk for thermal injury? A. Bipolar coagulation B. Monopolar coagulation C. Monopolar cutting D. Monopolar blended

Answer: B A short-duration, high-voltage discharge of current (coagulation current) provides extremely rapid tissue heating. Lowervoltage, higher-wattage current (cutting current) is better for tissue desiccation and vaporization. When the surgeon desires tissue division with the least amount of thermal injury and least coagulation necrosis, a cutting current is used. With bipolar electrosurgery, the electrons flow between two adjacent electrodes. The tissue between the two electrodes is heated and desiccated. There is little opportunity for tissue cutting when bipolar current is used alone, but the ability to coapt the electrodes across a vessel provides the best method of small-vessel coagulation without thermal injury to adjacent tissues. (See Schwartz 11th ed., p. 465.)

7. Which of the following represents advantages from robotic surgery compared to laparoscopic surgery? A. Increased dexterity B. Tremor elimination C. Scaling movement D. All of the above

Answer: D The major revolution in robotic surgery was the development of a master-slave surgical platform that returned the wrist to laparoscopic surgery and improved manual dexterity by developing an ergonomically comfortable work station, with 3-D imaging, tremor elimination, and scaling of movement (eg, large, gross hand movements can be scaled down to allow suturing with microsurgical precision) (Fig. 14-1). (See Schwartz 11th ed., p. 467.)

Brunicardi_Ch14_p115-118.indd 116

04/07/22 1:39 PM

117 CHAPTER 14 Minimally Invasive Surgery

FIG. 14-1.  Robotic instruments and hand controls. The surgeon is in a sitting position, and the arms and wrists are in an ergonomic and relaxed position.

8. To date, what is the only surgery that has demonstrated better outcomes using robotic versus laparoscopic or open techniques? A. Nissen fundoplication B. Cholecystectomy C. Prostatectomy D. Roux-en-Y gastric bypass

Answer: C The tidal wave of enthusiasm for robotic surgery came when most minimally invasive urologists declared robotic prostatectomy to be preferable to laparoscopic and open prostatectomy. The great advantage—it would appear—of robotic prostatectomy is the ability to visualize and spare the pelvic nerves responsible for erectile function. (See Schwartz 11th ed., p. 467.)

9. Which of the following are TRUE regarding safe laparoscopic surgery in pregnancy? A. The patient should be positioned slightly on the left lateral position. B. Open abdominal access (Hasson) is recommended versus direct puncture laparoscopy (Veress needle). C. The surgery should be performed during the second trimester if possible. D. All of the above.

Answer: D Concerns about the safety of laparoscopic cholecystectomy or appendectomy in the pregnant patient have been thoroughly investigated and are readily managed. Access to the abdomen in the pregnant patient should take into consideration the height of the uterine fundus, which reaches the umbilicus at 20 weeks. In order not to damage the uterus or its blood supply, most surgeons feel that the open (Hasson) approach should be used in favor of direct puncture laparoscopy. The patient should be positioned slightly on the left side to avoid compression of the vena cava by the uterus. Because pregnancy poses a risk for thromboembolism, sequential compression devices are essential for all procedures. Fetal acidosis induced by maternal hypercarbia also has been raised as a concern. (See Schwartz 11th ed., p. 473.)

Brunicardi_Ch14_p115-118.indd 117

04/07/22 1:39 PM

This page intentionally left blank

Brunicardi_Ch14_p115-118.indd 118

04/07/22 1:39 PM

CHAPTER

15

Molecular and Genomic Surgery

1. The process that occurs during translational control of eukaryotic gene expression is: A. Protein degradation. B. RNA processing. C. Posttranslational control. D. Transcription.

Answer: A See Schwartz 11th ed., Figure 15-4, p. 483.

2. In the transcription of prokaryotes, binding of RNA polymerase to the specific promoter region is achieved by: A. Sigma factors B. Operon C. Elongation factors D. Rho factors

Answer: A Initiation of transcription in prokaryotes (bacteria) begins with the recognition of DNA sequences by RNA polymerase. First, the bacterial RNA polymerase catalyzes RNA synthesis through loose binding to any region in the double-stranded DNA and then through specific binding to the promoter region with the assistance of accessory proteins called σ factors (sigma factors). A promoter region is the DNA region upstream of the transcription initiation site. RNA polymerase binds tightly at the promoter sites and causes the double-stranded DNA structure to unwind. Consequently, few nucleotides can be base-paired with the DNA template to begin transcription. Once transcription begins, the σ factor is released. The growing RNA chain may begin to peel off as the chain elongates. This occurs in such a way that there are always about 10 to 12 nucleotides of the growing RNA chains that are base-paired with the DNA template. (See Schwartz 11th ed., Figure 15-4, p. 483.)

3. All of the following transcription mechanisms occur in eukaryotes EXCEPT: A. Chromatin structure changes to allow DNA to be accessible to the polymerase. B. Three separate RNA polymerases are involved. C. Proteins or initiation factors are not required. D. Often packaged with histone and nonhistone proteins into chromatins.

Answer: C The unique features of eukaryotic transcription are as follows: (a) Three separate RNA polymerases are involved in eukaryotes: RNA polymerase I transcribes the precursor of 5.8S, 18S, and 28S rRNAs; RNA polymerase II synthesizes the precursors of mRNA as well as microRNA; and RNA polymerase III makes tRNAs and 5S rRNAs. (b) In eukaryotes, the initial transcript is often the precursor to final mRNAs, tRNAs, and rRNAs. The precursor is then modified and/or processed into its final functional form. RNA splicing is one type of processing to remove the noncoding introns (the region between coding exons) on an mRNA. (c) In contrast to bacterial DNA, eukaryotic DNA often is packaged with histone and nonhistone proteins into chromatins. Transcription will only occur when the chromatin structure changes in such a way that

119

Brunicardi_Ch15_p119-126.indd 119

04/07/22 5:46 PM

120 CHAPTER 15

DNA is accessible to the polymerase. (d) RNA is made in the nucleus and transported into cytoplasm, where translation occurs. Therefore, unlike bacteria, eukaryotes undergo uncoupled transcription and translation. (See Schwartz 11th ed., Figure 15-4, p. 483.)

Molecular and Genomic Surgery

4. Which of the following statements is incorrect? A. A codon is a triplet of three bases that codes for a single amino acid. B. More than one triplet codes for the same amino acid. C. Codons are mRNA sequentially recognized by tRNA adaptor proteins. D. Protein synthesis proceeds in the carboxy-to-aminoterminus direction.

Answer: D A codon, a triplet of three bases, codes for one amino acid. In this case, random combinations of the four bases form 4 × 4 × 4, or 64 codes. Because 64 codes are more than enough for 20 amino acids, most amino acids are coded by more than one codon. The start codon is AUG, which also corresponds to methionine; therefore, almost all proteins begin with this amino acid. The sequence of nucleotide triplets that follows the start codon signal is termed the reading frame. The codons on mRNA are sequentially recognized by tRNA adaptor proteins. Specific enzymes termed aminoacyl-tRNA synthetases link a specific amino acid to a specific tRNA. The translation of mRNA to protein requires the ribosomal complex to move stepwise along the mRNA until the initiator methionine sequence is identified. In concert with various protein initiator factors, the methionyl-tRNA is positioned on the mRNA and protein synthesis begins. Each new amino acid is added sequentially by the appropriate tRNA in conjunction with proteins called elongation factors. Protein synthesis proceeds in the amino-to-carboxy-terminus direction. (See Schwartz 11th ed., p. 484.)

5. The process of decoding information on mRNA to synthesize proteins is called: A. Transcription. B. Translation. C. Replication. D. Signaling.

Answer: B DNA directs the synthesis of RNA; RNA in turn directs the synthesis of proteins. Proteins are variable-length polypeptide polymers composed of various combinations of 20 different amino acids and are the working molecules of the cell. The process of decoding information on mRNA to synthesize proteins is called translation (see Fig. 15-1). Translation takes place in ribosomes composed of rRNA and ribosomal proteins. (See Schwartz 11th ed., p. 483.)

Nucleus

Cytoplasm

DNA

Transcription RNA transcript

mRNA turnover

Protein turnover

RNA degradation

Protein degradation

Nuclear envelope

RNA processing

mRNA

mRNA

Translation

Protein

Posttranslational modification

Active protein

RNA transport

Transcriptional control

Posttranscriptional control

Translational control

Posttranslational control

FIG. 15-1.  Four major steps in the control of eukaryotic gene expression. Transcriptional and posttranscriptional control determine the level of messenger RNA (mRNA) that is available to make a protein, while translational and posttranslational control determine the final outcome of functional proteins. Note that posttranscriptional and posttranslational controls consist of several steps.

Brunicardi_Ch15_p119-126.indd 120

04/07/22 5:46 PM

121

7. If chronic kidney disease (CKD) is to a cell as an engine is to a car, then cyclins and cyclin-dependent kinase inhibitors.(CKI) are: A. The key and ignition, respectively. B. The gas pedal and brakes, respectively. C. The distributor and the spark plug, respectively. D. The windows and the tires, respectively.

Answer: B The cell cycle is connected with signal transduction pathways as well as gene expression. Although the S and M phases rarely are subjected to changes imposed by extracellular signals, the G1 and G2 phases are the primary periods when cells decide whether to move on to the next phase. During the G1 phase, cells receive green- or red-light signals, S phase entry or G1 arrest, respectively. Growing cells proliferate only when supplied with appropriate mitogenic growth factors. Cells become committed to entry of the cell cycle only toward the end of G1. Mitogenic signals stimulate the activity of early G1 CDKs (eg, cyclin D/CDK4) that inhibit the activity of pRb protein and activate the transcription factor called E2F to induce the expression of batteries of genes essential for G1-S progression. Meanwhile, cells also receive antiproliferative signals such as those from tumor suppressors. These antiproliferative signals also act in the G1 phase to stop cells’ progress into the S phase by inducing CKI production. For example, when DNA is damaged, cells will repair the damage before entering the S phase. Therefore, G1 contains one of the most important checkpoints for cell cycle progression. If the analogy is made that CDK is to a cell as an engine is to a car, then cyclins and CKI are the gas pedal and brake, respectively. Accelerated proliferation or improper cell cycle progression with damaged DNA would be disastrous. Genetic gain-of-function mutations in oncogenes (that often promote expression or activity of the cyclin/CDK complex) or loss-of-function mutations in tumor suppressor (that stimulate production of CKI) are causal factors for malignant transformation. (See Schwartz 11th ed., p. 486.)

8. The cell cycle period during which DNA is replicated is: A. S. B. G1. C. M. D. G2.

Answer : A See Figure 15-2: The cell cycle and its control system. M is the mitosis phase, when the nucleus and the cytoplasm divide; S is the phase when DNA is duplicated; G1 is the gap between

Molecular and Genomic Surgery

Answer: C The human genome has an estimated 25,000 to 30,000 genes, and overall it is 99.9% identical in all people. Approximately 3 million locations where single-base DNA differences exist have been identified and termed single nucleotide polymorphisms. Single nucleotide polymorphisms may be critical determinants of human variation in disease susceptibility and responses to environmental factors. (See Schwartz 11th ed., p. 485.)

CHAPTER 15

6. The human genome contains approximately: A. 35,000 to 40,000 genes. B. 20,000 to 25,000 genes. C. 25,000 to 30,000 genes. D. 30,000 to 35,000 genes.

B/CDK1 Mitosis M G2

G1

FIG. 15-2.  The cell cycle and its control system. M is the mitosis phase, when the nucleus and the cytoplasm divide; S is the phase when DNA is duplicated; G1 is the gap between M and S; G2 is the gap between S and M. A complex of cyclin and cyclin-dependent kinase (CDK) controls specific events of each phase. Without cyclin, CDK is inactive. Different cyclin/CDK complexes are shown around the cell cycle. A, B, D, and E stand for cyclin A, cyclin B, cyclin D, and cyclin E, respectively.

Brunicardi_Ch15_p119-126.indd 121

S

A/CDK1

DNA replication

A/CDK2

D/CDK4 D/CDK6

E/CDK2

04/07/22 5:46 PM

122 CHAPTER 15

M and S; G2 is the gap between S and M. A complex of cyclin and cyclin-dependent kinase (CDK) controls specific events of each phase. Without cyclin, CDK is inactive. Different cyclin/CDK complexes are shown around the cell cycle. A, B, D, and E stand for cyclin A, cyclin B, cyclin D, and cyclin E, respectively. (See Schwartz 11th ed., p. 486.)

Molecular and Genomic Surgery

9. In cellular apoptosis, the release of cytochrome c activates the: A. FAS receptor. B. A membrane bound death receptor. C. Tumor necrosis factor (TNF) receptor. D. Caspase cascade.

Answer: D See Figure 15-3. (See Schwartz 11th ed., Figure 15-8, p. 487.)

Death signal (e.g., TNF or Fas) Plasma membrane

Death receptor

Mitochondrion

FIG. 15-3.  A simplified view of the apoptosis pathways. Extracellular death receptor pathways include the activation of Fas and tumor necrosis factor (TNF) receptors and consequent activation of the caspase pathway. Intracellular death pathway indicates the release of cytochrome c from mitochondria, which also triggers the activation of the caspase cascade. During apoptosis, cells undergo DNA fragmentation and nuclear and cell membrane breakdown and are eventually digested by other cells.

Death receptor signaling pathway

Activation of caspase cascade

10. All of the following are cell-surface receptors EXCEPT: A. Transmitter-gated ion channels. B. Seven-transmembrane-G-protein-coupled receptors. C. Enzyme-linked receptors. D. Adhesive receptors.

Brunicardi_Ch15_p119-126.indd 122

Cytochrome c release

Nucleus

Apoptotic target cell

Normal target cell

Answer: D There are three major classes of cell-surface receptors: transmitter-gated ion channels, seven-transmembrane G-protein– coupled receptors (GPCRs), and enzyme-linked receptors. The superfamily of GPCRs is one of the largest families of proteins, representing over 800 genes of the human genome. Members of this superfamily share a characteristic seventransmembrane configuration. The ligands for these receptors are diverse and include hormones, chemokines, neurotransmitters, proteinases, inflammatory mediators, and even sensory signals such as odorants and photons. Most GPCRs signal through heterotrimeric G proteins, which are guanine nucleotide regulatory complexes. Thus the receptor serves as the receiver, the G protein serves as the transducer, and the enzyme serves as the effector arm. Enzyme-linked receptors possess an extracellular ligand-recognition domain and a cytosolic domain that either has intrinsic enzymatic activity or directly links with an enzyme. Structurally, these receptors usually have only one transmembrane-spanning domain. Of at least five forms of enzyme-linked receptors classified by the nature of the enzyme activity to which they are coupled, the growth factor receptors such as tyrosine kinase receptor or serine/threonine kinase receptors mediate diverse cellular

04/07/22 5:46 PM

123

Answer: B Resistance to TGF-β’s anticancer action is one hallmark of human cancer cells. TGF-β receptors and SMADs are identified as tumor suppressors. The TGF-β signaling circuit can be disrupted in a variety of ways and in different types of human tumors. Some lose TGF-β responsiveness through downregulation or mutations of their TGF-β receptors. The cytoplasmic SMAD4 protein, which transduces signals from ligand-activated TGF-β receptors to downstream targets, may be eliminated through mutation of its encoding gene. The locus encoding cell cycle inhibitor p15INK4B may be deleted. Alternatively, the immediate downstream target of its actions, cyclin dependent kinase 4 (CDK4), may become unresponsive to the inhibitory actions of p15INK4B because of mutations that block p15INK4B binding. The resulting cyclin D/CDK4 complexes constitutively inactivate tumor suppressor pRb by hyperphosphorylation. Finally, functional pRb, the end target of this pathway, may be lost through mutation of its gene. For example, in pancreatic and colorectal cancers, 100% of cells derived from these cancers carry genetic defects in the TGF-β signaling pathway. Therefore, the antiproliferative pathway converging onto pRb and the cell division cycle is, in one way or another, disrupted in a majority of human cancer cells. Besides cancer, dysregulation of TGF-β signaling also has been associated with other human diseases such as Marfan syndrome and thoracic aortic aneurysm. (See Schwartz 11th ed., p. 489.)

12. All of the following are true regarding type 2 diabetes EXCEPT: A. More than 90% of individuals have insulin resistance. B. Genetic mutation in the cell-surface insulin receptors (InsR) cause the disease. C. Majority of cases may result from defects in downstream-signaling components in the insulin-signaling pathway. D. Phosphoryl group is added by the insulin receptor substrate (IRS).

Answer: D See Figure 15-4. Insulin is a peptide growth factor that binds to and activates the heterotetrameric receptor complex (InsR). InsR possesses protein tyrosine kinase activity and is

FIG. 15-4.  Insulin-signaling pathway. Insulin is a peptide growth factor that binds to and activates the heterotetrameric receptor complex (InsR). InsR possesses protein tyrosine kinase activity and is able to phosphorylate the downstream insulin receptor substrate (IRS). Phosphorylated IRS serves as a scaffold and controls the activation of multiple downstream pathways for gene expression, cell survival, and glucose metabolism. Inactivation of the insulin pathway can lead to type 2 diabetes.

Brunicardi_Ch15_p119-126.indd 123

Insulin receptor (InsR)

Insulin

Adaptor

IRS

MAPK cascade

Lipid & glucose metabolism

Molecular and Genomic Surgery

11. Dysregulation of transforming growth factor-β (TGF-β) signaling is associated with all EXCEPT: A. Cancer. B. Inguinal hernias. C. Marfan syndrome. D. Thoracic aortic aneurysm.

CHAPTER 15

events including cell growth, differentiation, metabolism, and survival/apoptosis. Dysregulation (particularly mutations) of these receptors is thought to underlie conditions of abnormal cellular proliferation in the context of cancer. The following sections will further review two examples of growth factor signaling pathways and their connection with human diseases. (See Schwartz 11th ed., p. 488.)

Plasma membrane

PI3K

Cell survival

Nucleus

Gene expression

04/07/22 5:46 PM

124 CHAPTER 15

able to phosphorylate the downstream IRS. Phosphorylated IRS serves as a scaffold and controls the activation of multiple downstream pathways for gene expression, cell survival, and glucose metabolism. Inactivation of the insulin pathway can lead to type 2 diabetes. (See Schwartz 11th ed., Figure 15-10, p. 489.)

Molecular and Genomic Surgery

13. Which of the following statements are TRUE? A. In normal cells, oncogenes promote cell growth by activating cell cycle progression. B. In cancer cells, oncogenes promote cell growth by activating cell cycle progression. C. Tumor suppressors enhance oncogene function. D. Oncogenes do not play an important role in maintaining controlled state of cell growth.

Answer: A There are two classes of cancer genes in which alteration has been identified in human and animal cancer cells: oncogenes, with dominant gain-of-function mutations, and tumor suppressor genes, with recessive loss-of-function mutations. In normal cells, oncogenes promote cell growth by activating cell cycle progression, whereas tumor suppressors counteract oncogenes’ functions. Therefore, the balance between oncogenes and tumor suppressors maintains a well-controlled state of cell growth. (See Schwartz 11th ed., p. 490.)

14. All of the following are correct about human embryonic stem cells (hESCs) EXCEPT: A. They are derived from early preimplantation embryos. B. They are derived from blastocysts. C. They can generate all differential germ layers. D. They are considered multipotent.

Answer: D hESCs are derived from early preimplantation embryos called blastocysts (5 days postfertilization) and are capable of generating all differentiated germ layers in the body by chimera assays or 2-D/3-D differentiation in a dish—ectoderm, mesoderm, and endoderm—and therefore are considered pluripotent. There are two pluripotent states associated with hESCs, one of which is the classic culture with basic fibroblast growth factor (bFGF) and knock out replacer (KSR), termed as “primed” pluripotent state. More recently, “naive” hESC culture methods have been introduced based on mouse studies, by supplementing 2i inhibitors (MEK1 and GSK3β inhibitors) into the medium in addition to bFGF. (See Schwartz 11th ed., p. 492.)

15. Gene expression detection method that provides information regarding mRNA size is: A. Polymerase chain reaction. B. Southern blot hybridization. C. Northern blot hybridization. D. Immunoblotting.

Answer: C Northern blotting refers to the technique of size fractionation of RNA in a gel and the transferring of an RNA sample to a solid support (membrane) in such a manner that the relative positions of the RNA molecules are maintained. The resulting membrane then is hybridized with a labeled probe complementary to the mRNA of interest. Signals generated from detection of the membrane can be used to determine the size and abundance of the target RNA. In principle, Northern blot hybridization is similar to Southern blot hybridization (and hence its name), with the exception that RNA, not DNA, is on the membrane. Although reverse-transcriptase PCR has been used in many applications, Northern analysis is the only method that provides information regarding mRNA size and has remained a standard method for detection and quantitation of mRNA. The process of Northern hybridization involves several steps, as does Southern hybridization, including electrophoresis of RNA samples in an agarose-formaldehyde gel, transfer to a membrane support, and hybridization to a radioactively labeled DNA probe. Data from hybridization allow quantification of steady-state mRNA levels and, at the same time, provide information related to the presence, size, and integrity of discrete mRNA species. Thus, Northern blot analysis, also termed RNA gel blot analysis, commonly is used in molecular biology studies relating to gene expression. (See Schwartz 11th ed., p. 494.)

Brunicardi_Ch15_p119-126.indd 124

04/07/22 5:46 PM

125

17. All of the following are TRUE regarding Bifunctional RNAi Technology EXCEPT: A. Field has worked to define oncogene and nononcogene addiction. B. It advances the understanding of discrimination between driver and passenger genes. C. The field made strides in appreciation of complexity of network interaction. D. The knowledge in the field has been effectively and reproducibly clinically translated.

Answer: D Over the last 20 years, the field has worked to define oncogene and nononcogene addiction, discriminate between driver and passenger genes, and appreciate the complexity of complex, robust, network interactions. These insights have led to a preliminary understanding of therapeutically relevant sensitivity and resistance pathway signal patterns requiring multiple target modulation. However, this knowledge has not been effectively or reproducibly clinically translated. Clinical response is usually far greater when a combination of singletarget molecular therapy is administered. However, it must also be realized that targeting two or more pathways may also increase the toxicity profile, particularly if target specificity is limited. When attempted, off-target toxicity has been demonstrated with combination small-molecule therapy. In contrast, multitargeting bifunctional short hairpin (bi-shRNA) DNA vectors are designed to limit off-target effect given the high specificity for the genes they are designed to target. (See Schwartz 11th ed., p. 505.)

18. Which of the following about CRISPR is TRUE? A. CRISPR-Cas9 technology can be used to edit single genes through gene knockout, mutation, and addition of an epitope tag. B. A typical CRISPR region contains a cluster of DNA repeats interspersed with spacers. C. CRISPR-Cas9 mediated gene editing involves DNA break and repair. D. All of the above.

Answer: D CRISPR stands for clustered regularly interspaced short palindromic repeats. It is a region on the genomic DNA first discovered in the microbes as an adapted immune system against exogenous DNA. A typical CRISPR region contains a cluster of short (21–48 bp) DNA repeats (ranging from 2 to 100) interspaced by nonrepetitive sequences called spacers. Within a CRISPR region, while each spacer has its unique sequence, the sequence of the repeats is highly conserved. Several genes, called the CRISPR-associated (Cas) genes, are almost always found directly flanking the CRISPR region. Currently, CRISPR-Cas9 is most used for editing single genes, through gene knockout, gene mutation, or the addition of an epitope tag to a native gene, for functional characterization of the gene of interest. For example, oncogenes or tumor suppressor genes can be knocked out to identify the causative gene for a particular cancer type; point mutations in functional domains may illustrate the mechanism of action of a protein; for proteins without available antibodies, epitope tags can be inserted onto the native gene for the detection of the native protein. (See Schwartz 11th ed., p. 506.)

Brunicardi_Ch15_p119-126.indd 125

Molecular and Genomic Surgery

Answer: A. One of the most exciting applications of immunotherapy has come from the identification of certain tumor targets called antigens and the aiming of an antibody at these targets. This was first used as a means of localizing tumors in the body for diagnosis and was more recently used to attack cancer cells. Trastuzumab (Herceptin) is an example of such a drug. Trastuzumab is a monoclonal antibody that neutralizes the mitogenic activity of cell-surface growth factor receptor HER2, which is overexpressed in approximately 25% of breast cancers. HER-2–overexpressing tumors tend to grow faster and generally are more likely to recur than tumors that do not overproduce HER-2. Trastuzumab is designed to attack cancer cells that overexpress HER-2 by slowing or preventing the growth of these cells, resulting in increased survival of HER-2–positive breast cancer patients. (See Schwartz 11th ed., p. 491.)

CHAPTER 15

16. Which of the following drugs is an example of an immunotherapy that targets an oncogene? A. Trastuzumab B. Methotrexate C. Adriamycin D. Gleevec

04/07/22 5:46 PM

This page intentionally left blank

Brunicardi_Ch15_p119-126.indd 126

04/07/22 5:46 PM

PART II

Specific Consideration

Brunicardi_Ch16_p127-136.indd 127

30/06/22 11:21 AM

Brunicardi_Ch16_p127-136.indd 128

30/06/22 11:21 AM

CHAPTER

16

The Skin and Subcutaneous Tissue

1. The area most amenable to salvage by resuscitative and wound management techniques following thermal injury is called the: A. Zone of hyperemia. B. Zone of coagulation. C. Zone of stasis.

Answer: C The management of thermal wounds is initially guided by the concept of three distinct zones of injury. The focus of thermal injury that has already undergone necrosis is known as the zone of coagulation. Well outside the zone of coagulation is the zone of hyperemia, which exhibits signs of inflammation but will likely remain viable. In between these two zones is a zone of stasis with questionable tissue viability, and it is this area at which proper burn care can salvage viable tissue and decrease the extent of injury. (See Schwartz 11th ed., pp. 521–522.)

2. Initial treatment of extensive cellulitis without abscess is: A. Vancomycin. B. β-lactam. C. Linezolid. D. Clindamycin.

Answer: B Extensive cellulitis is managed in a similar fashion as simple cellulitis. Initial treatment consists of intravenous antibiotics that cover β-hemolytic streptococcus, such as cephalosporins, with the addition of methicillin-resistant Staphylococcus aureus (MRSA) coverage if there is no improvement in symptoms. Vancomycin is typically the first choice for MRSA coverage, but this drug is inferior to β-lactams for coverage of methicillin-sensitive S. aureus (MSSA). Alternative antibiotics that are typically effective against MRSA are linezolid, daptomycin, tigecycline, and telavancin. Clindamycin is approved for use against MRSA, but resistance rates are increasing, and its use is discouraged if institutional rates of clindamycin resistance are >15%. (See Schwartz 11th ed., p. 525.)

3. A 3-mm, nodular-type basal cell carcinoma (BCC) of the skin of the trunk should be treated with: A. Mohs micrographic surgery. B. Dermatologic laser vaporization. C. Excision with a 4-mm margin of normal tissue. D. Electrodesiccation.

Answer: C Treatment of BCC varies according to size, location, type, and high or low risk. Treatment options include surgical excision and medical or destructive therapies. Surgical excision should include 4 mm margins for low-risk lesions. Mohs microsurgical is indicated for high-risk lesions, especially in cosmetically sensitive areas. In high-risk lesions in which Mohs micrographic surgery is not available, excision with 10 mm margin of normal tissue can be considered. (See Schwartz 11th ed., p. 529.)

129

Brunicardi_Ch16_p127-136.indd 129

30/06/22 11:21 AM

130

CHAPTER 16 The Skin and Subcutaneous Tissue

4. The primary risk factor for the development of squamous cell carcinoma (SCC) is: A. Ultraviolet (UV) radiation exposure. B. Tobacco use. C. Exposure to chemical agents. D. Chronic, nonhealing wounds.

Answer: A The primary risk factor for the development of SCC is UV radiation exposure; however, other risks include light Fitzpatrick skin type (I or II), environmental factors such as chemical agents, physical agents (ionizing radiation), psoralen, HPV-16 and -18 infections, immunosuppression, smoking, chronic wounds, burn scars, and chronic dermatoses. Heritable risk factors include xeroderma pigmentosum, epidermolysis bullosa, and oculocutaneous albinism. (See Schwartz 11th ed., p. 529.)

5. Using the ABCDE initialism for melanoma, at what diameter does a nevus become concerning for malignant transformation? A. 2 mm B. 4 mm C. 6 mm D. 10 mm

Answer: C Melanoma most commonly manifests as cutaneous disease, and clinical characteristics of malignant transformation are often remembered by the initialism ABCDE. These lesions are typically asymmetric with irregular borders, color variations, a diameter > 6 mm, and are undergoing some sort of evolution or change. Other key clinical characteristics include a pigmented lesion that has enlarged, ulcerated, or bled. Amelanotic lesions appear as raised pink, purple, or fleshcolored skin papules and are often diagnosed late. (See Schwartz 11th ed., p. 530.)

6. What is the most common site of distant metastasis of cutaneous melanoma? A. Brain B. Lung C. Bone D. Distant skin

Answer: B The most common sites of metastasis of melanoma are the lung and liver. These are followed by the brain, gastrointestinal tract, distant skin, and subcutaneous tissue. (See Schwartz 11th ed., p. 533.)

7. Which aggressive subtype of melanoma is characterized by an early vertical growth pattern, often leading to laterstage diagnosis? A. Superficial spreading B. Lentigo maligna C. Acral lentiginous D. Nodular

Answer: D The most common subtype of melanoma is superficial spreading (Fig. 16-1). This accounts for 50% to 70% of melanomas and typically arises from a precursor melanocytic nevus. Nodular subtype accounts for 15% to 30% of melanomas, and typically arises de novo, most commonly in men and on the trunk (Figs. 16-2 and 16-3). This subtype is aggressive with an early vertical growth pattern and is often diagnosed at a later stage. Up to 5% of these lesions will lack melanin and can be mistaken for other cutaneous lesions. Lentigo maligna represents 10% of melanoma cases and is a less aggressive subtype of melanoma in situ that typically arises on sun-exposed areas of the head and neck. Acral lentiginous

FIG. 16-1.  Primary cutaneous melanoma seen in the scalp of a 61-year-old man.

Brunicardi_Ch16_p127-136.indd 130

30/06/22 11:21 AM

131 melanoma accounts for 29% to 72% of melanomas in darkskinned individuals, is occasionally seen in Caucasians, and is found on palmar, plantar, and subungual surfaces. This subtype is not thought to be due to sun exposure. (See Schwartz 11th ed., p. 530.)

CHAPTER 16 The Skin and Subcutaneous Tissue

FIG. 16-2.  Nodular melanoma seen in the leg of a 55-year-old man.

A

B

C

FIG. 16-3.  A. AP view of advanced melanoma in a 59-year-old man. B. Lateral view. C. After resection and reconstruction with skin grafting.

Brunicardi_Ch16_p127-136.indd 131

30/06/22 11:21 AM

132 Answer: B SLNB is a standard staging procedure to evaluate the regional nodes for patients with clinically node-negative malignant melanoma. Detecting subclinical nodal metastasis may benefit from lymphadenectomy or adjuvant therapy. This technique identifies the first draining lymph node from the primary lesion and has shown excellent accuracy and significantly less morbidity compared to complete resection of nodal basins. It is almost always performed at the time of initial wide excision, as SLN mapping after lymphatic violation from surgical excision could decrease the accuracy of the test. Recently, the results of MSLT-1, an international, multicenter, phase III trial were published. This study randomized clinically node negative patients to either SLNB at the time of primary melanoma excision (and completion lymphadenectomy if positive) or nodal basin monitoring (and delayed complete lymphadenectomy for recurrent lymph node disease). The results of this study demonstrated that SLNB, with immediate lymphadenectomy if positive, improved disease-free survival by 7% and 10% in patients with intermediate thickness (1.2–3.5 mm) and thick (>3.5 mm) lesions, respectively. The latest NCCN guidelines suggest consideration of SLNB in clinically stage I/II patients with Breslow depth > 1.0 mm or >0.8 mm with high-risk features (ulceration, mitotic index > 2/mm2, or lymphovascular invasion). A patient with clinically positive nodes would not be a candidate for screening SLNB. (See Schwartz 11th ed., p. 531.)

9. Which surgical margins are appropriate for excision of a primary cutaneous melanoma that has a Breslow depth of 1.5 mm? A. 5 mm margins B. 10 mm margins C. 30 mm margins D. 1.5 mm margins

Answer: B The appropriate excision margin for cutaneous melanoma is based on primary tumor thickness. Several retrospective studies suggest that for melanoma in situ, 0.5 to 1 cm margins are sufficient. 1-cm margins should be obtained in anatomically feasible areas given the possibility of an incidental finding of a small invasive component in permanent sections. Several studies compared 1- to 3-cm margins and 2- to 5-cm margins in melanoma 2 mm thick by showing that 1-cm margins provide worse outcomes compared to 3-cm margins. Tumors 2 mm thick require 2-cm margins. (See Schwartz 11th ed., p. 532.)

10. What is the source of regenerative keratinocytes in the epithelialization of partial thickness wounds of the skin? A. Langerhans cell conversion to keratinocytes B. Fibroblast conversion to keratinocytes C. Extravasation of stem cells from the blood stream D. Keratinocytes from the lining of epidermal appendages

Answer: D The stratum basale, or germinative layer, is a deep, single layer of asynchronous, continuously replicating cuboidal to columnar epithelial cells and is the beginning of the life cycle of the keratinocytes of the epidermis. This layer is bound to its basement membrane by complexes made of keratin filaments and anchoring structures called hemidesmosomes. They are bound to other keratinocytes by structures called desmosomes. High mitotic activity and thus large nuclei and basophilic staining characterize the stratum basale on light microscopy. This layer also lines the epidermal appendages that reside largely within the substance of the dermis and later serves as a regenerative source of epithelium in the event of partial thickness wounds. (See Schwartz 11th ed., p. 514.)

CHAPTER 16

8. Which of the answer choices does NOT represent an indication for sentinel lymph node biopsy (SLNB) in the setting of malignant cutaneous melanoma? A. Breslow depth 0.5 mm with superficial ulceration B. Clinically positive lymph nodes C. Breslow depth 1.5 mm without superficial ulceration D. Breslow depth 1.0 mm with lymphovascular invasion

The Skin and Subcutaneous Tissue

Brunicardi_Ch16_p127-136.indd 132

30/06/22 11:21 AM

133

12. What type of gland is typically influenced by sex hormones and undergoes a secretion process that involves decapitation of the part of the cell? A. Hair follicle B. Sebaceous gland C. Apocrine sweat gland D. Eccrine sweat gland

Answer: C One type of sweat gland, known as the apocrine sweat gland, is found around the axilla, anus, areola, eyelid, and external auditory canal. The cells in this gland undergo an excretion process that involves decapitation of part of the cell. These apocrine glands are typically activated by sex hormones and thus activate around the time of puberty. The secretion from apocrine glands is initially odorless, but bacteria in the region may cause an odor to develop. Pheromone production may have been a function of the apocrine glands, but this may now be vestigial. While eccrine sweat glands are activated by the cholinergic system, apocrine glands are activated by the adrenergic system. (See Schwartz 11th ed., p. 515.)

13. In the Hurley classification of hidradenitis suppurativa, which stage is characterized by diffuse disease with interconnected sinus tracts and recurrent abscesses? A. Hurley stage IV B. Hurley stage III C. Hurley stage II D. Hurley stage I

Answer: B The diagnosis of hidradenitis is clinical, and the presentation is most commonly categorized by the Hurley classification system, divided into three stages. Single or multiple nodules or abscesses without any sinus tracts or scarring would be classified as stage I disease. As abscesses recur and sinus tracts and scarring form, the disease is classified as Hurley stage II. Stage III is the most advanced stage, with diffuse disease and interconnected sinus tracts and abscesses. Hurley stage IV is nonexistent. (See Schwartz 11th ed., p. 517.)

14. What mucocutaneous disorder, thought to be a reaction to various drugs, is characterized by mucocutaneous destruction at the dermoepidermal junction affecting 105). Antibiotic prophylaxis after a human bite is recommended as it has

Brunicardi_Ch16_p127-136.indd 133

The Skin and Subcutaneous Tissue

Answer: A Of the cells in the epidermis, 3% to 6% are immune cells known as Langerhans cells. Typically found within the stratum spinosum, these mobile, dendritic cells interdigitate between keratinocytes of the epidermis to create a dense network, sampling any antigens that attempt to pass through the cutaneous tissue. Through use of their characteristic rod- or racketshaped Birbeck granules, they take up antigens for presentation to T-cells. These monocyte-derived cells represent a large part of the skin’s adaptive immunity. (See Schwartz 11th ed., p. 515.)

CHAPTER 16

11. What cell type are characterized by rod- or racketshaped Birbeck granules, takes up antigens for presentation to T-cells, and makes up 3% to 6% of the cells of the epidermis? A. Langerhans cell B. Melanocyte C. Merkel cell D. Mast cell

30/06/22 11:21 AM

134 been shown to significantly decrease the rate of infection. A course of 3 to 7 days of amoxicillin/clavulanate is typically used. Alternatives are doxycycline or clindamycin with ciprofloxacin. (See Schwartz 11th ed., p. 519.)

CHAPTER 16 The Skin and Subcutaneous Tissue

16. Following caustic injury with a strong alkaline solution, what is the appropriate treatment? A. Neutralization with a weakly acidic solution for 30 minutes B. Irrigation with normal saline for 30 minutes C. Irrigation with normal saline for 120 minutes D. Immediate coverage with light compressive dressing

Answer: C Treatment for acidic or alkaline chemical burns is first and foremost centered around dilution of the offending agent, typically using distilled water or saline for 30 minutes for acidic burns and 2 hours for alkaline injuries. Attempting to neutralize the offending agent is typically discouraged, as it does not offer an advantage over dilution and the neutralization reaction could be exothermic, increasing the amount of tissue damage. After removal of the caustic agent, the burn is treated like other burns and is based on the depth of tissue injury. (See Schwartz 11th ed., p. 521.)

17. How long does it take to get irreversible tissue damage with external pressure greater than double capillary perfusion pressure? A. 1 hour B. 2 hours C. 3 hours D. 4 hours

Answer: B The average perfusion pressure of the microcirculation is about 30 mm Hg, and pressures greater than that cause local tissue ischemia. In animal models, pressure greater than twice the capillary perfusion pressure produces irreversible tissue necrosis in just 2 hours. Tissues with a higher metabolic demand are typically susceptible to insult from tissue hypoperfusion more rapidly than tissues with a lower metabolic demand. Because of this, it is possible to have muscle necrosis beneath cutaneous tissue that has yet to develop signs of irreversible damage. (See Schwartz 11th ed., p. 523.)

18. Pressure-induced ischemia to cutaneous tissue resulting in nonblanching erythema with intact epidermis represents which stage of injury? A. Does not yet qualify as pressure-induced tissue injury B. Stage I C. Stage II D. Stage III

Answer: B Pressure ulcers can present in several ways depending on the stage at presentation. They are typically grouped into four stages: stage I, nonblanching erythema over intact skin; stage II, partial-thickness injury with blistering or exposed dermis; stage III, full-thickness injury extending down to, but not including, fascia and without undermining of adjacent tissue; and stage IV, full-thickness skin injury with destruction or necrosis of muscle, bone, tendon, or joint capsule. (See Schwartz 11th ed., p. 523.)

19. What type of necrotizing soft tissue infection (NSTI) is characterized by a monomicrobial source of β-hemolytic Streptococcus or Staphylococcus species? A. Type I B. Type II C. Type III D. Type IV

Answer: B There are three types of NSTIs when classified by the offending agent. The most common is type 1, which is caused by a polymicrobial source including gram-positive cocci, gramnegative rods, and anaerobic bacteria, specifically Clostridium perfringens and C. septicum. Type 2 is caused by a monomicrobial source of β-hemolytic Streptococcus or Staphylococcus species, with methicillin-resistant Staphylococcus aureus (MRSA) contributing to the increasing number of communityacquired NSTIs. A history of trauma is often elicited and can be associated with toxic shock syndrome. Type III is a rare but fulminant subset resulting from a Vibrio vulnificus infection of traumatized skin exposed to a body of salt water. (See Schwartz 11th ed., p. 525.)

Brunicardi_Ch16_p127-136.indd 134

30/06/22 11:21 AM

135

21. What cystic lesion is a benign, congenital occurrence caused by persistent epithelium across embryonic fusion lines, most commonly between the forehead and tip of the nose? A. Trichilemmal cyst B. Epidermoid cyst C. Dermoid cyst D. Nevus

Answer: C Dermoid cysts are congenital variants that occur as the result of persistent epithelium within embryonic lines of fusion. They occur most commonly between the forehead and nose tip, and the most frequent site is the eyebrow. They can lie in the subcutaneous tissue or intracranially, and often communicate with the skin surface via a small fistula. These cystic structures contain epithelial tissue, hair, and a variety of epidermal appendages. Treatment for these cystic structures includes surgical excision with care taken to remove the cyst lining to prevent recurrence. (See Schwartz 11th ed., p. 527.)

22. What subtype of basal cell carcinoma (BCC) is the most common, characterized by raised, pearly pink papules with telangiectasias and occasionally a depressed tumor center with raised borders? A. Micronodular B. Infiltrating C. Superficial spreading D. Nodular

Answer: D There are multiple variants of BCC, and presentation can range from red, flesh-colored, or white macule or papule to nodules and ulcerated lesions. Growth patterns of these lesions can either be well-circumscribed or diffuse and the most common types of BCC are nodular and micronodular, superficial spreading, and infiltrative. The most common subtype is the nodular variant, characterized by raised, pearly pink papules with telangiectasias and occasionally a depressed tumor center with raised borders giving the classic “rodent ulcer” appearance. Superficial spreading BCC is confined to the epidermis as a flat, pink, scaling or crusting lesion, often mistaken for eczema, actinic keratosis, fungal infection, or psoriasis. This subtype typically appears on the trunk or extremities and the mean age of diagnosis is 57 years. The infiltrative form appears on the head and neck in the late 60s, often at embryonic fusion lines, with an opaque yellowwhite color that blends with surrounding skin and has no raised edges. The morpheaform subtype represents 2% to 3% of all BCC and is the most aggressive subtype. It usually presents as an indurated macule or papule with the appearance of an enlarging scar. The clinical margins are often indistinct, and the rate of positive margins after excision is high. There is also a pigmented variant of BCC that can be difficult to distinguish from certain melanoma subtypes. (See Schwartz 11th ed., pp. 528–529.)

Brunicardi_Ch16_p127-136.indd 135

The Skin and Subcutaneous Tissue

Answer: A Mucosal HPV types cause lesions in the mucosal or genital areas and behave like sexually transmitted infections. The most common mucosal types are HPV-6, -11, -16, -18, -31, and -33. These lesions present as condylomata acuminata, genital or veneral warts, papules that occur on the perineum, external genitalia, anus, and can extend into the mucosal surfaces of the vagina, urethra, and rectum. These lesions are at risk for malignant transformation, with types 6 and 11 conferring low risk, and types 16, 18, 31, and 33 conferring a high risk. The recently developed quadrivalent HPV vaccine, targeting HPV types -6, -11, -16, and -18, is now available to both males and females age 9 to 26 and is associated with an up to 90% reduction of infections from those HPV types. (See Schwartz 11th ed., p. 526.)

CHAPTER 16

20. What subtypes of the human papilloma virus (HPV) confer the highest risk for malignancy? A. Types 16 and 18 B. Types 6 and 11 C. Types 6 and 8 D. Types 3 and 4

30/06/22 11:21 AM

136 23. Which of these skin lesions can represent a precursor lesion to squamous cell carcinoma? A. Seborrheic keratosis B. Actinic keratosis C. Congenital nevus D. Acrochordons

CHAPTER 16 The Skin and Subcutaneous Tissue Brunicardi_Ch16_p127-136.indd 136

Answer: B Actinic keratoses are neoplasms of epidermal keratinocytes that represent a range in a spectrum of disease from sun damage to squamous cell carcinoma. They typically occur in fair-skinned, elderly individuals in primarily sun-exposed areas, and ultraviolet (UV) radiation exposure is the greatest risk factor. There are multiple variants, and they can present as erythematous and scaly to hypertrophic, keratinized lesions. They can become symptomatic, causing bleeding, pruritus, and pain. They can regress spontaneously, persist without change, and transform into invasive squamous cell carcinoma. It is estimated that approximately 10% of actinic keratoses will transform into invasive squamous cell carcinoma, and that progression takes about 2 years on average. About 60% to 65% of squamous cell carcinomas are believed to originate from actinic keratoses. Congenital nevi can rarely transform into malignant melanoma. Seborrheic keratoses and acrochordons do not carry malignant potential. (See Schwartz 11th ed., p. 528.)

30/06/22 11:21 AM

CHAPTER

17 Breast

1. Which of the following changes in the breast is not associated with pregnancy? A. Accumulation of lymphocytes, plasma cells, and eosinophils within the breast B. Enlargement of breast alveoli C. Release of colostrum D. Accumulation of secretory products in minor duct lumina

Answer: C With pregnancy, the breast undergoes proliferative and developmental maturation. As the breast enlarges in response to hormonal stimulation, lymphocytes, plasma cells, and eosinophils accumulate within the connective tissues. The minor ducts branch and alveoli develop. Development of the alveoli is asymmetric, and variations in the degree of development may occur within a single lobule. With parturition, enlargement of the breasts occurs via hypertrophy of alveolar epithelium and accumulation of secretory products in the lumina of the minor ducts. Alveolar epithelium contains abundant endoplasmic reticulum, large mitochondria, Golgi complexes, and dense lysosomes. Two distinct substances are produced by the alveolar epithelium: (a) the protein component of milk, which is synthesized in the endoplasmic reticulum (exocrine secretion); and (b) the lipid component of milk (apocrine secretion), which forms as free lipid droplets in the cytoplasm. Milk released in the first few days after parturition is called colostrum and has low lipid content but contains considerable quantities of antibodies. (See Schwartz 11th ed., p. 548.)

2. Which of the following statements is incorrect? A. Level I lymph nodes are those that are lateral to the pectoralis minor muscle. B. Level II lymph nodes are located deep to the pectoralis minor muscle. C. Level Ill lymph nodes are located medial to the pectoralis minor muscle. D. Level IV lymph nodes are the ipsilateral internal mammary lymph nodes.

Answer: D Axillary lymph node groups are as follows: Level I includes lymph nodes located lateral to the pectoralis minor muscle; level II includes lymph nodes located deep to the pectoralis minor; and level Ill includes lymph nodes located medial to the pectoralis minor. (See Schwartz 11th ed., p. 547.)

3. Concerning gynecomastia, which of the following is TRUE? A. During senescence gynecomastia is usually unilateral. B. During puberty gynecomastia is usually bilateral. C. Gynecomastia is associated with breast cancer in Klinefelter syndrome patients. D. Gynecomastia is classified as per a three-grade system.

Answer: C In gynecomastia, the ductal structures of the male breast enlarge, elongate, and branch with a concomitant increase in epithelium. During puberty, the condition often is unilateral and typically occurs between ages 12 and 15 years. In contrast, senescent gynecomastia is usually bilateral. Gynecomastia generally does not predispose the male breast to cancer. However, the hypoandrogenic state of Klinefelter syndrome (XXY), in which gynecomastia is usually evident, is associated with an increased risk of breast cancer. Gynecomastia is graded based on the degree of breast enlargement, the position

137

Brunicardi_Ch17_p137-142.indd 137

30/06/22 11:21 AM

138 of the nipple with reference to the inframammary fold and the degree of breast ptosis and skin redundancy: Grade 1: Mild breast enlargement without skin redundancy; Grade Ila: Moderate breast enlargement without skin redundancy; Grade llb: Moderate breast enlargement with skin redundancy; and Grade 3: Marked breast enlargement with skin redundancy and ptosis. (See Schwartz 11th ed., p. 549.)

CHAPTER 17

Answer: C Sclerosing adenosis is prevalent during the childbearing and premenopausal years and has no malignant potential. Multiple intraductal papillomas, which occur in younger women and are less frequently associated with nipple discharge, are susceptible to malignant transformation. Individuals with a diagnosis of ADH are at increased risk for development of breast cancer and should be counseled appropriately regarding risk reduction strategies. ALH results in minimal distention of lobular units with cells that are similar to those seen in lobular carcinoma in situ (LCIS). (See Schwartz 11th ed., pp. 552–553.)

5. Risk factors for the development of breast cancer include: A. Early menarche B. Nulliparity C. Late menopause D. Longer lactation periods

Answer: D Increased exposure to estrogen is associated with an increased risk for developing breast cancer, whereas reducing exposure is thought to be protective. Correspondingly, factors that increase the number of menstrual cycles, such as early menarche, nulliparity, and late menopause are associated with increased risk. Moderate levels of exercise and a longer lactation period, factors that decrease the total number of menstrual cycles, are protective. (See Schwartz 11th ed., p. 555.)

6. All of the following are TRUE concerning breast lobular carcinoma in situ (LCIS) EXCEPT: A. Develops only in the female breast. B. Cytoplasmic mucoid globules are a distinctive cellular feature. C. Frequency of LCIS cannot be reliably determined. D. The average age at diagnosis is 65 to 70 years.

Answer: D LCIS originates from the terminal duct lobular units and develops only in the female breast. Cytoplasmic mucoid globules are a distinctive cellular feature. The frequency of LCIS in the general population cannot be reliably determined because it usually presents as an incidental finding. The average age at diagnosis is 45 years, which is approximately 15 to 25 years younger than the age at diagnosis for invasive breast cancer. (See Schwartz 11th ed., pp. 553–554.)

7. Which of the following concerning breast cancer staging is correct? A. Stage I tumors have no metastases to either lymph nodes or distant sites. B. Stage Ill tumors include some with distant metastases (M1 disease). C. Inflammatory carcinoma is considered T4 disease. D. N4 disease includes metastases to highest contralateral axillary nodes.

Answer: C See Schwartz 11th ed., pp. 576–578.

8. Which of the following statements about the management of distal carcinoma in situ (DCIS) is TRUE? A. DCIS treated by mastectomy has a local recurrence rate of 4 cm of disease or disease in more than one quadrant) usually require mastectomy. For women with limited disease, lumpectomy and radiation therapy are generally recommended. For no palpable DCIS, needle localization or other image-guided techniques are used to guide the surgical resection. Specimen mammography is performed to ensure that all visible evidence of cancer is excised. Adjuvant tamoxifen therapy is considered for DCIS patients with estrogen-receptor

Breast

4. Lesions with malignant potential include all of the following EXCEPT: A. lntraductal papilloma. B. Atypical ductal hyperplasia (ADH). C. Sclerosing adenosis. D. Atypical lobular hyperplasia (ALH).

Brunicardi_Ch17_p137-142.indd 138

30/06/22 11:21 AM

139 (ER)-positive disease. The gold standard against which breast conservation therapy for DCIS is evaluated is mastectomy. Women treated with mastectomy have local recurrence and mortality rates of 70 years. B. Radiation. C. Male gender. D. Excessive corticosteroid therapy.

Answer: C Intubation-related risk factors include: prolonged intubation; high tracheostomy through the first tracheal ring or cricothyroid membrane; transverse rather than vertical incision on the trachea; oversized tracheostomy tube; prior tracheostomy or intubation; and traumatic intubation. Stenosis is also more common in older patients, in females, after radiation, or after excessive corticosteroid therapy, and in the setting of concomitant diseases such as autoimmune disorders, severe reflux disease, or obstructive sleep apnea and the setting of severe respiratory failure. However, even a properly placed tracheostomy can lead to tracheal stenosis because of scarring and local injury. Mild ulceration and stenosis are frequently seen after tracheostomy removal. Use of the smallest tracheostomy tube possible, rapid downsizing, and a vertical tracheal incision minimize the risk for posttracheostomy stenosis. (See Schwartz 11th ed., p. 663.)

Brunicardi_Ch19_p145-156.indd 146

04/07/22 1:51 PM

147 4. The tracheobronchial tree consists of approximately 23 airway divisions to the level of the alveoli. Mucus production in the airways is derived from which cell type? A. Clara cells B. Kulchitsky cells C. Goblet cells D. Type 1 pneumocytes

CHAPTER 19 Chest Wall, Lung, Mediastinum, and Pleura

The lung can be conveniently viewed as two linked components: The tracheobronchial tree (or conducting airways component) and the alveolar spaces (or gas exchange component). The tracheobronchial tree consists of approximately 23 airway divisions to the level of the alveoli. It includes the main bronchi, lobar bronchi, segmental bronchi (to designated bronchopulmonary segments), and terminal bronchioles (ie, the smallest airways still lined by bronchial epithelium and without alveoli). The tracheobronchial tree is normally lined by pseudostratified ciliated columnar cells and mucous (or goblet) cells, which both derive from basal cells (Fig. 19-3). Ciliated cells predominate. Goblet cells, which release mucus, can significantly increase in number in acute bronchial injury, such as exposure to cigarette smoke. The normal bronchial epithelium also contains bronchial submucosal glands, which are mixed salivary-type glands containing mucous cells, serous cells, and neuroendocrine cells called Kulchitsky cells, which are also found within the surface epithelium. The bronchial submucosal glands can give rise to salivary gland– type tumors, including mucoepidermoid carcinomas and adenoid cystic carcinomas. (See Schwartz 11th ed., p. 668.)

A

FIG. 19-3.  Normal lung histology. A. Pseudostratified ciliated columnar cells and mucous cells normally line the tracheobronchial tree. B. A Kulchitsky cell is depicted (arrow).

Brunicardi_Ch19_p145-156.indd 147

B

04/07/22 1:51 PM

148

CHAPTER 19 Chest Wall, Lung, Mediastinum, and Pleura

5. Stepwise pathologic progression of normal lung cellular architecture to invasive adenocarcinoma follows which of the following patterns? A. Adenocarcinoma in-situ --> Minimally invasive adenocarcinoma --> Lepidic predominate adenocarcinoma --> Invasive adenocarcinoma B. Minimally invasive adenocarcinoma --> Adenocarcinoma in-situ --> Lepidic predominate adenocarcinoma --> Invasive adenocarcinoma C. Adenocarcinoma in-situ --> Lepidic predominate adenocarcinoma --> Minimally invasive adenocarcinoma --> Invasive adenocarcinoma D. Minimally invasive adenocarcinoma --> Lepidic predominate adenocarcinoma --> Adenocarcinoma insitu --> Invasive adenocarcinoma

Answer: A The incidence of adenocarcinoma in the lung has increased over the last several decades and is now the most common type of lung cancer. It occurs more frequently in females than in males and is the most frequent histologic subtype in women, patients under the age of 45, and Asian populations. Adenocarcinoma can be divided into mucinous and nonmucinous types. Based on consensus, the international working group proposed a multidisciplinary approach, with standardized criteria and terminology for diagnosis in cytologic and small biopsy specimens, and routine molecular testing for known mutations, such as estimated glomerular filtration rate (EGFR) and KRAS mutations. The new classification system delineated a stepwise pathologic progression, from Atypical adenomatous hyperplasia (AAH) to invasive adenocarcinoma based on the predominant histologic growth patterns; the terms bronchioloalveolar carcinoma and mixed subtype adenocarcinoma were eliminated in favor of more biologically driven classification. (See Schwartz 11th ed., p. 669.)

6. The grade of neuroendocrine carcinoma (NEC) that is associated with hemoptysis, pneumonia, and tumor cells arranged in cords and clusters is: A. Grade VI NEC. B. Grade IV NEC. C. Grade II NEC. D. Grade I NEC.

Answer: D Grade I NEC (classic or typical carcinoid) is a low-grade NEC; 80% arise in the epithelium of the central airways. It occurs primarily in younger patients. Because of the central location, it classically presents with hemoptysis, with or without airway obstruction and pneumonia. Histologically, tumor cells are arranged in cords and clusters with a rich vascular stroma. This vascularity can lead to life-threatening hemorrhage with even simple bronchoscopic biopsy maneuvers. Regional lymph node metastases are seen in 15% of patients, but rarely spread systemically or cause death. (See Schwartz 11th ed., p. 672.)

7. CT imaging is used routinely in diagnosis of malignant lung cancer. CT imaging findings associated with malignancy include all the following EXCEPT: A. Size > 2 cm. B. Irregular, lobulated, or spiculated edges. C. Corona radiata sign. D. Stippled or eccentric calcifications.

Answer: A Spiral (helical) CT imaging is the gold standard for definitive diagnosis of lung cancer. CT findings characteristic of malignancy include growth over time. Increasing density on CT scan (40% to 50% of partial solid lesions are malignant compared to only 15% of subcentimeter solid or nonsolid nodules). Size > 3 cm. Irregular, lobulated, or spiculated edges. The finding of the corona radiata sign (consisting of fine linear strands extending 4 to 5 mm outward and appearing spiculated on radiographs). Calcification that is stippled, amorphous, or eccentric is usually associated with cancer. (See Schwartz 11th ed., p. 678.)

8. The most common pattern of benign calcification in hamartomas is: A. Solid. B. Diffuse. C. Central. D. Popcorn.

Answer: D CT findings characteristic of benign lesions include small size, calcification within the nodule, and stability over time. Four patterns of benign calcification are common: diffuse, solid, central, and laminated or “popcorn.” Granulomatous infections such as tuberculosis can demonstrate the first three patterns, whereas the popcorn pattern is most common in hamartomas. In areas of endemic granulomatous disease, differentiating benign versus malignant can be challenging. Infectious granulomas arising from a variety of organisms account for 70% to 80% of this type of benign solitary nodules; hamartomas are the next most common single cause, accounting for about 10%. (See Schwartz 11th ed., p. 685.)

Brunicardi_Ch19_p145-156.indd 148

04/07/22 1:51 PM

149

10. Which of the following is NOT a known predictive or prognostic tumor marker for adenocarcinoma? A. Estimated glomerular filtration rate (EGFR) B. KRAS mutation C. AFP (Alpha feto protein) D. EML4-ALK fusion gene

Answer: C Lung cancer management. Role of histologic diagnosis and molecular testing. Establishing a clear histologic diagnosis early in the evaluation and management of lung cancer is critical to effective treatment. Molecular signatures are also key determinants of treatment algorithms for adenocarcinoma and will likely become important for squamous cell carcinoma as well. Currently, differentiation between adenocarcinoma and squamous cell carcinoma in cytologic specimens or small biopsy specimens is imperative in patients with advanced stage disease, as treatment with pemetrexed or bevacizumab-based chemotherapy is associated with improved progression-free survival in patients with adenocarcinoma but not squamous cell cancer. Furthermore, life-threatening hemorrhage has occurred in patients with squamous cell carcinoma who were treated with bevacizumab. Finally, EGFR mutation predicts response to EGFR tumor kinase inhibitors and is now recommended as first-line therapy in advanced adenocarcinoma. Because adequate tissue is required for histologic assessment and molecular testing, each institution should have a clear, multidisciplinary approach to patient evaluation, tissue acquisition, tissue handling/processing, and tissue analysis (Fig. 19-4). In many cases, tumor morphology differentiates adenocarcinoma from the other histologic subtypes. If no clear morphology can be identified, then additional testing for one immunohistochemistry marker for adenocarcinoma and one for squamous cell carcinoma will usually enable differentiation. Immunohistochemistry for neuroendocrine markers is reserved for lesions exhibiting neuroendocrine morphology. Additional molecular testing should be performed on all adenocarcinoma specimens for known predictive and prognostic tumor markers (eg, EGFR, KRAS, and EML4ALK fusion gene). Ideally, use of tissue sections and cell block material is limited to the minimum necessary at each decision point. This emphasizes the importance of a multidisciplinary approach; surgeons and radiologists must work in direct cooperation with the cytopathologist to ensure that tissue samples are adequate for morphologic diagnosis as well as providing sufficient cellular material to enable molecular testing. (See Schwartz 11th ed., p. 683.)

Brunicardi_Ch19_p145-156.indd 149

Chest Wall, Lung, Mediastinum, and Pleura

Answer: D Nonpulmonary thoracic symptoms are often due to tumor invasion into surrounding thoracic structures. This can lead to symptoms of Horner syndrome (Pancoast tumors), pericardial tamponade, back pain, and recurrent laryngeal nerve palsy leading to hoarseness and coughing. Phrenic nerve palsy is due to invasion of a tumor into the phrenic nerve. The phrenic nerve traverses the hemithorax along the mediastinum, parallel and posterior to the superior vena cava and anterior to the pulmonary hilum. Tumors at the medial lung surface or anterior hilum can directly invade the nerve; symptoms include referred shoulder pain, hiccups, and dyspnea with exertion because of diaphragm paralysis. Radiographically, unilateral diaphragm elevation on chest radiograph is present. The diagnosis can be confirmed by fluoroscopic examination of the diaphragm with paradoxical motion with breathing and sniffing, also known as the “Sniff Test.” (See Schwartz 11th ed., p. 680.)

CHAPTER 19

9. A patient diagnosed with lung cancer presents with symptoms of dyspnea with exertion, hiccups, and right shoulder pain. The most likely location of this tumor is: A. Left lower lobe central squamous cell carcinoma. B. Right upper lobe apical adenocarcinoma. C. Right lower lobe peripheral large cell carcinoma. D. Right middle lobe central squamous cell carcinoma.

04/07/22 1:51 PM

150 STEP 1 POSITIVE BIOPSY (FOB, TBBx, Core, SLBx) POSITIVE CYTOLOGY (effusion, aspirate, washings, brushings)

NE morphology, large cells, NE IHC+

NSCLC, LCNEC

NE morphology, small cells, no nucleoli, NE IHC+, TTF-1 +/–, CK+

SCLC

Keratinization, pearls and/or intercellular bridges

CHAPTER 19 Chest Wall, Lung, Mediastinum, and Pleura

Histology: Lepidic, papillary, and/or acinar architecture(s) Cytology: 3-D arrangements, delicate foamy/vacuolated (translucent) cytoplasm, Fine nuclear chromatin and often prominent nucleoli Nuclei are often eccentrically situated

Classic Morphology: SQCC

No clear ADC or SQCC morphology: NSCLC-NOS NSCLC, favor SQCC

Classic morphology: ADC ADC marker and/or Mucin +ve; SQCC marker –ve (or weak in same cells)

SQCC marker +ve ADC marker –ve/or Mucin –ve

STEP 2 Apply ancillary panel of One SQCC and one ADC marker +/OR Mucin

IHC –ve and Mucin –ve

ADC marker or Mucin +ve; as well as SQCC marker +ve in different cells

NSCLC, favor ADC NSCLC NOS NSCLC, NOS, possible adenosquamous ca

Molecular analysis: eg, EGFR mutation† STEP 3

If tumor tissue inadequate for molecular testing, discuss need for further sampling — back to Step 1

FIG. 19-4.  Algorithm for adenocarcinoma diagnosis in small biopsies and/or cytology. Step 1: When positive biopsies (fiberoptic bronchoscopy [FOB], transbronchial [TBBx], core, or surgical lung biopsy [SLBx]) or cytology (effusion, aspirate, washings, and brushings) show clear adenocarcinoma (ADC) or squamous cell carcinoma (SQCC) morphology, the diagnosis can be firmly established. If there is neuroendocrine (NE) morphology, the tumor may be classified as small cell carcinoma (SCLC) or non-small-cell lung carcinoma (NSCLC), probably large cell neuroendocrine carcinoma (LCNEC) according to standard criteria (+ = positive, − = negative, and ± = positive or negative). If there is no clear ADC or SQCC morphology, the tumor is regarded as NSCLC -not otherwise specified (NOS). Step 2: NSCLC-NOS can be further classified based on (a) immunohistochemical stains, (b) mucin (DPAS or mucicarmine) stains, or (c) molecular data. If the stains all favor ADC-positive ADC marker(s) (ie, TTF-1 and/or mucin positive) with negative SQCC markers, then the tumor is classified as NSCLC, favor ADC. If SQCC markers (ie, p63 and/or CK5/6) are positive with negative ADC markers, the tumor is classified as NSCLC, favor SQCC. If the ADC and SQCC markers are both strongly positive in different populations of tumor cells, the tumor is classified as NSCLC-NOS, with a comment it may represent adenosquamous carcinoma. If all markers are negative, the tumor is classified as NSCLC-NOS. †EGFR mutation testing should be performed in (1) classic ADC, (2) NSCLC, favor ADC, (3) NSCLC-NOS, and (4) NSCLC-NOS, possible adenosquamous carcinoma. In NSCLC-NOS, if EGFR mutation is positive, the tumor is more likely to be ADC than SQCC. Step 3: If clinical management requires a more specific diagnosis than NSCLC-NOS, additional biopsies may be indicated. CD = cluster designation; CK = cytokeratin; DPAS = diastase-periodic acid Schiff; DPAS +ve = periodic-acid Schiff with diastase; EGFR = epidermal growth factor receptor; IHC = immunohistochemistry; NB = of note; TTF-1 = thyroid transcription factor-1; -ve = negative; +ve = positive. (Reproduced with permission from Travis WD, Brambilla E, Noguchi M, et al: Diagnosis of lung cancer in small biopsies and cytology: implications of the 2011 International Association for the Study of Lung Cancer/American Thoracic Society/European Respiratory Society classification, Arch Pathol Lab Med. 2013;137(5):668–684.)

Brunicardi_Ch19_p145-156.indd 150

04/07/22 1:51 PM

151

12. A 57-year-old non-small-cell lung cancer patient with a potentially resectable tumor found on CT scan who can walk on a flat surface indefinitely without oxygen or stopping to rest, secondary to dyspnea will most likely tolerate: A. Lobectomy. B. Pneumonectomy. C. Single-lung ventilation. D. Wedge resection.

Answer: A Assessment of functional status. Patients with potentially resectable tumors require careful assessment of their functional status and ability to tolerate either lobectomy or pneumonectomy. The surgeon should first estimate the likelihood of pneumonectomy, lobectomy, or possibly sleeve resection, based on the CT images. A sequential process of evaluation then unfolds. A patient’s history is the most important tool for gauging risk. Specific questions regarding performance status should be routinely asked. If the patient can walk on a flat surface indefinitely, without oxygen and without having to stop and rest secondary to dyspnea, he will be very likely to tolerate lobectomy. If the patient can walk up two flights of stairs (up two standard levels), without having to stop and rest secondary to dyspnea, she will likely tolerate pneumonectomy. Finally, nearly all patients, except those with carbon dioxide (CO2) retention on arterial blood gas analysis, will be able to tolerate periods of single-lung ventilation and wedge resection. (See Schwartz 11th ed., p. 691.)

Brunicardi_Ch19_p145-156.indd 151

Chest Wall, Lung, Mediastinum, and Pleura

Answer: D Mediastinal lymph node staging by PET scanning appears to have greater accuracy than CT scanning. PET staging of mediastinal lymph nodes has been evaluated in two metaanalyses. The overall sensitivity for mediastinal lymph node metastasis was 79% (95% confidence interval [CI] 76%–82%), with a specificity of 91% (95% CI 89%–93%) and an accuracy of 92% (95% CI 90%–94%). In comparing PET with CT scans in patients who also underwent lymph node biopsies, PET had a sensitivity of 88% and a specificity of 91%, whereas CT scanning had a sensitivity of 63% and a specificity of 76%. Combining CT and PET scanning may lead to even greater accuracy. In one study of CT, PET, and mediastinoscopy in 68 patients with potentially operable NSCLC, CT correctly identified the nodal stage in 40 patients (59%). It understaged the tumor in 12 patients and overstaged it in 16 patients. PET correctly identified the nodal stage in 59 patients (87%). It understaged the tumor in five patients and overstaged it in four. For detecting N2 and N3 disease, the combination of PET and CT scanning yielded a sensitivity, specificity, and accuracy of 93%, 95%, and 94%, respectively. CT scan alone yielded 75%, 63%, and 68%, respectively. Studies examining combined PET-CT consistently show improved accuracy compared to PET or CT alone; accuracy for PET-CT nodal positivity confirmed by mediastinoscopy is approximately 75%, with a negative predictive value of approximately 90%. (See Schwartz 11th ed., p. 687.)

CHAPTER 19

11. Evaluation of mediastinal lymphadenopathy by noninvasive imaging is best performed by which of the following modalities? A. Positron emission tomography (PET) scan B. CT scan C. MRI(magnetic resonance imaging) D. PET-CT

04/07/22 1:51 PM

152

CHAPTER 19 Chest Wall, Lung, Mediastinum, and Pleura

13. A 45-year-old man with 20 pack-year history of smoking is diagnosed with squamous cell carcinoma of the right middle lobe. Clinically the patient has symptoms of hoarseness and coughing when drinking liquids. The T staging for this tumor is: A. T2a. B. T2b. C. T3. D. T4.

Answer: D The staging of solid epithelial tumors is based on the TNM staging system. The primary tumor “T” status provides information about tumor size and relationship to surrounding structures; the “N” status provides information about regional lymph nodes; and the “M” status provides information about the presence or absence of metastatic disease. The designation of lymph nodes as N1, N2, or N3 requires familiarity with the lymph node mapping system. Based on clearly delineated anatomic boundaries, accurate and reproducible localization of thoracic lymph nodes is possible, facilitating detailed nodal staging for individual patients and standardization of nodal assessment between surgeons. In the case described, the patient’s symptoms of dysphagia and hoarseness suggests involvement of the recurrent laryngeal nerve, which confers him a T4 designation in the TNM classification system. (See Schwartz 11th ed., p. 689.)

14. Patients with resectable tumors require assessment of their pulmonary functional status to determine if they are able to tolerate surgical resection. The two most reliable predictors of pulmonary functional capacity are: A. FEV1 and DLCO. B. RV and TLC. C. RV and DLCO. D. FEV1 and RV.

Answer: A Pulmonary function studies are routinely performed when any resection greater than a wedge resection will be performed. Of all the measurements available, the two most valuable are forced expiratory volume in 1 second (FEV1) and carbon monoxide diffusion capacity (DLCO). General guidelines for the use of FEV1 in assessing the patient’s ability to tolerate pulmonary resection are as follows: Greater than 2.0 L can tolerate pneumonectomy, and >1.5 L can tolerate lobectomy. It must be emphasized that these are guidelines only. It is also important to note that the raw value is often imprecise because normal values are reported as “percent predicted” based on corrections made for age, height, and gender. Notably it is not uncommon to encounter patients with significant reductions in FEV1 and DLCO that are inconsistent with the patient’s functional status. In these patients, . exercise testing that yields maximal oxygen consumption (VO2max) has emerged as a valuable decision-making technique to help patients with abnormal FEV1 and DLCO. (See Schwartz 11th ed., p. 692.)

15. Which of the following is an indication for surgical drainage of a lung abscess? A. Abscess > 3 cm in diameter B. Hemoptysis C. Failure to decrease in size after 1 week of antibiotic therapy D. Persistent fever

Answer: B Surgical drainage of lung abscesses is uncommon since drainage usually occurs spontaneously via the tracheobronchial tree. Indications for intervention are listed in Table 19-1. (See Schwartz 11th ed., p. 708.) TABLE 19-1

Indications for surgical drainage procedures for lung abscesses

1. 2. 3. 4. 5. 6.

Failure of medical therapy Abscess under tension Abscess increasing in size during appropriate treatment Contralateral lung contamination Abscess > 4–6 cm in diameter Necrotizing infection with multiple abscesses, hemoptysis, abscess rupture, or pyopneumothorax 7. Inability to exclude a cavitating carcinoma

Brunicardi_Ch19_p145-156.indd 152

04/07/22 1:51 PM

153

17. The population most at risk for developing active tuberculosis is: A. Elderly. B. Minorities. C. Urban residents. D. HIV infection.

Answer: D Epidemiology. Tuberculosis is a widespread problem that affects nearly one-third of the world’s population. Between 8.3 and 9 million new cases of tuberculosis and 12 million prevalent cases (range 10–13 million) were estimated worldwide in 2011 according to the World Health Organization. Only 10,521 new cases were reported to the World Health Organization in the United States in 2011. HIV infection is the strongest risk factor for developing active tuberculosis. The elderly, minorities, and recent immigrants are the most common populations to have clinical manifestations of infection, yet no age group, sex, or race is exempt from infection. In most large urban centers, reported cases of tuberculosis are more numerous among the homeless, prisoners, and drugaddicted populations. Immunocompromised patients additionally contribute to an increased incidence of tuberculosis infection, often developing unusual systemic as well as pulmonary manifestations. (See Schwartz 11th ed., p. 710.)

18. The fungi associated with the highest mortality rate due to invasive mycoses in the United States is: A. Aspergillus. B. Cryptococcus. C. Candidia. D. Mucor.

Answer: A Aspergillosis. The genus Aspergillus comprises over 150 species and is the most common cause of mortality due to invasive mycoses in the United States. It is typically acute in onset and life-threatening and occurs in the setting of neutropenia, chronic steroid therapy, or cytotoxic chemotherapy. It can also occur in the general intensive care unit population of critically ill patients, including patients with underlying chronic obstructive pulmonary disease (COPD), postoperative patients, patients with cirrhosis or alcoholism, and postinfluenza patients, without any of these factors present. The species most commonly responsible for clinical disease include A. fumigatus, A. flavus, A. niger, and A. terreus. Aspergillus is a saprophytic, filamentous fungus with septate hyphae. Spores (2.5–3 μm in diameter) are released and easily inhaled by susceptible patients; because the spores are microns in size, they are able to reach the distal bronchi and alveoli. (See Schwartz 11th ed., p. 711.)

Brunicardi_Ch19_p145-156.indd 153

Chest Wall, Lung, Mediastinum, and Pleura

Answer: C Microbiology. Normal oropharyngeal secretions contain many more Streptococcus species and more anaerobes (approximately 1 × 108 organisms/mL) than aerobes (approximately 1 × 107organisms/mL). Pneumonia that follows from aspiration, with or without abscess development, is typically polymicrobial. An average of two to four isolates present in large numbers have been cultured from lung abscesses sampled percutaneously. Overall, at least 50% of these infections are caused by purely anaerobic bacteria, 25% are caused by mixed aerobes and anaerobes, and 25% or fewer are caused by aerobes only. In nosocomial pneumonia, 60% to 70% of the organisms are gram-negative bacteria, including Klebsiella pneumoniae, Haemophilus influenzae, Proteus species, Pseudomonas aeruginosa, Escherichia coli, Enterobacter cloacae, and Eikenella corrodens. Immunosuppressed patients may develop abscesses because of the usual pathogens as well as less virulent andopportunistic organisms such as Salmonella species, Legionella species, Pneumocystis carinii, atypical mycobacteria, and fungi. (See Schwartz 11th ed., p. 708.)

CHAPTER 19

16. The most likely cause of aspiration pneumonia is: A. A mixture of aerobes and anaerobes. B. Aerobes only. C. Anaerobes only. D. Gram-negative bacteria.

04/07/22 1:51 PM

154

CHAPTER 19 Chest Wall, Lung, Mediastinum, and Pleura

19. Previously, patients with severe primary pulmonary hypertension were listed for lung transplantation due to respiratory failure. The use of the following medication has significantly altered the need for transplantation: A. Epoprostenol. B. Midodrine. C. Dobutamine. D. Nitrates.

Answer: A Currently, virtually all patients with primary pulmonary hypertension are now treated with intravenous epoprostenol. Prostacyclin is a member of the endogenous prostanoid family and is produced from arachidonic acid by the enzymes prostacyclin synthase and cyclooxygenase (COX). It acts to increase pulmonary vasodilation within the lungs. Espoprostenol must be given by continuous infusion. (See Schwartz 11th ed., p. 719.)

20. The most common benign chest wall tumor is: A. Chondromas. B. Osteochondromas. C. Desmoid tumors. D. Fibrous dysplasia.

Answer: A Chondroma. Chondromas, seen primarily in children and young adults, are one of the more common benign tumors of the chest wall. They usually occur at the costochondral junction anteriorly and may be confused with costochondritis, except that a painless mass is present. Radiographically, lesion is lobulated and radiodense; it may have diffuse or focal calcifications; and it may displace the bonycortex without penetration. Chondromas may grow to huge sizes if left untreated. Treatment is surgical resection with a 2-cm margin. Large chondromas may harbor well-differentiated chondrosarcoma and should be managed with a 4-cm margin to prevent local recurrence. (See Schwartz 11th ed., p. 722.)

21. An “onion-peel” appearance of a rib on computed tomography is suggestive of: A. Chondroma. B. Ewing sarcoma. C. Plasmacytoma. D. Osteosarcoma.

Answer: B Primitive neuroectodermal tumors (PNETs) and Ewing sarcoma. PNETs (neuroblastomas, ganglioneuroblastomas, and ganglioneuromas) derive from primordial neural crest cells that migrate from the mantle layer of the developing spinal cord. Histologically, PNETs and Ewing sarcomas are small, round cell tumors; both possess a translocation between the long arms of chromosomes 11 and 22 within their genetic makeup. They also share a consistent pattern of protooncogene expression and have been found to express the product of the MIC2 gene. Ewing sarcoma occurs in adolescents and young adults who present with progressive chest wall pain, but without the presence of a mass. Systemic symptoms of malaise and fever are often present. Laboratory studies reveal an elevated erythrocyte sedimentation rate and mild white blood cell elevation. Radiographically, the characteristic onion peel appearance is produced by multiple layers of periosteum in the bone formation. Evidence of bony destruction is also common. The diagnosis can be made by a percutaneous needle biopsy or an incisional biopsy. (See Schwartz 11th ed., p. 725.)

22. Desmoid tumors are associated with which of the following gene? A. KRAS B. RET C. P53 D. Adenomatous polyposis coli (APC)

Answer: D Desmoid tumors. Soft tissue neoplasms arising from fascial or musculoaponeurotic structures, desmoid tumors consist of proliferations of benign appearing fibroblastic cells, abundant collagen, and few mitoses. Desmoid tumors possess alterations in the APC/β-catenin pathway. Cyclin D1 dysregulation is thought to play a significant role in their pathogenesis. Associations with other diseases and conditions are well documented, especially those with similar alterations in the APC pathway, such as familial adenomatous polyposis (Gardner syndrome). Other conditions with increased risk of desmoid tumor formation include increased estrogen states (pregnancy) and trauma. Surgical incisions (abdominal and thorax) have been the site of desmoid development, either in or near the scar. (See Schwartz 11th ed., p. 722.)

Brunicardi_Ch19_p145-156.indd 154

04/07/22 1:51 PM

155

24. A patient with an anterior mediastinal mass and elevated serum alpha fetoprotein most likely has: A. A teratoma. B. A nonseminomatous germ-cell tumor. C. A seminomatous germ-cell tumor. D. Metastatic hepatocellular carcinoma.

Answer: B The use of serum markers to evaluate a mediastinal mass can be invaluable in some patients. For example, nonseminomatous and seminomatous germ-cell tumors can frequently be diagnosed and often distinguished from one another by the levels of α-fetoprotein (AFP) and human chorionic gonadotropin (hCG). In >90% of nonseminomatous germ-cell tumors, either the AFP or the hCG level will be elevated. Results are close to 100% specific if the level of either AFP or hCG is >500 ng/mL. Some centers institute chemotherapy based on this result alone, without biopsy confirmation of the diagnosis. In contrast, the AFP level in patients with mediastinal seminoma is always normal; only 10% will have elevated hCG, which is usually 110 mg/100 mL, a chylothorax is almost certainly present (a 99% accuracy rate). If the triglyceride level is 40. D. Male gender.

Answer: B Malignant mesothelioma is the most common type of primary tumor of the pleura, with approximately 3000 cases per year in the United States. Other, less common tumors include benign and malignant fibrous tumors of the pleura, lipomas, and cysts. The only known risk factor for mesothelioma is exposure to asbestos, identified in >50% of cases. Exposure is typically work-related in industries using asbestos in the manufacturing process, such as shipbuilding and brake pad linings. The risk extends to family members who are exposed to the dust of the clothing or to the work environment. Asbestos exposure and smoking synergistically increase the risk for lung cancer, but smoking does not increase risk for malignant mesotheliomas. Male predominance is 2:1, and it occurs most commonly after the age of 40. (See Schwartz 11th ed., p. 743.)

Chest Wall, Lung, Mediastinum, and Pleura

26. A chylothorax is likely to be present in a patient whose pleural fluid analysis results show a triglyceride level of: A. 80 mg/100 mL. B. 100 mg/100 mL. C. 45 mg/100 mL. D. 130 mg/100 mL.

Brunicardi_Ch19_p145-156.indd 156

04/07/22 1:51 PM

CHAPTER

20

Congenital Heart Disease

1. The most common form of atrial septal defect (ASD) is: A. Sinus venosus defect. B. Ostium primum defect. C. Ostium secundum defect. D. Combined primum and secundum defect.

A

Answer: C ASDs can be classified into three different types (Fig. 20-1): (a) ostium secundum type defect (Fig. 20-1B,C) (deficiency of septum primum), which are the most prevalent subtype, comprising 80% of all ASDs; (b) ostium primum defects (Fig. 20-1A), which may also be described as partial or transitional AV canal defect; and (c) sinus venosus type defects, comprising approximately 5% to 10% of all ASDs. (See Schwartz 11th ed., p. 752.)

B

FIG. 20-1.  A. Echocardiogram of a patient with primum type artial septal defect (`*’ points to the atrial septal defect). B. Echocardiogram of a large secundum type ASD (`*’ points to the defect). C. Intraoperative picture during repair of atrial septal defect. A large fenestrated atrial septum is seen. Bicaval venous cannulation has been performed and a right atriotomy provides exposure to the atrial septum.

C

157

Brunicardi_Ch20_p157-170.indd 157

06/07/22 9:41 AM

158 Answer: C In general, ASDs are closed when patients are between 4 and 5 years of age. Children of this size can usually be operated on without the use of blood transfusion and have excellent outcomes. Patients who are symptomatic may require repair earlier, even in infancy. Some surgeons advocate routine repair in infants and children especially in cases where prematurityrelated lung disease may accelerate damage to the pulmonary vascular bed, though this philosophy may not be widespread. In a review by Reddy and colleagues, 116 neonates weighing less than 2500 g who underwent repair of simple and complex cardiac defects with the use of cardiopulmonary bypass were found to have no intracerebral hemorrhages, no long-term neurologic sequelae, and a low operative mortality rate (10%). These results correlated with the length of cardiopulmonary bypass and the complexity of repair. These investigators also found an 80% actuarial survival at 1 year and, more importantly, that growth following complete repair was equivalent to weight-matched neonates free from cardiac defects. (See Schwartz 11th ed., p. 754.)

3. Which of the following is not acceptable treatment for aortic valve stenosis with a hypoplastic left ventricle? A. Balloon valvotomy B. Intubation and initiation of prostaglandin C. Surgical valvotomy D. Norwood procedure

Answer: A The first decision that must be made in the neonate with critical left ventricular outflow tract (LVOT) obstruction is whether the patient is a candidate for biventricular or univentricular repair. Central to this decision is assessment of the degree of hypoplasia of the LV and other left-sided structures. Alsoufi and colleagues have described a rational approach to the neonate with critical LVOT obstruction. The options vary depending on whether the infant follows a single or a biventricular pathway. The options for a single ventricle include the Norwood operation, a hybrid strategy (initial ductal stent and bilateral pulmonary artery bands followed by later completion of the Norwood operation) or heart transplantation. The options for a biventricular heart include balloon valvuloplasty, surgical valvotomy, neonatal Ross operation, or a Yasui operation. Often valvotomy is accompanied by LV rehabilitation techniques, including endocardial fibroelastosis (EFE) resection and mitral valve interventions. Fetal aortic valvotomy, which is now offered at specialized centers, is another promising strategy to decompress the LV in fetal life and potentially allow growth of the left-sided structures sufficient to permit a biventricular circulation. Regardless of whether the baby is triaged to a single or biventricular strategy, any infant with severe aortic stenosis (AS) requires urgent intervention. Preoperative stabilization, however, has dramatically altered the clinical algorithm and outcomes for this patient population. The preoperative strategy begins with endotracheal intubation and inotropic support. Prostaglandin infusion is initiated to maintain ductal patency, and confirmatory studies are performed prior to operative intervention. Therapy is generally indicated in the presence of a transvalvular gradient of 50 mm Hg with associated symptoms including syncope, congestive heart failure (CHF), or angina, or if a gradient of 50 to 75 mm Hg exists with concomitant electrocardiography (ECG) evidence of LV strain or ischemia. In the critically ill neonate, a gradient across the aortic valve may not be present because of poor LV function. However, the decision regarding treatment options must be based on a complete understanding

CHAPTER 20

2. The most common age to close asymptomatic atrial septal defects (ASDs) is: A. In the immediate newborn period. B. After the child reaches 10 kg in weight. C. Age 4–5 years. D. During puberty.

Congenital Heart Disease

Brunicardi_Ch20_p157-170.indd 158

06/07/22 9:41 AM

159

Answer: B Coarctation of the aorta (COA) is defined as a luminal narrowing in the aorta that causes an obstruction to blood flow. This narrowing is most commonly located distal to the left subclavian artery. The embryologic origin of COA is a subject of some controversy. One theory holds that the obstructing shelf, which is largely composed of tissue found within the ductus, forms as the ductus involutes. The other theory holds that a diminished aortic isthmus develops secondary to decreased aortic flow in infants with enhanced ductal circulation. (See Schwartz 11th ed., pp. 761– 762.)

5. Which of the following is a TRUE surgical emergency in a newborn? A. Tetralogy of Fallot B. Truncus arteriosus C. Total anomalous pulmonary venous connection D. Coarctation of the aorta

Answer: C Total anomalous pulmonary venous connection (TAPVC) occurs in 1% to 2% of all cardiac malformations and is characterized by abnormal drainage of the pulmonary veins into the right heart, whether through connections into the right atrium or into its tributaries. Accordingly, the only mechanism by which oxygenated blood can return to the left heart is through an ASD, which is almost uniformly present with TAPVC. Unique to this lesion is the absence of a definitive form of palliation. Thus, TAPVC with concomitant obstruction (Fig. 20-2) represents one of the only true surgical emergencies across the entire spectrum of congenital heart surgery. (See Schwartz 11th ed., pp. 765– 768.)

Congenital Heart Disease

4. The most common location for a coarctation of the aorta is: A. Aortic arch. B. Distal to the left subclavian artery. C. At the diaphragm. D. At the level of the renal arteries.

CHAPTER 20

of associated defects. For example, in the presence of a hypoplastic LV (left ventricular end-diastolic volume < 20 mL/m2) or a markedly abnormal mitral valve, isolated aortic valvotomy should not be performed because studies have demonstrated high mortality in this population following isolated valvotomy. Patients who have an LV capable of providing systemic output are candidates for intervention to relieve AS, generally through balloon valvotomy. (See Schwartz 11th ed., p. 757.)

FIG. 20-2.  Infracardiac type of TAPVR. Note the stenosis (`*’) of the descending vertical vein as it drains into the portal system.

Brunicardi_Ch20_p157-170.indd 159

06/07/22 9:41 AM

160 6. The Bidirectional Glenn procedure is used to correct: A. Tricuspid atresia. B. Patent ductus arteriosus. C. Transposition of the great arteries. D. Total anomalous pulmonary venous connection.

CHAPTER 20 Congenital Heart Disease

Answer: A Recognizing the inadequacies of the initial repairs, Glenn described the first successful cavopulmonary anastomosis, an end-to-side right pulmonary artery-to-superior vena cava shunt in 1958, and later modified this to allow flow to both pulmonary arteries. This end-to-side right pulmonary artery-to-superior vena cava anastomosis was known as the bidirectional Glenn, and it is the first stage to final Fontan repair in widespread use today (Fig. 20-3). The Fontan repair was a major advancement in the treatment of congenital heart defect (CHD), as it essentially bypassed the right heart and allowed separation of the pulmonary and systemic circulations. It was first performed by Fontan in 1971 and consisted of a classic Glenn anastomosis, ASD closure, and direct connection of the right atrium to the proximal end of the left pulmonary artery using an aortic homograft. The main pulmonary artery was ligated, and a homograft valve was inserted into the orifice of the inferior vena cava. (See Schwartz 11th ed., pp. 770– 773.)

FIG. 20-3.  Angiogram showing a widely patent Glenn. The SVC (`*’) is seen draining into the central pulmonary artery.

7. Hypoplastic left heart syndrome (HLHS) is surgically treated with: A. Bilateral pulmonary artery banding and stent placement in the patent ductus arteriosus. B. Norwood procedure with a Blalock-Taussig (B-T) shunt. C. Norwood procedure with a right ventricle to pulmonary artery conduit (Sano shunt). D. All of the above.

Brunicardi_Ch20_p157-170.indd 160

Answer: D In 1983, Norwood and colleagues described a two-stage palliative surgical procedure for relief of HLHS that was later modified to the currently used three-stage method of palliation. Stage 1 palliation, also known as the modified Norwood procedure, bypasses the LV by creating a single outflow vessel, the neoaorta, which arises from the RV. The current technique of arch reconstruction involves completion of a connection between the pulmonary root, the native ascending aorta, and a piece of pulmonary homograft used to augment the diminutive native aorta. There are several modifications of this anastomosis, most notably the Damus-Kaye-Stansel (DKS) anastomosis, which involves dividing both the aorta and the pulmonary artery at the sinotubular junction. The proximal aorta is anastomosed to the proximal pulmonary artery, creating a “double-barreled” outlet from the heart. This outlet is anastomosed to the distal aorta, which can be augmented with homograft material if there is an associated coarctation. At the completion of arch reconstruction, a 3.5- or 4-mm shunt is placed from the

06/07/22 9:41 AM

161 innominate artery to the right pulmonary artery. The interatrial septum is then widely excised, thereby creating a large interatrial communication and preventing pulmonary venous hypertension. (See Schwartz 11th ed., pp. 773–775.)

9. Which of the following is NOT one of the components of the tetralogy of Fallot (TOF)? A. Atrial septal defect B. Ventricular septal defect (VSD) C. Right ventricular hypertrophy D. Right ventricular outflow obstruction

Answer: A The original description of tetralogy of Fallot (TOF) by Ettienne Louis Fallot, as the name implies, included four abnormalities: a large perimembranous VSD adjacent to the tricuspid valve; an overriding aorta; a variable degree of right ventricular outflow tract (RVOT) obstruction, which might include hypoplasia and dysplasia of the pulmonary valve as well as obstruction at the subvalvar and pulmonary artery level; and right ventricular hypertrophy. More recently, the Van Praagh et al pointed out that TOF could be more correctly termed monology of Fallot, since the four components are explained by the malposition of the infundibular septum. When the infundibular septum is displaced anteriorly and leftward, the RVOT is narrowed and its anterior displacement results in failure of fusion of the ventricular septum between the arms of the trabeculo-septo-marginalis. (See Schwartz 11th ed., p. 784.)

10. What is the best predictor of spontaneous closure of a ventricular septal defect (VSD)? A. Size B. Age at diagnosis C. Gestational age D. Lack of electrocardiogram changes

Answer: B VSDs may close or narrow spontaneously, and the probability of closure is inversely related to the age at which the defect is observed. Thus, infants at 1 month of age have an 80% incidence of spontaneous closure, whereas a child at 12 months of age has only a 25% chance of closure. This has an important impact on operative decision-making because a small or moderate-size VSD may be observed for a period of time in the absence of symptoms. Large defects and those in severely symptomatic neonates should be repaired during infancy to relieve symptoms and because irreversible changes in pulmonary vascular resistance may develop during the first year of life. (See Schwartz 11th ed., pp. 786–787.)

Brunicardi_Ch20_p157-170.indd 161

Congenital Heart Disease

Answer: A The most important consideration is the timing of surgical repair because arterial switch should be performed within 2 weeks after birth, before the left ventricle (LV) loses its ability to pump against systemic afterload. In patients presenting later than 2 weeks, the LV can be retrained with preliminary pulmonary artery banding and aortopulmonary shunt followed by definitive repair. Alternatively, the unprepared LV can be supported following arterial switch with a mechanical assist device for a few days while it recovers ability to manage systemic pressures. Echocardiography can be used to assess left ventricular performance and guide operative planning in these circumstances. (See Schwartz 11th ed., pp. 780–781.)

CHAPTER 20

8. The arterial switch operation for transposition of the great vessels is best performed: A. Within 2 weeks of birth. B. At 1 year of age. C. At 10 kg of weight. D. In adolescence.

06/07/22 9:41 AM

162 Answer: C The size of the VSD determines the initial pathophysiology of the disease. Large VSDs are classified as nonrestrictive and are at least equal in diameter to the aortic annulus. These defects allow free flow of blood from the left ventricle (LV) to the right ventricle (RV), elevating right ventricular pressures to the same level as systemic pressure. Consequently, the pulmonary-to-systemic flow ratio (Qp to Qs) is inversely dependent on the ratio of pulmonary vascular resistance to systemic vascular resistance. Nonrestrictive VSDs produce a large increase in pulmonary blood flow, and the afflicted infant will present with symptoms of congestive heart failure. However, if untreated, these defects will cause pulmonary hypertension with a corresponding increase in pulmonary vascular resistance. This will lead to a reversal of flow (a rightto-left shunt), which is known as Eisenmenger syndrome. Small restrictive VSDs offer significant resistance to the passage of blood across the defect, and therefore right ventricular pressure is either normal or only minimally elevated and the ratio of Qp to Qs rarely exceeds 1.5. These defects are generally asymptomatic because there are few physiologic consequences. However, there is a long-term risk of endocarditis because endocardial damage from the jet of blood through the defect may serve as a possible nidus for colonization. (See Schwartz 11th ed., pp. 786–787.)

12. Beyond early childhood, high pulmonary blood flow is most apt to produce: A. Cyanosis on exercise. B. Diminished exercise tolerance. C. Periodic episodes of hemoptysis. D. Right ventricular hypertrophy.

Answer: B High pulmonary blood flow beyond infancy may produce surprisingly little disability for a period of time, and the diminished exercise tolerance may be subtle. Cyanosis, hemoptysis, and pneumonia are not anticipated. With the volume overloading in the right ventricle, ventricular dilatation is more common than ventricular hypertrophy. (See Schwartz 11th ed., p. 751.)

13. During left thoracotomy for repair of patent ductus arteriosus the blood pressure is 70/22. Immediately after placement of a clip across the duct the blood pressure is: A. 70/22. B. 70/40. C. 90/22. D. 90/40.

Answer: B The hemodynamic consequences of an unrestrictive ductal shunt are left ventricular volume overload with increased left atrial and pulmonary artery pressures and right ventricular strain from the augmented afterload. These changes result in increased sympathetic discharge, tachycardia, tachypnea, and ventricular hypertrophy. The diastolic shunt results in lower aortic diastolic pressure and increases the potential for myocardial ischemia and underperfusion of other systemic organs, while the increased pulmonary flow leads to increased work of breathing and decreased gas exchange. Unrestrictive ductal flow may lead to pulmonary hypertension within the first year of life (See Schwartz 11th ed., pp. 759–760.)

CHAPTER 20

11. Flow across a ventricular septal defect (VSD) is dependent upon: A. Size of defect. B. Left and right ventricular pressure and size of the defect. C. Pulmonary and systemic vascular resistance and defect size. D. Pulmonary and systemic vascular resistance.

Congenital Heart Disease Brunicardi_Ch20_p157-170.indd 162

06/07/22 9:41 AM

163

15. Which structure must be identified prior to ligation and/ or division of a patent ductus arteriosus? A. Recurrent laryngeal nerve B. Left superior pulmonary vein C. Left bronchial artery D. Phrenic nerve

Answer: A Surgical closure can be achieved via either open or videoassisted approaches. The open approach employs a musclesparing posterior lateral thoracotomy in the third or fourth intercostal space on the side of the aorta (generally the left). The lung is then retracted anteriorly. In the neonate, the patent ductus arteriosus (PDA) is singly ligated with a surgical clip or permanent suture. Care must be taken to avoid the recurrent laryngeal nerve, which courses around the PDA. The PDA can also be ligated via a median sternotomy; however, this approach is generally reserved for patients who have additional cardiac or great vessel lesions requiring repair. Occasionally, a short, broad ductus, in which the dimension of its width approaches that of its length, will be encountered. In this case, division between vascular clamps with oversewing of both ends is advisable. In extreme cases, the use of cardiopulmonary bypass (CPB) to decompress the large ductus during ligation is an option. (See Schwartz 11th ed., pp. 759–760.)

16. Which of the following treatment paradigms most aptly describes the routine management of aortic coarctation in a 4-month-old child? A. Balloon dilation followed by stent placement for recoarctation B. Endovascular stent deployment with serial balloon dilations C. Observation D. Surgical repair followed by catheter based interventions for recoarctation

Answer: D Although operative repair is still the gold standard, treatment of coarctation of the aorta (COA) by catheter-based intervention has become more widespread for older children and adults. Both balloon dilatation and primary stent implantation have been used successfully. The most extensive study of the results of balloon angioplasty reported on 970 procedures: 422 native and 548 recurrent COAs. Mean gradient reduction was 74% ± 24% for native and 70% ± 31% for recurrent COA. This demonstrated that catheter-based therapy could

Brunicardi_Ch20_p157-170.indd 163

Congenital Heart Disease

Answer: A Subvalvular aortic stenosis (AS) occurs beneath the aortic valve and may be classified as discrete or tunnel-like (diffuse). A thin, fibromuscular diaphragm immediately proximal to the aortic valve characterizes discrete subaortic stenosis. This diaphragm typically extends for 180° or more in a crescentic or circular fashion, often attaching to the mitral valve as well as the interventricular septum. The aortic valve itself is usually normal in this condition, although the turbulence imparted by the subvalvular stenosis may affect leaflet morphology and valve competence. Diffuse subvalvular AS results in a long, tunnel-like obstruction that may extend to the left ventricular apex. In some individuals, there may be difficulty in distinguishing between hypertrophic cardiomyopathy and diffuse subaortic stenosis. Operation for subvalvular AS is indicated with a gradient exceeding 30 mm Hg, in the presence of aortic valve insufficiency, or when symptoms indicating left ventricular outflow tract (LVOT) obstruction are present. Given that repair of isolated discrete subaortic stenosis can be done with low rates of morbidity and mortality, some surgeons advocate repair in all cases of discrete AS to avoid progression of the stenosis and the development of aortic insufficiency, although more recent data demonstrate that subaortic resection should be delayed until the LV gradient exceeds 30 mm Hg because most children with an initial LV gradient less than 30 mm Hg have quiescent disease. Diffuse AS is a more complex lesion and often requires aortoventriculoplasty. Results are generally excellent, with operative mortality less than 5%. (See Schwartz 11th ed., pp. 755–758.)

CHAPTER 20

14. A 5-day-old man undergoes echocardiography during preoperative work-up of a tracheoesophageal fistula. The patient is found to have subvalvular aortic stenosis with a left ventricular (LV) gradient of 20 mm Hg but is otherwise asymptomatic and without aortic valve insufficiency. What is the next best step in management of his congenital heart defect (CHD)? A. Balloon valvotomy B. Observation with follow-up C. Aortic 1-patch repair (Doty procedure) D. Aortoventriculoplasty

06/07/22 9:41 AM

164

CHAPTER 20

produce equally effective results both in recurrent and in primary COA, a finding with far-reaching implications in the new paradigm of multidisciplinary treatment algorithms for congenital heart defect (CHD). In the valvuloplasty and angioplasty of congenital anomalies (VACA) report, higher preangioplasty gradient, earlier procedure date, older patient age, and the presence of recurrent COA were independent risk factors for suboptimal procedural outcome. In summary, children younger than age 6 months with native COA should be treated with surgical repair, while those requiring intervention at later ages may be ideal candidates for balloon dilatation or primary stent implantation. Additionally, catheter-based therapy should be employed for those cases of restenosis following either surgical or primary endovascular management. (See Schwartz 11th ed., pp. 761–763.)

Congenital Heart Disease

17. Which type of aortic coarctation repair is most associated with aneurysm formation at the repair site? A. Resection with end-to-end anastomosis B. Resection with extended end-to-end anastomosis C. Endovascular stenting D. Dacron patch aortoplasty

Answer: D The most common complications after coarctation of the aorta (COA) repair are late restenosis (Fig. 20-4) and aneurysm formation at the repair site. Aneurysm formation is particularly common after patch aortoplasty when using Dacron material. In a large series of 891 patients, aneurysms occurred in 5.4% of the total, with 89% occurring in the group who received Dacron-patch aortoplasty and only 8% occurring in those who received resection with primary end-to-end anastomosis. A further complication, although uncommon, is lower-body paralysis resulting from ischemic spinal cord injury during the repair. This dreaded outcome complicates 0.5% of all surgical repairs, but its incidence can be lessened with the use of some form of distal perfusion, preferably left heart bypass with the use of femoral arterial or distal thoracic aorta for arterial inflow and the femoral vein or left atrium for venous return. These techniques are generally reserved for older patients with complex coarctations that may need prolonged aortic cross clamp times for repair, often in the setting of large collateral vessels and/or previous surgery. (See Schwartz 11th ed., pp. 762–763.)

FIG. 20-4.  Reformatted images obtained from a CT angiogram after recurrent coarctation repaired by an extra anatomic bypass (`*’ points to the bypass graft).

Brunicardi_Ch20_p157-170.indd 164

06/07/22 9:41 AM

165

19. Which type of anomalous venous connection (according to the Darling classification) is most commonly associated with pulmonary venous obstruction? A. Supracardiac connection B. Cardiac connection C. Infracardiac connection D. Connection at multiple levels

Answer: C Darling and colleagues classified total anomalous pulmonary venous connection (TAPVC) according to the site or level of connection of the pulmonary veins to the systemic venous system: type I (45%), anomalous connection at the supracardiac level; type II (25%), anomalous connection at the cardiac level; type III (25%), anomalous connection at the infracardiac level; and type IV (5%), anomalous connection at multiple levels. Within each category, further subdivisions can be implemented, depending on whether pulmonary venous obstruction exists. Obstruction to pulmonary venous drainage is a powerful predictor of adverse natural outcome and occurs most frequently with the infracardiac type, especially when the pattern of infracardiac connection prevents the ductus venosus from bypassing the liver. (See Schwartz 11th ed., pp. 765–767.)

20. Which of the following factors confers the highest postoperative mortality after total anomalous pulmonary venous connection (TAPVC) repair? A. Postoperative atrial arrythmias B. Patient age at operation C. Pulmonary venous sclerosis D. Postoperative left ventricular (LV) dysfunction

Answer: C The most significant postoperative complication of TAPVC repair is pulmonary venous obstruction (Fig. 20-5), which occurs 9% to 11% of the time, regardless of the surgical technique employed. Mortality varies between 30% and 45%, and alternative catheter interventions do not offer definitive solutions. Recurrent pulmonary venous obstruction can be localized at the site of the pulmonary venous anastomosis (extrinsic), which usually can be cured with patch enlargement or balloon dilatation, or it may be secondary to endocardial thickening of the pulmonary venous ostia frequently resulting in diffuse pulmonary venous sclerosis (intrinsic), which carries a 66% mortality rate because few good solutions exist. More commonly, post repair left ventricular dysfunction can occur as the noncompliant LV suddenly is required to handle an increased volume load from redirected pulmonary venous return. This can manifest as an increase in pulmonary artery pressure but is distinguishable from primary

Brunicardi_Ch20_p157-170.indd 165

Congenital Heart Disease

Answer: D Truncus arteriosus was first managed with pulmonary artery banding as described by Armer and colleagues in 1961. However, this technique led to only marginal improvements in 1-year survival rates because ventricular failure inevitably occurred. In 1967, however, complete repair was accomplished by McGoon and his associates based on the experimental work of Rastelli, who introduced the idea that an extracardiac valved conduit could be used to restore ventricular-topulmonary artery continuity. Over the next 20 years, improved survival rates led to uniform adoption of complete repair even in the youngest and smallest infants. Surgical correction entails the use of cardiopulmonary bypass (CPB). Repair is completed by separation of the pulmonary arteries from the aorta, closure of the aortic defect (occasionally with a patch) to minimize coronary flow complications, placement of a valved cryopreserved allograft or jugular venous valved conduit (Contegra) to reconstruct the RVOT, and VSD closure. Important branch pulmonary arterial stenosis should be repaired at the time of complete repair and can usually be accomplished with longitudinal allograft patch arterioplasty. Severe truncal valve insufficiency occasionally requires truncal valve repair or even replacement, which can be accomplished with a cryopreserved allograft. (See Schwartz 11th ed., p. 764.)

CHAPTER 20

18. In a patient with truncus arteriosus, which of the following choices best describes definitive repair? A. Pulmonary artery banding B. Division of the main pulmonary artery (PA), aortic arch reconstruction with PA homograft, and creation of an innominate artery to right pulmonary artery shunt C. Mobilization of the coronary arteries, excision and transposition of the pulmonary valve into aortic position, reimplantation of the coronary arteries, and right ventricular outflow tract (RVOT) reconstruction D. Mobilization of the pulmonary arteries, patch repair of the aorta and ventricular septal defect (VSD), and extracardiac reconstruction of the RVOT

06/07/22 9:41 AM

166

CHAPTER 20 Congenital Heart Disease

FIG. 20-5.  Angiogram showing the discrete stenosis (`*’) of the right-sided pulmonary veins after conventional repair for supracardiac-type TAPVC.

pulmonary hypertension (another possible postoperative complication following repair of TAPVC) from the elevated left atrial pressure and LV dysfunction along with echocardiographic evidence of poor LV contractility. In pulmonary hypertension, the left atrial pressure may be low, the LV may appear “underfilled” (by echocardiography), and the RV may appear dilated. In either case, postoperative support for a few days with extracorporeal membrane oxygenation may be lifesaving, and TAPVC should be repaired in centers that have this capacity. (See Schwartz 11th ed., pp. 765–767.) 21. An 8-month-old female infant is admitted for her third respiratory infection. She is noted by her parents to otherwise be exhibiting age-appropriate behavior, normal stooling, and normal feeding. A chest XRAY is performed which demonstrates unilateral, left lung hyperinflation. Ultimately the patient undergoes a computed tomography angiography (CTA) and echocardiography confirming the presence of a pulmonary artery sling. Which of the following is the next best step after confirming the diagnosis? A. Bronchoscopy B. Cardiac catheterization C. Intubation and mechanical ventilation D. Barium swallow

Brunicardi_Ch20_p157-170.indd 166

Answer: A The symptoms associated with vascular rings include respiratory distress, barking cough, stridor, apnea, dysphagia, and recurrent respiratory tract infections. The diagnosis often requires a high index of suspicion. Minor respiratory tract infections may precipitate serious respiratory distress. The work-up includes chest X-rays, echocardiography, bronchoscopy, CT scan (Fig. 20-6), magnetic resonance imaging (MRI) (Fig. 20-7), and, rarely, cardiac catheterization. Chest X-rays show the relationship of the aortic arch to the trachea. Tracheal compression can be better evaluated using lateral films. Unilateral hyperinflation of the lung is sometimes seen and is often associated with a pulmonary artery sling (Fig. 20-8). Pulmonary artery (PA) slings (Fig. 20-9) are often associated with complete tracheal rings necessitating a bronchoscopy when this diagnosis is made (Fig. 20-10). Patients with dysphagia require a barium esophagogram as a part of their work-up (Fig. 20-11). (See Schwartz 11th ed., p. 769.)

06/07/22 9:41 AM

167

CHAPTER 20

FIG. 20-7.  MRI showing a double aortic arch.

FIG. 20-8.  Unilateral hyperinflation of the left lung associated with a rare vascular ring: left ascending aorta and right-sided descending aorta.

FIG. 20-9.  CT angiogram showing a PA sling. Note the LPA wrapping around behind the trachea.

Congenital Heart Disease

FIG. 20-6.  CT angiogram showing the four artery sign classic of double aortic arch.

FIG. 20-10.  Rigid bronchoscopy showing complete tracheal rings in a the patient with pulmonary artery sling.

Brunicardi_Ch20_p157-170.indd 167

06/07/22 9:41 AM

168

CHAPTER 20 Congenital Heart Disease

FIG. 20-11.  Barium esophagogram showing posterior indentation of the esophagus caused by a vascular ring (right aortic arch, aberrant left subclavian artery, and left ligamentum).

22. In the setting of tricuspid atresia without a concurrent ventricular septal defect (VSD), which of the following interventions is likely to stabilize an infant who presents with worsening cyanosis? A. Pulmonary artery banding B. Ligation of a patent ductus arteriosus C. Closure of atrial septal defect D. Prostaglandin E (PGE) administration

Brunicardi_Ch20_p157-170.indd 168

Answer: D The main pathophysiology in tricuspid atresia is that of a univentricular heart of left ventricular morphology. That is, the left ventricle (LV) must receive systemic blood via the interatrial communication and then distribute it to both the pulmonary circulation and the systemic circulation. Unless there is a ventricular septal defect (VSD) (as is found in some cases), pulmonary flow is dependent on the presence of a patent ductus arteriosus (PDA). As the ductus begins to close shortly after birth, infants become intensely cyanotic. Reestablishing ductal patency (with PGE1) restores pulmonary blood flow and stabilizes patients for surgical intervention. Pulmonary hypertension is unusual in tricuspid atresia. However, occasional patients have a large VSD between the LV and the infundibular portion of the right ventricle (RV) (just below the pulmonary valve). If there is no obstruction at the level of this VSD or at the valve, these infants may actually present with heart failure from excessive pulmonary blood flow. Regardless of whether these infants are “ductal-dependent” for pulmonary blood flow or have pulmonary blood flow provided across a VSD, they will be cyanotic since the obligatory right-to-left shunt at the atrial level will provide complete mixing of systemic and pulmonary venous return so that the LV ejects a hypoxemic mixture into the aorta. (See Schwartz 11th ed., pp. 770–772.)

06/07/22 9:41 AM

169

24. Wolff-Parkinson-White (WPW) is a conduction abnormality commonly associated with which form of congenital heart defect (CHD)? A. Tricuspid Atresia B. Hypoplastic left heart syndrome C. Ebstein Anomaly D. Aortic stenosis

Answer: C A WPW syndrome (Fig. 20-12) type of accessory pathway with associated preexcitation is present in 15% of patients. The surgical approach in widespread use today for patients surviving infancy was described by Danielson and colleagues in 1992. This procedure entails excision of redundant right atrial tissue and patch closure of any associated atrial septal defect (ASD), plication of the atrialized portion

Congenital Heart Disease

Answer: A The main pathophysiology in tricuspid atresia is that of a univentricular heart of left ventricular (LV) morphology. That is, the LV must receive systemic blood via the interatrial communication and then distribute it to both the pulmonary circulation and the systemic circulation. Unless there is a ventricular septal defect (VSD) (as is found in some cases), pulmonary flow is dependent on the presence of a PDA. As the ductus begins to close shortly after birth, infants become intensely cyanotic. Reestablishing ductal patency (with PGE1) restores pulmonary blood flow and stabilizes patients for surgical intervention. Pulmonary hypertension is unusual in tricuspid atresia. However, occasional patients have a large VSD between the LV and the infundibular portion of the RV (just below the pulmonary valve). If there is no obstruction at the level of this VSD or at the valve, these infants may actually present with heart failure from excessive pulmonary blood flow. Regardless of whether these infants are “ductal-dependent” for pulmonary blood flow or have pulmonary blood flow provided across a VSD, they will be cyanotic since the obligatory right-to-left shunt at the atrial level will provide complete mixing of systemic and pulmonary venous return so that the LV ejects a hypoxemic mixture into the aorta. (See Schwartz 11th ed., pp. 773–5.)

CHAPTER 20

23. Which of the following best describes the approach to surgical palliation of hypoplastic left heart syndrome? A. Separation of the systemic and pulmonary circulations along with progressive off-loading of the right ventricle (RV) B. Elimination of intracardiac shunting C. Relief of hypoxia through patent ductus arteriosus (PDA) closure D. Augmentation of left ventricular outflow obstruction to increase coronary blood flow

FIG. 20-12.  EKG of a newborn with Ebsteins anomaly and WPW syndrome. Note the pre-excitation (arrow).

Brunicardi_Ch20_p157-170.indd 169

06/07/22 9:41 AM

170

CHAPTER 20

of the ventricle with obliteration of the aneurysmal cavity, posterior tricuspid annuloplasty to narrow the tricuspid annulus, reconstruction of the tricuspid valve if the anterior leaflet is satisfactory, or replacement of the tricuspid valve if necessary. If the tricuspid valve is not amenable to reconstruction, valve replacement should be considered. Care must be taken when performing the posterior annuloplasty, or during the conduct of tricuspid valve replacement, to avoid the conduction system, because complete heart block can complicate this procedure. In addition, patients who demonstrated preoperative evidence of preexcitation should undergo electrophysiologic mapping and ablation. (See Schwartz 11th ed., pp. 777–779.)

Congenital Heart Disease

25. After accounting for size and location of the accompanying ventricular septal defect (VSD), which additional factor is a critical determinant of the clinical presentation and management of double-outlet right ventricle (DORV)? A. Left ventricular hypertrophy B. Posterior location of the aorta C. Anterior location of the aorta D. Presence of right ventricular outflow tract (RVOT) obstruction

Answer: D Patients with DORV typically present with one of the following three scenarios: (a) those with doubly committed or subaortic VSD present with congestive heart failure and a high propensity for pulmonary hypertension, much like infants with a large single VSD; (b) those with a subaortic VSD and pulmonary stenosis present with cyanosis and hypoxia, much like infants with tetralogy of Fallot; and (c) those with subpulmonic VSD present with cyanosis, much like those with D-TGA, because streaming directs desaturated systemic venous blood to the aorta and oxygenated blood to the pulmonary artery. Thus, the three critical factors influencing the clinical presentation and subsequent management of infants with DORV are the size and location of the VSD, the presence or absence of important RVOT obstruction, and the presence of other anomalies (especially associated hypoplasia of leftsided structures sometimes seen with subpulmonic VSD). (See Schwartz 11th ed., p. 783.)

26. Which anatomic consideration may complicate relief of severe right ventricular outflow tract (RVOT) obstruction in a patient with tetralogy of Fallot (TOF)? A. Large ratio of pulmonary artery diameter to aorta B. Posterior location of the aorta C. Small ventricular septal defect (VSD) D. Left anterior descending artery originating from the right coronary

Answer: D The morphology of TOF is markedly heterogeneous and includes an absent pulmonary valve, concomitant atrioventricular (AV) septal defects, and pulmonary atresia with major aortopulmonary collaterals. The present discussion will focus only on the so-called classic presentation of TOF without coexisting intracardiac defects. Anomalous coronary artery patterns, related to either origin or distribution, have been described in TOF. However, the most surgically important coronary anomaly occurs when the left anterior descending artery arises as a branch of the right coronary artery. This occurs in approximately 3% of cases of TOF and may preclude placement of a transannular patch, as the left anterior descending coronary artery crosses the RVOT at varying distances from the pulmonary valve annulus. (See Schwartz 11th ed., pp. 784–785.)

Brunicardi_Ch20_p157-170.indd 170

06/07/22 9:41 AM

CHAPTER

21

Acquired Heart Disease

1. New York Heart Association Classification (NYHA) is: A. A functional classification system that is strongly correlated with mortality. B. A functional classification system that does not correlate with mortality. C. Is an angina classification system similar to that of the Canadian Cardiovascular Society (CCS) angina classification. D. Only calculated after cardiac catheterization.

Answer: A The NYHA functional class is a widely used classification system in categorizing patients based on their functional status (Table 21-1). The NYHA classification has become one basis by which to compare patient populations in many studies. Although less commonly used, the CCS angina classification is also used to incorporate anginal symptoms into the functional assessment for prognostic value (Table 21-2). (See Schwartz 11th ed., p. 803.)

TABLE 21-1

New York Heart Association (NYHA) functional classification

Class

Description

I

Physical activity not limited by symptoms: fatigue, palpitations, or dyspnea.

II

Comfortable at rest. Slight limitation of physical activity. Fatigue, palpitations, or dyspnea with ordinary physical activity.

III

Comfortable at rest. Marked limitation of physical activity. Fatigue, palpitations, or dyspnea with less than ordinary physical activity.

IV

Inability to carry out any physical activity. Symptoms may be present at rest and increase with activity.

TABLE 21-2

Canadian Cardiovascular Society (CCS) angina classification

Class

Description

I

Ordinary physical activity (walking, climbing stairs) does not cause angina. Angina occurs with strenuous, rapid, or prolonged exertion during work or recreation.

II

Slight limitation of ordinary activity. Angina occurs with climbing stairs rapidly, walking uphill in the wind, under emotional stress, in the cold, or after meals. Walking more than 2 blocks or climbing one flight of stairs causes angina.

III

Marked limitation of ordinary physical activity (climbing a flight of stairs or walking 1 to 2 blocks at a normal pace).

IV

Inability to carry out any physical activity without discomfort. Angina may be present at rest.

171

Brunicardi_Ch21_p171-180.indd 171

04/07/22 2:10 PM

172

CHAPTER 21

Answer: B Patient characteristics can be classified by the status of the patient’s cardiac disease, comorbid conditions, and functional capacity. Patients are considered to be at major perioperative clinical risk if they have one or more of the following active cardiac conditions: acute coronary syndrome, decompensated heart failure, significant arrhythmias, or severe valvular heart disease. In these patients, intensive evaluation and treatment prior to surgery (unless emergent) are warranted, prior to proceeding with the noncardiac surgery. If the patient does not have any of the previously mentioned active cardiac conditions, the perioperative risk of MACE should be estimated. Both the operation performed and the patient’s risk factors are predictive of MACE, and the ACC/AHA guidelines recommend the use of either the American College of Surgeons’ NSQIP risk calculator or the Revised Cardiac Risk Index for the estimation of patientspecific risk. Patients at low (20% of the myocardium is necrosed, there is irreversible progression to ventricular dilation and failure. Once heart failure develops after postinfarction remodeling, the 1-year mortality reaches 32% despite current therapies. The classic aneurysm is a 4 to 6 mm thick scar, which bulges outward in paradoxical motion as the LV contracts during systole. More than 80% develop in the anteroseptal and apical portions of the left ventricle as a result of left anterior descending artery occlusion. The rest are inferior in location and the result of circumflex or right coronary occlusion. (See Schwartz 11th ed., p. 833.)

16. The most common arrhythmia worldwide is: A. Atrial flutter. B. Paroxysmal supraventricular tachycardia (PSVT). C. Wolff-Parkinson-White (WPW) syndrome. D. Atrial fibrillation.

Answer: D Atrial fibrillation (AF) remains the most common arrhythmia in the world with an overall incidence of 0.4% to 1% that increases to 8% in those older than 80 years. The most serious complication of AF is thromboembolism with resultant stroke, but serious morbidity and mortality may also result from hemodynamic compromise due to loss of atrial contraction, exacerbations of congestive heart failure from atrioventricular asynchrony and tachycardia-induced cardiomyopathy. (See Schwartz 11th ed., p. 838.)

Brunicardi_Ch21_p171-180.indd 176

04/07/22 2:10 PM

177

A

Answer: A The Cox-Maze IV uses a combination of bipolar radiofrequency (RF) ablation and cryoablation to effectively replace the majority of incisions that comprise the Cox-Maze III while significantly shortening cross-clamp time and reducing operative complexity. The Cox-Maze IV is performed on cardiopulmonary bypass through either a median sternotomy, often in combination with other cardiac surgery or a right minithoracotomy. In most cases, the right atrial lesion set performed on the beating heart, whereas the left atrial lesions are performed during cardioplegic arrest (Fig. 21-1). (See Schwartz 11th ed., p. 838.)

CHAPTER 21

17. Which of the following is FALSE regarding the CoxMaze IV procedure? A. It is indicated for ventricular tachycardia. B. It is indicated for treatment of Atrial Fibrillation (AF). C. It includes surgical management of left atrial appendage and radiofrequency or cryotherapy ablation. D. It is indicated in patients who are poor candidates for catheter-based ablation, including patients with large left atria and long-standing persistent AF.

B

Acquired Heart Disease FIG. 21-1.  The Cox-Maze IV Lesion Set. A. The left atrial lesion set is comprised of right and left pulmonary vein isolation, connecting lesions between the left and right superior and inferior pulmonary veins, a lesion from the left atrial appendage excision site to the pulmonary vein, and a lesion to the mitral valve annulus. B. The right atrial lesion set consists of lines of ablation along the superior and inferior vena cavae, the free wall of the right atrium, and down to the tricuspid valve annulus. (Reproduced with permission from Damiano RJ Jr, Schwartz FH, Bailey MS, et al: The Cox maze IV procedure: predictors of late recurrence. J Thorac Cardiovasc Surg. 2011;141(1):113–21.)

18. Which of the following is not TRUE in comparing CoxMaze IV and pulmonary vein isolation (PVI)? A. Both procedures require cardiopulmonary bypass. B. PVI has poorer results than Cox-Maze IV procedure. C. PVI has been shown to be superior to catheter-based ablation procedures. D. PVI is performed thoracoscopically.

Brunicardi_Ch21_p171-180.indd 177

Answer: A PVI is an attractive therapeutic option because it can be performed off of cardiopulmonary bypass (CPB) through small or thoracoscopic incisions. The results of PVI have been variable and highly dependent on patient selection since outcomes are consistently worse in patients with long-standing persistent atrial fibrillation (AF). In a study from Edgerton et al, only 56% of patients were free from AF at 6 months (35% off antiarrhythmic drugs), and with concomitant procedures, the success rate of PVI has been even lower. Several devices are available to close the left atrial appendage (LAA) at the time of PVI. These include staplers and epicardial clips that can be placed without the need for CPB. While surgical PVI has had poorer results than a Cox-Maze procedure, it has had superior results to catheter-based PVI. The Atrial Fibrillation Catheter Ablation Versus Surgical Ablation Treatment (FAST) Trial, which was a two-center, randomized clinical trial, compared catheter-based ablation to thoracoscopic PVI in patients with antiarrhythmic drugrefractory AF and either left atrial dilatation and hypertension or failed prior catheter-ablation. (See Schwartz 11th ed., p. 839.)

04/07/22 2:10 PM

178

CHAPTER 21

Answer: C Classic physical exam findings include jugular venous distention with Kussmaul’s sign, diminished cardiac apical impulses, peripheral edema, ascites, pulsatile liver, a pericardial knock, and, in advanced disease, signs of liver dysfunction, such as jaundice or cachexia. The “pericardial knock” is an early diastolic sound that reflects a sudden impediment to ventricular filling, similar to an S3 but of higher pitch. Several findings are characteristic on noninvasive and invasive testing. Central venous pressure (CVP) is often elevated 15 to 20 mm Hg or higher. Electrocardiogram (ECG) commonly demonstrates nonspecific low voltage QRS complexes and isolated repolarization abnormalities. Chest X-ray may demonstrate calcification of the pericardium, which is highly suggestive of constrictive pericarditis in patients with heart failure, but this is present in only 25% of cases. Cardiac CT or MRI (cMRI) typically demonstrate increased pericardial thickness (>4 mm) and calcification, dilation of the inferior vena cava, deformed ventricular contours, and flattening or leftward shift of the ventricular septum. Pericardial adhesions may also be seen on tagged cine MRI studies. As discussed, it is most important to distinguish pericardial constriction from restrictive cardiomyopathy, which is best done with either echocardiography or right heart catheterization. Findings favoring constriction on echocardiography include respiratory variation of ventricular septal motion and mitral inflow velocity, preserved or increased mitral annulus early diastolic filling velocity, and increased hepatic vein flow reversal with expiration. Cardiac catheterization will show increased atrial pressures, equalization of end-diastolic pressure, and early ventricular diastolic filling with a subsequent plateau, called the “square-root sign.” Additional findings upon catheterization that would favor constriction include respiratory variation in ventricular filling and increased ventricular interdependence, manifest as a discordant change in the total area of the left ventricular (LV) and right ventricular (RV) systolic pressure curve with respiration. (See Schwartz 11th ed., p. 840.)

20. Pericarditis is usually treated with: A. A short course of nonsteroidal anti-inflammatory agents. B. Use of steroids or IV antibiotics. C. Surgical exploration and drainage. D. Observation.

Answer: A The preferred treatment depends on the underlying cause of the pericarditis. The disease usually follows a self-limited and benign course and can be successfully treated with a short course of nonsteroidal anti-inflammatory agents (NSAIDs). Some patients may require judicious use of steroids or intravenous (IV) antibiotics. In cases of purulent pyogenic pericarditis, surgical exploration and drainage are occasionally necessary. Rarely, accumulation of fluid in the pericardium may lead to tamponade, requiring prompt evacuation of the pericardial space. While pericardiocentesis will typically suffice, surgical drainage may be required for thick, viscous, or clotted fluid or in patients with significant scarring from previous surgeries. More commonly, surgical intervention is required to manage recurrent disease. (See Schwartz 11th ed., p. 840.)

Acquired Heart Disease

19. A patient presents with a history of fatigue and dyspnea. He is found to have hepatomegaly, ascites, and an elevated jugular venous pulse. Heart sounds are normal, no murmurs are present, and the heart is of normal size. The pulse pressure is decreased by palpation. Electrocardiography is normal except for low voltage. The most likely diagnosis is: A. Right atrial myxoma. B. Tricuspid valve disease. C. Constrictive pericarditis. D. Primary pulmonary artery hypertension.

Brunicardi_Ch21_p171-180.indd 178

04/07/22 2:10 PM

179

22. Destination left ventricular assist device (LVAD) and total artificial heart is indicated. Only in patients with contraindications to heart transplantation they include: A. Irreversible renal failure. B. New York Heart Association Class III or Class IV. C. Ejection fraction < 25%. D. High oxygen requirements.

Answer: A Patients in need of ventricular assist devices (VADs) may have preexisting chronic heart failure, refractory ventricular arrhythmias, or acute heart failure following an MI, cardiopulmonary arrest, viral illness, pregnancy, or cardiotomy. Device therapy is intended to preserve end-organ perfusion and function and may be categorized as short- or long-term support for the left heart, the right heart, or both. In general, VADs may be used rarely for support while the heart recovers (bridge to recovery, BTR), while the patient waits for a heart transplant (bridge to transplant, BTT) or increasingly more commonly to treat a chronic heart failure patient who is not a transplant candidate (destination therapy, DT). Current eligibility criteria for mechanical support as destination therapy include (a) New York Heart Association Classification (NYHA) class III or IV heart failure despite guideline-directed medical therapy including cardiac resynchronization therapy if indicated; (b) peak oxygen consumption 65 years, irreversible pulmonary hypertension, chronic renal failure, insulin-dependent diabetes with endorgan damage, or other clinically significant comorbidities). Once a patient has an LVAD inserted as DT, close and intensive follow-up by a multidisciplinary heart failure team is required in order to optimize medical therapy, reduce device-related morbidity, and improve survival. (See Schwartz 11th ed., p. 837.)

23. The most common cardiac tumor is: A. Papillary fibroelastoma. B. Lymphangioma. C. Myxoma. D. Metastatic tumor.

Answer: C Cardiac myxomas are the most common cardiac tumor and are characterized by several distinguishing features. About 75% of the time, they arise from the interatrial septum near the fossa ovalis in the left atrium. Most others will develop in the right atrium, but, less commonly, they can arise from valvular surfaces and the walls of other cardiac chambers. Macroscopically, these tumors are pedunculated with a gelatinous consistency, and the surface may be smooth (65%), villous, or friable. Size varies greatly with these tumors and ranges from 1 to 15 cm in diameter. Internally, myxomas are heterogeneous and often contain hemorrhage, cysts, necrosis, or calcification. Histologically, these tumors contain cells that arise from a multipotent mesenchyme and are contained within a mucopolysaccharide stroma. (See Schwartz 11th ed., p. 842.)

Brunicardi_Ch21_p171-180.indd 179

Acquired Heart Disease

Answer: B IABP is the most commonly used device for mechanical circulatory support, and it may be easily deployed in the catheterization laboratory, in the operating room or at the bedside. The device is inserted percutaneously through the femoral artery into the thoracic aorta. It is synchronized so that the balloon is inflated during diastole and deflated during systole, resulting in augmentation of diastolic perfusion of the coronary arteries and decreased afterload. Typically, this improves cardiac index and decreases both preload and myocardial oxygen consumption. (See Schwartz 11th ed., p. 835.)

CHAPTER 21

21. Each of the following effects is anticipated after insertion of an intra-aortic balloonpump (IABP) EXCEPT: A. Preload decrease. B. Increased total myocardial oxygen consumption. C. Improvement in cardiac index. D. Coronary blood flow increase.

04/07/22 2:10 PM

180 24. Which of the following is NOT true of rhabdomyomas? A. They often require resection. B. They are often multicentric in the ventricles. C. They are the most common primary cardiac tumor in children. D. They often disappear spontaneously.

CHAPTER 21

Answer: A In children, rhabdomyomas are the most common primary cardiac tumor, whereas fibromas are the most commonly resected cardiac tumor. Rhabdomyomas are myocardial hamartomas that are often multicentric in the ventricles. About 50% of cases are associated with tuberous sclerosis, and while resection is occasionally necessary, most disappear spontaneously. Fibromas are congenital lesions that one-third of the time are found in children younger than 1-year old. These tumors, conversely, are ordinarily solitary lesions found in the inner interventricular septum, and they may present with heart failure, cyanosis, arrhythmias, syncopal episodes, chest pain, or sudden cardiac death. (See Schwartz 11th ed., p. 843.)

Acquired Heart Disease Brunicardi_Ch21_p171-180.indd 180

04/07/22 2:10 PM

CHAPTER

22

Thoracic Aneurysms and Aortic Dissection

1. Which of the following describes aortic aneurysms that are true aneurysms? A. Fusiform B. Saccular C. Pseudoaneurysm D. Mega-aorta

Answer: C Aortic aneurysms can be either “true” or “false.” True aneurysms can take two forms: fusiform and saccular. Fusiform aneurysms are more common and can be described as symmetrical dilatations of the aorta. Saccular aneurysms are localized outpouchings of the aorta. False aneurysms, also called pseudoaneurysms, are leaks in the aortic wall that are contained by the outer layer of the aorta and/or the periaortic tissue; they are caused by disruption of the aortic wall and lead blood to collect in pouches of fibrotic tissue. (See Schwartz 11th ed., p. 853.)

2. Which of the following is the most common cause of thoracic aortic aneurysms? A. Atherosclerosis B. Marfan syndrome C. Takayasu’s arteritis D. Nonspecific medial degeneration

Answer: D Nonspecific medial degeneration is the most common cause of thoracic aortic disease. Histologic findings of mild medial degeneration, including fragmentation of elastic fibers and loss of smooth muscle cells, are expected in the aging aorta. However, an advanced, accelerated form of medial degeneration leads to progressive weakening of the aortic wall, aneurysm formation, and eventual dissection, rupture, or both. The underlying causes of medial degenerative disease remain unknown. (See Schwarz 11th ed., p. 855.)

3. Marfan syndrome is an autosomal dominant genetic disorder characterized by connective tissue defect that leads to aneurysm formation. Which of the following statements is FALSE? A. Marfan syndrome is also associated with joint hypermobility and eye lens disorders. B. The disease causes fragmentation of elastic fibers, causing the aorta wall to be less elastic. C. Abnormal fibrillin and degeneration of aortic wall matric causes abnormal elasticity. D. Most Marfan syndrome patients have dilation of ascending aorta and dilation of the aortic annulus.

Answer :B Marfan syndrome is an autosomal dominant genetic disorder characterized by a specific connective tissue defect that leads to aneurysm formation. The phenotype of patients with Marfan syndrome typically includes a tall stature, high palate, joint hypermobility, eye lens disorders, mitral valve prolapse, and aortic aneurysms. Abnormal fibrillin causes degeneration of the aortic wall matrix by increasing the activity of transforming growth factor beta (TGF-β). Between 75% and 85% of patients with Marfan syndrome have dilatation of the ascending aorta and annuloaortic ectasia (dilatation of the aortic sinuses and annulus). Marfan syndrome also is frequently associated with aortic dissection, and aortic complications are the most common cause of death among patients with Marfan syndrome. (See Schwartz 11th ed., p. 855.)

181

Brunicardi_Ch22_p181-188.indd 181

06/07/22 9:46 AM

182 Answer: C Ehlers-Danlos syndrome includes a spectrum of inherited disorders of collagen synthesis. The subtypes represent differing defective steps of collagen production. Vascular-type Ehlers-Danlos syndrome is characterized by an autosomal dominant defect in type III collagen synthesis, which can have life-threatening cardiovascular manifestations. Spontaneous arterial rupture, usually involving the mesenteric vessels, is the most common cause of death in these patients. Thoracic aortic aneurysms and dissections are less commonly associated with Ehlers-Danlos syndrome, but when they do occur, they pose a particularly challenging surgical problem because of the reduced integrity of the aortic tissue. (See Schwartz 11th ed., p. 855.)

5. The most common cause of death in patients with type IV Ehlers-Danlos syndrome is: A. Myocardial infarction. B. Aortic dissection. C. Ruptured visceral artery. D. Pulmonary emboli.

Answer: C Ehlers-Danlos syndrome includes a spectrum of inherited connective tissue disorders of collagen synthesis. The subtypes represent differing defective steps of collagen production. Vascular-type Ehlers-Danlos syndrome is characterized by an autosomal dominant defect in type III collagen synthesis, which can have life-threatening cardiovascular manifestations. Spontaneous arterial rupture, usually involving the mesenteric vessels, is the most common cause of death in these patients. Thoracic aortic aneurysms and dissections are less commonly associated with Ehlers-Danlos syndrome, but when they do occur, they pose a particularly challenging surgical problem because of the reduced integrity of the aortic tissue. An Ehlers-Danlos variant of periventricular heterotopia associated with joint and skin hyperextensibility and aortic dilation has been described as being caused by mutations in the gene encoding filamin A (FLNA), an actin-binding protein that links the smooth muscle cell contractile unit to the cell surface. (See Schwartz 11th ed., p. 855.)

6. Bovine aortic arch: A. Is a normal variant where the innominate and left common carotid arteries have a common origin. B. Is associated with bicuspid aortic disease. C. Is associated with increased risk of aortic dissection. D. Does not appear to be associated with increased risk of aneurysm.

Answer: A Bovine aortic arch—a common origin of the innominate and left common carotid arteries—has been considered a normal anatomic variant. Studies from Yale University have identified a higher prevalence of bovine aortic arch in patients with thoracic aortic disease; an association was found between this anomaly and a generalized increase in aortic aneurysmal disease (without any predisposition to a particular aortic region). However, bovine aortic arch was not associated distinctly with bicuspid aortic valve or aortic dissection, but with a higher mean aortic growth rate: 0.29 cm per year in patients with bovine aortic arch, compared with 0.09 cm per year in controls. Therefore, bovine aortic arch may be better characterized as a precursor of aortic aneurysm than as a simple normal anatomic variant. Further studies are needed to delineate the underlying mechanism for this association. (See Schwartz 11th ed., p. 856.)

7. Mycotic aneurysms: A. Are associated exclusively with fungal infections of the aorta. B. Can occur via bacterial invasion with endocarditis, endothelial trauma, or infected laminar clot. C. Is an acute complication of syphilis. D. Are generally fusiform type.

Answer: B Primary infection of the aortic wall resulting in aneurysm formation is rare. Although these lesions are termed mycotic aneurysms, the responsible pathogens usually are bacteria rather than fungi. Bacterial invasion of the aortic wall may result from bacterial endocarditis, endothelial trauma caused by an aortic jet lesion, or extension from an infected laminar

CHAPTER 22

4. Vascular-type Ehlers-Danlos syndrome is characterized by a defect in: A. Elastin. B. Metalloproteinase. C. Type III collagen synthesis. D. Fibrillin.

Thoracic Aneurysms and Aortic Dissection

Brunicardi_Ch22_p181-188.indd 182

06/07/22 9:46 AM

183

Answer: A An analysis by Elefteriades of data from 1600 patients with thoracic aortic disease has helped quantify these wellrecognized risks. Average expansion rates were 0.07 cm per year in ascending aortic aneurysms and 0.19 cm per year in descending thoracic aortic aneurysms. As expected, aortic diameter was a strong predictor of rupture, dissection, and mortality. For thoracic aortic aneurysms > 6 cm in diameter, annual rates of catastrophic complications were 3.6% for rupture, 3.7% for dissection, and 10.8% for death. Critical “hinge-point” diameters, at which the incidence of expected complications significantly increased, were 6 cm for aneurysms of the ascending aorta and 7 cm for aneurysms of the descending thoracic aorta; the corresponding risks of rupture after reaching these diameters were 31% and 43%, respectively. (See Schwartz 11th ed., p. 857.)

9. The most common complication of extensive repair for distal aortic aneurysms is: A. Spinal cord ischemia. B. Renal failure. C. Pulmonary dysfunction. D. Left recurrent laryngeal nerve injury.

Answer: C Although spinal cord ischemia and renal failure receive the most attention, several other complications warrant consideration. The most common complication of extensive repairs is pulmonary dysfunction. With aneurysms adjacent to the left subclavian artery, the vagus and left recurrent laryngeal nerves are often adherent to the aortic wall and thus are susceptible to injury. (See Schwartz 11th ed., p. 871.)

10. Which of the following is NOT TRUE regarding anastomotic pseudoaneurysms? A. These can arise from deterioration of aortic tissue due to infection. B. These have increased in incidence with an influx of cardiovascular surgery. C. These commonly occur in patients with Marfan syndrome. D. These are associated with high incidence of morbidity and rupture.

Answer: B Anastomotic pseudoaneurysms can be caused by technical problems or by deterioration of the native aortic tissue, graft material, or suture. Commonly, they occur in patients with Marfan syndrome. Tissue deterioration usually is related to either progressive degenerative disease or infection. Improvements in sutures, graft materials, and surgical techniques have decreased the incidence of thoracic aortic pseudoaneurysms. Should thoracic aortic pseudoaneurysms occur, they typically require expeditious surgical or other intervention because they are associated with a high incidence of morbidity and rupture. (See Schwartz 11th ed., p. 857.)

Brunicardi_Ch22_p181-188.indd 183

Thoracic Aneurysms and Aortic Dissection

8. Aortic diameters at increased risk for rupture, dissection, and mortality include: A. Critical hinge-point, the diameter where the risk of rupture increases dramatically, diameters are 6 cm for ascending aorta and 7 cm for descending aorta. B. Fusiform aneurysm is more likely to rupture than saccular aneurysm. C. Disease etiology rather than diameter is more relevant than diameter. D. Average expansion rates are about 1 cm per year for both ascending and descending thoracic aorta aneurysms.

CHAPTER 22

clot within a preexisting aneurysm. The most common causative organisms are Staphylococcus aureus, Staphylococcus epidermidis, Salmonella, and Streptococcus. Unlike most other causes of thoracic aortic aneurysms, which generally produce fusiform aneurysms, infection often produces saccular aneurysms located in areas of aortic tissue destroyed by the infectious process. Although syphilis was once the most common cause of ascending aortic aneurysms, the advent of effective antibiotic therapy has made syphilitic aneurysms a rarity in developed nations. In other parts of the world, however, syphilitic aneurysms remain a major cause of morbidity and mortality. The spirochete Treponema pallidum causes an obliterative endarteritis of the vasa vasorum that results in medial ischemia and loss of the elastic and muscular elements of the aortic wall. The ascending aorta and arch are the most commonly involved areas. Because syphilitic aortitis often presents 10 to 30 years after the primary infection, the incidence of associated aneurysms may increase in the near future. (See Schwartz 11th ed., p. 856.)

06/07/22 9:46 AM

184

CHAPTER 22 Thoracic Aneurysms and Aortic Dissection

11. The most common presenting symptom in patients with an ascending thoracic aneurysm is: A. Anterior chest pain. B. Posterior chest pain. C. Aortic valve insufficiency. D. Sudden death.

Answer: A Initially, aneurysmal expansion and impingement on adjacent structures cause mild, chronic pain. The most common symptom in patients with ascending aortic aneurysms is anterior chest discomfort; the pain is frequently precordial in location but may radiate to the neck and jaw, mimicking angina. Aneurysms of the ascending aorta and transverse aortic arch can cause symptoms related to compression of the superior vena cava, the pulmonary artery, the airway, or the sternum. Rarely, these aneurysms erode into the superior vena cava or right atrium, causing acute high-output failure. (See Schwartz 11th ed., p. 857.)

12. The primary modes to study for thoracic aorta disease is/are: A. Invasive aortography. B. Cardiac catheterization. C. Cross-sectional imaging (computed tomography [CT] and magnetic resonance angiography [MRA]). D. Abdominal ultrasound.

Answer: C Although catheter-based contrast aortography was previously considered the gold standard for evaluating thoracic aortic disease, cross-sectional imaging (ie, CT and MRA) has largely replaced this modality. Technologic improvements have enabled CT and MRA to provide excellent aortic imaging while causing less morbidity than catheter-based studies do, so CT and MRA are now the primary modes for evaluating thoracic aortic disease. Today, the use of invasive aortography in patients with thoracic aortic disease is generally limited to those undergoing endovascular therapies or when other types of studies are contraindicated or have not provided satisfactory results. (See Schwartz 11th ed., p. 859.)

13. Endovascular repair of thoracic artery aneurysms is generally not recommended for: A. Descending thoracic aortic aneurysm and dissection. B. Blunt aortic injury. C. Penetrating aortic ulcers. D. Repair of aneurysms of proximal aorta.

Answer: D Experience with purely endovascular treatment of proximal aortic disease remains limited and only investigational. Endovascular therapy has become an accepted treatment for descending thoracic aortic aneurysm. Its role in treating proximal aortic disease and thoracoabdominal aortic aneurysm remains experimental; nonetheless, endoluminal stenting is approved by the US Food and Drug Administration for the treatment of isolated descending thoracic aortic aneurysm, and several different devices have been approved for the treatment of blunt aortic injury and penetrating aortic ulcer. (See Schwartz 11th ed., pp. 868, 872.)

14. Initial assessment and management of aortic dissection: A. Differs depending on the location and type of the dissection. B. Requires aggressive pharmacologic treatment. C. Requires repair within 24 hours. D. Is not altered by signs of organ dysfunction.

Answer: B Regardless of the location of the dissection, the initial treatment is the same for all patients with suspected or confirmed acute aortic dissection (see Fig. 22-1). Furthermore, because of the potential for rupture before the diagnosis is confirmed, aggressive pharmacologic management is started once there is clinical suspicion of dissection, and this treatment is continued during the diagnostic evaluation. The goals of pharmacologic treatment are to stabilize the dissection and prevent rupture. (See Schwartz 11th ed., p. 878.)

Brunicardi_Ch22_p181-188.indd 184

06/07/22 9:46 AM

185

CHAPTER 22

15. Mortality rates for operative repair of an aortic arch aneurysm have been significantly reduced intraoperatively by: A. Deep hypothermia to allow circulatory arrest. B. Innominate and left carotid artery cannulation to permit oxygenation of the brain. C. Right heart to left subclavian artery bypass to continue brain perfusion. D. Use of an intra-aortic balloon pump to maintain distal circulation.

16. Endoleaks: A. Type I and Type IV generally require early and aggressive intervention. B. Are uncommon. C. Can during the initial procedure or over time. D. Are categorized by leak site.

Brunicardi_Ch22_p181-188.indd 185

Answer: A Like the operations themselves, perfusion strategies used during proximal aortic surgery depend on the extent of the repair. Aneurysms that are isolated to the ascending segment can be replaced by using standard cardiopulmonary bypass and distal ascending aortic clamping. This provides constant perfusion of the brain and other vital organs during the repair. Aneurysms involving the transverse aortic arch, however, cannot be clamped during the repair, which necessitates the temporary withdrawal of cardiopulmonary bypass support; this is called circulatory arrest. To protect the brain and other vital organs during the circulatory arrest period, hypothermia must be initiated before pump flow is stopped. However, hypothermia is not without risk, and coagulopathy is associated with deep levels of hypothermia (70% stenosis in the celiac trunk. Endovascular treatment of mesenteric artery stenosis or short segment

Arterial Disease

9. The ideal characteristics of an aneurysm for endovascular abdominal aortic aneurysm repair include all of the following EXCEPT: A. Neck length more than 20 mm. B. Neck diameter >18 mm, and 7 mm. D. Aortic neck angle < 60 degrees.

Brunicardi_Ch23_p189-196.indd 192

08/07/22 11:41 AM

193

Answer: D Obstructive lesions of the renal artery can produce hypertension and it is reported in about 5% to 10% of all hypertensive patients in the United States. Approximately 80% of all renal artery occlusive lesions are caused by atherosclerosis, which typically involves a short segment of the renal artery ostia and represent spillover disease from a severely atheromatous aorta. The second most common cause of renal artery stenosis is fibromuscular dysplasia, which accounts for 20% of cases and is most frequently encountered in young, often multiparous women. Fibromuscular dysplasia of the renal artery presents a heterogeneous group of lesions that can produce histopathological changes in the intima, media or adventitia. The most common variety consists of medial fibroplasia, in which thickened fibromuscular ridges alternate with attenuated media producing the classic angiographic “string of beads” appearance. Indications for renal arterial revascularization include ≥70% stenosis of one or both renal arteries and at least one of the following clinical criteria: Inability to adequately control hypertension despite appropriate antihypertensive regime; chronic renal insufficiency related to bilateral renal artery occlusive disease or stenosis to a solitary functioning kidney; dialysis-dependent renal failure in a patient with renal artery stenosis but without another definite cause of end-stage renal disease and recurrent congestive heart failure or flash pulmonary edema not attributable to active coronary ischemia. (See Schwartz 11th ed., pp. 935–940.)

12. Which of the following is TRUE about atherosclerotic aortoiliac occlusive disease? A. Classified into three times with type I occurring in 5% to 10% to patient extending into the lower extremities below the knee. B. Type II aortoiliac disease represents diffuse aortoiliac disease above the inguinal ligament. C. Type III represents multisegment occlusive disease involving the aortoiliac area and extending to the supra renal aorta. D. Type I is more frequently found in men.

Answer: B The distal abdominal aorta and the iliac arteries are common sites affected by atherosclerosis. Based on the atherosclerotic disease pattern, aortoiliac occlusive disease can be classified into three types. Type I aortoiliac disease, which occurs in 5% to 10% of patient, is confined to the distal abdominal aorta and common iliac vessels. Due to the localized nature of this type of aortic obstruction and formation of collateral blood flow around the occluded segment, limb-threatening symptoms are rare in the absence of more distal disease. This type of aortoiliac occlusive disease occurs in a relatively young group

Brunicardi_Ch23_p189-196.indd 193

Arterial Disease

11. Which of the following is TRUE about renal artery disease? A. Renal artery occlusive disease affects 5% to 10% of all hypertensive patients in the United States. B. The most common cause of renal artery occlusive disease is atherosclerosis in 80% of patients followed by fibromuscular dysplasia. C. Renal artery revascularization is indicated in patients with recurrent congestive heart failure or flash pulmonary edema not attributable to active coronary ischemia in the presence of >70% stenosis of the renal artery. D. All of the above.

CHAPTER 23

occlusion by balloon dilatation or stent placement represent a less invasive therapeutic alternative to open surgical intervention particularly in patients whose medical comorbidities place them at a higher operative surgical risk. Endovascular therapy is also suited in patients with recurrent disease or anastomotic stenosis following previous open mesenteric revascularization. Prophylactic mesenteric revascularization is rarely performed in the asymptomatic patient undergoing an aortic procedure for other indications. However, the natural history of untreated chronic mesenteric ischemia may justify revascularization in some minimally symptomatic or asymptomatic patients if the operative risks are acceptable, since the first clinical presentation may be acute intestinal ischemia in as many as 50% of patients, with a mortality rate that ranges from 15% to 70%. Mesenteric angioplasty and stenting is particularly suited for this patient subgroup given its low morbidity and mortality. (See Schwartz 11th ed., pp. 928–933.)

08/07/22 11:41 AM

194

CHAPTER 23

of patients. Patients with type I disease pattern have a lower incidence of hypertension and diabetes with a significant frequency of abnormal blood lipid levels particularly type IV hyperlipoproteinemia. Type II aortoiliac disease represents a more diffuse atherosclerotic progression and involves predominantly the abdominal aorta with disease extension into the common iliac artery. This disease pattern affects approximately 25% of patients with aortoiliac occlusive disease. Type III aortoiliac occlusive disease, which affects approximately 65% patient with aortoiliac occlusive disease, is widespread disease that is seen above and below the inguinal ligament. Patients with multilevel disease are older, more commonly with a male to female ratio of 6:1 and much more likely to have diabetes, hypertension, and associated atherosclerotic disease involving cerebral, coronary, and basilar arteries. (See Schwartz 11th ed., pp. 941–942.)

Arterial Disease

13. In regard to obturator bypass, which of the following is TRUE? A. It is indicated in patients with groin hematoma and fever. B. The conduit of Dacron, polytetrafluoroethylene (PTFE), or autologous vein is tunneled through the posteromedial portion of the obturator membrane. C. The obturator artery and nerve pass in the anteromedial portion of the membrane. D. 5-year patency of 57% has been reported.

Answer: D An obturator bypass is used to reconstruct arterial anatomy in patients with a groin sepsis resulting from prior prostatic grafting, intra-arterial drug abuse, growing neoplasm, or damage from prior groin irradiation. This bypass can originate from the common iliac artery, external iliac artery, or uninvolved limb of an aortobifemoral bypass. The conduit is tunneled through the anteromedial portion of the obturator membrane to the distal superficial femoral artery or popliteal artery. The membrane must be divided sharply to avoid injury to adjacent structures. Care must be taken to identify the obturator artery and nerve that pass posterolaterally. There have been very good results in terms of patency and limb salvage for obturator bypass. Some authors have reported 57% 5-year patency and 77% 5-year limb salvage rates, whereas others have shown a higher rate of reinfection and low patency requiring reintervention. (See Schwartz 11th ed., p. 946.)

14. In patients with acute limb ischemia, the following is TRUE EXCEPT: A. Immediate anticoagulation is indicated. B. Hypercoagulable workup should be performed prior to heparin initiation in all patients. C. There is no clear superiority for thrombolysis over surgery in terms of a 30-day limb salvage on mortality. D. In the United States, surgery is used three- to five fold more frequently than thrombolysis.

Answer: B Acute lower extremity ischemia manifesting with the five Ps: Pain, pallor, paresthesia, paralysis, and pulselessness. The sixth P, “poikilothermia” or “perishing cold,” is added. In the absence of any significant contraindication, the patient with an ischemic lower extremity should be immediately anticoagulated. This will prevent propagation of the clot into the unaffected vascular beds. Intravenous fluid should be started and a Foley catheter inserted to monitor urine output. Baseline labs should be obtained but hypercoagulable workup should be performed only prior to initiation of heparin if there is a sufficient suspicion. There is no clear superiority for thrombolysis over surgery in terms of a 30-day limb salvage on mortality. Access to each treatment option is a major issue in the decision-making process, as time is often critical. National registry data from the United States revealed that surgery is used in three- to five fold more frequently than thrombolysis. (See Schwartz 11th ed., p. 954-955.)

Brunicardi_Ch23_p189-196.indd 194

08/07/22 11:41 AM

195

16. The following is TRUE about fibromuscular dysplasia EXCEPT: A. The characteristic beaded appearance of fibromuscular dysplasia (FMD) is due to areas of medial thinning alternating with areas of stenosis. B. The most commonly affected are the medium-sized arteries. C. The commonest involved arteries are the internal carotid arteries. D. FMD occurs most frequently in women.

Answer: C FMD is a vasculopathy of uncertain etiology that is characterized by segmental arterial involvement. Histologically, fibrous tissue proliferation, smooth muscle cell hyperplasia, and elastic fiber destruction alternate with mural thinning. The characteristic beaded appearance of fibromuscular dysplasia is due to areas of medial thinning alternating with areas of stenosis. The most commonly affected arteries are the mediumsized arteries, including the renal, internal carotid, vertebral, subclavian, mesenteric, and iliac arteries. The internal carotid artery is the second most common site of involvement after the renal arteries. FMD occurs most frequently in women (90%) and is recognized at approximately 55 years of age. Only 10% of patients with FMD will have complications attributable to the disease. Pathologically, FMD is heterogeneous group of four distinct types of lesions that are subgrouped based on the predominant site of involvement within the vessel wall. Of the four types (medial fibroplasia, intimal fibroplasia, medial hyperplasia, and perimedial dysplasia), medial fibroplasia is the most common pathological type, affecting the internal carotid artery and the renal artery and occurring in 85% of the reported cases. (See Schwartz 11th ed., pp. 972–973.)

Brunicardi_Ch23_p189-196.indd 195

Arterial Disease

Answer: C If a vein is unavailable, polytetrafluoroethylene (PTFE) or Dacron can be used for above-knee bypass or in some cases below the knee. The addition of rings to the PTFE did not confer benefit in a single prospective, randomized clinical trial. For infrageniculate prosthetic bypasses, use of a vein patch, cuff, or other venous anastomotic modifications can improve patency. Methods to improve prosthetic graft performance have consisted of altering the geometry at the distal anastomosis to get the benefit obtained with vein cuffs and covalently binding agents onto the luminal surface with anticoagulant, anti-inflammatory, and antiproliferative characteristics. A comparison of precuffed PTFE versus PTFE with a vein cuff showed a 1 year and 2 years primary patency rates were 52% and 49% in the precuffed group and 62% and 44% in the vein cuff group, respectively. Another approach for improving outcomes using prosthetic for bypass grafts involves binding anticoagulants to the conduit. The Gore propaten graft has heparin-bonded into the luminal surface of the PTFE graft using Carmeda bioactive surface technology. The heparin-binding does not alter the micro structure and handling characteristic of the PTFE. A prospective randomized trial suggested that heparin-bonded Dacron or PTFE was superior to plain PTFE for above-knee popliteal bypass. The 3-year primary patency rate for the heparin-bonded graft was 55% compared with 42% for the PTFE. But both of these patency rates are inferior to greater saphenous vein grafts. (See Schwartz 11th ed., p. 967-968.)

CHAPTER 23

15. In the absence of a suitable vein for lower extremity bypass, prosthetic grafts can be used. Which of the following can be used to improve patency of prosthetic grafts? A. Altering the geometry at the distal anastomosis by increasing the size of the arteriotomy B. Bonding agents into the outer surface of the graft C. Bonding anticoagulants to the conduit to improve patency of the graft D. Applying rings to the graft to improve patency

08/07/22 11:41 AM

196 17. Which of the following is TRUE about adventitial cystic disease of the popliteal artery? A. It occurs in about 5% usually in the popliteal artery. B. The disease affects men in a ratio approximately of 5:1. C. Patients present in their 40s and 50s. D. Computed tomography angiogram (CTA) of the lower extremities is the best diagnostic modality.

CHAPTER 23 Arterial Disease Brunicardi_Ch23_p189-196.indd 196

Answer: B The adventitial cystic disease of the popliteal artery was first described in 1954. It is a rare arterial condition occurring at an incidence of 0.1%, usually in the popliteal artery. This disease affects men in a ratio of approximately 5:1. The incidence is approximately 1 in 1200 cases of claudication or 1 and 1000 peripheral arteriograms. The disease may affect other vascular sites, such as the femoral, external iliac, radial, ulnar, and brachial arteries. The diagnosis should be considered in young patients with a mass in a nonaxial vessel in proximity to a related joint. The synovial-like mucin-filled cyst reside in the subadventitial layer of the vessel wall and have a similar microscopical appearance to a ganglion cyst. Despite the similarity and suggestion of a joint origin for these lesions, histochemical markers have failed to link the cystic lining to synovium. Patient presenting at a young age, with bilateral lower extremity claudication and minimal risk factors for atheroma formation should be evaluated for adventitial cystic disease as well as for popliteal artery entrapment syndrome and Buerger’s disease. Peripheral pulses may be present in the limb when extended, but then can disappear during knee-joint flexion. Noninvasive studies may suggest arterial stenosis with elevated velocities. Color-flow duplex scanning followed by T2-weighted magnetic resonance imaging (MRI) now appears to be the best diagnostic choice. Angiography will demonstrate a smooth, well-defined, crescent-shaped filling defect, the classic “scimitar” sign. There may be associated calcifications in the cyst wall and no other evidence of atherosclerotic occlusive disease. The recommended treatments are excision of the cyst with the cystic wall, enucleation, or simple aspiration when the artery is stenotic. Retention of the cystic lining leads to continued secretion of the cystic fluid and recurrent lesions. And 30% of patients who have an occluded artery, resection of the affected artery, followed by an interposition graft using autogenous saphenous vein, is recommended. (See Schwartz 11th ed., p. 973.)

08/07/22 11:41 AM

CHAPTER

24

Venous and Lymphatic Disease

1. All of the following regarding venous anatomy are TRUE EXCEPT: A. Veins are thin-walled, collapsible, and highly distensible to a diameter several times greater than that in the supine position. B. The small saphenous vein (SSV) is accompanied by the sural nerve. C. The radial, ulnar, brachial, anterior tibial, posterior tibial, peroneal, and popliteal veins usually exist as paired veins. D. The subclavian vein courses anterior to the anterior scalene muscle.

Answer: C Veins are thin-walled, highly distensible, and collapsible. Their structure specifically supports the primary functions of veins to transport blood toward the heart and serve as a reservoir to prevent intravascular volume overload. Lower extremity veins are divided into superficial, deep, and perforating veins. The superficial venous system lies above the uppermost fascial layer of the leg and thigh and consists of the great saphenous vein (GSV) and SSV and their tributaries. The GSV originates from the dorsal pedal venous arch and courses cephalad and medially, anterior to the medial malleolus, entering the common femoral vein approximately 4 cm inferior and lateral to the pubic tubercle. The saphenous nerve accompanies the GSV medially from the ankle to the level of the knee and supplies cutaneous sensation to the medial leg and ankle. The SSV originates laterally from the dorsal pedal venous arch and courses cephalad in the posterior calf. Most often, it penetrates the popliteal fossa, between the medial and lateral heads of the gastrocnemius muscle, to join the popliteal vein. The termination of the SSV may be quite variable, however, with a proximal extension of the SSV (the vein of Giacomini) connecting with the deep femoral vein or GSV. The sural nerve accompanies the SSV laterally along its course and supplies cutaneous sensation to the lateral malleolar region. The deep veins follow the course of major arteries in the extremities. In the lower leg, paired veins parallel the course of the anterior tibial, posterior tibial, and peroneal arteries, to join behind the knee forming the popliteal vein. Venous bridges connect the paired axial tibial veins in the lower leg. The popliteal vein continues through the adductor hiatus to become the femoral vein. In the proximal thigh, the femoral vein joins with the deep femoral vein to form the common femoral vein, becoming the external iliac vein at the inguinal ligament. As in the lower extremity, there are deep and superficial veins in the upper extremity. Deep digital veins form the palmar venous arches of the hand and empty into the paired radial and ulnar veins. These follow the named arteries in the arm and are known as the venae comitantes. They become the brachial veins most often near the antecubital fossa and then combine to contribute to forming the axillary vein. Superficial

197

Brunicardi_Ch24_p197-206.indd 197

08/07/22 11:47 AM

198

CHAPTER 24

veins of the upper extremity are the cephalic and basilic veins and their tributaries. The cephalic vein originates at the lateral wrist and courses over the lateral ventral surface of the forearm. In the upper arm, the cephalic vein terminates in the infraclavicular fossa, piercing the clavipectoral fascia to empty into the axillary vein. The basilic vein runs medially along the forearm and penetrates the deep fascia as it courses past the elbow in the upper arm. It then joins with the deep brachial veins to become the axillary vein, a landmark for identification of the axillary vein. The median antecubital vein joins the cephalic and the basilic veins on the ventral surface of the elbow. The axillary vein becomes the subclavian vein at the lateral border of the first rib. At the medial border of the scalenus anterior muscle, the subclavian vein joins with the internal jugular vein to become the brachiocephalic vein, with the subclavian vein coursing anterior to the scalenus anterior muscle. The left and right brachiocephalic veins join to become the superior vena cava, which empties into the right atrium. (See Schwartz 11th ed., p. 981.)

Venous and Lymphatic Disease

2. The target organ(s) of chronic venous insufficiency is/are: A. Perforator veins. B. The great saphenous veins. C. The skin. D. Lymphatics. E. Venous capillaries.

Brunicardi_Ch24_p197-206.indd 198

Answer: C Chronic venous insufficiency (CVI) may lead to characteristic changes in the skin and subcutaneous tissues in the affected limb. CVI results from incompetence of venous valves, venous obstruction, or both. Most CVI involves venous reflux, and severe CVI often reflects a combination of reflux and venous obstruction. It is important to remember that although CVI originates with abnormalities of the veins, the target organ of CVI is the skin, and the underlying physiologic and biochemical mechanisms leading to the cutaneous abnormalities associated with CVI are poorly understood. A typical leg affected by CVI will be edematous, with edema increasing over the course of the day. The leg may also be indurated and pigmented with eczema and dermatitis. These changes are associated with excessive proteinaceous capillary exudate and deposition of a pericapillary fibrin cuff that may limit nutritional exchange. In addition, an increase in white blood cell trapping within the skin microcirculation in CVI patients may lead to microvascular congestion and thrombosis. Subsequently, white blood cells may migrate into the interstitium and release necrotizing lysosomal enzymes, potentially leading to tissue destruction and eventual ulceration. Fibrosis can eventually develop from impaired nutrition, chronic inflammation, and fat necrosis (­lipodermatosclerosis). Hemosiderin deposition due to the extravasation of red cells and subsequent lysis in the skin contributes to the characteristic pigmentation of chronic venous disease (Fig. 24-1). Ulceration can develop with long-standing venous hypertension and is associated with alterations in microcirculatory and cutaneous lymphatic anatomy and function. The most common location of venous ulceration is approximately 3 cm proximal to the medial malleolus (Fig. 24-2). (See Schwartz 11th ed., p. 983.)

08/07/22 11:47 AM

199

CHAPTER 24 Venous and Lymphatic Disease

FIG. 24-1.  Characteristic hyperpigmentation of chronic venous insufficiency.

FIG. 24-2.  Venous ulceration located proximal to the medial malleolus.

Brunicardi_Ch24_p197-206.indd 199

08/07/22 11:47 AM

200 Answer: D The incidence of VTE is approximately 100 per 100,000 people per year in the general population, with 20% of the diagnoses made within 3 months of a surgical procedure. Of the symptomatic patients, one-third will present with pulmonary embolism (PE) and two-thirds with deep vein thrombosis (DVT). The estimated number of cases of VTE may well be over 600,000 per year in the United States, making it a major US health problem. Furthermore, death occurs in 6% of DVT and 12% of PE cases within 1 month of diagnosis. Not only does VTE pose a veritable threat to life, but it also places patients at higher risk for recurrence and post-VTE sequelae such as pulmonary hypertension and postthrombotic ­syndrome, with 4% and up to 30% incidence, respectively. (See Schwartz 11th ed., p. 984.)

4. Which of the following factors is the most important in the development of spontaneous deep vein thrombosis? A. Stasis B. Endothelial Damage C. Hypercoagulability D. All of the above

Answer: C Three conditions, first described by Rudolf Virchow in 1862, contribute to venous thromboembolism (VTE) formation: stasis of blood flow, endothelial damage, and hypercoagulability. Of these risk factors, relative hypercoagulability appears most important in most cases of spontaneous VTE, or ­so-called idiopathic VTE, whereas stasis and endothelial damage likely play a greater role in secondary VTE, or so-called provoked VTE, occurring in association with ­ ­transient risk factors such as immobilization, surgical procedures, and trauma. (See Schwartz 11th ed., p. 984.)

5. All of the following are acquired risk factors for venous thromboembolism (VTE) EXCEPT: A. Nephrotic syndrome. B. Factor V Leiden. C. Malignancy. D. Pregnancy. E. Obesity. F. Varicose veins.

Answer: B The more common acquired VTE risk factors include older age (>40 years), hospitalization and immobilization, hormone replacement and oral contraceptive therapy, pregnancy and the recently postpartum state, prior VTE, malignancy, major surgery, obesity, nephrotic syndrome, trauma and spinal cord injury, long-haul travel (>6 hours), varicose veins, antiphospholipid syndrome, myeloproliferative disorders, and polycythemia. Heritable risk factors include male sex, factor V Leiden mutation; prothrombin 20210A gene variant; antithrombin, protein C, and protein S deficiencies; and dysfibrinogenemias. In some patients, the cause of the thrombophilia may have both a heritable and an acquired component. These mixed causes include homocysteinemia; factors VII, VIII, IX, and XI elevation; hyperfibrinogenemia; and activated protein C resistance in the absence of factor V Leiden. There may be a synergistic effect when particular multiple inherited and acquired risk factors are present in the same patient. Other patient-specific factors associated with venous thrombosis include the traditional cardiovascular risk factors of obesity, hypertension, and diabetes. VTE is more common in whites and African Americans than Asians and Native Americans. Certain gene variants (single nucleotide polymorphisms) are also associated with a mildly increased risk for VTE, and their presence may interact with other risk ­factors to increase the overall risk for venous thrombosis. (See Schwartz 11th ed., Figure 24-3, p. 984.)

CHAPTER 24

3. Venous thromboembolism (VTE) is associated with all of the following EXCEPT: A. Increased morbidity and mortality. B. Pulmonary hypertension. C. Postthrombotic syndrome. D. No change in future risk of VTE.

Venous and Lymphatic Disease

Brunicardi_Ch24_p197-206.indd 200

08/07/22 11:47 AM

201

7. All of the following are absolute contraindications to catheter-directed thrombolysis EXCEPT: A. Prior history of ischemic or hemorrhagic stroke within 3 months. B. Traumatic cardiopulmonary resuscitation within 3 weeks. C. Known intracranial neoplasm. D. Age > 75 years.

Answer: D There are contraindications to thrombolytic therapy. Absolute contraindications include prior history of ischemic or hemorrhagic stroke within 3 months, head trauma within 3 months, neurologic surgery within 6 months, known intracranial neoplasm, internal bleeding within 6 weeks, active or known bleeding disorder, traumatic cardiopulmonary resuscitation within 3 weeks or suspected aortic dissection. Fortunately, serious remote bleeding is uncommon, and intracranial hemorrhage rarely occurs. The majority of bleeding complications are limited to the venous access site. Symptomatic pulmonary embolism occurs uncommonly and is very rarely fatal. (See Schwartz 11th ed., pp. 990–991.)

8. Phlegmasia cerulea dolens is best described as: A. Asymptomatic, but extensive deep vein thrombosis (DVT). B. Isolated popliteal vein thrombosis. C. Extensive DVT of the major axial deep venous channels of the lower extremity potentially complicated by venous gangrene and/or the need for amputation. D. Painless lower extremity swelling.

Answer: C Clinical symptoms may worsen as DVT propagates and involves the major proximal deep veins. Extensive DVT of the major axial deep venous channels of the lower extremity with relative sparing of collateral veins causes a condition called phlegmasia cerulea dolens (Fig. 24-3). This condition is characterized by pain and pitting edema with associated cyanosis. When the thrombosis extends to the collateral veins, massive fluid sequestration and more significant edema ensue, resulting in a condition known as phlegmasia alba dolens. The affected extremity in phlegmasia alba dolens is extremely painful and edematous and pale secondary to arterial insufficiency from dramatically elevated below lower knee compartment pressures. Both phlegmasia cerulean dolens and phlegmasia alba dolens can be complicated by venous gangrene and the need for amputation. (See Schwartz 11th ed., p. 986.)

Venous and Lymphatic Disease

Answer: A Anatomic factors may also contribute to development of DVT. At the site where the right iliac artery crosses over the left iliac vein, the left iliac vein may become chronically narrowed predisposing to iliofemoral venous thrombosis, so-called MayThurner syndrome. External compression of major veins by masses of various types can also lead to venous thrombosis. (See Schwartz 11th ed., p. 985.)

CHAPTER 24

6. May-Thurner syndrome is an anatomical factor associated with increased deep vein thrombosis (DVT) formation, and is characterized by which of the following? A. Narrowing of the left iliac vein at the site where the right iliac artery crosses over it B. Narrowing of the left renal vein as it traverses beneath the superior mesenteric artery C. Subclavian vein narrowing due to repetitive upper extremity effort D. A rapidly expanding hemangioma

FIG. 24-3.  Phlegmasia cerulea dolens of the left leg. Note the bluish discoloration.

Brunicardi_Ch24_p197-206.indd 201

08/07/22 11:47 AM

202 9. According to the American College of Chest Physicians, Answer: C the recommended duration of long-term antithrombotic Table 24-1 (See Schwartz 11th ed., Table 24-4, p. 990.) therapy after provoked deep vein thrombosis (DVT) is: A. 2 weeks. B. 1 month. TABLE 24-1 Summary of American College of Chest Physicians recommendations C. 3 months. regarding duration of long-term antithrombotic therapy for deep vein D. 6 months. thrombosis (DVT)

CHAPTER 24

Clinical Subgroup

Antithrombotic Treatment Duration

First episode DVT/transient risk/surgery

VKA or LMWH for 3 months

First episode DVT/unprovoked

VKA or LMWH for 3 months Consider for long-term therapy if: • Proximal DVT • Minimal bleeding risk • Stable coagulation monitoring

Venous and Lymphatic Disease

Distal DVT/unprovoked • Symptomatic • Asymptomatic and no risk factors for progression Second episode DVT/unprovoked DVT and cancer

VKA for 3 months Serial imaging in 2 weeks, if progression VKA for 3 months VKA for extended therapy LMWH for extended therapy over VKA

LMWH = low molecular weight heparin; VKA = vitamin K antagonist. Data from Kearon C, Akl EA, Comerota AJ, et al: Antithrombotic therapy for VTE disease: Antithrombotic Therapy and Prevention of Thrombosis, 9th ed: American College of Chest Physicians Evidence-Based Clinical Practice Guidelines, Chest. 2012; 141(2 Suppl): e419S-e496S.

10. Which of the following statements related to inferior vena cava (IVC) filters is TRUE? A. Placement of an IVC filter allows for a reduced duration of anticoagulation therapy. B. In patients with a proximal DVT, placement of an IVC filter reduces rate of pulmonary embolism, and also prolongs early and late survival. C. The rate of fatal complications related to IVC filters is 97%. The ability of DUS to assess isolated calf vein DVT varies greatly, with sensitivities ranging from 50% to 93% and specificities approaching 100%. (See Schwartz 11th ed., pp. 986–987)

FIG. 24-5.  Duplex ultrasound of a femoral vein containing thrombus demonstrating no flow within the femoral vein.

R FVP

FIG. 24-6.  B-mode ultrasound of the femoral vein in cross-section. The femoral vein does not collapse with external compression (arrows).

Brunicardi_Ch24_p197-206.indd 204

No compression

R FVP

Compression

08/07/22 11:47 AM

205

14. Direct thrombin inhibiting medications include which of the following? A. Warfarin B. Enoxaparin C. Argatroban D. Fondaparinux

Answer: C Direct thrombin inhibitors (DTIs) include recombinant hirudin, argatroban, and bivalirudin. These antithrombotic agents bind to thrombin, inhibiting the conversion of fibrinogen to fibrin as well as thrombin-induced platelet activation. These actions are independent of antithrombin. The DTIs should be reserved for (a) patients in whom there is a high clinical suspicion or confirmation of HIT, and (b) patients who have a history of HIT or test positive for heparin-associated antibodies. In patients with established HIT, DTIs should be administered for at least 7 days, or until the platelet count normalizes. Warfarin may then be introduced slowly, overlapping therapy with a DTI for at least 5 days. (See Schwartz 11th ed., p. 988.)

15. All of the following are components of the treatment of lymphedema EXCEPT: A. Extremity compression and elevation. B. Prophylactic antibiotics. C. Intermittent pneumatic compression therapy. D. Lymphatic massage.

Answer: B Bed rest and leg elevation. Elevation is an important aspect of controlling lower extremity swelling and is often the first recommended intervention. However, continuous elevation throughout the day can interfere with quality of life more than lymphedema itself. Elevation is an adjunct to lymphedema therapy but is not the mainstay of treatment. Intermittent pneumatic compression therapy. The use of IPC with a single-chamber or multichamber pump temporarily reduces edema and provides another adjunct to the use of compression stockings. These devices have been shown to be effective in reducing limb volume; however, use of compression stockings is necessary to maintain the volume reduction when the patient is no longer supine because fluid transport is not associated with the transport of macromolecules (proteins) from the tissue. Typically, IPC is used for 4 to 6 hours per day at home when the patient is supine, with pressure ranges between 30 and 60 mm Hg demonstrated to be most effective. Lymphatic massage. Manual lymphatic drainage is a form of massage developed by Vodder that is directed at reducing edema. In combination with the use of compression stockings, manual lymphatic drainage is associated with a longterm reduction in edema and fewer infections per patient per year.

Brunicardi_Ch24_p197-206.indd 205

Venous and Lymphatic Disease

Answer: B HIT results from heparin-associated antiplatelet antibodies (HAAbs) directed against platelet factor 4 complexed with heparin. HIT occurs in 1% to 5% of patients being treated with heparin. In patients with repeat heparin exposure (such as vascular surgery patients), the incidence of HAAbs may be as high as 21%. HIT occurs most frequently in the second week of therapy and may lead to disastrous venous or arterial thrombotic complications. Therefore, platelet counts should be monitored periodically in patients receiving continuous heparin therapy. HIT is diagnosed based on previous exposure to heparin, platelet count < 100,000, and/or platelet count decline of 50% following exposure. All heparin must be stopped and alternative anticoagulation initiated immediately to avoid thrombotic complications, which may approach 50% over the subsequent 30 days in affected individuals. (See Schwartz 11th ed., p. 988.)

CHAPTER 24

13. Heparin-induced thrombocytopenia (HIT) is characterized by which of the following? A. Diagnosis based on prior exposure to heparin with platelet count 24-hour pH study->surgery B. Endoscopy->PPI->24-hour pH study->surgery C. PPI->24-hour pH study->endoscopy->surgery D. 24-hour pH study->endoscopy->PPI->surgery

Answer: A Traditionally, a stepwise approach is used for the treatment of GERD. First-line therapy entails antisecretory medication, usually proton pump inhibitors (PPIs), in most patients. Failure of medication to adequately control GERD symptoms suggests either that the patient may have relatively severe disease or a non-GERD cause for his or her symptoms. Endoscopic examination at this stage of the patient’s evaluation is recommended and will provide the opportunity to assess the degree of mucosal injury and presence of Barrett esophagus (BE). Treatment options for these patients entail either long-term PPI use versus antireflux surgery. Laparoscopic antireflux surgery in these patients achieves long-term control of symptoms in 85% to 90%. The measurement of esophageal acid exposure via 24-hour pH should be undertaken when patients are considered for surgery. The status of the lower esophageal sphincter (LES) and esophageal body function with esophageal manometry should also be performed at this stage. These studies will serve to establish the diagnosis and assess esophageal body dysfunction. (See Schwartz 11th ed., p. 1037.)

21. An antireflux surgery should ideally do all of the following EXCEPT: A. Create a flap valve to prevent regurgitation into the esophagus. B. Double the resting gastric pressure. C. Reconstruct the valve to a length 8 cm in length, abnormal axis of the esophagus on a barium radiogram, more than four enlarged lymph nodes (LNs) on computed tomography (CT), a weight loss >20%, and loss of appetite. Studies indicate that there are several favorable parameters associated with tumors 8 cm in length. Consequently, the finding of a tumor >8 cm in length should exclude curative resection; the finding of a smaller tumor should encourage an aggressive approach. (See Schwartz 11th ed., p. 1073.)

28. The top three most common complications (in order of most common to least common) following an minimally invasive surgery (MIS) three-field esophagectomy are: A. Pneumonia, atrial fibrillation, anastomotic leak. B. Anastomotic leak, pneumonia, deep vein thrombosis (DVT). C. Atrial fibrillation, thoracic duct injury, pneumonia. D. Pneumonia, anastomotic leak, pneumothorax. E. Atrial fibrillation, anastomotic leak, pneumonia.

Answer: A The MIS transthoracic two-field esophagectomy is slightly different. In this operation, the abdominal portions of the operation are done first, including placement of the feeding tube, the creation of the conduit, and the sewing of the tip of the conduit to the fully dissected gastroesophageal junction (GEJ). The patient is then rolled into the left lateral decubitus position and, through right thoracoscopy, the esophagus is dissected and divided 10 cm above the tumor. Once freed, the specimen is pulled out through the mini-thoracotomy, and an end-to-end anastomosis stapler is introduced through the high corner of the gastric conduit and out a stab wound along the greater curvature. The anvil of the stapler is placed in the proximal esophagus and held with a purse-string, the stapler is docked, the anastomosis is created, and a gastrotomy is then closed with another firing of the gastrointestinal anastomosis (GIA) stapler. The three-field esophagectomy has the advantage of placing the anastomosis in the neck where

Brunicardi_Ch25_p207-222.indd 221

The Esophagus and Diaphragmatic Hernia

Answer: D The third issue—and one that has been long debated—is the question of whether an antireflux procedure should be added to a surgical myotomy. Excellent results have been reported following meticulously performed myotomy without an antireflux component. Retrospective studies, with long-term follow-up of large cohorts of patients undergoing Heller myotomy demonstrated that, after 10 years, >50% of patients had reflux symptoms without a fundoplication. In a recent randomized clinical trial, 7% of patients undergoing Dor fundoplication following lower esophageal sphincter (LES) myotomy had abnormal 24-hour pH probes, and 42% of patients with a myotomy only had abnormal reflux profiles. If an antireflux procedure is used as an adjunct to esophageal myotomy, a complete 360° fundoplication should be avoided. Rather, a 270° Belsey fundoplication, a Toupet posterior 180° fundoplication, or a Dor anterior 180° fundoplication should be used to avoid the long-term esophageal dysfunction secondary to the outflow obstruction afforded by the fundoplication itself. (See Schwartz 11th ed., pp. 1055–1056.)

CHAPTER 25

26. Patients who present with achalasia should NOT be offered which of the following treatment options? A. Heller myotomy with 270o Belsey fundoplication B. Heller myotomy with Toupet posterior 180o fundoplication C. Heller myotomy with Dor anterior 180o fundoplication D. Heller myotomy with complete 360o fundoplication

04/07/22 2:39 PM

222

CHAPTER 25

leakage is unlikely to create a severe systemic consequence. On the other hand, placement of the anastomosis in the high chest minimizes the risks of injury to structures in the neck, particularly the recurrent laryngeal nerve. Although the leak of the intrathoracic anastomosis may be more likely to bear septic consequences, the incidence of leak is diminished. Other complications of this approach relate to pulmonary and cardiac status. In many series, the most common complication is pneumonia, the second is atrial fibrillation, and the third is anastomotic leak. (See Schwartz 11th ed., p. 1076.)

The Esophagus and Diaphragmatic Hernia Brunicardi_Ch25_p207-222.indd 222

04/07/22 2:39 PM

CHAPTER

26

Stomach

1. All of the following statements about complicated peptic ulcer disease requiring hospitalization are TRUE EXCEPT: A. Bleeding is the most common cause of ulcer-related death. B. Perforation is the most common indication for operation. C. Endoscopic treatment is usually effective in achieving definitive hemostasis. D. Obstruction rarely requires operation.

Answer: D Bleeding is the most common cause of ulcer-related death, but only rarely do patients with bleeding gastric or duodenal ulcer require operation today. The success of endoscopic treatment and medical therapy for bleeding peptic ulcer disease (PUD) has resulted in the selection of a small subgroup of high-risk patients for today’s surgeon. (See Schwartz 11th ed., p. 1131.) Perforation is the second most common complication of peptic ulcer, but nowadays it is a much more common indication for operation than bleeding. (See Schwartz 11th ed., p. 1134.) Most patients with significant obstruction from chronic ulceration will require some sort of substantial intervention. Endoscopic balloon dilation can often transiently improve obstructive symptoms, but many of these patients ultimately fail and come to operation. (See Schwartz 11th ed., p. 1136.)

2. Which of the following inhibits gastrin secretion? A. Histamine B. Acetylcholine C. Amino acids D. Acid

Answer: D Luminal peptides and amino acids are the most potent stimulants of gastrin release, and luminal acid is the most potent inhibitor of gastrin secretion. The latter effect is predominantly mediated in a paracrine fashion by somatostatin released from antral D cells. Gastrin-stimulated acid secretion is significantly blocked by H2 antagonists, suggesting that the principal mediator of gastrin-stimulated acid production is histamine from mucosal enterochromaffin-like (ECL) cells and not direct stimulation of parietal cells by gastrin. (See Schwartz 11th ed., p. 1109.)

3. Eradication of Helicobacter pylori infection after treatment may be confirmed by all of the following EXCEPT: A. Carbon-labeled urea breath test. B. Serology. C. Fecal antigen test. D. Gastric mucosal biopsy.

Answer: B A positive serologic test is presumptive evidence of active infection if the patient has never been treated for H. pylori. Histologic examination of gastric mucosal biopsy using special stains is the gold standard test for helicobacter infection. Other sensitive tests include commercially available rapid urease tests, which assay for the presence of urease in mucosal biopsy specimens (strong presumptive evidence of infection). Urease is an omnipresent enzyme in H. pylori strains that colonize the gastric mucosa. The carbon-labeled urea breath test has become the standard test to confirm eradication of

223

Brunicardi_Ch26_p223-232.indd 223

08/07/22 11:47 AM

224

CHAPTER 26

H. pylori following appropriate treatment. In this test, the patient ingests urea-labeled with nonradioactive 13C or 14C. The labeled urea is acted upon by the urease present in the H. pylori and converted into ammonia and carbon dioxide. The radiolabeled carbon dioxide is excreted from the lungs and can be detected in the expired air. It can also be detected in a blood sample. The fecal antigen test also is quite sensitive and specific for active H. pylori infection and may also be used to confirm cure after treatment. (See Schwartz 11th ed., p. 1117.) Because H. pylori induces a strong immunologic response, serological testing is useful but may not be as accurate as the urea breath test or the stool antigen test, and a positive serology persists after eradication of H. pylori infection, so serology is not useful to confirm successful treatment of ­Helicobacter infection. (See Schwartz 11th ed., p. 1119.)

Stomach

4. A patient is referred 2 years following distal gastrectomy and Roux-en-Y reconstruction for nonhealing benign gastric ulcer. His chief complaint is epigastric pain relieved by vomiting undigested food. He has lost 15% of his body weight in the past 6 months. Solid gastric emptying scan shows 20% gastric emptying at 4 hours, and upper endoscopy is normal except for retained food in the stomach. Treatment options include all the following EXCEPT: A. Transthoracic vagotomy. B. Oral erythromycin. C. High subtotal gastrectomy. D. Feeding jejunostomy.

Answer: A Gastric stasis following surgery on the stomach may be due to a problem with gastric motor function or caused by an obstruction. The gastric motility abnormality could have been preexisting and unrecognized by the operating surgeon. Alternatively, it may be secondary to deliberate or unintentional vagotomy, or resection of the dominant gastric pacemaker. An obstruction may be mechanical (eg, anastomotic stricture, efferent limb kink from adhesions or constricting mesocolon, or a proximal small-bowel obstruction) or functional (eg, retrograde peristalsis in a Roux limb). Gastric stasis presents with vomiting (often of undigested food), bloating, epigastric pain, and weight loss. Once mechanical obstruction has been ruled out, medical treatment is successful in most cases of motor dysfunction following previous gastric surgery. This consists of dietary modification and promotility agents. (If operation is required) gastroparesis following subtotal gastric resection is best treated with near-total (95%) or total gastric resection and Roux-en-Y reconstruction. (See Schwartz 11th ed., pp. 1157–1158.) Three prokinetic (metoclopramide, erythromycin, domperidone) may be used to treat delayed gastric emptying. Typical doses and mechanism of action are shown in Table 26-1. (See Schwartz 11th ed., p. 1115.)

TABLE 26-1

Typical Adult Dose

Mechanism of Action

Metoclopramide

10 mg PO four times a day

Dopamine antagonist

Erythromycin

250 mg PO four times a day

Motilin agonist

Domperidone

10 mg PO four times a day

Dopamine antagonist

5. Which of the following is secreted by gastric parietal cells? A. Pepsinogen B. Intrinsic factor C. Gastrin-releasing peptide D. Ghrelin E. Histamine

Brunicardi_Ch26_p223-232.indd 224

Drugs that accelerate gastric emptying

Agent

Answer: B Activated parietal cells secrete intrinsic factor in addition to hydrochloric acid. Presumably the stimulants are similar, but acid secretion and intrinsic factor secretion may not be linked. Intrinsic factor binds to luminal vitamin B12, and the complex is absorbed in the terminal ileum via mucosal receptors. Vitamin B12 deficiency can be life-threatening, and patients with total gastrectomy or pernicious anemia (ie, patients with no parietal cells) require B12 supplementation. (See Schwartz 11th ed., pp. 1108–1109.)

08/07/22 11:47 AM

225 6. The most accurate diagnostic test for Zollinger-Ellison syndrome (ZES) is: A. Fasting serum gastrin. B. Computed tomography scan. C. Endoscopy. D. Secretin stimulation test.

Stomach

Elevated serum gastrin (off PPI + H2RA)

CHAPTER 26

Answer: D All patients with gastrinoma have an elevated gastrin level, and hypergastrinemia in the presence of elevated basal acid output (BAO) strongly suggests gastrinoma. Patients with gastrinoma usually have a BAO > 15 mEq/h or >5 mEq/h if they have had a previous procedure for peptic ulcer. Acid secretory medications should be held for several days before gastrin measurement, because acid suppression may falsely elevate gastrin levels. Causes of hypergastrinemia can be divided into those associated with hyperacidity and those associated with hypoacidity (see Fig. 26-1). The diagnosis of ZES is confirmed by the secretin stimulation test. An intravenous (IV) bolus of secretin (2 U/kg) is given, and gastrin levels are checked before and after injection. An increase in serum gastrin of 200 pg/mL or greater suggests the presence of gastrinoma. (See Schwartz 11th ed., p. 1137.)

Measure BAO and gastric pH

BAO low pH > 2

BAO > 10 mEq/h pH < 2

Renal insuff

Secretin stimulation test

Pernicious anemia

Previous GI surgery • Vagotomy? • Massive SB resection

Atrophic gastritis

Confirm with EGD/Bx test/treat H. pylori give B12

Significant elevation in serum gastrin in response to IV secretin?

Yes

No

Zollinger-Ellison syndrome

G-cell +/or ECL cell hyperplasia

Octreotide scan R/O MEN-1

Confirm with EGD + Bx

Consider resection

Loop GJ with antral alkalinization

Antral stasis

S/P gastrectomy B2

Takedown GJ Or TV and A

Consider TV + antrectomy

Retained antrum on “duodenal stump”

Resect retained antrum Or Convert B2 to B1

FIG. 26-1.  Algorithm for diagnosis and management of hypergastrinemia. BAO = basal acid output; B1 = Billroth 1; B2 = Billroth 2; Bx = biopsy; ECL = enterochromaffin-like; EGD = esophagogastroduodenoscopy; GJ = gastrojejunostomy; H2RA = histamine 2 receptor antagonist; insuff = insufficiency; MEN1 = multiple endocrine neoplasia type I; PPI = proton pump inhibitor; R/O = rule out; SB = small bowel; S/P = status post; TV = truncal vagotomy; TV and A = truncal vagotomy and antrectomy.

Brunicardi_Ch26_p223-232.indd 225

08/07/22 11:47 AM

226 7. In the patient with a secretin stimulation test suggesting gastrinoma, which of the following is the preoperative imaging study of choice? A. Computed tomography (CT) B. Magnetic resonance imaging (MRI) C. Endoscopic ultrasound (EUS) D. Angiographic localization E. Somatostatin receptor scintigraphy

CHAPTER 26

Answer: E About 80% of primary tumors are found in the gastrinoma triangle (Fig. 26-2), and many tumors are small (2 cm in size, and MRI is comparable. EUS is more sensitive than noninvasive imaging tests, but it still misses many smaller lesions or lesions in inaccessible locations (eg, the pancreatic tail). Somatostatin receptor scintigraphy (the octreotide scan) or Gallium-68 dotatate positron emission tomograph/computed tomography (PET/CT) are sensitive and specific when the pretest probability of gastrinoma is high and may identify sites of regional or distant metastatic disease. (See Schwartz 11th ed., p. 1137.)

Stomach FIG. 26-2.  Gastrinoma triangle.

8. A patient on chronic steroids and nonsteroidal antiinflammatory drugs (NSAIDs) for rheumatoid arthritis requires operation for bleeding duodenal ulcer. Patients like this: A. Have a 90-day mortality risk of 5%. B. Far outnumber patients requiring operation for perforated duodenal ulcer. C. Should be considered for lifelong acid suppression if vagotomy is omitted. D. Are usually treated with distal gastrectomy.

Brunicardi_Ch26_p223-232.indd 226

Answer: C Bleeding is the most common cause of ulcer-related death, but only rarely do patients with bleeding gastric or duodenal ulcer require operation today. The success of endoscopic treatment and medical therapy for bleeding peptic ulcer disease (PUD) has resulted in the selection of a small subgroup of high-risk patients for today’s surgeon. It is likely that patients currently coming to operation for bleeding PUD are at higher risk for a poor outcome than ever before. The mortality rate for surgery for bleeding peptic ulcer is around 20%. (See Schwartz 11th ed., pp. 1131–1133.) Today, most patients undergoing emergent operation have simple patch of a perforated ulcer or oversewing of a bleeding ulcer. (See Schwartz 11th ed., p. 1127.) Long-term maintenance proton pump inhibitor (PPI) therapy should be considered in all patients admitted to hospital with ulcer complications, all high-risk patients on NSAIDs or aspirin (the elderly or debilitated), and all patients requiring anticoagulation or antiplatelet agents or those with a h ­ istory of recurrent ulcer or bleeding. (See Schwartz 11th ed., p. 1125.)

08/07/22 11:48 AM

227

10. Which of the following options is the least preferable reconstruction for patients undergoing antrectomy for peptic ulcer disease? A. Bilroth I. B. Bilroth II. C. Roux-en-Y gastrojejunostomy. D. All are equally preferable.

Answer: C Following antrectomy, gastrointestinal (GI) continuity may be reestablished with a Billroth I gastroduodenostomy (Fig. 26-3) or a Billroth II loop gastrojejunostomy (Fig. 26-4). Since antrectomy routinely leaves a 60% to 70% gastric remnant, routine

Stomach

Answer: B Surgery is almost always indicated for ulcer perforation, although occasionally nonsurgical treatment can be used in the stable patient without peritonitis in whom radiologic studies document a sealed perforation. Simple patch closure, currently the most commonly performed operation for perforated peptic ulcer, is the procedure of choice in patients with hemodynamic instability and/or exudative peritonitis signifying a perforation >24 hours old. (See Schwartz 11th ed., p. 1134.)

CHAPTER 26

9. A 50-year-old homeless man is brought to the emergency room (ER) complaining of 2 days of severe abdominal pain. He states that he “has had ulcer trouble for 30 years.” Temp = 102°F, BP = 80/50, P = 130. He has abdominal distention and generalized peritonitis on examination. Serum lactate is elevated. He is given intravenous (IV) antibiotics and 2 L of crystalloid then started on norepinephrine drip. Nasogastric (NG) drains 300 mL of bilious nonbloody fluid. Computed tomography (CT) scan immediately after injection of 100 mL oral contrast via NG shows extravasation from the duodenal bulb, copious-free peritoneal fluid, and air. BP is now 110/70, serum lactate is normal, and urine output in the past hour is 20 mL. You strongly suspect perforated duodenal ulcer and you would recommend: A. Endoscopy and biopsy. B. Immediate closure of perforation with Graham patch. C. Nonoperative management. D. Immediate closure of perforation with definitive ulcer operation. E. An additional 8 hours of ICU resuscitation, followed by laparoscopic patch and highly selective vagotomy.

A

B

FIG. 26-3.  A and B. Billroth I gastroduodenostomy. (­Reproduced with permission from Zinner MJ. Atlas of G ­ astric Surgery. New York, NY: Elsevier/Churchill ­Livingstone; 1992.)

Brunicardi_Ch26_p223-232.indd 227

08/07/22 11:48 AM

228 reconstruction as a Roux-en-Y gastrojejunostomy should be avoided (Fig. 26-5). Although the Roux-en-Y operation is an excellent procedure for keeping duodenal contents out of the stomach and esophagus, in the presence of a large gastric remnant, this reconstruction will predispose to marginal ulceration and/or gastric stasis. (See Schwartz 11th ed., p. 1129.)

CHAPTER 26 Stomach A

B

C

FIG. 26-4.  A through C. Billroth II antecolic gastrojejunostomy. (Reproduced with permission from Zinner MJ, Schwartz SI, Ellis H. Maingot’s Abdominal Operations, 10th ed. Vol. I. Stamford, CT: Appleton & Lange; 1997.)

16 regional lymph nodes and adjuvant chemoradiotherapy. E. Endoscopic resection.

08/07/22 11:48 AM

230

CHAPTER 26 Stomach

15. A fifty-year-old man undergoes resection of a 5 cm gastrointestinal stromal tumor (GIST) involving the fundus of the stomach. Pathology indicates 12 mitoses/50 HPF. Molecular testing shows a KIT mutation. Subsequent management should include: A. Clinical follow-up alone. B. Clinical follow-up with periodic surveillance imaging alone. C. Adjuvant Imatinib for 1 year. D. Adjuvant Sunitinib for at least 3 years. E. Adjuvant Imatinib for at least 3 years.

Answer: E Risk of GIST recurrence after resection reflects the location of the tumor (gastric tumors are less likely to recur than nongastric tumors), size, and mitotic rate. Mitotic rate >5/50 HPF is the strongest predictor of recurrence. Patients at higher risk of recurrence should be treated with adjuvant Imatinib barring the presence of a PDGFRA D842V mutation, which is associated with Imatinib resistance. Duration of treatment for 3 years proved superior to 1 year in the Scandinavian Sarcoma Group XV111 trial and there is increasing momentum for indefinite treatment after resection of high-risk disease. (See Schwartz 11th ed., pp. 1149–1151.)

16. Which of the following options is the best initial management of a low-grade mucosa-associated lymphoid tissue (MALT) lymphoma of the gastric antrum? A. Helicobacter pylori eradication B. Chemotherapy +/– radiation therapy C. Wedge resection D. Antrectomy

Answer: A Low-grade MALT lymphoma, essentially a monoclonal proliferation of B cells, presumably arises from a background of chronic gastritis associated with H. pylori. These relatively innocuous tumors then undergo degeneration to high-grade lymphoma, which is the usual variety seen by the surgeon. Remarkably, when the H. pylori is eradicated and the gastritis improves, the low-grade MALT lymphoma often disappears. Thus, low-grade MALT lymphoma is not a surgical lesion. Careful follow-up is necessary particularly in those lesions with a t (11:18) translocation, thought to be a risk factor for a more aggressive MALT lesion. (See Schwartz 11th ed., p. 1149.)

17. Type III gastric carcinoid tumors: A. Often do not require resection. B. Are associated with hypergastrinemia. C. Are sporadic lesions. D. Have better outcomes than type I and II tumors.

Answer: C Type III gastric neuroendocrine tumors are sporadic. They are most often solitary (usually >2 cm) and occur more commonly in men. They are not associated with hypergastrinemia. Most patients have regional nodal or distant metastases at the time of diagnosis, and some present with symptoms of carcinoid syndrome. (See Schwartz 11th ed., p. 1151.)

18. In the patient with a normal liver and endoscopic diagnosis of watermelon stomach (gastric antral vascular ectasia [GAVE]), chronic blood loss requiring multiple transfusions is best treated by: A. Proton pump inhibitors and Carafate. B. Beta blockers. C. Distal gastrectomy. D. Total gastrectomy.

Answer: C The parallel red stripes atop the mucosal folds of the distal stomach give this rare entity its name. Histologically, gastric antral vascular ectasia (GAVE) is characterized by dilated mucosal blood vessels that often contain thrombi, in the lamina propria. Mucosal fibromuscular hyperplasia and hyalinization often are present (Fig. 26-6). The histologic appearance can resemble portal hypertensive gastropathy, but the latter usually affects the proximal stomach, whereas watermelon stomach predominantly affects the distal stomach.

FIG. 26-6.  Gastric antral vascular ectasia (watermelon stomach). (Reproduced with permission from Godlman H, Hayek J, Federman M. Gastrointestinal Mucosal Biopsy. New York, NY: Churchill Livingstone; 1996.)

Brunicardi_Ch26_p223-232.indd 230

08/07/22 11:48 AM

231

Answer: C A variety of hormonal aberrations have been observed in early dumping, including increased serum levels of vasoactive intestinal polypeptide (VIP), cholecystokinin (CCK), neurotensin, peripheral hormone peptide YY, renin-angiotensinaldosterone, and decreased atrial natriuretic peptide. Late dumping is associated with hypoglycemia and hyperinsulinemia. Medical therapy for the dumping syndrome consists of dietary modification and somatostatin analogue (octreotide). (See Schwartz 11th ed., p. 1157.)

20. Ménétrier disease is characterized by: A. Hypertrophic gastric folds and hypoproteinemia. B. A tortuous submucosal congenital arteriovenous malformation. C. Gastric antral vascular ectasia. D. Epithelial hyperplasia and hypergastrinemia.

Answer: A There are two clinical syndromes characterized by epithelial hyperplasia and giant gastric folds: Zollinger-Ellison syndrome (ZES) and Ménétrier disease. The latter is characteristically associated with protein-losing gastropathy and hypochlorhydria. There are large rugal folds in the proximal stomach, and the antrum is usually spared. Mucosal biopsy shows diffuse hyperplasia of the surface mucus-secreting cells and usually decreased parietal cells (Fig. 26-7). It has recently been suggested that Ménétrier disease is caused by local overexpression of transforming growth factor-A in the gastric mucosa, which stimulates the epidermal growth factor receptor, a receptor tyrosine kinase, on gastric surface epithelial cells (SECs). This results in the selective expansion of surface mucous cells in the gastric body and fundus. A few patients with this unusual disease have been successfully treated with the epidermal growth factor receptor blocking monoclonal antibody cetuximab. (See Schwartz 11th ed., pp. 1153–1154.)

Stomach

19. Treatment for severe early dumping after gastrectomy that is persistent despite an antidumping diet and fiber is: A. Expectant management. B. Oral glucose for symptoms. C. Octreotide. D. Surgical conversion to a Roux-en-Y drainage.

CHAPTER 26

Beta blockers and nitrates, useful in the treatment of portal hypertensive gastropathy, are ineffective in patients with gastric antral vascular ectasia. Patients with GAVE are usually elderly women with chronic gastrointestinal (GI) blood loss requiring transfusion. Most have an associated autoimmune connective tissue disorder, and at least 25% have chronic liver disease. Nonsurgical treatment options include estrogen and progesterone, and endoscopic treatment with the neodymium yttrium-aluminum garnet (Nd:YAG) laser or argon plasma coagulator. Antrectomy may be required to control blood loss, and this operation is quite effective but carries increased morbidity in this elderly patient group. Patients with portal hypertension and antral vascular ectasia should be considered for transjugular intrahepatic portosystemic shunt (TIPSS). (See Schwartz 11th ed., p. 1154.)

FIG. 26-7.  Mucosal biopsy in Ménétrier’s disease. (Reproduced with permission from Ming S-C, Goldman H. Pathology of the Gastrointestinal Tract, 2nd ed. Baltimore, MD: Williams & Wilkins; 1998.)

Brunicardi_Ch26_p223-232.indd 231

08/07/22 11:48 AM

This page intentionally left blank

Brunicardi_Ch26_p223-232.indd 232

08/07/22 11:48 AM

CHAPTER

27

The Surgical Management of Obesity

1. What body mass index [BMI or wt (kg)/height (m2)] definition of obesity serves as the standard indication for bariatric surgery when medical therapy has failed and comorbid conditions exist? A. Overweight (BMI 25.0–29.9) B. Class I obesity (BMI 30.0–34.9) C. Class II obesity (BMI 35.0-39.9) D. Class III obesity (BMI ≥ 40)

Answer: C A 1991 NIH Consensus Conference recommended that bariatric surgery was indicated for a BMI of 35.0 to 39.9 when medical therapy has failed and comorbid conditions exist. When no comorbid conditions exist, a BMI of ≥40 is required. This standard continues to be used by insurers, although recent studies have shown benefit in lower BMI groups. (See Schwartz 11th ed., pp. 1169–1170 and Table 27-2 p. 1175.)

2. Which of the following bariatric procedures is primarily intended to induce weight loss through malabsorption of ingested nutrients? A. Jejunoileal bypass B. Sleeve gastrectomy C. Roux-en-Y gastric bypass D. Duodenal switch

Answer: A Bariatric procedures are classified as restrictive, malabsorptive, or a combination of restrictive and malabsorptive in the mechanism of weight loss that they induce. Restrictive operations include adjustable gastric band and gastric sleeve, malabsorptive operations include jejunoileal bypass and its derivatives, and combined restrictive and malabsorptive procedures include the Roux-en-Y gastric bypass. (See Schwartz 11th ed., p. 1168.)

3. Complications of adjustable gastric banding which have diminished its popularity as a bariatric procedure include all of the following EXCEPT: A. Mortality risk. B. Slippage of the band. C. Failure to lose weight. D. Port and tubing complications.

Answer: A The adjustable gastric band procedure, usually performed laparoscopically, has the lowest cost and mortality risk of all the bariatric procedures, but is the least effective for weight loss. In addition, slippage and erosion of the band and complications related to the maintenance and use of the port for adjusting the size of the band contribute to its loss of popularity. (See Schwartz 11th ed., p. 1188.)

4. Early postoperative complications after a Roux-en-Y gastric bypass (RYGB) procedure include all of the following EXCEPT: A. Hematemesis. B. Dilation of the distal gastric remnant. C. Pulmonary embolus. D. Hyperglycemia.

Answer: D Early postoperative complications after RYGB include hematemesis due to bleeding from the gastrojejunal staple or suture line, gastric remnant dilation due to down-stream obstruction, and pulmonary complications such as atelectasis and pulmonary embolus. Diabetes, if present, usually improves promptly after RYGB and hyperglycemia is unlikely to be problematic. (See Schwartz 11th ed., p. 1185.)

233

Brunicardi_Ch27_p233-234.indd 233

30/06/22 11:24 AM

234

CHAPTER 27 The Surgical Management of Obesity

5. Small bowel obstruction after Roux-en-Y gastric bypass (RYGB) should be treated as an urgent surgical emergency because: A. It is frequently due to an incarcerated internal hernia which can progress to bowel necrosis and perforation. B. Abdominal distension risks disruption of suture lines. C. Signs and symptoms of peritonitis, such as pain, fever, and leukocytosis, are usually masked in the obese. D. Nasogastric intubation will not decompress the distal gastric remnant.

Answer: A Small bowel obstruction after RYGB is frequently due to an incarcerated internal hernia at the location of the closure, or lack thereof, of the mesenteric defect. This can progress rapidly to strangulation and necrosis of the bowel with subsequent perforation. Adverse outcomes with this complication have resulted in the uniform recommendation that small bowel obstruction in this setting should be regarded as a surgical emergency. Abdominal distention and difficulties with nasogastric intubation are not relevant concerns. (See Schwartz 11th ed., p. 1184.)

6. A late complication of Roux-en-Y gastric bypass is the development of post-gastric bypass hypoglycemia (PGBH) due to inappropriately elevated insulin levels. Recommended treatments of this condition include all of the following EXCEPT: A. Very low carbohydrate diet. B. Anti-secretory agents such as diazoxide and somatostatin. C. GLP-1 receptor antagonists. D. Pancreatic resection.

Answer: D PGBH occurs in some patients after Roux-en-Y gastric bypass due to exaggerated or excessive insulin levels. It is thought to be a consequence of greatly increased GLP-1 levels seen in some patients. A very low carbohydrate diet may provide relief in some patients, and diazoxide and somatostatin have been used in those who fail diet therapy. Recently GLP-1 receptor antagonists have been reported to be successful. Although pancreatic resection was initially reported in patients with severe hypoglycemia, it is no longer recommended. Partial pancreatectomy results in a recurrence of the problem, and total pancreatectomy exchanges one severe condition for another. (See Schwartz 11th ed., pp. 1204–1205.)

7. Complications which are associated with the gastric sleeve procedure include: A. Leak from the gastric staple line. B. Gastroesophageal reflux. C. Stenosis of the gastric sleeve. D. All of the above.

Answer: D The gastric sleeve procedure creates a high pressure tube. This places the gastric staple line at risk for leak, and additional buttress sutures are frequently used to reinforce the staple line. The high pressure also increases the probability of gastroesophageal reflux disease (GERD) so patients with a history of GERD are less favorable candidates for the gastric sleeve procedure. The gastric tube itself is usually constructed over a 40-French bougie. If a smaller bougie is used, the risk of leak and reflux are increased and stenosis of the gastric sleeve or tube may occur. (See Schwartz 11th ed., p. 1186.)

8. In addition to the effects of weight loss, the resolution of type 2 diabetes mellitus after the gastric sleeve procedure and Roux-en-Y gastric bypass (RYGB) is thought to be contributed to by: A. Reduced ghrelin production. B. Increased secretion of GLP-1. C. Appetite suppression. D. All of the above.

Answer: D Gastrectomy removes much of the ghrelin-producing portion of the stomach and this mechanism is thought to contribute to weight loss after both gastric sleeve and RYGB procedures. GLP-1, the enteric hormone which augments insulin release, is dramatically increased after RYGB, and is increased after the gastric sleeve procedure as well. A profound suppression of appetite and food craving has been found to follow these procedures, presumably due to the altered hormonal status of peptides which affect the satiety centers of the central ­nervous system. (See Schwartz 11th ed., p. 1175.)

9. Adolescent patients with morbid obesity are increasingly being referred for consideration of bariatric procedures due to failure of medical management and the risks associated with a lifetime of obesity. What nutritional deficiencies require lifelong treatment after Roux-enY gastric bypass (RYGB), the most common procedure performed in this age group? A. Pernicious anemia due to vitamin B12 deficiency B. Iron deficiency anemia C. Deficiencies of vitamins A, E, D, and K D. All of the above

Answer: D Loss of intrinsic factor produced in the gastric fundus, impaired iron absorption, and a deficiency of the fat-soluble vitamins present lifelong risks after RYGB and other malabsorptive bariatric procedures. Vitamin replacement and nutritional monitoring are therefore mandatory in bariatric patients. (See Schwartz 11th ed., pp. 1191–1192.)

Brunicardi_Ch27_p233-234.indd 234

30/06/22 11:24 AM

CHAPTER

28

Small Intestine

1. How long is the small intestine? A. 4 to 6 ft B. 10 to 12 ft C. 4 to 6 m D. 10 to 12 m

Answer: C The small intestine’s length varies depending on whether radiologic, surgical, or autopsy measurements are made, but it is thought to measure 4 to 6 m in living subjects. (See Schwartz 11th ed., p. 1219.)

2. Which of the following features is characteristic of the ileum, as opposed to the jejunum? A. The presence of valvulae conniventes B. The presence of Peyer patches C. Larger vasa recta D. Less fatty mesentery

Answer: B The entire small intestine contains valvulae conniventes, also known as plicae circularis. The jejunum has larger vasa recta, a larger diameter, and a less fatty mesentery. The ileum contains prominent lymphoid follicles called Peyer patches. (See Schwartz 11th ed., p. 1220.)

3. A pocket- or sock-like outpouching on the anti-­ mesenteric side of the distal ileum, called a Meckel diverticulum, is caused by: A. Excessive traction on the intestine during childbirth. B. Increased intraluminal pressure. C. A persistent vitelline duct. D. A mutation of the c-Mec gene.

Answer: C The embryonic gut communicates with the yolk sac by mean of the vitelline duct. Failure of this structure to obliterate by the end of gestation can result in a Meckel diverticulum. (See Schwartz 11th ed., pp. 1246–1247.)

4. The most common presentations of symptomatic Meckel diverticuli include all of the following EXCEPT: A. Bleeding. B. Intestinal obstruction. C. Inguinal pain. D. Diverticulitis.

Answer: C Bleeding, intestinal obstruction, and diverticulitis are the most common presenting symptoms of Meckel diverticuli. Most are discovered incidentally at surgical procedures. Inguinal pain is not a typical symptom. (See Schwartz ­ 11th ed., p. 1247.)

5. How much fluid normally enters the adult small intestine each day? A. 2 L B. 4 L C. 6 L D. 8 L

Answer: D About 8 to 9 L of fluid enters the small intestine daily, of which over 80% is absorbed. This includes 2 L from oral intake, 1.5 L of saliva, 2.5 L of gastric juice, 1.5 L of biliopancreatic s­ ecretions, and 1 L of fluid secreted by the small intestine. (See Schwartz 11th ed., p. 1222.)

235

Brunicardi_Ch28_p235-240.indd 235

30/06/22 11:24 AM

236

CHAPTER 28 Small Intestine

6. How are the digestion products of carbohydrates, such as glucose, galactose, and fructose, absorbed through the intestine? A. By passive diffusion across enterocyte plasma membranes B. By facilitated diffusion via specific transporters such as SGLT1, GLUT2, and GLUT5 C. By endocytosis of enterocytes on the villus D. By facilitated diffusion through tight junctions between enterocytes

Answer: B The three terminal products of carbohydrate digestion are transported through the enterocyte brush border membrane via facilitative transporter proteins such as the sodiumglucose co-transporter 1 (SGLT1), glucose transporter 2 (GLUT2) and glucose transporter 5 (GLUT5). There is evidence of overexpression of these transporters, particularly SGLT1, in diabetes and obesity, and new therapeutic approaches for these conditions are designed to inhibit these transporters. (See Schwartz 11th ed., p. 1223.)

7. What does the “enterohepatic circulation” refer to? A. The superior mesenteric—portal venous circuit B. The secretion of cholesterol in the bile and its reabsorption in the distal ileum C. The secretion of bile acids by the liver and their reabsorption in the distal ileum D. The secretion of cholecystokinin by the jejunum and its stimulation of bile flow

Answer: C Bile acids act as detergents which increase the solubility of lipid micelles which are taken up by the brush border ­membrane of the jejunum, where >90% of fat is absorbed. The bile acids themselves remain in the intestinal lumen and are reabsorbed in the distal ileum where they enter the portal venous c­ irculation and are resecreted in the bile. (See Schwartz 11th ed., p. 1225.)

8. Components of the gut-associated lymphoid tissue (GALT) include all of the following EXCEPT: A. Plasma cells. B. Stellate cells. C. Dendritic cells. D. CD4+ and CD8+ T cells.

Answer: B The GALT includes Peyer patches, which are microscopic aggregates of B-cell follicles and intervening T-cell areas found in the lamina propria of the small intestine, primarily in the distal ileum. IgA-producing plasma cells and dendritic cells populate these regions and serve to protect against microbes and foreign pathogens. Stellate cells, which produce collagen when activated, are not contained within the GALT. (See Schwartz 11th ed., p. 1226.)

9. The radiologic evaluation of a suspected small bowel obstruction may require use of a contrast agent to confirm the location and possible etiology of the obstruction. Water-soluble contrast agents, such as gastrograffin, are preferable to barium for all of the following reasons EXCEPT: A. Gastrograffin is safer than barium if a perforation has occurred. B. Gastrograffin may facilitate the resolution of a partial small bowel obstruction. C. Gastrograffin small bowel studies are easier to perform. D. Gastrograffin may detect subtle mucosal abnormalities more sensitively than barium.

Answer: C Water-soluble contrast agents, such as gastrograffin, are less likely to provoke peritonitis if a perforation has occurred and may be more sensitive in the detection of mucosal abnormalities. Gastrograffin has also been shown to be therapeutic for the treatment of partial small bowel obstruction, but gastrograffin studies take longer and are more labor-intensive to perform. (See Schwartz 11th ed., pp. 1229–1231.)

10. A closed loop obstruction is particularly dangerous because: A. Intraluminal pressure rises high enough to cause ischemia and necrosis. B. The obstruction is painless. C. Bacterial overgrowth results in sepsis. D. The obstructive segment is not apparent on imaging studies.

Answer: A A closed loop obstruction, in which an intestinal segment is obstructed both proximally and distally, as in a volvulus, is particularly dangerous because intraluminal pressure rises quickly and can cause venous congestion and arterial obstruction which leads to necrosis of the intestinal wall and perforation. It classically presents with “pain out of proportion to the physical exam,” and is usually apparent on CT scan which frequently shows a U-shaped or C- shaped dilated bowel loop associated with a radial distribution of mesenteric vessels converging toward a torsion point. (See Schwartz 11th ed., p. 1229.)

Brunicardi_Ch28_p235-240.indd 236

30/06/22 11:24 AM

237

12. Laparoscopic surgery for bowel obstruction may be preferable to open surgery for all of the following reasons EXCEPT: A. Laparoscopic surgery is associated with fewer wound infections and a shorter length of hospital stay. B. Laparoscopic surgery is preferable in cases where a single adhesive band is causing the obstruction. C. Laparoscopic surgery is less likely to produce further adhesion formation which may cause recurrent obstruction. D. The risk of iatrogenic bowel injury is higher with laparoscopic surgery.

Answer: D The use of laparoscopic surgery for small bowel obstruction has been found to be highly successful when the obstruction is caused by a single adhesive band. The avoidance of an open procedure also reduces the risk of subsequent adhesion formation. The morbidity risks of laparoscopic surgery are less than with open surgery and the risk of iatrogenic injury to the bowel is similar. (See Schwartz 11th ed., p. 1232.)

13. Crohn’s disease affects the small intestine in 80% of cases, and the colon alone in 20%. Isolated perineal and anorectal disease occurs in 5% to 10% of patients. In addition, extraintestinal manifestations of Crohn’s disease occur in up to 25% of patients and may include which of the following? A. Erythema nodosum B. Interstitial lung disease C. Nephrolithiasis D. All of the above

Answer: D Extraintestinal manifestations of Crohn’s disease are diffuse and can involve dermatologic, rheumatologic, ocular, hepatobiliary, urologic, and pulmonary conditions. (See Schwartz 11th ed., p. 1237.)

14. The primary genetic defect associated with Crohn’s disease is a mutation of the NOD2 gene on chromosome 16. This gene encodes for a protein product which: A. Mediates the innate immune response to microbial pathogens. B. Activates stellate cells to produce collagen. C. Regulates the rate of crypt-to-villus enterocyte migration. D. Mediates the production of enterocyte alkaline phosphatase.

Answer: A The protein product of the NOD2 gene mediates the innate immune response to microbial pathogens. A variety of defects in immune regulatory mechanisms such as over responsiveness of mucosal T cells to enteric flora-derived antigens can lead to defective immune tolerance and sustained inflammation. (See Schwartz 11th ed., p. 1236.)

15. In the resection of a stenotic area of intestine in a patient with Crohn’s disease, the best approach is: A. A resection margin of 2 cm from gross disease. B. A resection margin of 12 cm from gross disease. C. A resection margin of 2 cm from microscopic disease on frozen section. D. A resection margin of 12 cm from microscopic ­disease on frozen section.

Answer: A There are no differences in the recurrence rates for resection with a 2 cm margin or a 12 cm margin from gross disease. The additional bowel lost may contribute to eventual short gut syndrome in a patient who requires multiple resections, so minimizing bowel loss is a priority. There is no benefit to achieving frozen section negative margins in the resection of Crohn’s strictures; positive margin resections have the same recurrence rate as negative margin resections. The effort to obtain a frozen section negative margin carries the risk of removing more intestine than is necessary. (See Schwartz 11th ed., p. 1239.)

Brunicardi_Ch28_p235-240.indd 237

Small Intestine

Answer: D Partial small bowel obstruction and early postoperative obstruction can mimic ileus and may respond to nonoperative therapy. Crohn disease usually responds to medical therapy, although recurrent obstruction is an indication for surgical correction. Obstruction due to an internal hernia requires prompt surgical intervention to avoid strangulation and necrosis. (See Schwartz 11th ed., p. 1231.)

CHAPTER 28

11. Therapy of a small bowel obstruction usually consists of prompt surgical correction. In patients with no evidence of closed loop obstruction, and in whom there is no fever or leukocytosis or tachycardia, a period of careful observation with nasogastric decompression may be successful in all of the following conditions EXCEPT: A. Partial small bowel obstruction. B. Obstruction in the early postoperative period. C. Obstruction due to Crohn disease. D. Obstruction due to an internal hernia.

30/06/22 11:24 AM

238

CHAPTER 28 Small Intestine

16. The failure of an enterocutaneous fistula to heal on a regimen of total parenteral nutrition and anti-secretory therapy may be due to which of the following? A. A foreign body in the fistula tract B. Epithelialization of the fistula tract C. Down-stream obstruction of the fistulized segment of intestine D. All of the above

Answer: D Factors which prevent healing of an entero-cutaneous fistula include foreign body, epithelialization of the fistula tract, down-stream obstruction, radiation enteritis, associated infection (abscess or sepsis), malignancy, and a short (95%) of GISTs express CD117 (KIT), and as such, are sensitive to tyrosine kinase inhibitors (TKIs), such as imatinib mesylate and sunitinib malate. Risk stratification is based on tumor size and mitotic activity, and 30% to 50% are malignant. Although small GISTs may be asymptomatic and discovered incidentally, larger lesions can cause bleeding, obstruction, or abdominal pain. Treatment of choice is surgical resection (either local excision or radical resection) with microscopically negative margins, if possible; however, local recurrence is common. For larger marginally resectable tumors, TKIs (imatinib) can be used to shrink the tumor. These agents can also be considered for adjuvant therapy after resection and are useful for treating metastatic disease. (See Schwartz 11th ed., p. 1303.)

Brunicardi_Ch29_p241-258.indd 249

Colon, Rectum, and Anus

Answer: C Total mesorectal excision (TME) is a technique that uses sharp dissection along anatomic planes to ensure complete resection of the rectal mesentery during low and extended low anterior resections. For upper rectal or rectosigmoid resections, a partial mesorectal excision of at least 5 cm distal to the tumor appears adequate. TME both decreases local recurrence rates and improves long-term survival rates. Moreover, this technique is associated with less blood loss and less risk to the pelvic nerves and presacral plexus than is blunt dissection. The principles of TME should be applied to all radical resections for rectal cancer. (See Schwartz 11th ed., p. 1300.)

Chapter 29

14. Which of the following statements about total mesorectal excision (TME) for rectal cancer is TRUE? A. TME improves local recurrence but not long-term survival. B. TME is not necessary for rectosigmoid tumors. C. TME is associated with less blood loss than a blunt dissection. D. TME carries a higher risk of injury to the pelvic nerves than a blunt dissection.

30/06/22 11:25 AM

250 Answer: A Melanoma. Anorectal melanoma is rare, comprising 50, are also concerning for malignancy. When such findings are identified, the patient should have their gallbladder removed, even if they are asymptomatic. Polyps that are not removed should be monitored on serial imaging. (See Schwartz 11th ed., p. 1421.)

25. A 55-year-old man presents with painless jaundice and is found to have a Bismuth-Corlette type II cholangiocarcinoma, which of the following is the next best step? A. Whipple B. Primary resection followed by an hepaticojejunostomy C. Local tumor excision with lymphadenectomy, cholecystectomy, common bile duct (CBD) excision, and bilateral Roux-en-Y hepaticojejunostomies D. Diagnostic laparoscopy

Answer: B Patients suspected of having resectable disease should first undergo diagnostic laparoscopy. Those who are found to have previously unidentified metastatic disease should undergo cholecystectomy and surgical bypass for biliary decompression. For curative resection, the location and local extension of the tumor dictates the extent of the surgery required. Distal bile duct tumors are often resectable but may require pancreaticoduodenectomy (Whipple procedure). For patients

Brunicardi_Ch32_p279-288.indd 286

04/07/22 5:43 PM

287

Brunicardi_Ch32_p279-288.indd 287

Answer: C Most patients with perihilar cholangiocarcinoma present with advanced, unresectable disease. Median survival in this population is between 5 and 8 months. The most common causes of death are hepatic failure and cholangitis. The overall 5-year survival rate for patients with resectable perihilar cholangiocarcinoma is between 10% and 30%, but for patients with negative margins, it may be as high as 40%. The operative mortality for perihilar cholangiocarcinoma is 6% to 8%. Patients with distal cholangiocarcinoma are more likely to have resectable disease and improved prognosis compared to perihilar cholangiocarcinoma. The overall 5-year survival rate for resectable distal disease is 30% to 50%, and the median survival is 32 to 38 months. Patients who receive liver transplantation for cholangiocarcinoma can experience 5-year disease-free survival rates as high as 68%. (See Schwartz 11th ed., p. 1423.)

The Gallbladder and Extrahepatic Biliary System

26. Which of the following scenarios can result in a 5-year disease survival rate as high as 68%? A. Resectable perihilar cholangiocarcinoma B. Resectable distal cholangiocarcinoma C. Cholangiocarcinoma treated with liver transplantation D. None of the above

CHAPTER 32

with distal bile duct cancer found to be unresectable on surgical exploration, Roux-en-Y hepaticojejunostomy, cholecystectomy, and gastrojejunostomy to prevent gastric outlet obstruction should be performed. Perihilar tumors involving the bifurcation or proximal common hepatic duct (BismuthCorlette types I or II) with no signs of vascular involvement are candidates for local tumor excision with portal lymphadenectomy, cholecystectomy, common bile duct excision, and bilateral Roux-en-Y hepaticojejunostomies. If the tumor involves the right or left hepatic duct (Bismuth-Corlette types IIIa or IIIb), right or left hepatic lobectomy, respectively, should also be performed. Frequently, resection of the adjacent caudate lobe is required because of direct extension into caudate biliary radicals or parenchyma. Type IV Klatskin tumors, those with more extensive involvement of both hepatic ducts and intrahepatic spread, are often considered unresectable or only treatable with liver transplantation. (See Schwartz 11th ed.,­ p. 1423.)

04/07/22 5:43 PM

This page intentionally left blank

Brunicardi_Ch32_p279-288.indd 288

04/07/22 5:43 PM

CHAPTER

33

Pancreas

1. From which vessel does the most common anatomic variant of the right (replaced) hepatic artery arise from? A. Left gastric artery B. Aorta C. Superior mesenteric artery D. Gastroduodenal artery

Answer: C In 15% to 20% of patients, the right hepatic artery will arise from the superior mesenteric artery and travel upward toward the liver along the posterior aspect of the head of the pancreas (referred to as a replaced right hepatic artery). It is important to look for this variation on preoperative computed tomographic (CT) scans and in the operating room so the replaced hepatic artery is recognized and injury is avoided. (See Schwartz 11th ed., p. 1433.)

2. According to Ranson’s criteria a 67-year-old female patient suspected of acute pancreatitis presenting to the operating room (OR) with sudden onset of severe abdominal pain, a serum aspartate aminotransferase (AST) > 250 U/dL, a WBC > 16,000/mm3, and a blood glucose > 200 mg/dL would receive a disease classification of: A. Severe. B. Mild, uncomplicated.

Answer: A See Table 33-1. (See Schwartz 11th ed., p. 1445, Table 33-7.) TABLE 33-1

Ranson’s prognostic signs of pancreatitis

Criteria for acute pancreatitis not due to gallstones At admission

During the initial 48 h

  Age > 55 y

  Hematocrit fall > 10 points

  WBC > 16,000/mm

  BUN elevation > 5 mg/dL

  Blood glucose > 200 mg/dL

  Serum calcium < 8 mg/dL

  Serum LDH > 350 IU/L

  Arterial PO2 < 60 mmHg

  Serum AST > 250 U/dL

  Base deficit > 4 mEq/L

3

  Estimated fluid sequestration > 6 L Criteria for acute gallstone pancreatitis At admission

During the initial 48 h

  Age > 70 y

  Hematocrit fall > 10 points

  WBC > 18,000/mm

  BUN elevation > 2 mg/dL

  Blood glucose > 220 mg/dL

  Serum calcium < 8 mg/dL

  Serum LDH > 400 IU/L

  Base deficit > 5 mEq/L

  Serum AST > 250 U/dL

  Estimated fluid sequestration > 4 L

3

Note: Fewer than three positive criteria predict mild, uncomplicated disease, whereas more than six positive criteria predict severe disease with a mortality risk of 50%. Abbreviations: AST = aspartate transaminase; BUN = blood urea nitrogen; LDH = lactate dehydrogenase; PO2 = partial pressure of oxygen; WBC = white blood cell count. Data from Ranson JHC. Etiological and prognostic factors in human acute pancreatitis: a review. Am J Gastroenterol. 1982;77:633 and from Ranson JH, Rifkind KM, Roses DF, et al. Prognostic signs and the role of operative management in acute pancreatitis. Surg Gynecol Obstet. 1974;139:69.

289

Brunicardi_Ch33_p289-296.indd 289

08/07/22 11:50 AM

290

CHAPTER 33 Pancreas

3. In the setting of acute pancreatitis, following aggressive fluid resuscitation, what is the most ideal nutritional approach? A. Patient should undergo strict bowel rest until resolution of inflammation. B. Enteral feeding can be started as early as 24 hours following stabilization. C. Parenteral nutrition is preferred, to be started early after stabilization. D. Tube feeding is inferior to parenteral nutrition during inflammation.

Answer: B In contrast to analgesia and fluid therapy, there is a sound evidence base for nutritional support in acute pancreatitis. It is no longer acceptable to “rest the pancreas” by avoiding enteral nutrition, now the mainstay of nutritional support. Parenteral nutrition is now known to be more expensive, riskier, and not more effective than enteral nutrition and should only be offered if the patient’s calculated nutritional requirements cannot be achieved by the enteral route. Early initiation of enteral nutrition (within the first 24 hours of admission) is not superior to delaying an oral diet until 72 hours. If this is not tolerated over 48 to 72 hours, then nasogastric tube feedings can be started and increased in step-wise fashion over 2 to 3 days. (See Schwartz 11th ed., p. 1446.)

4. What is the most common cause of chronic pancreatitis? A. Hyperparathyroidism B. Alcohol abuse C. Idiopathic D. Genetic

Answer: C There are a variety of chronic pancreatitis etiologies, the most common of which are idiopathic, genetic, heavy drinking, obstructive, hyperlipidemia, hyperparathyroidism, etc. Idiopathic and genetic predispositions together comprise of more than half of the root causes of chronic pancreatitis. A variety of genes have been identified to play a role in chronic pancreatitis, with the cystic fibrosis transmembrane conductance regulator (CFTR) gene being the most prevalent influencer (Fig. 33-1). (See Schwartz 11th ed., p. 1452, Figure 33-17.) Very heavy drinking (15%) Alcohol + genetic (3%)

Idiopathic (42%)

Genetic (24%)

CFTR (14%)

CFTR + SPINK1 (3%) SPINK1 (4%)

Gallstone / severe AP (3%)

Hyperlipidemia, Autoimmune, Obstructive Other (4%) (9%)

PRSS1 (3%)

FIG. 33-1.  Etiologies of chronic pancreatitis. (Reproduced with permission from Whitcomb DC: Going MAD: development of a “matrix academic division” to facilitate translating research to personalized medicine, Acad Med. 2011;86(11):1353–1359.)

5. In patients undergoing endoscopic retrograde cholangiopancreatography (ERCP) for diagnosis and staging of chronic pancreatitis, the population most at risk of developing procedure-induced pancreatitis are those with: A. Calculus disease. B. Intraductal lesions. C. Sphincter of Oddi dysfunction. D. High percentage of parenchymal calcification.

Brunicardi_Ch33_p289-296.indd 290

Answer: C For the diagnosis and staging of chronic pancreatitis, ERCP is considered to be the gold standard. It also serves as a vehicle that enables other diagnostic and therapeutic maneuvers, such as biopsy or brushing for cytology, or the use of stents to relieve obstruction or drain a pseudocyst. Unfortunately, ERCP also carries a risk of procedure-induced pancreatitis that occurs in approximately 5% of patients. Patients at increased risk include those with sphincter of Oddi

08/07/22 11:50 AM

291 dysfunction and those with a previous history of post-ERCP pancreatitis. Post-ERCP pancreatitis occurs after uncomplicated procedures, as well as after those that require prolonged manipulation. Severe pancreatitis and deaths have occurred after ERCP. It should be reserved for patients in whom the diagnosis is unclear despite the use of other imaging methods, or in whom a diagnostic or therapeutic maneuver is specifically indicated. (See Schwartz 11th ed., pp. 1459–1560.)

7. What is the most ideal modality for evaluation of chronic pancreatitis? A. Computed tomography (CT) abdomen pelvis B. Magnetic resonance cholengiopancreatography (MRCP) C. Endoscopic retrograde cholangiopancreatography (ERCP) D. Endoscopic ultrasonography (EUS)

Answer: D EUS has heavily impacted the evaluation and management of patients with chronic pancreatitis. Although it is more operator-dependent than transabdominal ultrasonography, EUS provides not only imaging capability but also adds the capacity to obtain cytologic and chemical samples of tissue and fluid aspirated with linear array monitoring (Fig. 33-2). EUS images obtained through a high-frequency (7.5- to 12.5-mHz) transducer are able to evaluate subtle changes in 2- to 3-mm structures within the pancreas and can detect indolent neoplasms in the setting of chronic inflammation. Small intraductal lesions, intraductal mucus, cystic lesions, and subtle ductular abnormalities are recognizable by EUS (Table 33-2). This allows ERCP to be reserved for these patients who require therapeutic maneuvers, or for the evaluation of more complex problems. EUS is comparable to ERCP in the detection of advanced changes in chronic pancreatitis and may be more sensitive than ERCP in the detection of mild disease. (See Schwartz 11th ed., p. 1457.)

Pancreas

Answer: D Pain from chronic pancreatitis has been ascribed to three possible etiologies. Ductal hypertension, due to strictures or stones, may predispose to pain that is initiated or exacerbated by eating. Chronic pain without exacerbation may be related to parenchymal disease or retroperitoneal inflammation with persistent neural involvement. Acute exacerbations of pain in the setting of chronic pain may be due to acute increases in duct pressure or recurrent episodes of acute inflammation in the setting of chronic parenchymal disease. Nealon and Matin have described these various pain syndromes as being predictive of the response to various surgical procedures. Pain that is found in association with ductal hypertension is most readily relieved by pancreatic duct decompression, through endoscopic stenting or surgical decompression. (See Schwartz 11th ed., p. 1460.)

CHAPTER 33

6. Pain from chronic pancreatitis can be caused by: A. Ductal hypertension. B. Parenchymal disease. C. Obstructive pancreatopathy. D. All of the above.

FIG. 33-2.  Endoscopic ultrasound of chronic pancreatitis. The endoscopic ultrasound appearance of the parenchyma is heterogeneous, and dilated ducts are seen, indicating early obstructive pancreatopathy. (Reproduced with permission from Mark Topazian, Division of Digestive Diseases, Department of Medicine, Mayo Clinic.)

Brunicardi_Ch33_p289-296.indd 291

08/07/22 11:50 AM

292 TABLE 33-2

Endoscopic ultrasound features of chronic pancreatitis

Endoscopic Ultrasound Feature

Implication

Ductal changes

CHAPTER 33

  Duct size > 3 mm

  Ductal dilation

  Tortuous pancreatic duct

  Ductal irregularity

  Intraductal echogenic foci

  Stones or calcification

  Echogenic duct wall

  Ductal fibrosis

  Side-branch ectasia

  Periductal fibrosis

Parenchymal changes

Pancreas

  Inhomogeneous echo pattern

 Edema

  Reduced echogenic foci (1–3 mm)

 Edema

  Enhanced echogenic foci

 Calcifications

  Prominent interlobular septae

 Fibrosis

  Lobular outer gland margin

  Fibrosis, glandular atrophy

  Large, echo-poor cavities (>5 mm)

 Pseudocyst

Reproduced with permission from Catalano MF, Lahoti S, Geenen JE, et al.: P­ rospective evaluation of endoscopic ultrasonography, endoscopic retrograde pancreatography, and secretin test in the diagnosis of chronic pancreatitis, Gastrointest Endosc. 1998;48(1):11-17.

8. Which component of the pancreas is associated with the long-term inflammatory process in chronic pancreatitis, subsequently addressed in surgical resection? A. The head B. The body C. The neck D. The tail

Answer: A The common element of these variations on the theme of LR-LPJ remains the excavation or “coring out” of the central portion of the pancreatic head. It remains uncertain, however, whether and to what degree the dichotomy needs to be extended into the body and tail. The logical conclusion of all of these efforts is that the head of the pancreas is the nidus of the chronic inflammatory process in chronic pancreatitis and that removal of the central portion of the head of the gland is the key to the successful resolution of pain in the long term. (See Schwartz 11th ed., p. 1478.)

9. Which of the following is the imaging modality of choice for initial diagnosis of exocrine tumors? A. Computed tomography (CT) with oral and intravenous (IV) contrast B. Positron emission tomography-CT (PET-CT) C. CT with four-phase contrast D. Magnetic resonance imaging (MRI)

Answer: C As with pancreatic exocrine tumors, the initial diagnostic imaging test of choice for pancreatic endocrine tumors is a multidetector CT scan with four phases of contrast and fine cuts through the pancreas and liver. Neuroendocrine tumors of the pancreas often enhance with contrast. EUS can be superior to CT in localizing these tumors, which can produce dramatic symptoms despite their small (500 pg/mL. Glucagon is a catabolic hormone, and most patients present with malnutrition. The rash associated with glucagonoma is thought to be caused by low levels of amino acids. Preoperative treatment usually includes control of the diabetes, parenteral ­nutrition, and octreotide. Like VIPomas, glucagonomas are more often in the body and tail of the pancreas and tend to be large tumors with metastases. Again, debulking operations are recommended in good operative candidates to relieve ­symptoms. (See Schwartz 11th ed., p. 1483.)

13. Which of the following have been proven to significantly reduce the rate of pancreatic leak following pancreaticoduodenectomy? A. Stent B. Glue C. Octreotide D. None of the above

Answer: D Considerable attention has been focused on the prevention of pancreatic leak after pancreas resection. Modifications of the anastomotic technique (end-to-side or end-to-end, ductto-mucosa, or invaginated), the use of jejunum or the stomach for drainage, the use of pancreatic duct stents, the use of octreotide, and various sealants have all been evaluated. Long-acting synthetic analogues of somatostatin have been evaluated as a pharmacologic therapy to reduce pancreatic secretion and the rate of pancreatic fistula after pancreatic resection. Some European studies supported benefit particularly in selected higher risk patients, while previous North

CHAPTER 33

11. Treatment of a 1 cm gastrinoma in the wall of the duodenum is best accomplished by: A. Enucleation. B. Full-thickness resection. C. Duodenectomy. D. Whipple procedure.

Pancreas Brunicardi_Ch33_p289-296.indd 294

08/07/22 11:50 AM

295

Answer: D See Fig. 33-4. (See Schwartz 11th ed., p. 1501, Figure 33-78.)

Pancreas

14. A pancreatic cystic neoplasms that is 9 g/dL D. >10 g/dL

Answer: D For chronic hemolytic anemias, a rise in hemoglobin levels to >10 g/dL without the need for transfusion signifies a successful response to splenectomy. By this criterion, splenectomy has been reported to be successful for the vast majority of patients with chronic hemolytic anemia. For hemolytic anemia due to spherocytosis, the success rate is usually higher, ranging from 90% to 100%. (See Schwartz 11th ed., pp. 1524–1525.)

13. Which of the following is NOT a location where accessory spleens can be found? A. Gastrocolic ligament B. Gerota fascia C. Large bowel mesentery D. Broad ligament

Answer: B The most common anomaly of splenic embryology is the accessory spleen. Present in up to 20% of the population, one or more accessory spleen(s) may occur in up to 30% of patients with hematologic disease. Over 80% of accessory spleens are found in the region of the splenic hilum and vascular pedicle. Other locations for accessory spleens in descending order of frequency are: the gastrocolic ligament, the tail of the pancreas, the greater omentum, the greater curve of the stomach, the splenocolic ligament, the small and large bowel mesentery, the left broad ligament in women, and the left spermatic cord in men. (See Schwartz 11th ed., p. 1519.)

14. Which of the following splenic ligaments is NOT an avascular plane? A. Gastrosplenic B. Splenocolic C. Phrenosplenic D. Splenorenal

Answer: A Of particular clinical relevance, the spleen is suspended in position by several ligaments and peritoneal folds to the colon (splenocolic ligament); the stomach (gastrosplenic ligament); the diaphragm (phrenosplenic ligament); and the kidney, adrenal gland, and tail of the pancreas (splenorenal ligament) (Fig. 34-1). Whereas the gastrosplenic ligament contains the short gastric vessels, the remaining ligaments are usually avascular, with rare exceptions, such as in a patient with portal hypertension. The relationship of the pancreas to the spleen also has important clinical implications. In cadaveric anatomic series, the tail of the pancreas has been demonstrated to lie within 1 cm of the splenic hilum 75% of the time and to actually abut the spleen in 30% of patients. (See Schwartz 11th ed., p. 1519.)

Brunicardi_Ch34_p297-306.indd 300

30/06/22 11:26 AM

301 Gastrosplenic ligament

Phrenicocolic ligament

Sustentaculum lienis

The Spleen

Splenocolic ligament

FIG. 34-1.  Suspensory ligaments of the spleen.

Greater omentum

15. All of the following are functions of the spleen EXCEPT: A. Clearance of damaged or aged red blood cells from the blood. B. Extramedullary site for hematopoesis and recycling iron. C. Initiation of adaptive immune response from filtration of lymph. D. Clearance of encapsulated bacteria from the blood stream.

Answer: C The spleen has both fast and slow circulation of blood. It is during slow circulation that blood travels through the reticular spaces and splenic cords where it is exposed to contact with splenic macrophages, which remove senescent blood cells. Through this process, the spleen is also able to remove erythrocyte inclusions such as Heinz bodies without lysing the cells. Through the reticuloendothelial system, the spleen clears encapsulated bacteria such as pneumococcus and Haemophilus influenzae, which are poorly opsonized from the hepatic reticuloendothelial system. In addition to these functions, the spleen serves as an extramedullary site for hematopoiesis and plays a functional role in the recycling of iron. While the white pulp of the spleen is important in the initiation of the adaptive immune response, material is delivered to the spleen through the blood and not the lymph. (See Schwartz 11th ed., pp. 1520–1521.)

16. Which of the following proteins is not altered in hereditary spherocytosis (HS)? A. Pyruvate kinase B. Spectrin C. Ankyrin D. Band 3 protein

Answer: A The underlying abnormality in HS is an inherited dysfunction or deficiency in one of the erythrocyte membrane proteins (spectrin, ankyrin, band 3 protein, or protein 4.2), which results in destabilization of the membrane lipid bilayer. This destabilization allows a release of lipids from the membrane, causing a reduction in membrane surface area and a lack of deformability, leading to sequestration and destruction of the spherocytic erythrocytes in the spleen. Although less common than glucose-6-phosphate dehydrogenase (G6PD) deficiency overall, pyruvate kinase deficiency is the most common red blood cell enzyme deficiency to cause congenital chronic hemolytic anemia. (See Schwartz 11th ed., p. 1524.)

Brunicardi_Ch34_p297-306.indd 301

CHAPTER 34

Lesser sac

30/06/22 11:26 AM

302 Answer: A The increase in infectious complications associated with splenectomy in thalassemia patients is thought to be due to a coexisting immune deficiency that is caused by iron overload. Iron overload is associated with both thalassemia as well as the transfusions that accompany treatment for thalassemia. Some investigators have tried partial splenectomy with some success in reducing mortality associated with splenectomy in these patients. In addition, splenectomy should be delayed until the patient is older than 4 years of age unless absolutely necessary. While transfusion to maintain a hemoglobin of >9 mg/dL is part of the treatment for thalassemia it does not reduce infectious complications associated with splenectomy in these patients. There is little evidence supporting efficacy of prophylactic antibiotics in asplenic patients in preventing infectious complications associated with splenectomy. (See Schwartz 11th ed., p. 1525.)

18. A 30-year-old woman presents to her primary care provider with complaints of bleeding gums while brushing her teeth as well as menorrhagia and several episodes of epistaxis within the past month. She has been previously healthy with no prior medical problems or surgeries. Examination reveals petechiae and ecchymosis over the lower extremities. Laboratory results show white blood cell count 7000/mm3, hemoglobin 14 g/dL, hematocrit 42%, and platelet count 28,000/mm3 with numerous megakaryocytes on peripheral smear. First-line therapy for this condition would be: A. Oral prednisone B. IV immunoglobulin C. Rituximab D. Splenectomy

Answer: A Idiopathic thrombocytopenic purpura is an autoimmune disorder characterized by a low platelet count and mucocutaneous and petechial bleeding. The usual first line of therapy for idiopathic thrombocytopenic purpura is oral prednisone with most responses occurring within the first 3 weeks after initiating therapy. IV immunoglobulin is given for internal bleeding with platelet counts < 5000/mm3, when extensive purpura exists, or to increase platelets preoperatively and is thought to work by impairing clearance of immunoglobulin G-coated platelets by competing for binding to tissue macrophage receptors. Both rituximab and thrombopoietin receptor antagonists are second-line treatment options. Splenectomy is an option for refractory immune thrombocytopenia (ITP) and can provide a permanent response in about 75% to 85% of patients. (See Schwartz 11th ed., pp. 1526–1527.)

19. The most common physical finding in a patient with hairy cell leukemia (HCL) is: A. Massive splenomegaly. B. Shortness of breath. C. Abdominal pain. D. Joint pain.

Answer: A HCL is an uncommon blood disorder, representing only 2% of all adult leukemias. HCL is characterized by splenomegaly, pancytopenia, and large numbers of abnormal lymphocytes in the bone marrow. These lymphocytes contain irregular hair-like cytoplasmic projections identifiable on the peripheral smear. Most patients seek medical attention because of symptoms related to anemia, neutropenia, thrombocytopenia, or splenomegaly. The most common physical finding is splenomegaly, which occurs in 80% of patients with HCL and is often palpable 5 cm below the costal margin. Many patients with HCL have few symptoms and require no specific therapy. Treatment is indicated for those with moderate to severe symptoms related to cytopenias, such as repeated infections or bleeding episodes, or to splenomegaly, such as pain or early satiety. Splenectomy does not correct the underlying disorder, but does return cell counts to normal in 40% to 70% of patients and alleviates pain and early satiety. Newer chemotherapeutic agents (the purine analogues 2ʹ-deoxycoformycin [2ʹ-DCF] and 2-chlorodeoxyadenosine [2-CdA]) are able to induce durable complete remission in most patients. (See Schwartz 11th ed., p. 1527.)

CHAPTER 34

17. The disproportionately high rate of overwhelming postsplenectomy infection in thalassemia patients is thought to be due to an immune deficiency. Which of the following strategies has been shown to reduce mortality? A. Partial splenectomy B. Prophylactic antibiotic therapy C. Delaying splenectomy until after 2 years of age D. Transfusion to maintain a hemoglobin of >9 mg/dL

The Spleen

Brunicardi_Ch34_p297-306.indd 302

30/06/22 11:26 AM

303

21. Which of the following is an indication for splenectomy in polycythemia vera? A. Failure of aspirin to prevent thrombotic complications B. Frequent need for phlebotomy C. Symptoms related to splenomegaly D. Prevention of progression to myeloid metaplasia

Answer: C Polycythemia vera (PV) is a clonal, chronic, progressive myeloproliferative disorder characterized by an increase in red blood cell mass, frequently accompanied by leukocytosis, thrombocytosis, and splenomegaly. Patients affected by PV typically enjoy prolonged survival compared to others affected by hematologic malignancies, but remain at risk for transformation to myelofibrosis or acute myeloid leukemia (AML). The disease is rare, with an annual incidence of 5 to 17 cases per million population. Although the diagnosis may be discovered by routine screening laboratory tests in asymptomatic individuals, affected patients may present with any number of nonspecific complaints, including headache, dizziness, weakness, pruritus, visual disturbances, excessive sweating, joint symptoms, and weight loss. Physical findings include ruddy cyanosis, conjunctival plethora, hepatomegaly, splenomegaly, and hypertension. The diagnosis is established by an elevated red blood cell mass (>25% of mean predicted value), thrombocytosis, leukocytosis, normal arterial oxygen saturation in the presence of increased red blood cell mass, splenomegaly, low serum erythropoietin (EPO) stores, and bone marrow hypercellularity. Treatment should be tailored to the risk status of the patient and ranges from phlebotomy and aspirin to chemotherapeutic agents. As in essential thrombocythemia (ET), splenectomy is not helpful in the early stages of disease and is best reserved for late-stage patients in whom myeloid metaplasia has developed and splenomegaly-related symptoms are severe. (See Schwartz 11th ed., p. 1529.)

22. Which of the following is the most common etiology of splenic cyst worldwide? A. Bacterial infection B. Trauma C. Parasitic infection D. Congenital anomaly

Answer: C Splenic cysts are rare lesions. The most common etiology for splenic cysts worldwide is parasitic infestation, particularly echinococcal. Symptomatic parasitic cysts are best treated with splenectomy, though selected cases may be amenable to percutaneous aspiration, instillation of protoscolicidal agent, and reaspiration. Nonparasitic cysts most commonly result from trauma and are called pseudocysts; however, dermoid, epidermoid, and epithelial cysts have been reported as well. The treatment of nonparasitic cysts depends on whether or not they produce symptoms. Asymptomatic nonparasitic cysts may be observed with close ultrasound follow-up to exclude significant expansion. Patients should be advised of the risk of cyst rupture with even minor abdominal trauma

Brunicardi_Ch34_p297-306.indd 303

The Spleen

Answer: C CML is a disorder of the primitive pluripotent stem cell in the bone marrow, resulting in a significant increase in erythroid, megakaryotic, and pluripotent progenitors in the peripheral blood smear. The genetic hallmark is a transposition between the bcr gene on chromosome 9 and the abl gene on chromosome 22. CML accounts for 7% to 15% of all leukemias, with an incidence of 1.5 in 100,000 in the United States. CML is frequently asymptomatic in the chronic phase, but symptomatic patients often present with the gradual onset of fatigue, anorexia, sweating, and left upper quadrant pain and early satiety secondary to splenomegaly. Enlargement of the spleen is found in roughly one half of patients with CML. Splenectomy is indicated to ease pain and early satiety. (See Schwartz 11th ed., p. 1528.)

CHAPTER 34

20. Which of the following is an indication for splenectomy in a patient with chronic myelogenous leukemia (CML)? A. Failure of chemotherapy to decrease splenomegaly B. Sequestration requiring transfusion C. Symptomatic relief of early satiety D. Presence of bcr gene mutation

30/06/22 11:26 AM

304 if they elect nonoperative management for large cysts. Small symptomatic nonparasitic cysts may be excised with splenic preservation, and large symptomatic nonparasitic cysts may be unroofed. Both of these operations may be performed laparoscopically. (See Schwartz 11th ed., p. 1530.)

CHAPTER 34

Answer: A Although rare, splenic artery aneurysm (SAA) is the most common visceral artery aneurysm. Women are four times more likely to be affected than men. The aneurysm usually arises in the middle to distal portion of the splenic artery. The risk of rupture is between 3% and 9%; however, once rupture occurs, mortality is substantial (35%–50%). According to a recent series, mortality is significantly higher in patients with underlying portal hypertension (>50%) than in those without it (17%). SAA is particularly worrisome when discovered during pregnancy, as rupture imparts a high risk of mortality to both mother (70%) and fetus (95%). Most patients are asymptomatic and seek medical attention based on an incidental radiographic finding. About 20% of patients with SAA have symptoms of left upper quadrant pain. Indications for treatment include presence of symptoms, pregnancy, intention to become pregnant, and pseudoaneurysms associated with inflammatory processes. For asymptomatic patients, size > 2 cm constitutes an indication for surgery. Aneurysm resection or ligation alone is acceptable for amenable lesions in the mid-splenic artery, but distal lesions in close proximity to the splenic hilum should be treated with concomitant splenectomy. An excellent prognosis follows elective treatment. Splenic artery embolization has been used to treat SAA, but painful splenic infarction and abscess may follow. (See Schwartz 11th ed., p. 1531.)

24. A 45-year-old man presents to the emergency department with emesis of bright red blood. Labs include HGB 10 g/dL, HCT 30%, platelets 300,000/mm3, INR 1.0, AST 30 U/L, ALT 45 U/L, and albumin 4.0 g/dL. After appropriate resuscitation, he undergoes esophagogastroduodenoscopy (EGD) which is notable for gastric varices. What is the appropriate treatment for his condition? A. Transjugular intrahepatic portosystemic shunt B. Variceal band ligation C. Splenorenal shunt D. Splenectomy

Answer: D While portal hypertension is most commonly a result of cirrhosis it can result from other causes such as splenic vein thrombosis. Patients with splenic vein thrombosis can present with bleeding from gastric varices in the setting of normal liver function and test results. These patients also often have a history of pancreatic disease. Portal hypertension secondary to splenic vein thrombosis is potentially curable with splenectomy. (See Schwartz 11th ed., p. 1531.)

25. Which of the following is NOT part of the triad seen with Felty syndrome? A. Rheumatoid arthritis (RA) B. Splenomegaly C. Neutropenia D. Thrombocytopenia

Answer: D The triad of RA, splenomegaly, and neutropenia is called Felty syndrome. It exists in approximately 3% of all patients with RA, two-thirds of which are women. Immune complexes coat the surface of white blood cells, leading to their sequestration and clearance in the spleen with subsequent neutropenia. This neutropenia (1.5 cm C. History of thrombocytopenia D. History of neutropenia

Brunicardi_Ch34_p297-306.indd 304

30/06/22 11:26 AM

305 been excellent, with over 80% of patients showing a durable increase in white blood cell count. More than one-half of patients who had infections prior to surgery did not have any infections after splenectomy. Besides symptomatic neutropenia, other indications for splenectomy include transfusiondependent anemia and profound thrombocytopenia. (See Schwartz 11th ed., p. 1532.)

27. All of the following are TRUE regarding laparoscopic splenectomy EXCEPT: A. It is associated with shorter hospital stays. B. It is associated with increased intraoperative blood loss. C. It is associated with decreased morbidity. D. Patients are positioned in the right lateral decubitus position or the 45o right lateral decubitus position.

Answer: B Laparoscopic splenectomy has become the favored procedure versus open splenectomy for elective splenectomy over the past two decades and is now considered the gold standard for elective splenectomy in patients with normal-sized spleens. With experienced surgeons, laparoscopic splenectomy is associated with decreased intraoperative blood loss, shorter hospital length of stay, and lower morbidity rates as compared to open splenectomy. Laparoscopic splenectomy is often performed with the patient in the right lateral decubitus position; patients are sometimes placed in a 45o right lateral decubitus position to facilitate easier access for concomitant procedures such as laparoscopic cholecystectomy. (See Schwartz 11th ed., p. 1534.)

28. What is the most common complication following open splenectomy? A. Pancreatitis B. Left lower lobe atelectasis C. Pleural effusion D. Wound infection

Answer: B Complications following splenectomy can be divided into pulmonary, hemorrhagic, infectious, pancreatic, and thromboembolic. Pulmonary complications include left lower lobe atelectasis, pleural effusion, and pneumonia with left lower lobe atelectasis being the most common complication overall. Hemorrhagic complications include intraoperative hemorrhage, postoperative hemorrhage, and subphrenic hematoma. Infectious complications include subphrenic abscess and wound infection. Placement of a drain in the left upper quadrant can be associated with postoperative subphrenic abscess and is therefore not routinely recommended. Pancreatic complications include pancreatitis, pseudocyst formation, and pancreatic fistula and often result from intraoperative

Brunicardi_Ch34_p297-306.indd 305

The Spleen

Answer: B Asplenic patients have an increased susceptibility to infection for the remainder of their lives and although the overall lifetime risk of overwhelming postsplenectomy infection (OPSI) is low the consequences can be devastating. Patients undergoing splenectomy for hematologic or malignant indications have a greater risk of OPSI than patients undergoing splenectomy for trauma or iatrogenic injury and OPSI is more common in children than adults. Providers need to have a high index of suspicion when evaluating asplenic patients for possible infection. Patient education and vaccinations against encapsulated pathogens is the mainstay of preventative therapy. Patients should be vaccinated 2 weeks prior to elective splenectomy in order to optimize antigen recognition and processing. If splenectomy is performed emergently, vaccinations are given postoperatively with an attempt to delay administration for 2 weeks to avoid the transient immunosuppression associated with surgery. There is little evidence supporting efficacy of prophylactic antibiotics in asplenic patients and vaccination remains the most effective prevention strategy. (See Schwartz 11th ed., p. 1532.)

CHAPTER 34

26. Which of the following is the most effective prevention strategy against overwhelming postsplenectomy infection? A. Vaccination 2 weeks after splenectomy B. Vaccination 2 weeks before splenectomy C. Daily antibiotic prophylaxis D. Carrying a reserve supply of antibiotics for selfadministration

30/06/22 11:26 AM

306 trauma to the pancreas during dissection of the splenic hilum. Thromboembolic complications include deep vein thrombosis and portal vein thrombosis. (See Schwartz 11th ed., p. 1539.)

CHAPTER 34

Answer: C While the all lifetime risk of OPSI is low (ranging from 5 cm. B. Sentinel node biopsy is a standard treatment. C. Leiomyosarcoma is the most common histology of breast sarcoma. D. Lumpectomy is a standard treatment for T1 breast sarcoma.

Answer: D Angiosarcoma of the breast accounts for about 50% of all sarcomas of the breast and has increasingly been associated with radiation therapy for treatment of primary breast cancer. Complete excision with negative margins is the primary therapy. Simple mastectomy confers no additional benefit if complete excision can be accomplished by segmental mastectomy. Because of low rates of regional lymphatic spread, axillary dissection is not routinely indicated. (See Schwartz 11th ed., p. 1584.)

Brunicardi_Ch36_p311-312.indd 312

30/06/22 11:26 AM

CHAPTER

37

Inguinal Hernias

Answer: B Approximately 75% of abdominal wall hernias occur in the groin. The lifetime risk of inguinal hernia is 27% in men and 3% in women. Of inguinal hernia repairs, 90% are performed in men and 10% in women. The incidence of inguinal hernias in men has a bimodal distribution, with peaks before the first year of age and after age 40. Abramson demonstrated the age dependence of inguinal hernias in 1978. Those age 25 to 34 years had a lifetime prevalence rate of 15%, whereas those age ≥ 75 years had a rate of 47% (Table 37-1). Approximately 70% of femoral hernia repairs are performed in women; however, inguinal hernias are five times more common than femoral hernias. The most common subtype of groin hernia in men and women is the indirect inguinal hernia. (See Schwartz 11th ed., p. 1599.)

1. The incidence of inguinal hernias in men has a bimodal distribution, which peaks before the: A. second year of life and after age 50. B. first year of life and after age 40. C. eighth year of life and after age 40. D. fifth year of life and after age 50.

TABLE 37-1

Inguinal hernia prevalence by age

Age (Y)

25–34

35–44

45–54

55–64

65–74

75+

Current prevalence (%)

12

15

20

26

29

34

Lifetime prevalence (%)

15

19

28

34

40

47

Current = repaired hernias excluded; lifetime = repaired hernias included.

2. According to the Nyhus classification system, which categorizes hernia defects by location, size, and type, type IIIC represents: A. Indirect hernia internal abdominal ring normal; typical in infants, children, small adults. B. Direct hernia; size is not taken into account. C. Recurrent hernia; modifiers A–D are sometimes added, which correspond to indirect, direct, femoral, and mixed. D. Femoral hernia.

Answer: D TABLE 37-2

Nyhus classification system

Type I

Indirect hernia; internal abdominal ring normal; typically in infants, children, small adults

Type II

Indirect hernia; internal ring enlarged without impingement on the floor of the inguinal canal; does not extend to the scrotum

Type IIIA

Direct hernia; size is not taken into account

Type IIIB

Indirect hernia that has enlarged enough to encroach upon the posterior inguinal wall; indirect sliding or scrotal hernias are usually placed in this category because they are commonly associated with extension to the direct space; also includes pantaloon hernias

Type IIIC

Femoral hernia

Type IV

Recurrent hernia; modifiers A–D are sometimes added, which correspond to indirect, direct, femoral, and mixed, respectively. (See Schwartz 11th ed., Table 37-2, p. 1602.)

313

Brunicardi_Ch37_p313-322.indd 313

30/06/22 11:27 AM

314 Answer: D Several studies have documented strenuous physical activity as a risk factor for acquired inguinal hernia. A casecontrolled study of over 1400 male patients with inguinal hernia revealed that a positive family history was associated with an eightfold lifetime incidence of inguinal hernia. Chronic obstructive pulmonary disease also significantly increases the risk of direct inguinal hernias, thought to be due to repeated instances of intra-abdominal pressure during coughing. Several studies have suggested a protective effect of obesity. In a large, population-based prospective study of American individuals (First National Health and Nutrition Examination Survey), the risk of inguinal hernia development in obese men was only 50% that of normal-weight men, while the risk in overweight men was 80% that of nonobese men. A possible explanation is the increased difficulty in detecting inguinal hernias in obese individuals. (See Schwartz 11th ed., Table 37-2, p. 1604.)

4. The high incidence of inguinal hernias in preterm babies is most often due to: A. Failure of the peritoneum to close. B. Familial history. C. Female gender. D. Developmental dysplasia of the hip.

Answer: A Inguinal hernias may be congenital or acquired. Most adult inguinal hernias are considered acquired defects in the abdominal wall although collagen studies have demonstrated a heritable predisposition. A number of studies have attempted to delineate the precise causes of inguinal hernia formation; however, the best-characterized risk factor is weakness in the abdominal wall musculature (Table 37-3). Congenital hernias, which make up the majority of pediatric hernias, can be considered an impedance of normal development, rather than an acquired weakness. During the normal course of development, the testes descend from the intra-abdominal space into the scrotum in the third trimester. Their descent is preceded by the gubernaculum and a diverticulum of peritoneum, which protrudes through the inguinal canal and becomes the processus vaginalis. Between 36 and 40 weeks of gestation, the processus vaginalis closes and eliminates the peritoneal opening at the internal inguinal ring. Failure of the peritoneum to close results in a patent processus vaginalis (PPV), hence the high incidence of indirect inguinal hernias

CHAPTER 37

3. All of the following are TRUE regarding an incidence of acquired inguinal hernias EXCEPT: A. Increases with strenuous activity. B. Increases with family history. C. Increases with chronic obstructive pulmonary disease (COPD). D. Increases with obesity.

Inguinal Hernias

TABLE 37-3

Presumed causes of groin herniation

Coughing Chronic obstructive pulmonary disease Obesity Straining  Constipation  Prostatism Pregnancy Birthweight 1% of primary hernia repair.

Answer: A Injury to spermatic cord structures may result in ischemic orchitis or testicular atrophy. Ischemic orchitis is most commonly caused by injury to the pampiniform plexus and not to the testicular artery. It usually manifests within 1 week of inguinal hernia repair as an enlarged, indurated, and painful testis, and it is almost certainly self-limited. It occurs in 80 g) or airway obstruction appears immanent. C. In patients with demonstrated poor compliance with or tolerance of anti-thyroid medications. D. All of the above.

Answer: D Near-total or total thyroidectomy is now preferred over subtotal thyroidectomy due to a lower recurrence rate. Surgery is preferred over medical therapy (radioactive iodine) in childbearing women who desire to have children in the near future, in non-compliant patients, or when airway obstruction appears likely. (See Schwartz 11th ed., p. 1637.)

9. What is the recommended course of action when fine needle aspiration biopsy (FNAB) of a thyroid nodule is interpreted as “suspicious for malignancy”? A. Repeat FNAB B. Lobectomy C. Lobectomy and isthmusectomy D. Total thyroidectomy

Answer: B Most papillary and other cancers can be diagnosed by FNAB but the features can be subtle and lobectomy or near-total thyroidectomy is recommended because 60% to 75% turn out to be malignant. Follicular neoplasms of the thyroid are less aggressive than papillary neoplasms, and an FNAB may be unable to differentiate between a follicular adenoma and a follicular carcinoma. For this reason unilateral lobectomy is recommended for this FNAB diagnosis. (See Schwartz 11th ed., p. 1644.)

10. Which diseases are associated with germline mutations in the RET tyrosine kinase receptor gene? A. Multiple endocrine neoplasia type 2A (MEN2A) B. Multiple endocrine neoplasia type 2B (MEN2B) C. Hirschsprung Disease D. All of the above

Answer: D Mutations in the extracellular domain of the RET tyrosine kinase receptor are associated with MEN2A, familial medullary thyroid cancer (FMTC), and Hirschsprung disease. Mutations in the intracellular domain are associated with MEN2B, FMTC, and Hirschsprung disease. (See Schwartz 11th ed., pp. 1645–1646.)

11. Children exposed to the Chernoble disaster in 1986 subsequently demonstrated an increased incidence of which thyroid cancer? A. Papillary thyroid cancer (PTC) B. Follicular thyroid cancer (FTC) C. Medullary thyroid cancer (MTC) D. Anaplastic thyroid cancer (ATC)

Answer: A Papillary thyroid cancer accounts for 80% of all thyroid malignancies and is the predominant thyroid cancer in children and individuals exposed to external radiation. (See Schwartz 11th ed., p. 1647.)

Brunicardi_Ch38_p323-328.indd 324

05/07/22 1:32 PM

325

13. An adolescent patient with a thyroid mass undergoes fine needle aspiration biopsy which returns as medullary carcinoma of the thyroid (MCT). What other diseases should be screened for before treatment is undertaken? A. Hyperparathryroidism B. Pheochromocytoma C. Mucocutaneous ganglioneuromas D. All of the above

Answer: D MCT can be spontaneous (in 75%) or familial (in 25%) in multiple endocrine neoplasia syndrome type 2 (MEN2). MEN2A is associated with pheochromocytoma and hyperparathyroidism, whereas MEN2B is associated with pheochromocytoma, Marfanoid habitus, and mucocutaneous ganglioneuromas. (See Schwartz 11th ed., p. 1655.)

14. An asymptomatic child with a normal physical examination is found to harbor a mutation in codon 918 of the RET tyrosine kinase receptor, compatible with MEN2B. Ultrasound of the neck is unremarkable and serum calcitonin levels are normal. What course is indicated? A. Repeat examination and ultrasound yearly B. Planned thyroidectomy in 3 to 5 years C. Total thyroidectomy D. Total thyroidectomy with bilateral neck dissection

Answer: C Children with mutations at codon 634 of the RET tyrosine kinase receptor gene (MEN2A) are advised to undergo thyroidectomy before age 5, whereas children with mutations at codon 918 (MEN2B) should undergo thyroidectomy before age 1. If ultrasound of the neck is normal and calcitonin levels are normal, a formal neck dissection can be avoided. (See Schwartz 11th ed., pp. 1656–1657.)

15. Postoperative complications of thyroid surgery which may require immediate reoperation include: A. Hypocalcemia. B. Dyspnea. C. Dystonia. D. Wound hematoma.

Answer: D Inadvertent injury (ischemia) or removal of the parathyroid glands can cause acute neuromuscular excitability due to hypocalcemia. This can be ameliorated with intravenous calcium. Dyspnea may be a consequence of pain or anxiety but rarely indicates airway compromise. Bilateral vocal cord dysfunction with airway compromise requires reintubation. An expanding hematoma in the neck may not cause bleeding from the wound but can compress the membranous portion of the trachea and cause dyspnea. The wound should be opened at the bedside and the patient returned to the operating room for wound exploration and reclosure. (See Schwartz 11th ed., p. 1663.)

16. A patient with primary hyperparathyroidism undergoes neck exploration where four small, normal appearing glands are found. What are the possible locations of an additional, supernumerary gland? A. In the thyroid gland B. In the thymus C. In the tracheoesophageal groove D. All of the above

Answer: D Supernumerary parathyroid glands occur in 7% to 13% of people, and may be located in the thymus (most commonly), within the parenchyma of the thyroid gland, or in the tracheoesophageal groove, the mediastinum, or elsewhere in the neck. (See Schwartz 11th ed., pp. 1663–1664.)

Brunicardi_Ch38_p323-328.indd 325

Thyroid, Parathyroid, and Adrenal

Answer: D Total thyroidectomy is the recommended treatment particularly in younger patients. Prophylactic neck dissection is not required unless nodal involvement is suspected. Recurrent disease can be treated with radioiodine therapy. (See Schwartz 11th ed., p. 1651.)

Chapter 38

12. The recommended treatment for an otherwise healthy 50-year-old man with a 2 cm follicular thyroid cancer in the left lobe diagnosed by fine needle aspiration biopsy (FNAB) is: A. Left lobectomy. B. Left lobectomy and isthmusectomy. C. Total left lobectomy and subtotal right lobectomy. D. Total thyroidectomy.

05/07/22 1:32 PM

326

Chapter 38 Thyroid, Parathyroid, and Adrenal

17. A 70-year-old woman with early dementia but otherwise good physical health has an elevated parathyroid hormone (PTH) level and a sestamibi scan which localizes a single focus of increased activity to the left lower neck. An ultrasound conforms an enlarged gland in the same area. What treatment is likely to provide the best outcome? A. Bilateral neck exploration under general anesthesia B. Unilateral, “mini-incision” parathyroidectomy under local anesthesia C. Minimally invasive videoscopic parathyroidectomy from a left axillary approach under general anesthesia D. Percutaneous alcohol ablation with ultrasound guidance under local anesthesia

Answer: B Localization studies such as sestamibi scans have been shown to allow more limited operations, including those utilizing “min-incisions” under local anesthesia, for patients who are not good risks for general anesthesia. Improved cosmesis, shorter lengths of stay, and reduced complications are benefits from this approach. (See Schwartz 11th ed., p. 1674.)

18. A 65-year-old woman with mild dementia and elevated calcium and parathyroid hormone levels undergoes bilateral neck dissection. Two upper pole parathyroid glands and one lower pole parathyroid gland are found but despite a diligent search, the fourth gland is not apparent. What steps are appropriate to locate the fourth gland? A. The thyrothymic ligament should be mobilized and the upper end of the thymus gently retracted into the field for dissection and removal if any mass is found. B. The carotid sheath should be opened from the bifurcation to the base of the neck and explored. C. Intraoperative ultrasonography can be employed to examine the thyroid lobe on the side of the missing parathyroid to detect an intrathyroidal location. D. All of the above.

Answer: D The position of the parathyroid glands can be quite variable. In addition to the tracheoesophageal groove adjacent to the posterior capsule of the thyroid, the gland may be intra-­thyroidal, intra-thymic, within the carotid sheath, retroesophageal, or in the posterior mediastinum. (See Schwartz 11th ed., p. 1675.)

19. Gastrointestinal complications of primary hyperparathyroidism include all of the following EXCEPT: A. Peptic ulcer disease. B. Acute pancreatitis. C. Cholelithiasis. D. Hepatitis.

Answer: D Primary hyperparathyroidism has been associated with peptic ulcer disease, both with and without an accompanying gastrinoma. It is sometimes a cause of acute pancreatitis although usually only when serum calcium levels are very high (>12.5 mg/dL). An increased incidence of cholelithiasis is also seen, presumably due to an increase in biliary calcium excretion, but hepatitis is not associated with the disease. (See Schwartz 11th ed., p. 1668.)

20. A 50-year-old man with chronic renal failure develops hypercalcemia (>11 mg/dL) and an elevated parathormone level (>800 pg/mL). Ultrasound and radionuclide studies of the neck reveal no localized mass. On bilateral neck exploration, four variably sized but enlarged parathyroid glands are found. Which of the following surgical options is NOT indicated? A. Total parathyroidectomy B. Subtotal parathyroidectomy with retention of onehalf of the smallest gland C. Total thyroidectomy with reimplantation of 1 mm cubes of one gland into the brachioradialis muscle of the nondominant forearm D. Subtotal parathyroidectomy with retention of onehalf of the smallest gland combined with upper thymectomy

Answer: A Total parathyroidectomy is contraindicated in patients who are candidates for renal transplantation. Subtotal parathyroidectomy with retention of one-half of the smallest gland, with or without upper thymectomy, is considered appropriate, as is total parathyroidectomy with auto-transplantation of a portion of the smallest gland, depending on surgeon experience and preference. Recurrent disease after this procedure can be managed with removal of the auto-transplanted gland avoiding repeat neck exploration. (See Schwartz 11th ed., pp. 1679–1680.)

Brunicardi_Ch38_p323-328.indd 326

05/07/22 1:32 PM

327

22. A 35-year-old woman undergoes an evaluation for infertility. She has gained almost 100 pounds in the past year, is hypertensive, and is borderline diabetic. She also complains of easy bruising. Her serum chemistries are normal with the exception of an elevated glucose. Imaging studies reveal a unilateral adrenal mass. What is the likely diagnosis? A. Secondary hypercortisolism (Cushing’s disease) B. Primary hypercortisolism (Cushing’s syndrome) C. Hyperaldosteronism (Conn’s syndrome) D. Pheochromocytoma

Answer: B Cushing’s syndrome refers to any cause of hypercortisolism caused by either an adrenal source or exogenous administration of steroids. Cushing’s disease refers only to an adrenocorticotropin hormone (ACTH)-secreting adenoma of the pituitary gland. Cushing’s syndrome due to an isolated adrenal adenoma is far less common than hypercortisolism due to a pituitary adenoma, but adrenalectomy is curative for primary adrenal tumors or for adrenal hyperplasia that persists despite efforts to resect a pituitary tumor. (See Schwartz 11th ed., pp. 1688–1689.)

23. All of the following imaging techniques are useful to localize a pheochromocytoma EXCEPT: A. Computed tomography (CT) scan. B. Magnetic resonance imaging (MRI) scan. C. Metaiodobenzylguanidine (MIBG) scan. D. Octreotide scan.

Answer: D Pheochromocytomas are solid tumors which appear on CT scan as soft tissue masses. They are detected with 85% to 95% accuracy, but it is important to avoid intravenous contrast enhancement when a pheochromocytoma is suspected; intravenous contrast can provoke a hypertensive crisis due to release of catecholamines. MRI is useful to identify pheochromocytomas, both because they identify soft tissue masses, but also because this tumor tends to enhance on T2-weighted images. Radio-labeled MIBG is taken up avidly by the pheochromocytoma because its structure is similar to norepinephrine. Therefore, the MIBG scan can localize an occult tumor. Octreotide scans are not used for pheochromocytoma as the tumor does not overexpress somatostatin receptors. (See Schwartz 11th ed., p. 1694.)

24. Pheochromocytomas are associated with all of the following diseases EXCEPT: A. Multiple endocrine neoplasia (MEN) type 2A. B. Multiple endocrine neoplasia (MEN) type 2B. C. Von Hippel Lindau (VHL) disease. D. Familial adenomatous polyposis (FAP) disease.

Answer: D Pheochromocytomas are caused by germline mutations in the RET proto-oncogene (the origin of both MEN 2A and MEN 2B) as well as the VHL gene mutation. Adrenal and extra-adrenal pheochromocytomas may occur spontaneously or as part of a neoplastic syndrome, but they are not associated with the FAP syndrome. (See Schwartz 11th ed., p. 1693.)

25. The preoperative preparation of a patient with pheochromocytoma should include all of the following EXCEPT: A. An alpha-adrenergic blocker such as phentolamine. B. A beta-adrenergic blocker such as propranolol. C. Intravenous hydration to avoid volume depletion. D. Systemic steroids to avoid adrenal insufficiency.

Answer: D The preoperative preparation of a patient with a catecholamine-secreting tumor includes alpha-adrenergic blockade to control hypertension, a beta-adrenergic blocker to prevent tachycardia, and volume replacement to avoid hypotension due to alpha- and beta-blockade. Steroids are not needed to prevent adrenal insufficiency. (See Schwartz 11th ed., pp. 1694–1695.)

Brunicardi_Ch38_p323-328.indd 327

Thyroid, Parathyroid, and Adrenal

Answer: C Primary aldosteronism, or Conn’s syndrome, is seen in about 1% of hypertensive patients. It is more common in middle aged individual and is usually associated with a single adenoma of the adrenal cortex. The hypertension is usually refractory to medical treatment, and is classically associated with hypokalemia, but may be seen in normokalemic individuals. (See Schwartz 11th ed., pp. 1685–1688.)

Chapter 38

21. A 50-year-old, healthy-appearing man undergoes evaluation of persistent hypertension. Serum chemistries reveal hypokalemia (90% of patients. Lesser operations result in unacceptably high recurrence rates, and recurrence is more frequent following infection. According to a recent review, factors predictive of recurrence included more than two infections prior to surgery, age < 2 years, and inadequate initial operation. (See Schwartz 11th ed., p. 1710.)

3. An infant comes to the emergency room with bilious emesis and irritability. Physical examination is notable for abdominal tenderness and erythema of the abdominal wall. Abdominal X-ray demonstrates dilated proximal bowel with air-fluid levels. What is the most appropriate next step in management, after resuscitation? A. Upper gastrointestinal series B. Barium enema C. Gastrostomy D. Laparotomy

Answer: D The cardinal symptom of intestinal obstruction in the newborn is bilious emesis. Prompt recognition and treatment of neonatal intestinal obstruction can truly be life-saving. Bilious vomiting is usually the first sign of volvulus, and all infants with bilious vomiting must be evaluated rapidly to ensure that they do not have intestinal malrotation with volvulus. The child with irritability and bilious emesis should raise particular suspicions for this diagnosis. If left untreated, vascular compromise of the midgut initially causes bloody stools, but eventually results in circulatory collapse. Additional clues to the presence of advanced ischemia of the intestine include erythema and edema of the abdominal wall, which progress to shock and death. It must be reemphasized that the index of suspicion for this condition must be high, since abdominal signs are minimal in the early stages. Abdominal films show a paucity of gas throughout the intestine with a few scattered air-fluid levels. When these findings are present, the patient should undergo immediate fluid resuscitation to ensure adequate perfusion and urine output followed by prompt exploratory laparotomy. (See Schwartz 11th ed., p. 1725.)

4. A 4-week-old full-term, otherwise healthy female is admitted with nonbilious projectile emesis. Ultrasound shows a pyloric channel length of 16 mm and a pyloric thickness of 4 mm. Labs show hypokalemic, hypochloremic metabolic alkalosis. What is the next best step in management? A. Fluid resuscitation with 5% dextrose and 0.45% saline with added potassium until correction of electrolyte abnormalities B. Fluid resuscitation with normal saline until achieving a urine output of >2 mL/kg per hour for 24 hours C. Immediate laparoscopic pyloromyotomy D. Emergent open pyloromyotomy

Answer: A Given frequent fluid and electrolyte abnormalities at time of presentation, pyloric stenosis is never a surgical emergency. Fluid resuscitation with correction of electrolyte abnormalities and metabolic alkalosis is essential prior to induction of general anesthesia for operation. For most infants, fluid containing 5% dextrose and 0.45% saline with added potassium of 2 to 4 mEq/kg over 24 hours at a rate of approximately 150 to 175 mL/kg for 24 hours will correct the underlying deficit. It is important to ensure that the child has an adequate urine output (>2 mL/kg per hour) as further evidence that rehydration has occurred. (See Schwartz 11th ed., p. 1722.)

5. The most common form of esophageal atresia (EA) is: A. Pure EA (no fistula). B. Pure tracheoesophageal fistula (TEF) (no atresia). C. EA with distal TEF. D. EA with proximal TEF.

Answer: C The five major varieties of EA and TEF are shown in Fig. 39-2. The most commonly seen variety is EA with distal TEF (type C), which occurs in approximately 85% of the cases in most series. The next most frequent type is pure EA (type A), occurring in 8% to 10% of patients, followed by TEF without EA (type E). This occurs in 8% of cases and is also referred to as an H-type fistula, based on the anatomic similarity to that letter (Fig. 39-3). EA with fistula between both proximal and distal ends of the esophagus and trachea (type D) is seen in approximately 2% of cases, and type B, EA with TEF between proximal esophagus and trachea, is seen in approximately 1% of all cases. (See Schwartz 11th ed., p. 1717.)

Brunicardi_Ch39_p329-336.indd 330

08/07/22 11:51 AM

331

Chapter 39 Pediatric Surgery

FIG. 39-2.  The five varieties of esophageal atresia and tracheoesophageal fistula. A. Isolated esophageal atresia. B. Esophageal atresia with tracheoesophageal fistula between proximal segment of esophagus and trachea. C. Esophageal atresia with tracheoesophageal fistula between distal esophagus and trachea. D. Esophageal atresia with fistula between both proximal and distal ends of esophagus and trachea. E. Tracheoesophageal fistula without esophageal atresia (H-type fistula).

FIG. 39-3.  Barium esophagram showing H-type tracheoesophageal fistula (arrow).

6. The predicted 4-year survival rate of a child with a Wilms tumor that is confined to one kidney and is grossly excised is: A. 24%. B. 38%. C. 68%. D. 97%.

Brunicardi_Ch39_p329-336.indd 331

Answer: D Following nephroureterectomy for Wilms tumor, the need for chemotherapy and/or radiation therapy is determined by the histology of the tumor and the clinical stage of the patient (Table 39-1). Essentially, patients who have disease confined to one kidney that is completely excised surgically receive a short course of chemotherapy and can expect a 97% 4-year survival, with tumor relapse rare after that time. Patients with more advanced disease or with unfavorable histology receive more intensive chemotherapy and radiation. Even in stage IV, cure rates of 80% are achieved. The survival rates are worse in the small percentage of patients considered to have unfavorable histology. (See Schwartz 11th ed., p. 1747.)

08/07/22 11:51 AM

332 TABLE 39-1

Staging of Wilms tumor

Chapter 39

Stage I: Tumor limited to the kidney and completely excised. Stage II: Tumor that extends beyond the kidney but is completely excised. This includes penetration of the renal capsule, invasion of the soft tissues of the renal sinus, or blood vessels within the nephrectomy specimen outside the renal parenchyma containing tumor. No residual tumor is apparent at or beyond the margins of excision.a Stage III: Residual nonhematogenous tumor confined to the abdomen. Lymph nodes in the abdomen or pelvis contain tumor. Peritoneal contamination by the tumor, such as by spillage or biopsy of tumor before or during surgery. Tumor growth that has penetrated through the peritoneal surface. Implants are found on the peritoneal surfaces. Tumor extends beyond the surgical margins either microscopically or grossly. Tumor is not completely resectable because of local infiltration into vital structures. The tumor was treated with preoperative chemotherapy with or without biopsy. Tumor is removed in greater than one piece. Stage IV: Hematogenous metastases or lymph node involvement outside the abdomino-pelvic region. Stage V: Bilateral renal involvement. International Neuroblastoma Staging System Stage 1: Localized tumor with complete gross resection, with or without microscopic residual disease Stage 2A: Localized tumor with incomplete gross excision; representative ipsilateral nonadherent lymph nodes negative for tumor Stage 2B: Localized tumor with or without complete gross excision, with ipsilateral nonadherent lymph nodes positive for tumor. Enlarged contralateral lymph nodes must be negative microscopically Stage 3: Unresectable unilateral tumor crossing midline, with or without regional lymph node involvement; or localized unilateral tumor with contralateral regional lymph node involvement; or midline tumor Stage 4: Any primary tumor with dissemination to distant lymph nodes, bone, bone marrow, liver, skin, and/or other organs Stage 4S: In infants 25 cm H2O). What is the next best step in management? A. Place the remaining bowel into a plastic silo and perform delayed reduction. B. Proceed with reduction but avoid primary closure. C. Decompress the stomach with an orogastric tube and the rectum with irrigations. D. Proceed with reduction and primary closure.

08/07/22 11:51 AM

336 reduction in many centers. In addition, hydrostatic reduction with barium may be useful if pneumatic reduction is unsuccessful. The overall success rate of radiographic reduction varies based on the experience of the center and is typically between 60% and 90%. (See Schwartz 11th ed., p. 1731.)

Chapter 39 Pediatric Surgery

17. A male infant is born with an imperforate anus. Prior to surgery, a VACTERL workup is performed. Which study is not part of the typical workup? A. Echocardiogram B. Renal ultrasound C. Spinal radiographs D. Brain MRI

Answer: D Approximately 60% of patients with imperforate anus have an associated malformation. The most common is a urinary tract defect, which occurs in approximately 50% of patients. Skeletal defects are also seen, and the sacrum is most commonly involved. Spinal cord anomalies especially tethered cored are common, particularly in children with high lesions. Gastrointestinal anomalies occur, most commonly esophageal atresia. Cardiac anomalies may be noted, and occasionally patients present with a constellation of defects as part of the VACTERL syndrome. (See Schwartz 11th ed., p. 1735.)

18. A newborn female with cystic fibrosis and meconium ileus without perforation has been managed with N-­acetylcysteine (Mucomyst) infused transanally every 12 hours. She remains obstructed and is taken to the operating room for intervention. What is the optimal surgical procedure? A. Enterectomy with ileostomy creation B. Simple enterotomy with lavage C. Ileocecectomy with primary anastomosis D. Colotomy with retrograde lavage

Answer: B If surgical intervention is required because of failure of contrast enemas to relieve obstruction in meconium ileus, operative irrigation with dilute contrast agent, N-acetylcysteine, or saline through a purse-string suture may be successful. Alternatively, resection of the distended terminal ileum is performed, and the meconium pellets are flushed from the distal small bowel. At this point, an end ileostomy may be created. The distal bowel may be brought up as a mucus fistula or sewn to the side of the ileum as a classic Bishop-Koop anastomosis. An end-to-end anastomosis may also be considered in the appropriate setting. (See Schwartz 11th ed., p. 1726.)

19. Standard volume of packed red blood cell transfusion in the newborn infant is: A. 10 mL/kg. B. 5 mL/kg. C. 20 mL/kg. D. 7 mL/kg.

Answer: A A useful guideline for estimating blood volume for the newborn infant is approximately 80 mL/kg of body weight. When packed red blood cells are required, the transfusion requirement is usually administered in 10 mL/kg increments, which is roughly equivalent to a 500-mL transfusion for a 70-kg adult. (See Schwartz 11th ed., p. 1707.)

20. Undescended testes are usually repaired by what age? A. 6 months B. 1 year of age C. 2 years of age D. 4 years of age

Answer: C Males with bilateral undescended testicles are often infertile. When the testicle is not within the scrotum, it is subjected to a higher temperature, resulting in decreased spermatogenesis. Mengel and coworkers studied 515 undescended testicles by histology and demonstrated a decreasing presence of spermatogonia after 2 years of age. Despite orchidopexy, the incidence of infertility is approximately two times higher in men with unilateral orchidopexy compared to men with normal testicular descent. Consequently, it is now recommended that the undescended testicle be surgically repositioned by 1 year of age. (See Schwartz 11th ed., p. 1744.)

Brunicardi_Ch39_p329-336.indd 336

08/07/22 11:51 AM

CHAPTER

40 Urology

1. Which of the following is TRUE about bladder cancer? A. For patients with bladder, cancer invading into the bladder muscle (T2 lesion) immediate radiotherapy followed by surgery offers the best chance of cure. B. Patients with limited lymph node involvement may be cured by surgery alone. C. Continent neobladders have yet to be successfully utilized in patients undergoing cystectomy. D. lntravesical chemotherapy prior to surgery is routinely used for bladder cancers invading into the bladder muscle (T2 lesion).

Answer: B For patients who have disease invading into bladder muscle (T2), immediate (within 3 months of diagnosis) cystectomy with extended lymph node dissection offers the best chance of survival. Patients with limited lymph node involvement may be cured with surgery alone. Patients have multiple reconstructive options, including continent and noncontinent urinary diversions. The orthotopic neobladder has emerged as a popular urinary diversion for patients without urethral involvement. This diversion type involves the detubularization of a segment of bowel, typically distal ileum, which is then refashioned into a pouch that is anastomosed to the proximal urethra (neobladder) or to the skin (continent cutaneous diversion). Patients with nonmuscle-invasive bladder cancer (confined to the bladder mucosa or submucosa) can be managed with transurethral resection alone and adjuvant intravesical (instilled into the bladder) chemotherapy/­ immunotherapy. (See Schwartz 11th ed., pp. 1770–1771.)

2. Which of the following are TRUE concerning testicular cancer? A. Most common malignancy in men aged 15–35. B. Most commonly presents as a painful enlarging mass. C. Initial work up includes chest, abdominal, and brain imaging. D. Most common site of metastases is to the lungs.

Answer: A Testicular cancer is the most common solid malignancy in men age 15 to 35 years. Chest and abdominal imaging must be performed to evaluate for evidence of metastasis. The most common site of spread is the retroperitoneal lymph nodes extending from the common iliac vessels to the renal vessels. (See Schwartz 11th ed., pp. 1771–1772.)

3. Which of the following statements about renal cell carcinoma (RCC) are FALSE? A. Lesions are usually solid but can be cystic. B. May be sporadic or hereditary. C. Surgical debulking can improve survival in patients who present with metastatic disease. D. Patients are not curable (and therefore should not be operated on) if tumor thrombus extends proximally into the vena cava.

Answer: D Renal tumors are usually solid, but they also can be cystic. Most cases of RCC are sporadic, but many hereditary forms have been described. Up to 20% to 30% of patients may present with metastatic disease, in which case, surgical debulking can improve survival, as shown in randomized controlled trials. Up to 10% of RCC invades the lumen of the renal vein or vena cava. The degree of venous extension directly impacts the surgical approach. Patients with thrombus below the level of the liver can be managed with cross-clamping above and below the thrombus and extraction from a cavotomy at the insertion of the renal veins. (See Schwartz 11th ed., pp. 1772–1773.)

337

Brunicardi_Ch40_p337-340.indd 337

30/06/22 11:27 AM

338

CHAPTER 40 Urology

4. Which of the following statements is FALSE concerning carcinoma of the prostate? A. Annual digital rectal examination and serum prostate-specific antigen (PSA) determinations are recommended beginning at age 55. B. Lung metastasis is less common than bone metastasis. C. Radical prostatectomy is associated with a 5% incidence of permanent urinary incontinence D. Once prostate cancer has spread, it is no longer curable but can be contained by lowering serum testosterone and/or by administration of androgen receptor blockers.

Answer: C The American Urological Association has advised screening for men 55 to 69 years of age. The most common site of spread of prostate cancer is to the pelvic lymph nodes and bone. Radical prostatectomy is associated with early incontinence and erectile dysfunction. Incontinence improves significantly with time, with 4 hours unrelated to sexual stimulation. Risk factors include sickle cell disease or trait, malignancy, medications, cocaine abuse, certain antidepressants, and total parenteral nutrition. Low-flow priapism can be confirmed with a penile blood gas of the cavernosal bodies demonstrating hypoxic, acidotic blood. Injection of phenylephrine (up to 200 mg in 20 mL normal saline) into the corporal bodies may be required. (See Schwartz 11th ed., pp. 1769–1770.)

7. Prostatitis is typified by all of the following EXCEPT: A. Pain, dysuria, urgency. B. Fever in both acute and chronic prostatitis. C. Treatment consists of long-term antibiotics. D. Workup may include culture of expressed prostatic secretions.

Answer: B Acute prostatitis is marked by fever, suprapubic or perineal pain, and new onset lower urinary tract symptoms, including dysuria, frequency, urgency, changes in stream caliber, or difficulty emptying the bladder. Treatment consists of a long-term course (4–6 weeks) of antibiotics. Fever is not observed in chronic prostatitis, and onset may occur over many months. Patients with chronic bacterial prostatitis may also report recurrent urinary tract infections (UTIs). Differentiation between acute and chronic etiologies requires culture of expressed prostatic secretion to confirm the presence or absence of bacteria. (See Schwartz 11th ed., p. 1761.)

8. Which of the following statements is correct about urolithiasis? A. Stone formation is associated with diabetes. B. Uric acid stones are the most common type. C. Magnetic resonance imaging (MRI) is the most accurate diagnostic test. D. Lithotripsy is best for stones located in the middle or distal ureter.

Answer: A Stone formation has been associated with obesity, metabolic syndrome, and diabetes mellitus. Stones are most commonly composed of calcium oxalate. A noncontrast computer tomography (CT) scan is the most sensitive (98%) and specific (97%) examination to detect urolithiasis. Ureteroscopy is the procedure of choice for patients with middle or distal ureteral stones. (See Schwartz 11th ed., pp. 1762–1763.)

Brunicardi_Ch40_p337-340.indd 338

30/06/22 11:27 AM

339

10. Which of the following statements is FALSE concerning Fournier’s gangrene (necrotizing fasciitis)? A. Mortality is 25%. B. Risk factors include diabetes, obesity, and alcoholism. C. Often polymicrobial. D. Prompt surgical debridement mandatory.

Answer: A Fournier’s gangrene is a necrotizing fasciitis of the male genitalia and perineum that can be rapidly progressive and fatal if not treated promptly. The mortality rate is as high as 67%. Risk factors for Fournier’s gangrene include perirectal abscesses, diabetes, obesity, and chronic alcoholism. The often polymicrobial infection spreads and can dissect along Scarpa and Colles fascia. Prompt and aggressive surgical debridement of nonviable tissue and broad spectrum antibiotics are necessary to prevent further spread. (See Schwartz 11th ed., p. 1769.)

11. Concerning testicular cancer, which of the following statements are TRUE? A. Second most common cancer in men ages 20–40. B. Nonseminomatous variants are the most common subtype. C. Radical inguinal orchiectomy is the “gold standard” of treatment. D. Workup includes biomarker assays for mutated p53; overexpression of matrix metalloproteinases, and estrogen receptor.

Answer: C Testicular cancer is the most common cancer in men age 20 to 40 years and the second most common cancer in young men age 15 to 19 years. Standard initial workup includes scrotal ultrasound and serum tumors markers (α-fetoprotein, quantitative human chorionic gonadotropin, and lactate dehydrogenase). Radical inguinal orchiectomy is the gold standard treatment for excision of the primary tumor. (See Schwartz 11th ed., pp. 1771–1772.)

12. All of the following statements are TRUE of renal cell carcinoma (RCC), EXCEPT: A. Renal CT imaging with contrast is the definitive radiologic test. B. Risk factors include smoking, obesity, and hypertension. C. Familial syndromes should be suspected in younger patients and in patients with bilateral tumors. D. Renal masses of any size, if suspicious for malignancy, require excision as the first therapeutic maneuver.

Answer: D Most patients diagnosed with RCC in the modern era typically present with an incidentally discovered renal mass on the abdominal radiographic imaging. Renal computed tomography (CT) imaging with intravenous contrast remains the single most important radiographic test to determine the nature of the mass. Major recognized risk factors for RCC include obesity and hypertension. Familial RCC subtypes with classical clinical manifestations are also well described. Management options for small renal masses (90% 5-year overall survival. D. Surgery is indicated for stages III and IV disease.

Answer: C Vaginal carcinoma is a rare gynecologic malignancy and accounts for about 3% of cancers affecting the female reproductive system. Rare clear cell carcinoma of the vagina is associated with in utero exposure to diethylstilbestrol (DES). Vaginal cancer is staged clinically by pelvic examination, chest X-ray, cystoscopy, and proctoscopy. External beam pelvic radiation is the mainstay of treatment for stages II, III, and IV and may be followed by intracavitary and/or interstitial brachytherapy. Prognosis for treated early stage disease is excellent with >90% 5-year survival rates. (See Schwartz 11th ed., pp. 1810–1811.)

Brunicardi_Ch41_p341-344.indd 343

Gynecology

Answer: D Sexually transmitted organisms, especially Neisseria gonorrhoeae and Chlamydia trachomatis, are implicated in many cases, although microorganisms that comprise the vaginal flora (eg, anaerobes, Gardnerella vaginalis, Haemophilus influenza, enteric Gram-negative rods, and Streptococcus agalactiae) have been implicated as well. Because of the psychosocial complexity associated with a diagnosis of PID; when possible, additional criteria should be used to enhance the specificity of the minimum clinical criteria. These include oral temperature >101° F (>38.3° C). Several outpatient parenteral and oral antimicrobial regimens have been effective in achieving clinical and microbiologic cure. Hospitalization for intravenous antibiotics may be necessitated in cases where surgical emergencies cannot be ruled out. (See Schwartz 11th ed., pp. 1803–1804.)

CHAPTER 41

9. Which of the following statements is FALSE concerning pelvic inflammatory disease (PID)? A. Etiology is sexually transmitted organisms. B. Sequellae include increased risk of ectopic pregnancy. C. Initial therapy requires intravenous antibiotics. D. Clinical criteria include an oral temperature >101° F.

30/06/22 11:28 AM

This page intentionally left blank

Brunicardi_Ch41_p341-344.indd 344

30/06/22 11:28 AM

CHAPTER

42

Neurosurgery

1. Which intracranial pathology is associated with the correct sign or symptom? A. Medialization of the uncus and compression of the midbrain leading to uncal herniation B. Increased intracranial pressure (ICP) causing mental status decline due to compression of the bilateral frontal lobes C. Choroid plexus production of cerebral spinal fluid in excess of 1L/day leading to subfalcine herniation D. ICP classically presenting as hypotension, bradycardia, and irregular respirations

Answer: A Increased ICP can injure the brain in several ways. Focal mass lesions cause shift and herniation. Temporal lesions push the uncus medially and compress the midbrain. This phenomenon is known as uncal herniation. Masses higher up in the hemisphere can push the cingulate gyrus under the falx cerebri. This process is known as subfalcine herniation. Diffuse increases in pressure in the cerebral hemispheres can lead to central, or transtentorial, herniation. Increased pressure in the posterior fossa can lead to upward central herniation or downward tonsillar herniation through the foramen magnum. Uncal, transtentorial, and tonsillar herniation can cause direct damage to the brain stem (Fig. 42-1). (See Schwartz 11th ed., Figure 42-4, p. 1831.)

1

3 2

FIG. 42-1.  Schematic drawing of brain herniation patterns. 1. Subfalcine herniation. The cingulate gyrus shifts across midline under the falx cerebri. 2. Uncal herniation. The uncus (medial temporal lobe gyrus) shifts medially and compresses the midbrain and cerebral peduncle. 3. Central transtentorial herniation. The diencephalon and midbrain shift caudally through the tentorial incisura. 4. Tonsillar herniation. The cerebellar tonsil shifts caudally through the foramen magnum. (Reproduced with permission from Wilkins RH, Rengachary SS: Neurosurgery, 2nd ed. New York, NY: McGraw Hill; 1996.)

4

345

Brunicardi_Ch42_p345-350.indd 345

04/07/22 2:47 PM

346 2. The lesion that can cause mass effect and rapidly kill the patient is: A. Inferior fossa lesions. B. Posterior fossa lesions. C. Progressive obtundation. D. Bradycardial lesions.

CHAPTER 42 Neurosurgery

Answer: B The posterior fossa (brain stem and cerebellum) requires special consideration because the volume of the posterior fossa within the cranial vault is small. Posterior fossa lesions such as tumors, hemorrhage, or stroke can cause mass effect that can rapidly kill the patient in two ways. Occlusion of the fourth ventricle can lead to acute obstructive hydrocephalus, raised increased intracranial pressure (ICP), herniation, and eventually death. This mass effect can also lead directly to brain stem compression (Fig. 42-2). Symptoms of brain stem compression include hypertension, agitation, and progressive obtundation, followed rapidly by brain death. A patient exhibiting any of these symptoms needs an emergent neurosurgical evaluation for possible ventriculostomy or suboccipital craniectomy (removal of the bone covering the cerebellum). This situation is especially critical, as expeditious decompression can lead to significant functional recovery. (See Schwartz 11th ed., p. 1833.)

FIG. 42-2.  Maturing cerebellar stroke seen as a hypodense area in the right cerebellar hemisphere (arrowhead) on head computed tomography in a patient with rapidly progressing obtundation 2 days after the initial onset of symptoms. Swelling of the infarcted tissue causes posterior fossa mass effect. The fourth ventricle is obliterated and not visible, and the brain stem is being compressed.

3. Post trauma day 4, a 19-year-old man with significant facial trauma and associated temporal bone articulates which of the following symptoms? A. Double vision and facial asymmetry B. Headache, light sensitivity, and fatigue C. Anosmia and rhinorrhea D. Vertigo, ipsilateral deafness, and facial paralysis

Brunicardi_Ch42_p345-350.indd 346

Answer: D Fractures of the skull base are common in head-injured patients, and they indicate significant impact. They are generally apparent on routine head CT, but they should be evaluated with dedicated fine-slice coronal-section CT scan to document and delineate the extent of the fracture and involved structures. If asymptomatic, they require no treatment. Skull base fractures requiring intervention include those with an associated cranial nerve deficit or cerebrospinal fluid (CSF) leak. A fracture of the temporal bone, for instance, can damage the facial or vestibulocochlear nerve, resulting in vertigo, ipsilateral deafness, or facial paralysis. A communication may be formed between the subarachnoid space and the middle ear, allowing CSF drainage into the pharynx via the Eustachian tube or from the ear (otorrhea). Extravasation of blood results in ecchymosis behind the ear, known as Battle’s sign.

04/07/22 2:47 PM

347 A fracture of the anterior skull base can result in anosmia (loss of smell from damage to the olfactory nerve), CSF drainage from the nose (rhinorrhea), or periorbital ecchymosis, known as raccoon eyes. (See Schwartz 11th ed., p. 1835.)

5. Which of the following closed brain injury is paired with the correct description? A. Contusion, bruise of the brain from breakdown of small blood vessels and extravasation into the brain most commonly affecting the posterior fossa B. Diffuse axonal injury, due to acceleration and deceleration leading to axonal disruption and/or detachment which is not visible on magnetic resonance imaging (MRI) C. Concussion, temporary neuronal dysfunction leading to susceptibility to secondary brain injury D. Traumatic brain injury, diffuse intracranial hypertension with parenchymal hyperemia visible on MRI

Answer: C A concussion is defined as temporary neuronal dysfunction following nonpenetrating head trauma. The head computed tomography (CT) is normal, and deficits resolve over minutes to hours. Definitions vary; some require transient loss of consciousness, while others include patients with any alteration of mental status. Memory difficulties, especially amnesia of the event, are very common. Studies have shown that the brain remains in a hypermetabolic state for up to a week after injury. The brain is also much more susceptible to injury from even minor head trauma in the first 1 to 2 weeks after concussion. This is known as second-impact syndrome, and patients should be informed that, even after mild head injury, they might experience memory difficulties or persistent headaches. Return to play guidelines after sports-related concussions are controversial and are under active debate. (See Schwartz 11th ed., p. 1836.)

6. A patient who withdraws from pain, is mumbling inappropriate words and opens his eyes to pain has a Glasgow Coma Scale score of: A. 3. B. 6. C. 9. D. 12.

Answer: C See Table 42-1. (See Schwartz 11th ed., Table 42-2, p. 1830.)

TABLE 42-1

Neurosurgery

Answer: B Copious clear drainage from the nose or ear makes the diagnosis of CSF leakage obvious. Often, however, the drainage may be discolored with blood or small in volume if some drains into the throat. In indeterminate cases, it is important to consider radiographic findings on the computed tomography (CT) scan near the fracture that suggest CSF leak, such as pneumocephalus, subarachnoid, or intraparenchymal blood at the fracture site. The “halo” test assesses for a double ring when a drop of the fluid is allowed to fall on an absorbent surface, but it has been shown to have poor clinical utility. The fluid can be sent for β-2 transferrin testing, a carbohydratefree isoform of transferrin exclusively found in the CSF; however, these tests often take 1 to 2 weeks to result and also can be difficult to incorporate into clinical practice. (See Schwartz 11th ed., p. 1835.)

CHAPTER 42

4. In regard to the halo test, a positive indicator for cerebrospinal fluid (CSF) is when tinged with blood will show which of the following when dropped on an absorbent tissue? A. A single ring with a darker center spot containing blood components surrounded by a light halo of CSF B. A double ring with a darker center spot containing blood components surrounded by a light halo of CSF C. A single ring with a lighter center spot containing CSF surrounded by a darker halo of blood components D. A double ring with a lighter center spot containing CSF surrounded by a darker halo of blood components

The Glasgow Coma Scale scorea

Motor Response

 

Verbal Response

 

Eye-Opening Response

 

Obeys commands

6

Oriented

5

Opens spontaneously

4

Localizes to pain

5

Confused

4

Opens to speech

3

Withdraws from pain

4

Inappropriate words

3

Opens to pain

2

Flexor posturing

3

Unintelligible sounds

2

No eye opening

1

Extensor posturing

2

No sounds

1

 

 

No movement

1

 

 

 

 

Add the three scores to obtain the Glasgow Coma Scale (GCS) score, which can range from 3 to 15. Add “T” after the GCS if intubated and no verbal score is possible. For these patients, the GCS can range from 3T to 10T. a

Brunicardi_Ch42_p345-350.indd 347

04/07/22 2:47 PM

348 7. The most common malignant tumor of the brain is: A. Ependymoma. B. Astrocytoma. C. Ganglioglioma. D. Teratoma.

CHAPTER 42 Neurosurgery A

Answer: B Astrocytoma is the most common primary central nervous system (CNS) neoplasm. The term glioma often is used to refer to astrocytomas specifically, excluding other glial tumors. Astrocytomas are graded from I to IV. Grades I and II are referred to as low-grade astrocytoma or low-grade glioma, grade III as anaplastic astrocytoma, and grade IV as glioblastoma multiforme (GBM). Prognosis varies significantly between grades I/II, III, and IV, but not between I and II. Median survival is 8 years after diagnosis with a lowgrade tumor, 2 to 3 years with an anaplastic astrocytoma, and roughly 1 year with a GBM. GBMs account for almost twothirds of all astrocytomas, anaplastic astrocytomas account for two-thirds of the rest, and low-grade astrocytomas the remainder. Figure 42-3 demonstrates the typical appearance of a GBM. (See Schwartz 10th ed., p. 1855.)

B

FIG. 42-3.  A. Postcontrast T1-weighted axial magnetic resonance imaging demonstrating a ring-enhancing lesion in the anteromedial right temporal lobe with central necrosis (dark area) consistent with glioblastoma multiforme. B. T2-weighted axial magnetic resonance imaging with extensive bright signal signifying peritumoral edema seen with glioblastoma multiformes.

8. Medical management of traumatic brain injury with an associated subdural hematoma requires which steps to minimize secondary injury including: A. High-dose methylprednisolone to reduce 6-month mortality, especially in severe traumatic brain injury. B. Antiseizure and peptic ulcer prophylaxis as well as blood glucose, vital sign, and temperature monitoring. C. Permissive hypotension to reduce intracranial hematoma growth. D. Prophylactic administration of platelets for patients with intracranial hemorrhage and exposure to aspirin.

Brunicardi_Ch42_p345-350.indd 348

Answer: B Several medical steps may be taken to minimize secondary injury and the systemic consequences of head injury. Patients with a documented closed head injury and evidence of intracranial hemorrhage or a depressed skull fracture should receive a 1 g Keppra loading dose, followed by 1 week of therapeutic maintenance Keppra, typically 500 mg twice a day. Antiseizure prophylaxis has been shown to decrease the incidence of early posttraumatic seizures. There is no evidence to support long-term use of prophylactic antiepileptic agents. Blood glucose levels should be closely monitored by free

04/07/22 2:47 PM

349

TABLE 42-2

Answer: C The cervical nerve roots exit the central canal above the pedicle of the same-numbered vertebra and at the level of the higher adjacent intervertebral disc. For example, the C6 nerve root passes above the C6 pedicle at the level of the C5–C6 discs. The cervical nerve roots may be compressed acutely by disc herniation, or chronically by hypertrophic degenerative changes of the discs, facets, and ligaments. Table 42-2 summarizes the effects of various disc herniations. Most patients with acute disc herniations will improve without surgery. Nonsteroidal anti-inflammatory drugs (NSAIDs) or cervical traction may help alleviate symptoms. Patients whose symptoms do not resolve or who have significant weakness should undergo decompressive surgery. The two main options for nerve root decompression are anterior cervical discectomy and fusion (ACDF) and posterior cervical foraminotomy (keyhole foraminotomy). ACDF allows more direct access to and removal of the pathology (anterior to the nerve root). However, the procedure requires fusion because discectomy causes a collapse of the interbody space and instability will likely occur. Figure 42-4 demonstrates a C6–C7 ACDF with the typical interposed graft and plating system. Keyhole foraminotomy allows for decompression without requiring fusion, but it is less effective for removing centrally located canal pathology. (See Schwartz 11th ed., p. 1862.)

Neurosurgery

9. The most common level of cervical radiculopathy from cervical disc herniation is: A. C4–C5. B. C5–C6. C. C6–C7. D. C7–T1.

CHAPTER 42

blood sugar checks and controlled with sliding scale insulin. Fevers also should be evaluated and controlled with antipyretics, as well as source-directed therapy when possible. Hyperglycemia and hyperthermia are toxic to injured neurons and contribute to secondary injury. Head-injured patients have an increased prevalence of peptic ulceration and gastrointestinal (GI) bleeding. Peptic ulcers occurring in patients with head injury or high increased intracranial pressure (ICP) are referred to as Cushing ulcers. Ulcer prophylaxis should be used. Compression stockings or athrombic pumps should be used when the patient cannot be mobilized rapidly for prophylaxis of deep venous thrombosis. (See Schwartz 11th ed., p. 1839.)

Cervical disc herniations and symptoms by level

Level

Frequency (%)

Root Injured

Reflex

Weakness

Numbness

C4–C5

2

C5



Deltoid

Shoulder

C5–C6

19

C6

Biceps

Biceps brachii

Thumb

C6–C7

69

C7

Triceps

Wrist extensors (wrist drop)

Second and third digits

C7–T1

10

C8



Hand intrinsics

Fourth and fifth digits

Adapted with permission from Greenberg MS. Handbook of Neurosurgery, 7th ed. New York, NY: Thieme; 2010.

Brunicardi_Ch42_p345-350.indd 349

04/07/22 2:47 PM

350

CHAPTER 42 Neurosurgery A

B

FIG. 42-4.  A. Anteroposterior cervical spine X-ray showing the position of an anterior cervical plate used for stabilization after C6–C7 discectomy. Patient presented with right triceps weakness and dysesthesias in the right fifth digit. Magnetic resonance imaging revealed a right paracentral C6–C7 herniated disc compressing the exiting C7 nerve root. B. Lateral cervical spine X-ray of the same patient clearly demonstrates the position of the plate and screws. The allograft bone spacer placed in the drilled-out disc space is also apparent.

10. Urinary retention, saddle anesthesia, and progressing leg weakness are: A. Most often secondary to compression of the thoracic spine secondary to massive disc herniation into the cauda equina. B. A surgical emergency and diagnosis should be made clinically without evaluation of or confirmation by preoperative imaging. C. Due to compression of the lumbar nerve roots, which require urgent intervention to preserve sphincter function and ambulation. D. A diagnosis of exclusion following failure to respond to systemic steroid treatment (steroid challenge).

Brunicardi_Ch42_p345-350.indd 350

Answer: C Cauda equina syndrome is due to compression of the cauda equina and may result from massive disc herniation, epidural hematoma (EDH), epidural abscess, tumor, or subluxation from trauma. Patients with cauda equina compression often present with urinary retention, saddle anesthesia, or progressing leg weakness. Saddle anesthesia is numbness in the perineum, genitals, buttocks, and upper inner thighs. Patients with suspected cauda equina syndrome should undergo immediate magnetic resonance imaging (MRI) of the lumbar spine to evaluate for a surgical lesion. Mass lesions should be removed urgently via laminectomy to preserve sphincter function and ambulation. (See Schwartz 11th ed., p. 1865.)

04/07/22 2:47 PM

CHAPTER

43

Orthopedic Surgery

1. Long bone fractures can be described as any of the ­following recognized types EXCEPT: A. Convoluted. B. Transverse. C. Oblique. D. Spiral.

Answer: A Musculoskeletal injuries resulting from trauma include ­fractures of bones, damage to joints, and injuries to soft tissues. Long bone fractures can be described as transverse, oblique, spiral, segmental, or comminuted. (See Schwartz 11th ed., p. 1881.)

2. Goals of fracture reduction include all of the following EXCEPT: A. Restore length. B. Restore marrow integrity. C. Restore rotation. D. Restore angulation.

Answer: B Reduction is performed with axial traction and reversal of the mechanism of injury in order to restore length, rotation, and angulation. (See Schwartz 11th ed., p. 1881.)

3. Which of the following is TRUE concerning compartment syndromes? A. It is due to decreased intracompartmental pressure. B. It is typified by hyperesthesia. C. It can be assessed by needles placed into affected compartment. D. Pain is relieved by passive muscle stretching.

Answer: C Compartment syndrome is an orthopedic emergency caused by significant swelling within a compartment of an injured extremity that jeopardizes blood flow to the limb. Increased pressure within the compartment compromises perfusion to muscles and can cause ischemia or necrosis. Patients complain of pain and numbness, and passive stretch of muscles within the compartment causes severe pain. While the diagnosis is based on clinical examination, pressures can be measured with needles placed into the compartment, which is necessary in unconscious patients who may not show these typical exam findings. (See Schwartz 11th ed., p. 1883.)

4. Each of the following is associated with pelvic fracture EXCEPT: A. Prompt operative intervention for pubic rami fractures. B. Life-threatening hemorrhage. C. Associated genitourinary (GU) injury. D. Displacement associated with two or more fractures in the pelvic ring.

Answer: A Pelvic fractures are indicative of high-energy trauma and are associated with head, chest, abdominal, and urogenital injuries. Hemorrhage from pelvic trauma can be life-threatening and patients can present with hemodynamic instability requiring significant fluid resuscitation and blood transfusions. The bleeding that occurs is often due to injury of the venous plexus in the posterior pelvis, though it can also be due to a large vessel injury such as a gluteal artery. Other associated injuries are bladder and urethral injuries that manifest with bleeding from the urethral meatus or blood in the bladder catheter, and need to be assessed with a retrograde urethrogram. The pelvis is a ring structure made up of the sacrum and the two innominate bones that are held together by strong ligaments. Because it is a ring, displacement can only occur if the

351

Brunicardi_Ch43_p351-354.indd 351

30/06/22 11:28 AM

352 ring is disrupted in two places. Displaced sacral fractures and iliac wing fractures are treated with screws or plates, while pubic rami fractures can usually be managed nonoperatively. (See Schwartz 11th ed., pp. 1887–1888.)

CHAPTER 43 Orthopedic Surgery

5. Chronic unremitting back pain suggests all of the following possibilities EXCEPT: A. Infection. B. Malignancy (primary). C. Spinal cord infarction. D. Metastatic disease.

Answer: C Back pain occurs in the majority of adults but is usually selflimited, resolving in 1 to 2 weeks. Chronic unremitting back pain suggests the possibility of infection, malignancy, or metastatic disease. (See Schwartz 11th ed., p. 1904.)

6. Concerning long bone fractures, which of the following is TRUE? A. These are classified into four categories (GustilloAnderson classification). B. Treatment of these fractures consists of immediate antibiotics, irrigation, debridement. C. External fixation needed for clean and contaminated wound. D. Distal tibial wounds can be covered by local muscle flaps.

Answer: B An open fracture occurs when the bone breaks through the skin. These injuries are classified into three types according to the Gustillo-Anderson classification. These injuries require immediate administration of antibiotics and irrigation and debridement of the wound. When the wound is contaminated, an external fixator is initially used. Early coverage of the wound is important to avoid infection. Usually a large wound in the proximal or middle third of the tibia can be covered using local muscle flaps, while the distal third of the tibia will require a free flap. (See Schwartz 11th ed., pp. 1881–1883.)

7. Which of the following is FALSE concerning humeral shaft fractures? A. Radial nerve injuries are primarily neuropraxias. B. Management is primarily nonsurgical. C. Gentle motion exercises are begun at 8 weeks. D. Radial nerve injury healing is monitored by ­electromyography (EMG).

Answer: C The majority of humeral shaft fractures can heal with nonsurgical management if they are within an acceptable degree of angulation. The radial nerve spirals around the humeral shaft and is at risk for injury. Most radial nerve injuries are neurapraxias, or stretching of the nerve, and function typically returns within 3 to 4 months. Gentle motion exercises are begun within 1 to 2 weeks. The patient should have an EMG to monitor recovery of the nerve. (See Schwartz 11th ed., pp. 1884–1885.)

8. Concerning knee dislocations, which of the following statements is FALSE? A. Anterior and posterior cruciate ligaments are torn. B. Common peroneal nerve is prone to injury. C. Immediate reduction is critical. D. Normal arterial pulses preclude need for further vascular assessment.

Answer: D When the knee dislocates, the anterior cruciate ligament and posterior cruciate ligament are torn. The neurovascular examination is extremely important, focusing on the common peroneal nerve and the vascular status of the extremity, followed by immediate reduction of the knee and subsequent neurovascular examination. If the pulses are normal, the ankle brachial index (ABI) should be measured. If the ABI is >0.5, the patient should be monitored with serial examination. If the ABI is less than 0.9, then a CT scan or an arterial duplex ultrasound should be performed. (See Schwartz 11th ed., p. 1890.)

9. Concerning shoulder dislocations, which of the following statements is FALSE? A. Association with impingement syndromes. B. Therapy with prolonged immobilization. C. Open stabilization procedures are the “gold standard” of treatment for majority of injuries associated with shoulder dislocation. D. Humeral head fractures common.

Answer: C Prolonged immobilization is not recommended because this will often lead to substantial stiffness in the shoulder and does not appreciably decrease the redislocation rate. Arthroscopic stabilization procedures have been the gold standard treatment for the majority of injuries related to shoulder dislocations. Posterior dislocations of the shoulder are rare and could be missed. This dislocation can occur due to electric shock or seizures. Radiographs are generally obtained to evaluate for concomitant body injuries, such as fractures of the humeral

Brunicardi_Ch43_p351-354.indd 352

30/06/22 11:28 AM

353

10. Management of arthritis includes which of the following? A. Injection B. Osteoplasty C. Arthroclasty D. Arthrotomy

Orthopedic Surgery

Brunicardi_Ch43_p351-354.indd 353

Answer: A Measures to treat arthritis include weight loss, rest, physical therapy, nonsteroidal antiinflammatory drugs (NSAIDs), bracing, and assistive devices such as cane or walker. Joint injections are commonly performed into the knee and shoulder. Common injections include corticosteroids and hyaluronic acid gels. Corticosteroid injections can decrease inflammation within the joint. These injections are usually administered in combination with a local anesthetic, such as lidocaine, in order to provide more immediate relief for both diagnostic and therapeutic purposes. (See Schwartz 11th ed., pp. 1905–1907.)

CHAPTER 43

head. After minor trauma or repetitive injury, patients may experience pain and discomfort, which can be due to irritation of the tissues in the subacromial space. In many cases, such shoulder impingement syndromes are caused by simple bursitis or tendonitis and the long head of the biceps or supraspinatus tendon. Occasionally, impingement syndromes can progress to tears of the supraspinatus tendon, which can be confirmed by magnetic resonance imaging (MRI) or ultrasound. (See Schwartz 11th ed., pp. 1895–1897.)

30/06/22 11:28 AM

This page intentionally left blank

Brunicardi_Ch43_p351-354.indd 354

30/06/22 11:28 AM

44

CHAPTER

Surgery of the Hand and Wrist

Answer: A Three principal nerves serve the forearm, wrist, and hand: the median, radial, and ulnar nerves. The most critical of these from a sensory standpoint is the median nerve. The median nerve begins as a terminal branch of the medial and lateral cords of the brachial plexus. It receives fibers from C5–T1. The ulnar nerve is a terminal branch of the medial cord of the brachial plexus. It receives innervation from C8 and T1 roots. The radial nerve is the larger of two terminal branches of the posterior cord of the brachial plexus. It receives fibers from C5–T1 nerve roots. It innervates all of the extensor muscles of the forearm and wrist. See also Fig. 44-1. (See Schwartz 11th ed., p. 1930.)

1. The median, radial, and ulnar nerves serve the forearm, wrist, and hand. Which of the following statements about these nerves are TRUE? A. Median nerve receives fibers from C5–T1 and travels through the carpal tunnel. B. The radial nerve receives fibers from C5–T1 and innervates the intrinsic muscles of the hand. C. The ulnar nerve receives fibers from C5–T1 and innervates the extrinsic muscles of the hand. D. The ulnar nerve with the median nerve travel through the carpal tunnel.

Ulnar A N

FCR Radial A

TCL

Median N

P 2

APL EPB

S

2

3

4 5

3

4

5

U

H

FPL

RL

EC

C

EC

/E

CR

B

Q

EP

ED

L EIP/EDC

FIG. 44-1.  Cross-section of the wrist at the midcarpal level. The relative geography of the neurologic and tendinous structures can be seen. The transverse carpal ligament (TCL) is the roof of the carpal tunnel, passing volar to the median nerve and long flexor tendons. The TCL is also the floor of the ulnar tunnel, or Guyon’s canal, passing dorsal to the ulnar artery and nerve. The wrist and digital extensor tendons are also seen, distal to their compartments on the distal radius and ulna. Bones: C = capitate; H = hamate; P = pisiform; S = scaphoid. Tendons (flexor digitorum superficialis is volar to flexor digitorum profundus within the carpal tunnel): 2 = index finger; 3 = middle finger; 4 = ring finger; 5 = small finger. A = artery; APL = abductor pollicis longus; ECRB = extensor carpi radialis brevis; ECRL = extensor carpi radialis longus; ECU = extensor carpi ulnaris; EDC = extensor digitorum communis; EDQ = extensor digiti quinti; EIP = extensor indices proprius; EPB = extensor pollicis brevis; EPL = extensor pollicis longus; FCR = flexor carpi radialis; FPL = flexor pollicis longus; N = nerve.

355

Brunicardi_Ch44_p355-360.indd 355

30/06/22 11:28 AM

356

Chapter 44 Surgery of the Hand and Wrist

2. For vascular injuries to the hand requiring tourniquet, the maximum time the tourniquet should be applied to prevent tissue necrosis is: A. 1 hour. B. 2 hours. C. 3 hours. D. 4 hours.

Answer: B Initial treatment for an actively bleeding wound should be direct local pressure for no less than 10 continuous minutes. If this is unsuccessful, an upper extremity tourniquet inflated to 100 mmHg above the systolic pressure should be used. One should keep this tourniquet time to 5% from that of the previous year is considered a criterion of frailty. A standard algorithm is used to calculate physical activity and is corrected for gender. Men who expend 10%–15% in the preceding 6 months may also be an indication for preoperative nutritional restoration. (See Schwartz 11th ed., p. 2051.)

7. Factors which adversely affect the cardiac function of older adults postoperatively include which of the following? A. Depletion of intravascular volume B. Age-related impairment in the response to catecholamines C. Increased myocardial relaxation time D. All of the above

Answer: D Aging has been shown to cause a 1% decrease in cardiac output per year, even in the absence of arrhythmias and cardiac ischemia. The inability to increase cardiac output during stress is dependent on ventricular dilatation and volume status. Incomplete emptying of the ventricle at end systole and reduced ejection fraction leads to impaired coronary perfusion and ischemia. (See Schwartz 11th ed., p. 2051.)

8. Medications which should be discontinued or avoided in the elderly postoperative patient include which of the following? A. Meperidine (Demerol) B. Diphenhydramine (Benadryl) C. Antihistamines D. All of the above

Answer: D Medications with psychotropic effects such as meperidine, diphenhydramine and anticholinergics may contribute to postoperative delirium and should be avoided. (See Schwartz 11th ed., p. 2052.)

9. Which forms of oral intake may be allowed up until 2 hours before general anesthesia? A. Clear liquids B. Light food and full liquids C. Fried and fatty foods D. Cooked meat

Answer: A The American Society of Anesthesiologists issued new guidelines in 2011 that reduced the fasting period prior to general anesthesia. Clear liquids were judged permissible until 2 hours before anesthesia, and light foods and full liquids were permissible until 6 hours before anesthesia. Fried and fatty foods and meats continue to be restricted for 8 hours before anesthesia, but other factors which can affect gastric emptying such as diabetes may require longer periods of preoperative fasting. (See Schwartz 11 ed., p. 2053.)

10. Prophylaxis against deep venous thrombosis has been shown to reduce the incidence of postoperative venous thromboembolism. For older adults undergoing nonorthopedic procedures which of the following regimens is recommended? A. Full-dose heparization B. Low molecular weight heparin and lower extremity compression devices C. Low molecular weight heparin only D. Intracaval filter placement

Answer: B Low molecular weight heparin (LMWH) together with lower extremity compression devices are recommended. Full-dose heparin and intracaval filter placement are accompanied by unnecessary risks and costs. (See Schwartz 11th ed., p. 2053.)

Brunicardi_Ch47_p365-368.indd 366

30/06/22 11:29 AM

367

12. What is the role of surgical palliative care? A. To avoid high-risk operations B. To minimize intervention in patients at high risk of immanent mortality C. To improve symptoms, reduce anxiety, and improve quality of life D. To support patient’s family members and caregivers

Answer: C The role of surgical palliative care is to offer the treatment which will most likely improve symptoms and reduce anxiety. Quality of life issues are paramount, even if for limited periods of time. The purpose of surgical palliative care is to provide the best care for the patient who is compromised by age, disease, or diminished function. (See Schwartz 11th ed., p. 2053.)

13. In frail nursing home residents, what percent of patients are alive 1 year after colon cancer resection? A. 70% B. 50% C. 30% D. 10%

Answer: B Among frail older adults, functional decline after surgery is substantial and sustained. In one study of frail nursing home patients only 53% were alive 1 year after colon resection for malignancy. The same was true for patients who underwent lower extremity bypass. Hip replacement was associated with a 1 year mortality rate of more than one-third of patients. (See Schwartz 11 ed., p. 2054.)

14. Cancer surgery in elderly patients is: A. Always less successful. B. Complicated by the fact that clinical trials usually do not include elderly subjects. C. Recommended equally as in younger patients. D. Does not change overall life expectancy.

Answer: B The frequency of referrals for surgical treatment of equivalent stage cancer is decreased in the elderly for virtually all tumors. Despite this, survival after surgery is nearly equivalent for same stage malignancy as in younger patients. The data derived from clinical trials of adjuvant and neoadjuvant therapy are less helpful for decision making in elderly patients because elderly subjects are usually not included in clinical studies. (See Schwartz 11 ed., p. 2055.)

15. Elderly patients with acute peritonitis may not present with typical symptoms of acute abdominal pain, fever, or leukocytosis due to an impaired immune response. A high index of suspicion is needed as the initial clinical diagnosis in elderly patients with acute appendicitis is correct in what percent of cases? A. Less than 50% B. 70%–80% C. 90%–95% D. 100%

Answer: A In elderly patients with acute appendicitis or acute cholecystitis, one-third lack symptoms of abdominal pain, one-third are afebrile, and one-third have a normal white blood cell count. Therefore an “unimpressive” abdominal examination is irrelevant in the evaluation of the elderly patient whose tolerance for food has suddenly changed. (See Schwartz 11th ed., p. 2055.)

16. In elderly patients undergoing heart valve replacement, bioprosthetic valves are preferred over synthetic valves because: A. There is less need for anticoagulation which is hazardous in the elderly. B. The operative time is shorter which reduces the risk of pulmonary complications. C. Synthetic valves have a higher incidence of manufacturing defects. D. The extent of hemolysis is less with bioprosthetic valves.

Answer: A Prolonged anticoagulation is more hazardous in the elderly where the risk of falls is increased. Even a fall from the standing position can result in a fatal intracranial bleed in an anticoagulated patient. (See Schwartz 11th ed., p. 2056.)

Brunicardi_Ch47_p365-368.indd 367

Surgical Considerations in the Elderly

Answer: D The Michigan Surgical Home and Optimization Program has included preoperative home-based walking; incentive spirometry; and education about nutrition, stress management, and care planning, The program has been found to reduce postoperative complications, length of hospital stay and costs. (See Schwartz 11 ed., p. 2053.)

CHAPTER 47

11. Preoperative prehabilitation programs have been shown to reduce complications in elderly compromised patients. What are the components of such a prehabilitation program? A. An at home-based walking program with daily follow up B. Incentive spirometry training beginning one week prior to surgery C. Education on stress management D. All of the above

30/06/22 11:29 AM

368

CHAPTER 47 Surgical Considerations in the Elderly

17. Transcatheter aortic valve implantation/replacement (TAVI/TAVR) is indicated in which of the following patients? A. Only those with high surgical risk B. Those with minimal aortic stenosis C. Elderly patients with aortic valve disease D. Patients with congenital bicuspid aortic valve disease

Answer: C TAVI/TAVR was initially considered as an alternative to operative replacement of aortic stenosis only in high risk patients. Recent trials have shown that the risks and outcomes are comparable in older low- and medium-risk patients as in younger patients. (See Schwartz 11th ed., p. 2056.)

18. What are the advantages of endovascular repair (EVAR) of an abdominal aortic aneurysm (AAA) in elderly patients? A. A reduction in blood loss and operative time B. A reduction in the need for postoperative intensive care C. The avoidance of abdominal and pulmonary complications associated with open AAA repair D. All of the above

Answer: D Randomized controlled trials of EVAR versus open AAA repair have shown equal or improved survival, fewer complications, and shorter length of stay. The procedure can be performed under epidural anesthesia in patients who are at high risk for general inhalational anesthesia and has comparable or improved outcomes. (See Schwartz 11th ed., p. 2056.)

Brunicardi_Ch47_p365-368.indd 368

30/06/22 11:29 AM

CHAPTER

48

Ethics, Palliative Care, and Care at the End of Life

1. Biomedical ethics is a system of analysis and deliberation which is intended to direct physicians and surgeons to moral “goodness” in patient care. It includes consideration of all of the following EXCEPT: A. Autonomy—the patient’s right to decide for himself/ herself what care will be provided. B. Beneficence—the concept that proposed treatments will benefit the patient. C. Nonmaleficence—the avoidance of treatments which may harm the patient. D. Equipoise—the lack of a preference for one treatment over another.

Answer: D The patient and the doctor decide together what treatment is in the best interest of the patient and share the benefits and the burdens of this joint decision-making. The physician’s role is to clarify the indications, risks, and benefits of the possible treatment courses; the patient’s role is to decide what course to take. Equipoise suggests that neither the patient nor the doctor has a preference, which is not the case. The fourth component of biomedical ethics is justice, meaning fairness where both the benefits and burdens are distributed equally. (See Schwartz 11th ed., p. 2061.)

2. What is implied by the “principalist approach” to medical ethics? A. The principal caregiver determines which course will be followed after discussions with the patient. B. Identify the principles, such as autonomy and beneficence, which appear to be in conflict and evaluate the conflict taking into consideration of values such as self-determination and quality of life. C. The guiding principle of “first do no harm” requires that risks to the patient be minimized. D. The fundamental principle is that the discussion of biomedical ethics needs to involve all members of the care team as well as the patient.

Answer: B In an apparent conflict or conundrum, the relevant principles of autonomy, beneficence, nonmaleficence, and justice are weighed together with the values at stake to determine jointly the best course of action. (See Schwartz 11th ed., p. 2061.)

3. Informed consent contains all of the following elements EXCEPT: A. The physician must document that the patient or the surrogate decision maker has the capacity to make a medical decision. B. The surgeon describes the risks and benefits of treatment options sufficiently for the patient to make an informed decision. C. A member of the patient’s family must witness the consent authorization. D. The patient authorizes a treatment plan without undue influence.

Answer: C Documentation that the patient or the surrogate decision maker has the capacity to make a decision is a primary element. To the extent known, the risks of any treatment options as well as the likelihood of functional recovery need to be explained in detail. In elective situations, patients are allowed time to consult with others before making a decision, but family members are not required to witness the authorization to proceed with care. (See Schwartz 11th ed., p. 2063.)

369

Brunicardi_Ch48_p369-372.indd 369

30/06/22 11:29 AM

370

CHAPTER 48 Ethics, Palliative Care, and Care at the End of Life

4. Living wills are documents which are meant to guide decision-making when: A. The patient’s family cannot be contacted. B. The patient is rendered incompetent or unresponsive by an illness judged to be terminal. C. Multiple attempts at resuscitation have failed. D. The patient’s family disagrees with the course of treatment.

Answer: B Living wills are intended to guide decisions by physicians, family members, and/or surrogate decision makers when the patient himself/herself is unable to render an opinion and the condition or disease is judged to be terminal or “hopeless.” (See Schwartz 11th ed., p. 2064.)

5. “Informed consent” implies all of the following, EXCEPT: A. The patient has been provided with the pertinent details of his/her diagnosis, prognosis, and the options for and risks of treatment. B. The information has been provided according to what a reasonable person would be expected to understand. C. The discussion of the options, risks, and possible hazards has been documented. D. There are witnesses to the discussion who also understand the discussion.

Answer: D The “reasonable person” standard for informed consent has precedent in a 1972 court case which rejected the notion that “simple consent” for treatment was sufficient. The court decided that the facts of diagnosis, treatment options, and risks that a “reasonable person” would want to know should be included in the discussion to obtain consent for treatment. (See Schwartz 11th ed., pp. 2062–2063.)

6. The durable power of attorney for health care is an alternative to “living wills” where a patient designates a specific individual to make decisions if and when the patient is unable to. All of the following statements about the durable power of attorney for health care are true EXCEPT: A. The durable power of attorney has the authority to make decisions regarding the provision or the withdrawal of care. B. The durable power of attorney is expected to provide decisions which are consistent with the patient’s preferences and judgment. C. Decisions made by durable power of attorneys are highly accurate in predicting the patient’s own decision. D. Respecting the decision of the durable power of attorney is a form of respecting the self-determination of the incapacitated patient.

Answer: C Studies indicate that individual identified as the durable power of attorney for health care makes decisions which are consistent with the patient’s own determination only about two-thirds of the time. (See Schwartz 11th ed., p. 2064.)

7. Surgical palliative care can be defined as “the treatment of suffering and the promotion of quality of life for seriously or terminally ill patients under the care of surgeons.” All of the following statements about surgical palliative care are true EXCEPT: A. Surgical intervention is prohibited in patients who are receiving palliative care. B. The lessening of pain is a priority for surgical palliative care. C. Psychosocial support for family members is part of surgical palliative care. D. Bereavement support for the patient care staff is included in surgical palliative care.

Answer: A Surgical interventions are not prohibited if the procedure provides relief of pain or distressing symptoms or otherwise improves the quality of life. Adequate treatment of pain is a fundamental pillar of surgical palliative care, and is also accompanied by psychosocial support for family member and for members of the care team upon the death of a patient. (See Schwartz 11th ed., pp. 2066–2067.)

Brunicardi_Ch48_p369-372.indd 370

30/06/22 11:29 AM

371

Ethics, Palliative Care, and Care at the End of Life

Brunicardi_Ch48_p369-372.indd 371

Answer: C Modification to an existing device or the development of a new device to be used in surgery requires approval of the Office of Product Evaluation and Quality of the Food and Drug Administration. IRB approval alone is not sufficient approval alone to authorize its use. Surgical innovations are part of managing operative challenges but when they are used on more than three patients they become surgical research and require IRB approval. Informed consent of patients is required. (See Schwartz 11th ed., p. 2073.)

CHAPTER 48

8. The distinction between surgical innovation and surgical research can be unclear and requires ethical considerations. All of the following statements about surgical innovation are true EXCEPT: A. A novel or revised part of an operation that a surgeon makes in order to best treat an individual patient is considered surgical innovation. B. An innovation introduced into an operation requires approval of Institutional Review Board (IRB) if it is used on more than three patients. C. An innovative modification of an existing surgical instrument or device can be used in patients with IRB approval. D. When employing an innovative modification to an operative procedure, explicit informed consent of the patient is required.

30/06/22 11:29 AM

This page intentionally left blank

Brunicardi_Ch48_p369-372.indd 372

30/06/22 11:29 AM

CHAPTER

49

Global Surgery

1. A 55-year-old East African woman presents with a palpable mass in her right breast that has been present for several years. You are concerned for cancer and know her case fatality rate is significantly higher than a similar patient in North America. Why? A. Patients present with later stages of cancer. B. Patients often decline screening. C. Cultural prohibitions preclude surgery as an acceptable treatment option. D. Improper treatment of surgically correctable disease.

Answer: A Previously thought to be a disease almost exclusive to highincome countries, nearly two-thirds of the 7.6 million cancer deaths worldwide occur in low- and middle-income countries (LMICs). Mortality from cancer correlates inversely with a country’s economy for certain treatable cancers, including breast, testicular, and cervical cancer—LMICs have higher case fatality rates. Breast cancer case fatality rates illustrate the great disparity in outcomes between regions. Case fatality rates in East Africa reach an unacceptable 59% compared to 19% in the United States. In LMICs, patients have very limited access to screening. They present for care with much later stages of cancer. The number and quality of training programs in surgical oncology are also inversely related to a country’s income, leaving LMICs with few adequately trained providers. Collaborative training programs between highincome countries (HIC) and LMIC centers, as well as teleteaching and mobile consultation, may address this shortage in a relatively low-cost, high-impact way. (See Schwartz 11th Ed., pp. 2082–2083.)

2. When considering surgical care in low- and middleincome countries (LMICs), “task sharing” is one approach that can provide patients with expanded access to surgical care. What does this term refer to? A. Transporting patients from lower to higher resource settings B. Training advanced care practitioners to deliver surgery and anesthesia services C. Establishing a system that allows select surgical disease to be treated medically D. Sharing training materials and tele-teaching between high-income countries (HIC) and LMIC centers

Answer: B Primary care physicians, nurses, midwives, or advanced care practitioners (ACPs) provide much of the basic surgical and anesthetic care in LMICs. Where regulations allow, “task sharing,” or training ACPs to deliver surgery and anesthesia services previously allowed only under the purview of fully trained specialists, can provide expanded access to care. Non-MD practitioners, known as assistant medical officers (AMOs) or tecnicos de cirurgia in Mozambique, often have extensive operative experience, including obstetrical care, and are the primary surgical providers in some regions. Task sharing with ACPs also occurs in the United States and other countries where they fill a need otherwise unmet by specialists even in major tertiary care centers. However, concerns about the quality of care, lack of adequate supervision, and the effect on prestige and professional development for specialists and ACPs, continue to be topics for debate. (See Schwartz 11th Ed., p. 2081.)

373

Brunicardi_Ch49_p373-376.indd 373

30/06/22 11:30 AM

374

CHAPTER 49

Answer: D The World Bank arbitrarily defined U.S. $100 per DALY averted per day in low-income countries as highly cost-effective. Compared to other public health initiatives, developing basic and emergency surgical care at the district level hospital is as cost-effective as, or more so, than typical public health programs such as retroviral treatments for HIV/AIDS or immunization for measles (Fig. 49-1). Male circumcision is one example of a well-documented preventative, minor surgical procedure, capable of reducing the transmission of HIV. The DCP3 adopted a working definition of essential surgical conditions as those that (a) are primarily or extensively treated by surgery; (b) have a large health burden; and (c) can be successfully treated by a surgical procedure that is cost-effective and feasible to promote globally. Using this definition, the DCP3 identified 44 essential procedures, most of which can be performed in first-level hospitals (Table 49-1). The first-level (district) hospital is the appropriate platform to provide essential surgical service. These procedures rank among the most cost-effective of all interventions and include those that treat injuries, obstetric complications (including fistulas), abdominal emergencies, cataracts, and congenital anomalies. (See Schwartz 11th Ed., p. 2086.)

3. You are planning to work abroad but a colleague mentions that “surgery is too expensive” in low- and middleincome countries (LMICs) and you should focus on providing mosquito netting instead. You disagree. Which of the following answers could help you support your case? A. Developing surgical care capabilities can be below the U.S. $100 per disability-adjusted life year (DALY) averted per day threshold for cost-effective care. B. Surgical treatment is often primary prevention for additional disease. C. Essential surgical procedures rank among the most cost-effective of all health interventions. D. All of the above.

Global Surgery

Orthopedic surgery trip Trauma center Cesarean delivery Hydrocephalus repair Trachoma surgery Cleft lip and palate repair Hernia repair Surgical hospital Cataract surgery Obstetric hospital

TABLE 49-1

AL Y /D

AL Y

$1 00 0

D $1 00 /

AL Y /D

$1

FIG. 49-1.  Cost-effectiveness of surgical interventions, compared to two key medical interventions. Note: DALY = disability-adjusted life year. (Illustration reproduced with permission from Intermountain Healthcare.)

$1 0

/D

AL Y

Measles vaccination Antiviral therapy for HIV

Range - Cost/DALY (2012 US$)

Essential surgical procedures that can be performed in first level (district) hospitals (DCP3)

Obstetric Complications Severe postpartum hemorrhage, obstructed labor, prolonged labor, eclampsia, prolapsed cord, fetal distress, tubal pregnancy, postabortion endometritis/myometritis, postabortion sepsis, intrauterine fetal death Trauma and Violence Major limb fracture/injury, joint dislocation, major soft tissue injury, pneumo/hemothorax, ruptured spleen Acute Surgical Emergencies Strangulated hernia, intestinal obstruction, intestinal perforation, appendicitis, liver abscess, major wound infection, osteomyelitis/septic arthritis Nonacute Surgical Conditions Congenital hernia, hernia, breast cancer, chronic osteomyelitis, hydrocele, urethral stricture, prostatic hypertrophy, cataract, eye injury

Brunicardi_Ch49_p373-376.indd 374

30/06/22 11:30 AM

375

FIG. 49-2.  Case fatality rates for severely injured people, 1998. Note: ISS = injury severity score. Income classifications are based on status at the time of the study. Ghana is now a lower-middle income country, and Mexico is an upper-middle-income country. (Illustration reproduced with permission from Intermountain Healthcare.)

5. You are developing a novel device to facilitate virtual laparoscopic skills training for surgeons in LMICs. Which of the following will help innovations such as yours be effective in resource-limited settings? A. Collaborate with a single, local manufacturer B. Utilize qualitatively superior technology to the status quo C. Establish a profit-driven business model D. Employ widely accessible technologies

Brunicardi_Ch49_p373-376.indd 375

Fatalities (percent) (ISS ≥ 9)

80

Kumasi, Ghana

Monterrey, Mexico

60

Global Surgery

Answer: D In 2015, the Lancet Commission on Global Surgery (LCGS) reported that maternal mortality was closely related to density of surgical, anesthetic, and obstetrical providers (SAOPs). They showed that maternal mortality throughout the world appeared to decrease—by 13.1% on average—for every 10 unit increase in SAOPs per 100,000 persons, a strong argument for addressing maldistribution of providers related to surgical disease. High-priority surgical procedures to improve maternal health include cesarean section, hysterectomy for postpartum bleeding and uterine rupture, management of ectopic pregnancy, and dilatation and curettage. In terms of nonobstetrical acute surgical emergencies, about 90% could be addressed by developing the capability to care for the 10 most common acute surgical conditions in any local region. Even common nonacute conditions can have significant impact on quality of life and socioeconomic status, such as cleft lip and cleft palate, hernias, and cataracts. Nearly 2 million lives could be saved each year if LMICs could design and implement simple trauma care initiatives that reduced the case fatality rates among seriously injured patients to equal those in high-income countries (HICs) (Fig. 49-2). Transplantation, though an area of great interest to people in poor countries, still eludes most of the developing world, where cultural and legal prohibitions preclude cadaveric and nonrelated organ donation. (See Schwartz 11th Ed., p. 2095.)

CHAPTER 49

4. Global surgery interventions can be prioritized by targeting diseases that impose the largest burden on a society and have a highly successful surgical outcome. The four broad, high-priority surgical areas that contribute most to the public health mission in low- and middle-income countries (LIMCs) include all of the following EXCEPT: A. Acute surgical emergencies. B. Nonacute surgical conditions. C. Obstetrical emergencies. D. Organ transplantation. E. Trauma care.

Seattle, Washington United States

40 20 0 Low

Middle Country income

High

Answer: D The pressing need for surgical care at all levels and the shortage of fully trained surgeons, anesthesiologists, and support personnel as well as equipment and supplies means that opportunities abound for innovation. Innovation that radically changes the way we do things and that changes a paradigm of a service or system is called “disruptive”; it abruptly changes an older and more expensive system in favor of a less expensive, more widely available technology or process. The ability for disruptive innovations to transform products and services into affordable realities requires three main factors: a sophisticated technology that simplifies, a low-cost business model, and an economically coherent value network (Fig. 49-3). Regulations and standards that vary between countries and locales can facilitate or impede disruptive change. While disruptions often are not qualitatively superior

30/06/22 11:30 AM

376 to the status quo, they make the process both less expensive and more accessible, and through multiple iterations, ultimately improve quality as they cycle through the transformative process. (See Schwartz 11th Ed., p. 2106.)

CHAPTER 49

Sophisticated technology that simplifies

Global Surgery

Regulations and standards that facilitate change Low-cost, innovative business models

Economically coherent value network

FIG. 49-3.  Elements of disruptive innovation. (Illustration reproduced with permission from Intermountain Healthcare.)

6. Patients in rural areas and in countries with poor infrastructure often lack access to safe, timely, and affordable surgical care. Support for Global Surgery has grown significantly in recent years due in part to all of the following EXCEPT: A. Trained surgeons migrating to areas of need. B. Improved control of acute infectious diseases previously the cause of significant morbidity. C. Technology allowing improved access to health care information and training. D. Recognition of the cost-effectiveness of surgery as a public health intervention.

Answer: A Disparities in care and outcomes are multidimensional, and no simple solution exists to improve access to appropriate and affordable surgical care. Yet, five major forces are reshaping priorities and strategies leading the charge for the globalization of surgical care: 1. The epidemiologic transition of diseases from primarily infectious to more chronic conditions 2. The mobile nature of the world’s populations, allowing people to move freely between more isolated areas of the world, leading to a more integrated global community 3. Ubiquitous information access exponentially enabling widespread participation in understanding and designing innovative opportunities for high-quality surgical care 4. A revolution for equity and human rights where the world’s poor are demanding benefits to surgical care similar to those found in high-income countries (HICs) 5. Recognition of the cost-effectiveness of surgical care and its potential to build economies, demonstrating the value of including surgery in global health strategies Trained practitioners commonly migrate to economically and culturally favorable locales, leaving low-resource countries underserved. Investments in training greater numbers of doctors in these countries, including surgical specialists, have been only partially successful in meeting demand in poor countries. Until economic conditions improve or opportunities for professional development increase, and incentives enticing migration of health care workers to HICs abate, it is unlikely that the most skilled practitioners will remain in resource-poor areas beyond their immediate obligations. (See Schwartz 11th Ed., p. 2078.)

Brunicardi_Ch49_p373-376.indd 376

30/06/22 11:30 AM

CHAPTER

50

Optimizing Perioperative Care: Enhanced Recovery and Chinese Medicine 1. The use of preoperative evaluation is associated with which of the following? A. Increased preoperative testing B. Decreased case cancellations C. Decreased time from clinic to surgery D. Decreased patient satisfaction

Answer: B The use of a preoperative evaluation results in identifying patients at elevated respiratory risk, a 55% decrease in preoperative testing, an 88% reduction in case cancellations, reduction in day of surgery delays, reduced total length of stay, a positive impact on hospital finances with cost reduction, and lower in-hospital mortality. (See Schwartz 11th ed., p. 2113.)

2. Preoperative immunonutrition supplements provide greater quantities of which of the following as compared to preoperative standard oral supplements? A. Trace minerals B. Arachidonic acid C. Arginine D. Amino acids

Answer: C Immunonutrition supplements have the addition of arginine to improve immunity and tissue repair and omega-3 fatty acids to mediate the inflammatory response. However, the dosages, duration, and frequency of immunonutrition needed to improve surgical outcomes are unknown. Both types of preoperative nutritional supplementation have been demonstrated to be beneficial, but there is no statistically significant difference in infectious complications, complications, or hospital length of stay between the two types. (See Schwartz 11th ed., p. 2115.)

3. A 65-year-old woman is undergoing an elective lowanterior resection for rectal cancer. Which of the following would not be recommended to counter the effects of insulin resistance perioperatively? A. Preoperative carbohydrate supplementation B. Use of a laparoscopic versus open approach C. Appropriate pain management D. Low glycemic index diet postoperatively

Answer: D Low glycemic index diets are not part of enhanced recovery pathways. Preoperative and postoperative fasting states trigger insulin resistance resulting in a catabolic state with gluconeogenesis and protein breakdown. Preoperative carbohydrate supplementation and early postoperative feeding are both recommended. Hyperglycemia that results from postoperative fasting can be corrected with exogenous insulin, and achievement of normoglycemia in the perioperative period normalizes the main components of metabolism. Both pain and magnitude of surgery are associated with increased insulin resistance; thus, adequate pain control and use of a minimally invasive approach are preferred. When epidural analgesia is added to preoperative carbohydrate supplementation, there is even greater improvement in insulin resistance. (See Schwartz 11th ed., pp. 2116–2117.)

377

Brunicardi_Ch50_p377-380.indd 377

30/06/22 11:30 AM

378

CHAPTER 50 Optimizing Perioperative Care: Enhanced Recovery and Chinese Medicine

4. Which of the following fluids used for perioperative resuscitation has been associated with increased complications? A. Normal saline B. Balanced crystalloids C. Albumin D. Synthetic colloids

Answer: A Normal saline can result in hyperchloremia and has been associated with postoperative morbidity and mortality when compared to balanced crystalloids in studies of cardiac and other surgical patients. A 2013 Cochrane review did not show any difference in outcomes when colloids were compared to crystalloids for fluid resuscitation. Thus, balanced crystalloids tend to be utilized. (See Schwartz 11th ed., p. 2119.)

5. Perioperative fluid management using goal-directed therapy seeks to achieve which of the following? A. Minimal crystalloid use B. Fixed fluid administration C. Zero fluid balance D. Normal cardiac output

Answer: C Goal-directed therapy aims to maintain zero fluid balance coupled with minimal weight gain or loss. Normovolemia is desired to maintain perfusion without volume overload. Goal-directed therapy does not mean that there is minimal fluid or fixed fluid administration. For some procedures, fluid requirements may be more than anticipated and for others, fluid requirements may be less than anticipated. Goaldirected therapy aims to avoid the complications associated with both hypo- and hypervolemia. (See Schwartz 11th ed., pp. 2118–2119.)

6. Which of the following is associated with increased postoperative opioid requirements? A. Intraoperative intravenous opioids B. Intraoperative ketamine infusion C. Neuraxial opioid analgesia D. Peripheral nerve blocks

Answer: A While opioid administration reduces pain immediately after administration, they worsen pain scores after they wear off. In fact, they can increase postoperative opioid requirements. Alternatively, there are non-opioid adjuncts that can be used in multimodal pain regimens to reduce opioid requirements such as non-steroidal anti-inflammatory drugs (NSAIDs), gabapentinoids, acetaminophen, local anesthetics, ketamine, and lidocaine. Neuraxial opioid analgesia can reduce postoperative pain scores, and regional blocks can be opioid sparing. (See Schwartz 11th ed., pp. 2119–2121.)

7. A 59-year-old woman who is planned for a total joint replacement tells her anesthesiologist that she has a history of severe postoperative nausea and vomiting. Which of the following would be associated with a decreased risk of nausea and vomiting? A. General anesthesia B. Total intravenous anesthesia C. Nitrous oxide and volatile agents D. Intravenous fentanyl infusion

Answer: B Strategies for avoiding postoperative nausea and vomiting include the avoidance of general anesthesia, the use of totally intravenous anesthesia, avoidance of nitrous oxide and volatile agents, minimizing intraoperative and postoperative opioids, and adequate hydration. (See Schwartz 11th ed., p. 2121.)

8. Which of the following strategies should be utilized to reduce the incidence of postoperative ileus after a laparoscopic colon resection? A. Placement of a nasogastric tube B. Administration of a mu-opioid receptor antagonist C. Maintenance of hypovolemia D. Use of multimodal pain strategies

Answer: D Multimodal pain strategies and neuraxial blocks reduce opioid use and therefore minimize nausea and improve early enteral nutrition. Routine use of nasogastric tubes is not recommended to prevent ileus; nasogastric tubes actually delay return of gastrointestinal activity. Administration of a perioperative mu-opioid receptor antagonist (alvimopan) has been associated with decreased ileus and length of stay in open not laparoscopic surgeries. Maintenance of normovolemia is desired as both fluid overload and dehydration are associated with negative effects on bowel function. Early resumption of enteral feeding is recommended. (See Schwartz 11th ed., pp. 2121–2122.)

Brunicardi_Ch50_p377-380.indd 378

30/06/22 11:30 AM

379

10. Traditional Chinese medicine techniques such as acupuncture can be used as an adjunct to Western medicine in achieving the goals of enhanced recovery. Which of the following is associated with the use of acupuncture? A. Increased intravenous opioid use B. Decreased anesthetic requirements C. Decreased nausea and vomiting D. Decreased fluid administration

Answer: C Acupuncture can be used to both prevent and treat postoperative nausea and vomiting, and it is associated with improved gastrointestinal function and decreased ileus. It can also be used for perioperative pain management, and there is evidence that it reduces opioid use. There is no evidence to suggest that acupuncture has an effect on anesthetic requirements, fluid administration, or other complications. (See Schwartz 11th ed., pp. 2124–2128.)

Brunicardi_Ch50_p377-380.indd 379

Optimizing Perioperative Care: Enhanced Recovery and Chinese Medicine

Answer: B The first step in setting up an ERAS program is to identify the current state and the goal state of care. After that, stakeholders should be identified and engaged; the stakeholders should work together to develop the protocol and identify facilitators and barriers to implementation of the protocol. Ultimately, development of standardized order sets and education of frontline providers will be necessary to ensure appropriate implementation. (See Schwartz 11th ed., p. 2124.)

CHAPTER 50

9. A surgical quality officer is tasked with setting up an enhanced recovery after surgery (ERAS) program. Which of the following should be her first step? A. Engagement of stakeholders B. Identification of current and goal states C. Development of standardized order sets D. Determination of facilitators and barriers

30/06/22 11:30 AM

This page intentionally left blank

Brunicardi_Ch50_p377-380.indd 380

30/06/22 11:30 AM

CHAPTER

51

Understanding, Evaluating, and Using Evidence for Surgical Practice 1. Of the study designs listed below, which is generally considered to be the strongest level of evidence? A. Systematic review B. Cross-sectional study C. Case-control study D. Animal studies E. Case series

Answer: A Randomized controlled trials (RCTs) are often considered the “gold standard” of evidence; however, RCTs are relatively rare in the surgical literature. In the absence of RCTs, a metaanalysis will likely provide the highest level of evidence for a given subject. Meta-analyses use a standardized method to search for, appraise, and pool published data in order to increase the overall statistical power compared to individual studies. New statistical conclusions can then be made using quantitative methods based on a larger sample size. Similar to meta-analyses, systematic reviews use standardized methods to search for and appraise studies; however, they do not utilize standardized quantitative methods to summarize the results. Therefore, they are generally considered lower quality evidence compared to a meta-analysis. Although the hierarchy of evidence remains largely intact (Fig. 51-1), many newer frameworks of evidence evaluation allow for movement of studies up or down based upon study design and potential unaddressed biases. (See Schwartz 11th ed., pp. 2139–2140.)

RCT Cohort study Case control study Case series Case reports Animal research In-vitro research

FIG. 51-1.  Evidence-based hierarchy.

Expert experience/opinion

381

Brunicardi_Ch51_p381-384.indd 381

04/07/22 2:50 PM

382

CHAPTER 51 Understanding, Evaluating, and Using Evidence for Surgical Practice

2. A type II error occurs when: A. The null hypothesis is rejected but is actually true. B. The null hypothesis is not rejected when it is actually false. C. The null hypothesis is not rejected when it is actually false. D. The incorrect statistical test was performed.

Answer: B A type II error is the failure to reject the null hypothesis when the null hypothesis is false. This error may also be referred to as a false negative. Alternatively, a type I error occurs when the null hypothesis is rejected but is actually true. The type II error is related to the power of the study and may also be referred to as the significance level, often a value of 0.05. This error may also be referred to as a false positive. The type of statistical test is unrelated to type I or II errors. (See Schwartz 11th ed., p. 2147.)

3. Which of the following is a properly constructed patient/ population, intervention, comparison and outcome (PICO) question? A. Does [surgery] provide a [good recovery] in [adults] with [acute appendicitis]? B. In [patients > 18 years old with uncomplicated acute appendicitis] do [antibiotics] compared to [laparoscopic appendectomy] result in fewer [major complications]? C. In [patients] do [antibiotics] work better than [surgery] for [appendicitis]? D. In [adult patients] are [antibiotics] superior for treating [acute appendicitis]?

Answer: B Effective and efficient use of search engines is enhanced by framing the clinical question in a format designed to improve the relevancy of search results. PICO is one such format and stands for: • Patient or population is the specific group of individuals for which the questions is being asked. • Intervention is the treatment or technique of interest for the defined patient or population. Intervention might be a procedure, such as “laparoscopic appendectomy” or be defined as an exposure of interest, such as “smoking.” • Comparison is the alternative treatment or technique to which you are comparing the intervention. Terms might include, for example, “open appendectomy” or “observation.” • Outcome of interest is the final step of the PICO format. Examples include “mortality,” “operative time,” and “wound infection.” Option B is written in the correct order with a well-defined population, intervention, comparison, and outcome. The remaining answers are either out of order, nonspecific, or missing a component of the PICO question. (See Schwartz 11th ed., p. 2139.)

4. A trauma surgeon at your hospital examines data from motor vehicle collisions and determines that there is an association between blood alcohol content and fatalities. This kind of study is an example of a: A. Randomized controlled trial. B. Case series. C. Case-control study. D. Cross-sectional study.

Brunicardi_Ch51_p381-384.indd 382

Answer: C In a case-control study, cohorts are determined by the presence or absence of a particular outcome of interest, in this case motor vehicle collision fatality. Case-control studies are considered to be a lower level of evidence for risk factors, given that they are more susceptible to multiple types of bias than cohort studies. However, well-designed and properly analyzed case-control studies can provide solid evidence, for instance on risk factors for specific conditions. This is in contrast to a cross-sectional study where cohorts are determined by presence or absence of an exposure. A case series involves reporting on a group of patients that share specified clinical features, but generally does not include a control group. (See Schwartz 11th ed., p. 2139.)

04/07/22 2:50 PM

383 Answer: C External validity or generalizability refers to how results of a study translate to “real-world” situations with greater heterogeneity within the potential target population. Involvement of multiple institutions across multiple cities would help increase the chances that a study could be generalized to the population of interest as compared to a study performed at a single institution. In general, external validity is related to the representativeness of the sample population, environment, and procedures to the target population at large. Alternatively, Internal validity describes the degree to which a study’s causal conclusion is warranted. Factors such as randomization, blinding, completeness of follow-up, equivalence among groups, and accuracy of analysis affect the internal validity of a study. (See Schwartz 11th ed., p. 2146.)

CHAPTER 51

5. Which of the following would increase the external validity of a study? A. Surgical interventions in the study are performed by a single surgeon. B. The study has strict inclusion criteria. C. Involvement of multiple institutions across a number of cities. D. Increasing the sample size. E. Performing the study at a single center.

Understanding, Evaluating, and Using Evidence for Surgical Practice

Brunicardi_Ch51_p381-384.indd 383

04/07/22 2:50 PM

This page intentionally left blank

Brunicardi_Ch51_p381-384.indd 384

04/07/22 2:50 PM

CHAPTER

52

Ambulatory Surgery

1. Based on the Centers of Medicare and Medicaid Services (CMS), ambulatory surgery centers (ASCs) are: A. Any Center that provides same day medical service with minor procedures. B. Any distinct entity that operates exclusively for the purpose of providing surgical services to patients not requiring hospitalization and which services will not exceed 24 hours following admission. C. Any office outside a hospital that provides invasive procedures to patients outside hospital setting. D. Does not need referral from a primary care physician.

Answer: B ASCs are independent health care facilities that offer patients the convenience of having surgery performed safely without admission to a hospital. According to the Centers for Medicare and Medicaid Services (CMS), effective May 18, 2009, ASCs are any distinct entity that operates exclusively for the purpose providing surgical services to patients not requiring hospitalization and which the expected duration of services would not exceed 24 hours following an admission. ASCs should not be confused with office-based surgery practices or with other outpatient centers that provide diagnostic services or primary health care, such as urgent care centers, community health centers, mobile diagnostic units, or rural health clinics. ASCs are distinguished from other health care facilities by (a) there use of a referral system for accepting patients and (b) their maintenance of a dedicated operating room. (See Schwartz 11th ed., p. 2153.)

2. The top five procedures performed at ambulatory surgery centers (ASCs) are: A. Cataract surgery, esophagogastroduodenoscopy with biopsy, colonoscopy with biopsy, and spine epidural injection foraminal. B. Laparoscopic cholecystectomy, inguinal hernia repair, epidural injection, diagnostic colonoscopy. C. Laparoscopic cholecystectomy, inguinal hernia repair, abdominal wall hernia repair, debridement of skin ulcers, skin lesion removal. D. Injection paravertebral facet joint, insertion of temporary bladder catheter, diagnostic colonoscopy, abdominal wall hernia repair, inguinal hernia repair.

Answer: A The top five in procedures they have formed at ASCs are cataract surgery with intraocular lens, esophagogastroduodenoscopy with biopsy, colonoscopy and biopsy, colonoscopy with lesion removal, and spine epidural injection foraminal. In contrast, the top five procedures performed at the hospital outpatient departments by volume are: Subcutaneous tissue debridement, esophagogastroduodenoscopy with biopsy, aspiration/injection of joint, cataract surgery with intra ocular lens implant, and colonoscopy and biopsy. (See Schwartz 11th ed., p. 2155.)

3. Cost of performing procedures at ambulatory surgery centers (ASCs) are lower than those performed in hospitals due to which of the following factors? A. Avoiding emergency procedures B. Fewer diagnostic tests and fewer medications C. Less staffing and less expensive instruments D. All of the above

Answer: D In many cases an outpatient procedure performed in an ASC is between 1/2 and 1/3 of their costs as the same procedure performed in a hospital. In large part, ASCs affect cost savings by eliminating overnight hospitalizations and emergency procedures. ASCs perform fewer extensive diagnostic tests and dispense fewer medications. These facilities are not staffed around the clock and are not encumbered by the need for expensive and highly specialized equipment as are hospitals. (See Schwartz 11th ed., p. 2157.)

385

Brunicardi_Ch52_p385-386.indd 385

30/06/22 11:30 AM

386 Answer: D Health care facilities in the United States are highly regulated by federal and state entities. ASCs are included in this oversight. Independent observers evaluate the safety and quality of care provided in ASC through three processes: Medicare certification, state licensure, and voluntary accreditation. To obtain Medicare certification, ASCs must meet the Medicare certification requirements, known as the conditions for coverage of these conditions include specifying standards for administration of anesthesia, quality evaluation, operating and recovery rooms, medical staff, nursing services, and other aspects of care. An ASC must have an inspection conducted by a state official or a representative of an organization authorized by the government. Each state determines the specific requirements ASCs must meet for licensure. An ASC does not have to be certified by Medicare in order to be accredited by JCAHO, but if they will be reimbursed by the Medicare they should meet their requirements. In 37 states, any party looking to open an ASC must demonstrate the need for ASC to exist. State licensure requirements generally exist for both health care facilities and health care professionals. (See Schwartz 11th ed., p. 2158.)

5. Unplanned admission to a hospital after a procedure in ambulatory surgery centers (ASCs) occur in 0.5% to 2% of cases with highest odds in the following EXCEPT: A. Medicaid patients. B. Patients with private insurance. C. Lower median household income. D. Greater preoperative comorbidity burden.

Answer: B Safe use of ASC is based on identification of patients who are unlikely to require admission to a hospital after their procedure. Thus far, ASCs do very well in this aspect. Unplanned admissions after ambulatory surgery occurring approximately 0.5% to 2.0% of cases. In the future, ASCs will be challenged to reduce this unplanned admissions rate even further. Patients with Medicaid insurance, lower median household income, and greater preoperative comorbidity burden have the highest odds of unplanned acute care use. These patients may benefit from interventions that enhance and streamline possible to follow-up. Additionally, the potential costs associated with postoperative acute care following procedures performed in ASCs are not insignificant. Patient-specific predictors of unplanned hospital admissions include age 65 years or older, anticipated operating time longer than 120 minutes, cardiac comorbidities, peripheral vascular disease, cerebral vascular disease, malignancy, positive for human immunodeficiency virus (HIV) and regional or general anesthesia use. The strongest predictor for unplanned inpatient hospital admission was the individual patient’s own history of previous hospitalizations, particularly among older adults. African American and Hispanic individuals also have had a markedly elevated risk of inpatient hospital admission, possible related to cultural or socioeconomic issues for access to care. (See Schwartz 11th ed., pp. 2159–2160.)

CHAPTER 52

4. Requirements to open an ambulatory surgery center (ASC) in any state include which of the following? A. Accreditation by regulating state bodies B. Certificate of need C. Accreditation by JCAHO D. None of the above

Ambulatory Surgery

Brunicardi_Ch52_p385-386.indd 386

30/06/22 11:30 AM

CHAPTER

53

Skills and Simulation

1. Virtual reality (VR) simulation has been used for training in laparoscopy, flexible endoscopy, sinuscopic surgery, and endovascular interventions. Which of the following is an advantage of VR simulator–based training as compared to physical laparoscopic video trainers? A. Significantly improved operative performance B. Ability to perform proficiency-based training C. Automated performance measurement D. Lower upfront costs for development

Answer: C VR simulators allow for automated performance measurement such as time, instrument motion, and electrosurgery use measurements as well as a tally of the occurrence of predefined errors. These measurements are not only automated but also free of observer bias. Both virtual reality and physical laparoscopic video trainers (“box” trainers) allow for proficiency-based training. Good training results can be obtained with proficiency-based training within a larger curriculum, regardless of training platform. Although there may be some advantage with the virtual reality trainer, no significant difference has been identified. (See Schwartz 11th ed., pp. 2169–2171.)

2. Which of the following is a nontechnical skill that can be taught using simulation training? A. Situational awareness B. Instrument selection C. Economy of movement D. Respect for tissue

Answer: A Nontechnical skills refer to the cognitive knowledge and teamwork-related abilities that must be integrated with psychomotor skills and abilities. Examples of nontechnical skills include situational awareness, communication, and teamwork. Examples of technical skills include respect for tissue, time and motion, instrument handling, and knowledge of instruments; these skills, for example, can be evaluated using a global ratings scale such as in the objective structured assessment of technical skills (OSATS). (See Schwartz 11th ed., pp. 2166–2167, 2175–2177.)

3. Which of the following strategies for simulation-based error training allows the instructor to engage in a broad assessment of learning needs? A. Error-enabled approach B. Forced-error approach C. Error-centric skills assessments D. Graduated autonomy approach

Answer: A The error-enabled approach is where the learner can make any of a variety of errors during the course of a task. This allows for a broad assessment of learning needs. The forcederror approach is where the learner experiences a specific, usually unexpected error and demonstrates error management as well as options for correction. This approach allows instructors to engage in specific error management skills and metrics. Error-centric assessments allow for evaluation and categorization of errors; these types of assessments allow for focused error training and feedback. While gradual increases in patient autonomy and autonomous decision-making are an important component of resident training, this is not a specific simulation-based strategy. (See Schwartz 11th ed., pp. 2177–2178.)

387

Brunicardi_Ch53_p387-388.indd 387

30/06/22 11:31 AM

388

CHAPTER 53 Skills and Simulation

4. Proficiency-based simulation training on central venous catheter insertion has been successful in improving which of the following outcomes? A. Iatrogenic pneumothoraces B. Misplaced central venous catheters C. Catheter-related bloodstream infections D. Failure of catheter insertion

Answer: C Proficiency-based simulation training on central venous catheter insertion is one of the best examples of evidence for improved patient outcomes with simulation. Multiple studies have demonstrated reductions in the incidence of catheter-related bloodstream infections. (See Schwartz 11th ed., pp. 2178–2179.)

5. A surgical instructor wishes to evaluate the effects of a simulation-based training program to reduce bile duct injuries during laparoscopic cholecystectomy. Which of the following would be the most feasible, relevant learner metric in assessing the success of this training? A. Perceptions about the training B. Knowledge about bile duct injuries C. Skills transfer to the clinical environment D. Institutional incidence of bile duct injuries

Answer: C While ideally, simulation training should lead to improved patient outcomes, the incidence of bile duct injury with laparoscopic cholecystectomy is uncommon. Large, unrealistic sample sizes would be necessary to demonstrate a clinically relevant difference. In evaluating the effectiveness of educational intervention efforts, the Kirkpatrick four-level scale is often used. The four levels are: Reaction, learning, behavioral change, and institutional impact. Since institutional impact cannot be assessed in this example, the next highest level would be behavioral change—did the knowledge, skills, and attitudes acquired during simulation transfer to the clinical environment? (See Schwartz 11th ed., p. 2178.)

Brunicardi_Ch53_p387-388.indd 388

30/06/22 11:31 AM

CHAPTER

54

Web-Based Education and Implications of Social Media 1. Which of the following terms describes computermediated platforms that are used for creating and sharing information, ideas, and other content? A. Websites B. Social media C. Simulators D. Applications

Answer: B Social media is a term describing websites and web-based applications that enable users to share ideas, information, and content through virtual networking. Commonly used social media platforms include Facebook, Twitter, and YouTube. (See Schwartz 11th ed., pp. 2187–2189.)

2. Which of the following is not a barrier to the more widespread adoption of web-based educational strategies? A. Lack of data for superior effectiveness B. Up-front costs for development C. Surgeon uptake of the technology D. Ability to have asynchronous learning

Answer: D Web-based education and coaching allow for asynchronous learning whereby students and instructors do not have to be on the same schedule. Additionally, web-based education allows for access to educational materials regardless of time of day or night. However, barriers include lack of highquality studies demonstrating that web-based education is superior to traditional educational interventions. Additionally, the high up-front costs for development and need for technical expertise are also barriers. Lastly, lack of surgeons’ uptake of the technology is a barrier. (See Schwartz 11th ed., pp. 2187–2188.)

3. Which of the following is not a social media strategy for disseminating information about peer-reviewed publications? A. Visual abstracts B. Online journal clubs C. Conference live-tweeting D. Journal postings

Answer: C Conference live-tweeting describes the posting of social media posts on Twitter about an event while it is ongoing. Usually, peer review and publication lag behind conference proceedings. Visual abstracts are graphical summaries of the main results of journal articles. They may be created by the author or the journal, and they typically accompany social media posts referencing the peer-reviewed publication. Online journal clubs are similar to in-person journal clubs that discuss peer-reviewed publications. Social media-based journal clubs, however, allow asynchronous discussions across multiple time zones and geographic locations. Journals often post links to the abstract and/or peer-reviewed publication on social media in order to draw attention to the article. These posts can be accompanied by a visual abstract. (See Schwartz 11th ed., pp. 2190–2192.)

389

Brunicardi_Ch54_p389-390.indd 389

30/06/22 11:31 AM

390

CHAPTER 54 Web-Based Education and Implications of Social Media

4. Which of the following increases the quality and accuracy of web-based educational materials for patients? A. Crowdsourcing B. Peer review C. Physician author D. Number of viewers

Answer: B One of the potential pitfalls associated with web-based education is inaccuracy in information in web-based educational materials. High-quality online resources should have peer-reviewed content or verifiable mechanisms for quality control of information. Crowdsourcing describes the process whereby information is gathered by enlisting the assistance of a large number of participants. Crowdsourcing often involves the general public or a loosely defined group of individuals. Physician authorship is not necessarily associated with increased accuracy of the online content. Number of viewers of the web-based content does not guarantee quality or accuracy. (See Schwartz 11th ed., pp. 2190–2193.)

5. Which of the following uses of social media does not require patient consent? A. Case report of a patient with an unusual complication B. Email between physician and established patient C. Online journal club about a published randomized trial D. Posting of a photo that does not have the patient’s face

Answer: C Another potential pitfall of web-based education is ethical considerations surrounding patient confidentiality and privacy. Appropriate safeguards must be taken to guard against violation of patient confidentiality and privacy. Although patients must consent to participate in a randomized trial, discussion of an already published and peer-reviewed randomized trial does not require additional patient consent. Case reports, whether published in a journal or posted on social media, require patient consent. Emails or other electronic communications should only be used by physicians in an established patient–physician relationship and with patient consent. Photos should not be posted that have any details that would result in the patient being identifiable. (See Schwartz 11th ed., pp. 2190–2193.)

Brunicardi_Ch54_p389-390.indd 390

30/06/22 11:31 AM

INDEX

Note: Page numbers followed by italic f or t denote figures or tables, respectively. A

A-A index, 55 AAA. See Abdominal aortic aneurysms AAH. See Atypical adenomatous hyperplasia ABCDE initialism, for melanoma, 130 Abdomen, blunt trauma to, 60, 60f Abdominal aortic aneurysms, 191–192, 368 Abdominal compartment syndrome, 69, 103, 112–113 Abdominal hemorrhage, 50 Abdominal hernias, 270 Abdominal rectopexy, 250 Abdominal wall anterior, 307 desmoid tumors of, 308 malignancy of, 307 Abdominal wall hernia, 307 ABI. See Ankle brachial index Abscess anorectal, 253 breast, 140 cellulitis without, 129 hepatic, 41–42 lung, surgical drainage of, 152, 152t perianal, 253 pyogenic, 41, 272, 272f retroperitoneal, 309 subphrenic, 51 Abstracts, visual, 389 Abundance mentality model, of conflict resolution, 4 Accessory spleen, 300 Accommodation response, 241 ACDF. See Anterior cervical discectomy and fusion Acetaminophen, 269 Achalasia, 212, 221 Acidic chemical burns, 134 Acidosis alkalosis versus, 17 metabolic. See Metabolic acidosis Acral lentiginous melanoma, 130–131

Brunicardi_Index_p391-420.indd 391

Acrochordons, 136 ACTH. See Adrenocorticotropin hormone Actinic keratoses, 136 Activated protein C, 22 Acupuncture, 379 Acute cholecystitis, 279–280, 283f, 283–284 Acute hypercalcemia, 18 Acute limb ischemia, 194 Acute liver failure, 269 Acute lung injury description of, 100 transfusion-related, 25–27 Acute myeloid leukemia, 303 Acute myocardial infarction, 33 Acute pancreatitis, 42, 290 Acute prostatitis, 338 Acute renal failure, 18 Acute respiratory distress syndrome, 32, 36, 100–101, 101t ADAM S13, 24 Adenocarcinoma in Barrett mucosa, 214f–215f duodenal, 238 gallbladder, 280 lung, 148–149, 150f metastatic Barrett esophagus and, 208–209 rectal, 256 Adenomas, liver, 275, 275f Adenomatous polyposis coli, 154 Adenosine diphosphate, 21 ADH. See Antidiuretic hormone; Atypical ductal hyperplasia Adjustable gastric banding, 233 ADP. See Adenosine diphosphate Adrenal glands, 8 Adrenal incidentaloma, 328 Adrenal insufficiency, 8–9 Adrenal mass, 327 Adrenalectomy, laparoscopic, 328 Adrenocorticotropin hormone, 8 Adventitial cystic disease, of popliteal artery, 196 Aerobic bacteria, 41 AF. See Atrial fibrillation

African Americans, 5 AG. See Anion gap AI. See Aortic insufficiency AIHA. See Autoimmune hemolytic anemia Air enema, 335 Air leak, 59 Airway pressure, 111 Airways interventions for, 57–58 mucus production in, 147 AJCC. See American Joint Committee on Cancer Albumin, 268 anion gap affected by, 16 fluid resuscitation uses of, 14 serum calcium and, 15 Aldosterone, 17 ALF. See Acute liver failure ALH. See Atypical lobular hyperplasia ALI. See Acute lung injury Alkaline chemical burns, 134 Alkalosis acidosis versus, 17 metabolic, 17 potassium change associated with, 15 Allen test, 107 Allergy, antibiotics, 45 Allografts, kidney, 91–92, 92f Allotransplant, 87 Alpha angle, 28 Alvimopan, 101, 378 Ambulatory surgery, 385–386 Ambulatory surgery centers, 385–386 Amelanotic lesions, 130 American Joint Committee on Cancer description of, 248 sarcoma staging, 312 American Society of Anesthesiologists physical status classification system, 363 Amino acids branched-chain, 9 gastrin secretion inhibition by, 223

30/06/22 11:32 AM

392

INDEX

Amino acids (Cont.): questions regarding, 8 in wound healing, 79 Aminoacyl-tRNA synthetases, 120 AML. See Acute myeloid leukemia AMOs. See Assistant medical officers Amoxicillin/clavulanate, for human bites, 134 Amsterdam I criteria, 247–248 Amsterdam II criteria, 247–248 Anal canal, 241 Anastomoses arterial, 92f Damus-Kaye-Stansel, 160 end-to-end, 244, 245f end-to-side, 244, 245f intestinal, 244–245 questions regarding, 244–245 side-to-side, 244, 245f venous, 92f, 93 wound healing, 75 Anastomotic pseudoaneurysms, 183 Anemia autoimmune hemolytic, 298–300 wound healing and, 78 Anesthesia American Society of Anesthesiologists physical status classification system, 363 epidural, 101 fasting before, 364, 366 general, 245, 363, 366 malignant hyperthermia after, 363 regional, 245 saddle, 350 Aneurysms aortic. See Aortic aneurysms aortic arch, 185 ascending thoracic, 184 endovascular repair of, 184, 192 left ventricular, 176 mycotic, 182–183 splenic, 304 splenic artery, 304 Angina pectoris, 171, 171t Angiography computed tomography. See Computed tomography angiography hepatic hemorrhage controlled using, 61 indications for, 50 Anion gap equation for calculating, 14 metabolic acidosis with, 13–14, 17 normal, 13–14, 16 Ankle brachial index, 352

Brunicardi_Index_p391-420.indd 392

Ankylosing spondylitis, 246 Anomalous coronary artery patterns, 170 Anorectal abscess, 253 Anorectal lymphatic drainage, 241 Anorectal melanoma, 250 Anosmia, 347 Anterior abdominal wall, 307 Anterior cerebral artery, 112 Anterior cervical discectomy and fusion, 349, 350f Anterior mediastinal mass, 155 Anterior mediastinal tumors, 155 Anthrax inhalational, 44 treatment of, 44 Antibiotic prophylaxis for colectomy, 246 for human bites, 133–135 for inguinal hernias, 316 for overwhelming postsplenectomy infection, 306 for surgical infections, 38–39, 45, 102 Antibiotics allergy to, 45 diverticulitis treated with, 247 lymphedema treated with, 206 methicillin-resistant Staphylococcus aureus treated with, 129 perioperative treatment with, 45 postappendectomy, 261 primary microbial peritonitis treated with, 41 pyogenic liver abscess treated with, 272 uncomplicated diverticulitis treated with, 247 Anticoagulation, in elderly, 367 Antidiuretic hormone as mesenteric vasoconstrictor, 31 sodium depletion caused by secretion of, 13 Antigen-presenting cells, 87 Antigens, 125 Antireflux, 214, 216–217, 217f Antithrombin, 205 Antithrombin III, 22 Antithrombotic therapy, for deep vein thrombosis, 202 Antrectomy, 227f–228f, 227–228, 231 Anus imperforate, 336 questions regarding, 241–257 Aorta chemoreceptors in, 30 coarctation of the, 159, 163–164 diameter of, 183

Aortic aneurysms abdominal, 191–192, 368 ascending, 183–184 distal, 183 false, 181 thoracic, 181 true, 181 Aortic arch aneurysms of, 185 bovine, 182 Aortic arch receptor, 30 Aortic dissection, 184, 185f, 186–187 Aortic insufficiency, 175 Aortic stenosis age-related, 175 congenital, 163 Aortic valve replacement of, 175 stenosis of, 158–159 Aortoiliac occlusive disease, 193–194 APC. See Activated protein C; Adenomatous polyposis coli APCs. See Antigen-presenting cells Apocrine sweat glands, 133 Apoptosis of cancer cells, 84, 122 pathways of, 122f Appendectomy, laparoscopic, 259–261 Appendiceal carcinoid tumors, 260 Appendicitis, 259–262, 260t, 262f, 367 Appendix, 259–262 ARDS. See Acute respiratory distress syndrome Arginine, 79 Arginine vasopressin, 33 Arrhythmia in laparoscopy, 115 questions regarding, 176 Arterial anastomosis, 92f Arterial disease computed tomography angiography of, 189 vascular stents for, 189–190 Arterial lines, 107–108 Arterial pH, 17–18 Arterial switch operation, 161 Arthritis inflammatory bowel disease and, 246 management of, 353 ASA physical status classification system. See American Society of Anesthesiologists physical status classification system Asbestos exposure, 156 Ascending aortic aneurysms, 183–184 Ascending aortic dissection, 187

30/06/22 11:32 AM

393

B

Back pain, 352 Balloon valvotomy, 158–159 Balloon-expandable stents, 189 Bare coronary stent placement, 364 Bariatric surgery gastric stasis after, 224 questions regarding, 233–234 Barium esophagogram of H-type tracheoesophageal fistula, 331f of vascular rings, 168f Barium swallow, for eosinophilic esophagitis, 220 Baroreceptors, 30 Barotrauma, 112 Barrett esophagus, 208–209 Barrett mucosa, 214f–215f Basal cell carcinoma, 129, 135 Basal energy expenditure, 11, 11t Base deficit, 36 Basic fibroblast growth factor, 124

Brunicardi_Index_p391-420.indd 393

Battle’s sign, 346 Baxter formula, 68 B-cell–mediated rejection, 95 BE. See Barrett esophagus Beck’s triad, 49, 55 Beclin I gene, 84 BEE. See Basal energy expenditure Belsey fundoplication, 221 Benign calcifications, in hamartomas, 148 Benign prostatic hyperplasia, 339 Betel nut chewing, 144 Bevacizumab, 149 bFGF. See Basic fibroblast growth factor Bicarbonate chemoreceptor sensitivity to levels of, 30 metabolic acidosis with anion gap caused by loss of, 13, 17 Bidirectional Glenn procedure, 160, 160f Bifunctional RNAi technology, 125 Bifunctional short hairpin, 125 Bile acids description of, 236, 268 reabsorption of, 283 Bile ducts cancer/carcinoma of, 246, 276 common, 279 stricture of, 282 Bile leak, 102 Bile salts, 268 Biliary atresia, 333 Biliary cancer, 281 Biliary injury, 282 Biliary scintigraphy, 283 Biliary stricture, 282 Bilious emesis, 330 Billroth I gastroduodenostomy, 227, 227f Billroth II gastroduodenostomy, 227, 228f Biloma, after laparoscopic cholecystectomy, 102 Biologic mesh, for abdominal wall hernia repair, 307 Biomarkers, cancer, 85 Biomedical ethics, 369 Biopsy fine needle aspiration, of thyroid nodules, 324 primitive neuroectodermal tumor diagnosis, 154 sentinel lymph node for breast cancer, 139 for melanoma, 132

Biosynthetic heart valves, 367 Bipolar electrosurgery, 116 Birbeck granules, 133 bi-shRNA. See Bifunctional short hairpin Bites, human, 133–134 Bladder cancer, 337 Bladder pressure, 112t Blastocysts, 124 Blastomyces dermatitidis, 90 Blood loss, 26, 32–33 Blood pressure intra-arterial monitoring system for, 106 pregnancy-related changes, 62 Blood transfusion complications of, 26 description of, 25 diseases not transmitted by, 27 indications for, 25–26 Blood urea nitrogen, 18 “Blooming artifact,” 189 Blotting, 124 Blunt trauma to abdomen, 60, 60f duodenal hematoma from, 53 to liver, 50 to pancreas, 53 transcriptomic response to, 10 BMI. See Body mass index Body mass index, 233 Bone healing, 77 Bone marrow suppression, 88 Bottleneck, 5 Bougie, 234 Bovine aortic arch, 182 Bowel preparation, 246 BPH. See Benign prostatic hyperplasia Brachiocephalic veins, 198 Bradycardia, 115 Brain herniation of, 345, 345f malignant tumors of, 348 traumatic injury of. See Traumatic brain injury Brain stem compression, 346 Branched-chain amino acids, 9 BRCA1, 140 BRCA2, 140 Breast abscess of, 140 anatomy of, 139 blood supply to, 139 Cooper suspension ligaments of, 139 hormonal effects on, 139 infection of, 140

INDEX

Ascending thoracic aneurysms, 184 ASCs. See Ambulatory surgery centers ASD. See Atrial septal defects Aspergillosis, 153 Aspergillus sp., 153 Aspiration pneumonia, 153 Aspirin, 23 Asplenia, 298 Assistant medical officers, 373 Astrocytoma, 348 Atherosclerotic aortoiliac occlusive disease, 193–194 AT-III. See Antithrombin III Atresia biliary, 333 esophageal, 330, 331f Atrial fibrillation, 174, 176 Atrial septal defects, 157f, 157–158 Atrophic gastritis, 229 Atypical adenomatous hyperplasia, 148 Atypical ductal hyperplasia, 137 Atypical lobular hyperplasia, 137 AUG, 120 Authoritative leader, 4 Autoimmune hemolytic anemia, 298–300 Automobile accidents, 50, 59 Autophagy, 9 Autotransplant, 87 Axillary lymph nodes, 137 Axillary vein, 198 Axonotmesis, 77 AZA. See Azathioprine Azathioprine, 88, 89t

30/06/22 11:32 AM

394

INDEX

Breast (Cont.): pregnancy-related changes in, 137 sarcoma of, 312 Breast cancer BRCA1/BRCA2, 140 distal carcinoma in situ, 137–138 estrogen exposure and, 137 lobular carcinoma in situ, 137 lymph node metastasis of, 139 mastectomy for, 140–141 in pregnancy, 139 radiation therapy for, 140 risk factors for, 137, 140 sentinel lymph node biopsy for, 139 staging of, 137 treatment of, 140–141 Breast lesions, 137 Bronchioles, 147 Bronchoscopy, 59, 167f Brush border membrane, 236 BUN. See Blood urea nitrogen Burch procedure, 343 Burn(s) caloric needs in, 69 caustic, 134 chemical, 134 complications of, 70 cyanide toxicity from, 66 depth classification of, 67 electrical, 67 eschar, 69 escharotomies for, 68 first-degree, 66, 68 fluid resuscitation for, 68 full-thickness, 67–68 heterotopic ossification associated with, 70 initial evaluation of, 65 Lactated Ringer solution for, 68 Lund and Browder chart for, 66 massive resuscitation of, 69 nutritional requirements, 69 partial-thickness, 67–68 resuscitation of, 69 “rule of nines” for, 65, 66f second-degree, 67 smoke inhalation injuries, 66 superficial, 66 third-degree, 67 thoracic compartment syndrome caused by, 68 total body surface area calculations, 65–66, 66f zone of coagulation for, 67, 129 zone of hyperemia, 67, 129

Brunicardi_Index_p391-420.indd 394

Burn disasters, 71 Burn wounds excision of, 70 silver sulfadiazine for, neutropenia caused by, 68–69 surgical treatment of, 70 C

CA-125, 342 Calcifications, in hamartomas, 148 Calcineurin inhibitors, 88, 89t Calcium hyperkalemia treated with, 15 serum levels of, 15 Calcium chloride, 15 Calcium gluconate, 15 Callus formation, 77 Canadian Cardiovascular Society angina classification, 171, 171t Cancer. See also Carcinoma; Tumor(s) biomarkers of, 85 bladder, 337 breast. See Breast cancer chemoprevention of, 84 chemotherapy agents for, 84 in chronic wounds, 80 colorectal, 123, 247–249 conservative resections in, 85 dormancy of, 84 epithelial ovarian, 342 esophageal, 210, 221 gallbladder, 276, 280 gastric, 229 hallmarks of, 83 head and neck, 143 hereditary, 83 hereditary nonpolyposis colorectal cancer, 247–248 immune-based therapies for, 85 invasion, 84 laryngeal, 144 lip, 143 in low- and middle-income countries, 373 lung. See Lung cancer management of, 85 metastasis of, 84–85 ovarian, 342 pancreatic, 123 papillary thyroid, 281, 324 prostate, 338 rectal. See Rectal cancer in situ, 84 splenectomy and, 299 testicular, 337, 339

tumorigenic transformation of cells in, 83 vaginal, 343 Cancer cells apoptosis of, 84 metastasis by, 84 transforming growth factor-ß resistance by, 123 tumorigenic transformation of, 83 Cancer genes, 124 Candida albicans, 41 Capnography, 111 Caput medusae, 270 Carbohydrate digestion of, 236 preoperative supplementation of, 377 Carbon monoxide diffusion capacity, 152 Carbonic acid, 17 Carcinoid tumors, 230, 260 Carcinoma. See also Cancer; Tumor(s) basal cell, 129, 135 bile duct, 246 hepatocellular, 276–277 neuroendocrine, 148 renal cell, 337, 339 squamous cell. See Squamous cell carcinoma vulvar, 342 Cardiac myxomas, 179 Cardiac output, 108, 366 Cardiac tamponade, 35, 49 Cardiac transplantation, 95 Cardiac tumors, 179–180 Cardiogenic shock, 29, 34, 55–56, 56f Cardiopulmonary bypass coagulation during, 25 Cox-Maze IV procedure, 177, 177f deleterious effects of, 173 Cardiopulmonary resuscitation, 363 Cardiovascular disease, renal failure and, 91 Caroli disease, 285 Carotid bodies, 30 Carotid body tumors, 191 Carotid endarterectomy, 190–191 Carpal tunnel syndrome, 357 Cas9, 125 Case fatality rates, 375f Case reports, 390 Case series, 382 Case-control study, 382 Caspases, 84 Catecholamines, for septic shock, 33

30/06/22 11:32 AM

395

Brunicardi_Index_p391-420.indd 395

Chemotherapy adjuvant, for colorectal cancer, 248–249 agents used in, 84 hairy cell leukemia treated with, 302 hepatocellular carcinoma treated with, 277 Chenodeoxycholic acid, 268 Chernobyl disaster, 324 Chest injury, penetrating, 53 Chest wall tumors, 154 Children. See Pediatric patients Child-Turcotte-Pugh score, 274, 274t Chin lift, 58 Chinese medicine, traditional, 379 Chlamydia trachomatis, 343 Chloroquine, 84 Cholangiocarcinoma, 276, 281, 286–287 Cholecystectomy indications for, 286 laparoscopic acute cholecystitis treated with, 284 anesthesia for, 364 biloma after, 102 safety concerns for, 117 Cholecystitis, acute, 279–280, 283f, 283–284 Cholecystokinin, 279 Choledochal cysts, 281, 285, 286f, 335 Cholestasis, 103 Cholic acid, 268 Chondromas, 154 Chondrosarcoma, 154 Chronic allograft nephropathy, 93 Chronic bacterial prostatitis, 338 Chronic liver disease, 270 Chronic lymphedema, 311 Chronic lymphocytic leukemia, 297 Chronic myelogenous leukemia, 303 Chronic pancreatitis, 290f–291f, 290–292, 292t Chronic venous insufficiency, 198, 199f Chvostek sign, 16 Chylothorax, 156 Cingulate gyrus, 345f Cirrhosis, 270–271, 276, 280 CKI. See Cyclin-dependent kinase inhibitors Claudication, 196 Clean wound, 39, 40t Clean/contaminated wound, 39–40, 40t CLL. See Chronic lymphocytic leukemia Clopidogrel, 22–23

Closed brain injury, 347 Closed loop obstruction, 236 Clostridium difficile, 259 Clostridium perfringens, 80 Clot formation of, 21 propagation of, 22 Clotting factors, 268 congenital deficiency of, 22 in intrinsic and extrinsic pathways, 21 CML. See Chronic myelogenous leukemia CMS. See Centers of Medicare and Medicaid Services CMV. See Cytomegalovirus COA. See Coarctation of the aorta Coaching, 3 Coagulation in cardiopulmonary bypass, 25 extrinsic pathway of, 21 intrinsic pathway of, 21 Coagulation cascade, 22 Coagulopathy, trauma-induced, 23 Coarctation of the aorta, 159, 163–164 Codon, 120 Coercive style, of leadership, 4 Colectomy bowel preparation for, 246 left, 244 right, 243 sigmoid, 244, 250 subtotal, 244 total, 244 transverse, 244 Collagen in inguinal hernias, 315, 316t synthesis of, steroid effects on, 78 in wound healing, 74, 76t Collagen disorders, 315 Collagenase, 74, 76t Colloid osmotic pressure, 16 Colloid resuscitation, 30 Colocolic anastomosis, 245f Colon hepatic flexure of, 54 laparoscopic resection of, 378 layers of, 256 leiomyoma of, 249 leiomyosarcoma of, 249 minimally invasive resection of, 243 penetrating injuries to, 54 questions regarding, 241–257 resection of, 243–244 Colonic pseudo-obstruction, 251 Colonoscopy, 252

INDEX

Catheter(s) indwelling intravascular, 43, 46–47 pulmonary artery, 109, 109f thrombosis and, 107 Catheter-based contrast aortography, 184 Catheter-directed thrombolysis, 201 Catheter-related bloodstream infection, 108 Cauda equina syndrome, 350 Cause of death medical errors as, 97 in pediatric patients, 333 Caustic injury, 134 CCK. See Cholecystokinin CD117, 249 CD4+ T cells cytokines produced by, 9 helper T cells, 10 CD8+ T cells, 9 CDC. See Centers for Disease Control and Prevention CDH. See Congenital diaphragmatic hernia Cecal volvulus, 250–251 Cell(s) apoptosis of, 84, 122 physiologic alterations of, in cancer, 83 tumorigenic transformation of, 83 Cell cycle, 83, 121f, 121–122 Cell-free mitochondrial DNA, 8 Cell-surface receptors, 122–123 Cellulitis, 129 Centers for Disease Control and Prevention, 100 Centers of Medicare and Medicaid Services, 385 Central line infections, 100 Central transtentorial herniation, 345f Central venous pressure, 55, 106–107, 178 Cephalic vein, 198 Cerebellar stroke, 346f Cerebral perfusion pressure, 53, 112 Cerebrospinal fluid leak/leakage, 144, 346–347 Cerebrovascular accident, 190 Cervical disc herniation, 349, 349t Cervical esophagus, 207, 213 Cervical radiculopathy, 349 Cesarean delivery, 341 Chagas’ disease, 27 Charcot triad, 284 Chemical burns, 134 Chemoprevention, of cancer, 84 Chemoreceptors, 30

30/06/22 11:32 AM

396

INDEX

Colorectal cancer adjuvant chemotherapy for, 248–249 description of, 123 in elderly, 367 hepatic metastasis of, 277 hereditary nonpolyposis, 247–248 lymph node metastasis of, 248–249 resection of, 348, 367 staging of, 248 TNM staging of, 248 Colostomy, 54, 256 Colostrum, 137 Columnar epithelium, 208–209 Command-and-control leadership, 3–5 Common bile duct arterial supply to, 279 lateral injury to, 281 Common carotid artery, 145, 145f Communication errors in, 4 in leadership, 3–4 Community-acquired pneumonia, 44 Compartment syndrome abdominal, 69, 103, 112–113 definition of, 351 description of, 51 Compensated shock, 35 Complement cascade, 32 Complicated diverticulitis, 247 Computed tomography adenomas on, 275f adrenal incidentaloma on, 328 appendicitis on, 262, 262f cerebellar stroke on, 346f cerebrospinal fluid leakage findings, 346 hernia recurrence evaluations, 319 inferior vena cava filters on, 203f liver lesions on, 268, 275f lung cancer diagnosis using, 148 magnetic resonance imaging versus, 269 mediastinal lymph node staging with, 151 mesenteric vein thrombosis on, 206 pheochromocytomas on, 327 pyogenic liver abscess on, 272f urolithiasis on, 338 Computed tomography angiogram of arterial disease, 189 of coarctation of the aorta, 164f of double aortic arch, 167f of extremities, 55 of neck, 52

Brunicardi_Index_p391-420.indd 396

of pulmonary artery slings, 167f of total anomalous pulmonary venous connection repair, 166f Concussion, 347 Conference live-tweeting, 389 Conflict resolution, 3t, 4 Congenital diaphragmatic hernia, 329 Congenital factor XIII deficiency, 22 Congenital heart disease aortic stenosis, 163 aortic valve stenosis, 158–159 arterial switch operation for, 161 atrial septal defects, 157f, 157–158 bidirectional Glenn procedure for, 160, 160f double aortic arch, 167f double-outlet right ventricle, 170 Ebstein’s anomaly, 169f hypoplastic left heart syndrome, 160–161, 169 management of, 163 patent ductus arteriosus, 162–163 pulmonary artery slings, 166, 167f–168f tetralogy of Fallot, 161, 170 total anomalous pulmonary venous connection, 159, 159f, 165–166, 166f transposition of the great vessels, 161 tricuspid atresia, 168 truncus arteriosus, 165 ventricular septal defect, 161–162 Wolff-Parkinson-White syndrome associated with, 169f, 169–170 Congenital inguinal hernias, 314–315, 315f Congenital nevi, 136 Conn’s syndrome, 327 Consent, informed, 369–370 Constructive pericarditis, 178 Contaminated wound, 40, 40t Continence, 241–242 Contusions pancreatic, 53 pulmonary, 58, 63 Cooper’s ligament of breast, 139 of inguinal region, 317 Corona radiata sign, 148 Coronary artery bypass, 173 Corticosteroids arthritis treated with, 353 immunosuppression uses of, 89t, 90 side effects of, 89t wound healing affected by, 78 Corticotropin-releasing hormone, 8 Cortisol, 8

Cox-Maze IV procedure, 177, 177f CPB. See Cardiopulmonary bypass CPP. See Cerebral perfusion pressure CPR. See Cardiopulmonary resuscitation C-reactive protein, 7 CRH. See Corticotropin-releasing hormone Cricoid cartilage, 146, 146f Cricopharyngeal muscle, 207 Cricopharyngeal sphincter, 212 Cricothyroidotomy, 56 CRISPR, 125 CRISPR-associated genes, 125 Critically ill patients hyperglycemia in, 33 insulin resistance in, 33 metabolism in, 11 treatment of, 33–34 Crohn’s disease description of, 237 rectovaginal fistulas caused by, 254 steatorrhea in, 268 Crossmatching, 25 Cross-sectional study, 382 Crowdsourcing, 390 CRP. See C-reactive protein Cruveilhier-Baumgarten murmur, 270 CT. See Computed tomography CTA. See Computed tomography angiogram CTS. See Carpal tunnel syndrome Cubital tunnel syndrome, 358 Cushing ulcers, 349 Cushing’s disease, 327 Cushing’s syndrome, 327 Cutaneous melanoma, 130, 132 CVD. See Cardiovascular disease CVI. See Chronic venous insufficiency CVP. See Central venous pressure Cyanide poisoning, 66 Cyanide toxicity, 66 Cyclin D1, 154 Cyclin-dependent kinase inhibitors, 121 Cyclins, 121 Cyclophilin, 88 Cyclosporin, 88, 89t, 90 Cyst(s) choledochal, 281, 285, 286f, 335 dermoid, 135 ganglion, 360 splenic, 303–304 thyroglossal duct, 323, 330 Cystic neoplasms, pancreatic, 295f Cytochrome c, 122

30/06/22 11:32 AM

397

D

Dabigatran, 24 Dacron patch aortoplasty, for coarctation of the aorta, 164 DALY. See Disability-adjusted life year Damage control resuscitation, 26 Damage-associated molecular patterns, 7–8, 10, 29, 37 DAMPs. See Damage-associated molecular patterns Damus-Kaye-Stansel anastomosis, 160 DCD. See Donation after cardiac death DCIS. See Distal carcinoma in situ DCP3, 374, 374t DCR. See Damage control resuscitation Death cardiac, organ donation after, 91 causes of, 97 medical errors as cause of, 97 Decompressive laparotomy, 69 Deep vein thrombosis, 200–202, 202t, 366 Defecation, 241 Delirium, postoperative, 365–366 Democratic leadership style, 3–4 Dermatitis, 294 Dermoid cysts, 135 DES. See Diffuse esophageal spasm Desarda repair, of inguinal hernias, 318, 318f Desmoid tumors of abdominal wall, 308 description of, 154 Desufflation, 115 Diabetes mellitus dermatitis and, 294 pancreatic endocrine tumors associated with, 294 questions regarding, 78, 123–124 Roux-en-Y gastric bypass effects on, 234 wound healing affected by, 78 Diagnostic peritoneal lavage, 59–60, 60t Diaphragm, penetrating trauma to, 59 Diaphragmatic hernia, 218, 329 DIC. See Disseminated intravascular coagulation Difficult airway algorithm, 364

Brunicardi_Index_p391-420.indd 397

Diffuse alveolar damage, 32 Diffuse axonal injury, 347 Diffuse esophageal spasm, 212 Digestion, carbohydrate, 236 Digit amputation, 356 Direct Coombs test, 298 Direct oral anticoagulants, 24 Direct thrombin inhibitors, 24, 205 Dirty wound, 40, 40t Disability-adjusted life year, 374 Disasters, 71 Dislocation knee, 352 shoulder, 352 Disseminated intravascular coagulation, 23 Distal aortic aneurysms, 183 Distal carcinoma in situ, 137–138 Distal ischemia, 107 Distal pancreatectomy, 54 Distant flaps, 361 Diversity, in surgery, 5 Diverticulectomy, 220 Diverticulitis, 247 Diverticulopexy, 220 Diverticulum Meckel, 235, 335 Zenker’s, 220 DKS anastomosis. See Damus-KayeStansel anastomosis DLCO. See Carbon monoxide diffusion capacity DNA eukaryotic, 119 replication of, 121–122 in RNA synthesis, 120 DO2. See Oxygen delivery DOACs. See Direct oral anticoagulants Dobutamine, 34 Dobutamine-stress echocardiography, 172 Donation after cardiac death, 91 Donation by non–heart-beating donors, 91 Donebedian model of measuring quality, 98 Dopamine, 34 Dor fundoplication, 221 Dormancy, 84 DORV. See Double-outlet right ventricle Double aortic arch, 167f “Double bubble” sign, 333, 334f Double-outlet right ventricle, 170 DPL. See Diagnostic peritoneal lavage Drug metabolism, 268

DTIs. See Direct thrombin inhibitors Dumping syndrome, 231 Duodenum adenocarcinoma of, 238 gastrinoma of, 294 hematoma of, 53 obstruction of, 333, 334f ulcers of, 226–227 Duplex ultrasonography, for venous thromboembolism, 204, 204f Durable power of attorney for health care, 370 DVT. See Deep vein thrombosis Dysphagia, 210

INDEX

Cytokines autophagy regulation of, 9 CD4+ T cell production of, 9 proinflammatory, 9 shock-related elevations of, 32 Th1, 9 Cytomegalovirus, 90

E

EA. See Esophageal atresia Ebstein’s anomaly, 169f EBV. See Epstein-Barr virus E-cadherin gene, 229 Eccrine sweat glands, 133 Echinococcal antigens, 273 Echocardiography dobutamine-stress, 172 transesophageal, 172–173 transthoracic, 172–173 Edmonton protocol, 94 EDP. See End-diastolic pressure EDS. See Ehler-Danlos syndrome EDV. See End-diastolic volume EE. See Eosinophilic esophagitis eFAST. See Extended focused abdominal sonography for trauma EGD. See Esophagogastroduodenoscopy EGFR. See Estimated glomerular filtration rate Ehler-Danlos syndrome, 75, 182, 315 Elderly abdominal aortic aneurysms in, 368 anticoagulation in, 367 appendicitis in, 367 cardiac function in, 366 colon cancer resection in, 367 deep venous thrombosis prophylaxis in, 366 falls in, 367 frailty in, 365 heart valve replacement in, 367 medications in, 366 necrotizing fasciitis in, 80 nutritional status in, 366 preoperative assessment and counseling in, 365

30/06/22 11:32 AM

398

INDEX

Elderly (Cont.): preoperative prehabilitation programs in, 367 surgery in, 365–368 transcatheter aortic valve implantation/replacement in, 368 trauma in, 63 Electrical burns, 67 Electrocardiography/ electrocardiogram electrical burn evaluations, 67 hyperkalemia signs, 13 hypocalcemia signs, 14 ICU monitoring uses of, 106 Elongation factors, 120 Emergency department thoracotomy, 49 Emergent orchiectomy, 320 Emergent tracheostomy, 56 Emesis, 330 Empyema, 102–103 EN. See Enteral nutrition; Epidermal necrolysis Encephalopathy hepatic, 269 transjugular intrahepatic portosystemic shunt as cause of, 273 End colostomy, 54 Endarterectomy, 92 End-diastolic pressure, 106–107 End-diastolic volume, 106–107 Endemic granulomatous disease, 148 Endoleaks, 185–186, 186t Endometriosis, 342–343 Endorectal advancement flap, for rectovaginal fistula, 253, 254f Endorectal ultrasound, 242–243 Endoscopic retrograde cholangiography, 281 Endoscopic retrograde cholangiopancreatography for chronic pancreatitis, 290–291 description of, 50, 53, 61, 102 for intraductal papillary mucinous neoplasms, 296 Endoscopic ultrasound of chronic pancreatitis, 291, 291f of esophageal cancer, 210 Endoscopy for hernia repair, 319 for ulcerative colitis, 257 for volvulus, 250–251 Endothelium platelet adherence to, 21 thrombomodulin from, 22

Brunicardi_Index_p391-420.indd 398

Endovascular repair of abdominal aortic aneurysms, 368 of thoracic artery aneurysms, 184, 192 End-tidal CO2, 111, 363 End-to-end anastomoses, 244, 245f End-to-side anastomoses, 244, 245f Enhanced recovery after surgery, 379 Enteral nutrition formulas, 9 hepatic failure treated with, 9 pancreatic necrosis and, 42 parenteral nutrition versus, 11 Enterocutaneous fistula, 238 Enterohepatic circulation, 236, 283 Enterolithotomy, laparoscopic, 284 Enzyme-linked receptors, 122 EOA. See Esophageal obturator airway Eosinophilic esophagitis, 220, 220f Eosinophilic granulomas, 155 Epidermal necrolysis, 133 Epidermis, 133 Epidural analgesia, 245 Epidural anesthesia, 101 Epinephrine anesthetic agents with, 356 questions regarding, 31 Epithelial ovarian cancer, 342 Epitope tags, 125 Epoprostenol, for pulmonary hypertension, 154 Epstein-Barr virus, 90 ERAS. See Enhanced recovery after surgery ERCP. See Endoscopic retrograde cholangiopancreatography Error-centric assessments, 387 Error-enabled approach, 387 Erythema nodosum, 247 Erythrocyte membrane proteins, 301 Erythromycin, 101 ESC. See Endoscopic retrograde cholangiography Eschar, 69 Escharotomies, 68 Escherichia coli, 272, 299 Esophageal atresia, 330, 331f Esophageal dilation, 219 Esophageal myotomy, 211 Esophageal obturator airway, 318 Esophagectomy description of, 209–210 minimally invasive surgery, 221–222 Esophagitis, 214, 214f, 220, 220f Esophagogastroduodenoscopy, 211

Esophagus antireflux, 214, 216–217, 217f blood supply to, 207, 208f cancer of, 210, 221 cervical, 207, 213 columnar-lined, 209 contractions of, 212 gastric juice exposure in, 215, 216f hernias of, 218, 218f–219f innervation of, 212f leiomyoma of, 211 lymphatic drainage of, 213f Mallory-Weiss tears of, 211 narrowing of, 207 nonperistalic contractions of, 212 pH monitoring of, 216f resection of, 209 rupture of, 211 Schatzki ring of, 219 squamous cell carcinoma of, 209 submucosa of, 213 thoracic, 213 tumors of, 209, 213 Essential thrombocythemia, 303 Estimated glomerular filtration rate, 148 Estrogen breast cancer and, 137 ductal development in breasts, 139 ET. See Essential thrombocythemia Ethics, 369–371, 390 Eukaryotes DNA, 119 gene expression, 119, 120f transcription in, 119 EUS. See Endoscopic ultrasound Evidence levels, 381, 381f Evidence-based hierarchy, 381f Ewing sarcomas, 154–155 Exocrine pancreatic secretions, 93 Experiential learning, 5 Extended focused abdominal sonography for trauma, 59 Extended left colectomy, 244 Extended right colectomy, 243 Extended simple mastectomy, 141 External sphincter, of rectum, 241–242 External validity, 383 Extracellular fluid volume deficit, 17–18 Extremities soft tissue sarcomas of, 312 vascular injuries of, 54–55, 55t Extrinsic pathway, 21 F

Facial fractures, 144 Factor IX, 21 Factor V, 22

30/06/22 11:32 AM

399

Brunicardi_Index_p391-420.indd 399

Fluid resuscitation albumin for, 14 for burns, 68 normal saline for, 18, 378 Fluid volume deficit, 17–18 Fluorodeoxyglucose-PET/CT, 269 FMD. See Fibromuscular dysplasia FNAB. See Fine needle aspiration biopsy FNH. See Focal nodular hyperplasia FOBT. See Fecal occult blood test Focal nodular hyperplasia, 275, 275f Focused assessment with sonography in trauma, 60 Fontan repair, 160 Forced expiratory volume in 1 second, 152 Forced-error approach, 387 Foreign bodies rectal, 255 retained, 99 Fournier’s gangrene, 339 Fourth-degree hemorrhoids, 252 Fractional inspiration of oxygen, 41 Fractional saturation of hemoglobin, 110 Fractures facial, 144 humeral shaft, 352 long bone, 351–352 pelvic, 351–352 reduction of, 351 rib, 63 sacral, 352 skull base, 346–347 Frailty, 365 Free flaps, 361 Free water deficit, 15 Fresh frozen plasma, for thrombotic thrombocytopenic purpura, 25 FTS. See Flexor tenosynovitis Full-thickness burns, 67–68 Full-thickness wounds, 75–76 Fundoplication Belsey, 221 Dor, 221 goal of, 217 Nissen, 116, 216 Fungi, 153 Furosemide, 18 G

G0 phase, of cell cycle, 83 G1 phase, of cell cycle, 83, 121 G2 phase, of cell cycle, 83, 121 Gain-of-function mutations, 121

Gallbladder acute cholecystitis of, 279–280 arterial anomalies of, 282 cancer of, 276, 280 lesions of, 281 lymphatics of, 280 polyps of, 286 wall layers of, 282 Gallstone(s) description of, 280 ileus, 284, 285f pancreatitis, 289 GALT. See Gut-associated lymphoid tissue Ganglion cyst, 360 Gangrenous appendicitis, 261f Gardner syndrome, 154 Gastrectomy dumping after, 231 partial, 62 Gastric antral vascular ectasia, 230–231, 231f Gastric banding, adjustable, 233 Gastric bypass hypoglycemia after, 234 Roux-en-Y. See Roux-en-Y gastric bypass Gastric cancer, 229 Gastric carcinoid tumors, 230 Gastric emptying, 224t Gastric injuries, 62 Gastric juice, esophageal exposure to, 215, 216f Gastric sleeve procedure, 234 Gastric stasis, after bariatric surgery, 224 Gastrin, 208, 223 Gastrinoma, 225–226, 294 Gastroduodenal arteries, 279 Gastroesophageal junction, 211, 216 Gastroesophageal reflux disease, 214, 216, 219, 234 Gastrografin, 236 Gastrointestinal anastomosis stapler, 221 Gastrointestinal bleeding, 238 Gastrointestinal stromal tumors, 230, 238, 249 Gastrointestinal tract, wound healing in, 75–76, 76t Gastroschisis, 335 Gastrosplenic ligament, 300, 301f GBM. See Glioblastoma multiforme GCS. See Glasgow Coma Scale GEJ. See Gastroesophageal junction Gene expression, 10, 124 General anesthesia, 245, 363, 366

INDEX

Factor VIII, 21–22 Factor X, 21 Factor Xa inhibitors, 24 Factor XI, 21 Factor XII, 21 Factor XIII deficiency, congenital, 22 Falciform ligament, 263, 263f Falls, 367 False aneurysms, 181 Familial adenomatous polyposis, 238, 308 FAP. See Familial adenomatous polyposis FAST. See Focused assessment with sonography in trauma Fasting, 364, 366 FBN-1 gene, 75 Fecal continence, 241–242 Fecal occult blood test, 256 Felon, 359, 359f Felty syndrome, 298, 304–305 Femoral hernia, 313, 317 Ferritin, 270 Fetal acidosis, 117 Fetal aortic valvotomy, 158 Fetal wound healing, 77 α-Fetoprotein, 155 FEV1. See Forced expiratory volume in 1 second FFP. See Fresh frozen plasma Fibrillin, 75, 181 Fibrin glue fixation, for inguinal hernia repair, 319 Fibrinogen, 21 Fibroblastic phase, of wound healing, 73f, 74 Fibroids, uterine, 341 Fibromas, 180 Fibromuscular dysplasia, 195 Fibroplasia, 78 Fine needle aspiration biopsy, of thyroid nodules, 324 First-degree burns, 66, 68 First-degree hemorrhoids, 252 Fistula enterocutaneous, 237 pancreatic, 294 rectovaginal, 253–255, 254f tracheoesophageal, 330, 331f FK506-binding proteins, 89 FKBPs. See FK506-binding proteins Flail chest, 58 Flaps, 361 Flexor tenosynovitis, 359 Fluid maintenance, 16

30/06/22 11:32 AM

400

INDEX

Generalizability, 383 Genomic surgery, 119–125 GERD. See Gastroesophageal reflux disease Geriatric patients necrotizing fasciitis in, 80 trauma in, 63 Geriatric syndromes, 365 GIA stapler. See Gastrointestinal anastomosis stapler GISTs. See Gastrointestinal stromal tumors Glasgow Coma Scale, 52, 52t, 63, 347, 347t Glenn procedure, bidirectional, 160, 160f Glioblastoma multiforme, 348, 348f Global surgery, 373–376 GLP-1, 234 Glucagon, 294 Glucagonoma, 294 Gluconeogenesis, 267 Glucose, hepatic production of, 267 Glucose transporter 2, 236 Glucose transporter 5, 236 Glucose-6-phosphate dehydrogenase deficiency, 300 GLUT2. See Glucose transporter 2 GLUT5. See Glucose transporter 5 Glutamine, 8 Glycogenolysis, 31, 267 Glycoprotein I/IX/V, 21 GnRH. See Gonadotropin releasing hormone Goal-directed therapy, 378 Goblet cells, 147, 147f, 209 Goleman, Daniel, 4 Gonadotropin releasing hormone, 343 Gore Propaten graft, 195 GP. See Glycoprotein I/IX/V GPCRs. See G-protein–coupled receptors G6PD. See Glucose-6-phosphate dehydrogenase deficiency G-protein–coupled receptors, 122 Graft loss, 93 Graft rejection, 90 Graft thrombosis, 93 Granuloma eosinophilic, 155 infectious, 148 Graves disease, 324 Great saphenous vein, 197 Greater omentum, 308 Groin hernia, 313, 314t Growth factor receptors, 122 GSV. See Great saphenous vein

Brunicardi_Index_p391-420.indd 400

Gunshot wounds, 50, 54 Gustillo-Anderson classification, 352 Gut-associated lymphoid tissue, 236, 259 G-value, 28 Gynecology, 341–343 Gynecomastia, 137–138, 140 H

Hair removal, 38 Hairy cell leukemia, 302 HAL. See Hand-assisted laparoscopy Halo test, 347 Halsted radical mastectomy, 141 Hamartomas, 148 Hand felon of, 359, 359f flexor tenosynovitis of, 359 high-pressure injection injuries to, 357 infections of, 359, 360f malignant tumor of, 360 necrotizing myositis of, 358 replantations, 356 squamous cell carcinoma of, 360 surgery of, 355–360 Hand-assisted laparoscopy, 243 Harris-Benedict equation, 69 Hasson approach, 117 HCC. See Hepatocellular carcinoma hCG. See Human chorionic gonadotropin HCL. See Hairy cell leukemia HD. See Hodgkin disease HDGC. See Hereditary diffuse gastric cancer Head and neck region, 143–144 Head injuries, 53 Healing bone, 77 wound. See Wound healing Health care facilities, 386 Heart disease aortic stenosis, 175 arrhythmias, 176 bypass conduits for, 173 congenital. See Congenital heart disease Cox-Maze IV procedure for, 177, 177f intra-aortic balloon pump for, 179 left ventricular assist devices for, 179 in Marfan syndrome, 175 mitral stenosis, 174–175 New York Heart Association Classification of, 171, 171f pericarditis, 178

pulmonary vein isolation for, 177 transesophageal echocardiography for, 172–173 transthoracic echocardiography for, 172–173 tricuspid stenosis, 176 Heart murmurs, 175 Heart transplantation, 95 Heart valve replacement, 174–175, 268, 367 Helicobacter pylori, 223–224, 230 Heller myotomy, 116, 221 Helper T cells, 10 Hemangiomas, 274–275, 362 Hematoma in bone healing, 77 duodenal, 53 Hemidesmosomes, 132 Hemodialysis, 18 Hemoglobin oxygen saturation of, 111 during shock resuscitation, 62 Hemolytic anemias, 298–300 Hemorrhage abdominal, 50 hepatic, angiography for, 61 intraperitoneal, 61 pelvic, 50 posttransplant, 92 variceal, 271 Hemorrhagic shock, 19, 30 Hemorrhoids, 252 Hemosiderin, 198 Hemostasis physiological events of, 21 primary, 21 Hemothorax, 53 Heparin, low molecular weight, 203, 366 Heparin-induced thrombocytopenia, 23–24, 173, 205 Hepatic abscess, 41–42 Hepatic artery anatomy of, 264, 265f anomalies of, 282 replaced right, 289 right, 282, 289 Hepatic duct confluence, 266, 267f Hepatic encephalopathy, 269 Hepatic failure, 9 Hepatic glycogenolysis, 31 Hepatic veins, 265, 266f Hepatitis C blood transfusion transmission of, 27 chronic carrier state, 44 chronic liver disease caused by, 270

30/06/22 11:32 AM

401

Brunicardi_Index_p391-420.indd 401

HPA axis. See Hypothalamicpituitary-adrenal axis HPV. See Human papillomavirus HS. See Hereditary spherocytosis HSV. See Herpes simplex virus HTS. See Hypertrophic scars Human bites, 133–134 Human chorionic gonadotropin, 155 Human embryonic stem cells, 124 Human genome, 121 Human immunodeficiency virus blood transfusion transmission of, 27 surgeon-to-patient transmission of, 43–44 transmission of, 27, 43–44 tuberculosis risks, 153 Human leukocyte antigen system, 87, 91–92 Human papillomavirus head and neck cancers, 143 malignancy risks, 135 vaccine for, 135, 342 Humeral shaft fractures, 352 Hurley classification, of hidradenitis suppurativa, 133 Hydatid disease, 273 Hydroxocobalamin, for cyanide poisoning, 66 Hydroxyethyl starch solutions, 14 Hyperacute rejection in transplantation, 88 Hyperbilirubinemia, 103 Hypercalcemia, acute, 18 Hyperchloremic metabolic acidosis, 18 Hypercoagulability, 30 Hypergastrinemia, 225, 225f Hyperglycemia in critically ill patients, 33 postoperative, 40 postoperative fasting as cause of, 377 Hyperkalemia signs and symptoms of, 13 treatment of, 15–16 Hyperreflexia, 16 Hypertonic saline, 14 Hypertriglyceridemia, 90 Hypertrophic scars, 80–81 Hypoalbuminemia, 16 Hypocalcemia, 14 asymptomatic, 16 hypomagnesemia and, 16, 19 Hypochloremic, hypokalemic metabolic alkalosis, 17 Hypocoagulability, 30 Hypoglycemia, post-gastric bypass, 234

Hypokalemia, 16 Hypomagnesemia, 16, 19 Hyponatremia, postoperative, 13 Hypoparathyroidism, 14 Hypoplastic left heart syndrome, 160–161, 169 Hypoplastic left ventricle, 159 Hypotension, in shock, 35 Hypothalamic-pituitary-adrenal axis, 8–9 Hypothermia, 40–41, 185 Hypovolemic shock, 29, 31–32 Hypoxia, intraoperative, 40–41

INDEX

Hepatobiliary iminodiacetic acid scan, 283, 283f Hepatocellular carcinoma, 276–277 Herbicide exposure, 311 Herceptin. See Trastuzumab Hereditary cancer, 83 Hereditary diffuse gastric cancer, 229 Hereditary hemochromatosis, 270 Hereditary nonpolyposis colorectal cancer, 247–248 Hereditary spherocytosis, 297, 301 Hernia abdominal, 270 abdominal wall, 307 congenital diaphragmatic, 329 diaphragmatic, 218 esophageal, 218, 218f–219f femoral, 313, 317 fibrin glue fixation for, 319 groin, 313, 314t hiatal, 218, 218f–219f incisional, 307–308 inguinal. See Inguinal hernias Nyhus classification of, 313, 313t paraesophageal, 218, 218f–219f recurrence of, 319, 321 Shouldice repair for, 321 transabdominal preperitoneal repair of, 321 Herpes simplex virus, 90 hESCs. See Human embryonic stem cells Heterotopic ossification, 70 Hiatal hernia, 218, 218f–219f HIDA scan. See Hepatobiliary iminodiacetic acid scan Hidradenitis suppurativa, 133 High reliability organization theory, 97 High-mobility group protein B1, 8 High-pressure injection injuries, 357 Hirschsprung disease, 334 Hispanics, 5 HIT. See Heparin-induced thrombocytopenia HLA system. See Human leukocyte antigen system HLHS. See Hypoplastic left heart syndrome HMGB1. See High-mobility group protein B1 HNPCC. See Hereditary nonpolyposis colorectal cancer HO. See Heterotopic ossification Hodgkin disease, 297 Holosystolic murmur, 175 Horner syndrome, 149

I

IABP. See Intra-aortic balloon pump IAP. See Intra-abdominal pressure ICP. See Intracranial pressure, increased Idarucizumab, 24 Idiopathic thrombocytopenic purpura, 23, 298, 302 IL-2. See Interleukin-2 IL-6. See Interleukin-6 IL-10. See Interleukin-10 IL-17. See Interleukin-17 Ileocolic anastomosis, 245f Ileocolostomy, 54 Ileum, 235 Ileus gallstone, 284, 285f postoperative, 101, 378 IM. See Intestinal metaplasia Imatinib, 230, 249 Iminodiacetic acid, 283 Immune response to injury, 9 platelet’s role in, 10 Immune thrombocytopenia, 23, 300, 302 Immune-based therapies, for cancer, 85 Immunonutrition supplements, 377 Immunosuppression cardiac transplantation, 95 drugs for, 88–90, 89t indications for, 87 lung abscess secondary to, 153 tuberculosis secondary to, 153 Immunotherapy, oncogene targeting by, 125 IMPDH. See Inosine monophosphate dehydrogenase Imperforate anus, 336 Impingement syndromes, 352–353 In situ cancers, 84 Incisional hernia, 307–308 Incontinence, stress, 342–343

30/06/22 11:32 AM

402

INDEX

Indwelling intravascular catheter infection, 43 Infection breast, 140 central line, 100 definition of, 37 hand, 359, 360f indwelling intravascular catheter, 43 after injury, 11 intravascular catheter, 43, 46–47 necrotizing, 46, 358 necrotizing fasciitis, 80 overwhelming postsplenectomy, 305–306 pancreatic, 42 postsplenectomy, 302, 305–306 posttransplantation, 90 retroperitoneal, 309 surgical site, 39t, 39–41, 47, 246 systemic manifestations of, 37 in transplant recipients, 90 wound, 102 Infectious granulomas, 148 Inferior thyroid artery, 207, 323 Inferior vena cava filters, 202, 203f suprahepatic, 265 Inflammatory bowel disease, 246–247 Inflammatory response injury-mediated, 8 in traumatic injury, 7–8 Informed consent, 369–370 Inguinal hernias acquired, 314 age-based incidence of, 313 antibiotic prophylaxis for, 316 collagen types in, 315, 316t congenital, 314, 315f Desarda repair of, 318, 318f extracellular matrix elements and, 315 fibrin glue fixation for, 319 incidence of, 313, 313t ischemic orchitis and, 320–321 McVay Cooper ligament repair of, 317, 317f nociceptive pain associated with, 320 Nyhus classification of, 313, 313t open repair of, 316 in pediatric patients, 334 in preterm infants, 314 prevalence of, 313t recurrence of, 319, 321 repair of, 316–317, 317f, 321, 334 risk factors for, 314–315 robotic-assisted repair of, 319

Brunicardi_Index_p391-420.indd 402

in scrotum, 316–317 Shouldice repair for, 321 transabdominal preperitoneal repair of, 321 Inhalational anthrax, 44 Injury complement cascade activation after, 32 gastric, 62 head, 53 immune response to, 9 infection after, 11 lactic acid production in response to, 17 life-threatening, 49, 59 metabolic support after, 7–11 metabolism after, 11 in pregnancy, 62 superior laryngeal nerve, 100 systemic response to, 7–11 Injury-mediated inflammatory response, 8 Inosine monophosphate dehydrogenase, 88 INR. See International normalized ratio Insensible fluid losses, 116 Institute of Medicine, 97 Institutional Review Board, 371 Insulin, 123–124 Insulin receptor substrate, 123–124 Insulin resistance in critically ill patients, 33 perioperative, 377 Insulin-signaling pathway, 123f Integrins, 84 Interatrial septum, 161 Interleukin-2, 9, 89 Interleukin-6, 7, 32 Interleukin-10, 9, 32 Interleukin-17, 11 Intermittent pneumatic compression therapy, 205 Internal hemorrhoids, 252 Internal sphincter, of rectum, 241 Internal thoracic artery, 173 International normalized ratio, 175, 268 Interstitial cells of Cajal, 279 Interstitial fluid compartments, 16 Intestinal ischemia, 251–252, 330 Intestinal malrotation, 308 Intestinal metaplasia, 209–210 Intestinal obstruction, 330 Intra-abdominal hypertension, 113 Intra-abdominal pressure, 112–113, 116

Intra-aortic balloon pump, 34, 179 Intra-arterial blood pressure monitoring system, 106 Intracranial pressure increased, 345, 349 monitoring of, 112 Intraductal papillary mucinous neoplasms, 296 Intraductal papillomas, 137 Intraperitoneal hemorrhage, 61 Intravascular catheter infections, 43, 46–47 Intravascular space, 14 Intravascular volume, pregnancyrelated changes in, 62 Intrinsic factor, 224, 234 Intrinsic pathway, 21 Intubation, 146 Intussusception, 335–336 IOM. See Institute of Medicine IPC. See Intermittent pneumatic compression therapy IPMNs. See Intraductal papillary mucinous neoplasms IRB. See Institutional Review Board IRS. See Insulin receptor substrate Ischemia acute limb, 194 distal, 107 intestinal, 251–252, 330 large bowel, 251–252 local tissue, 134 mesenteric, 192–193 pressure-induced, 134 small bowel, 251–252 Ischemic cardiomyopathy, 173 Ischemic colitis, 251–252 Ischemic load, 106 Ischemic orchitis, 320–321 Islet cell transplantation, 94–95 ITA. See Inferior thyroid artery ITP. See Idiopathic thrombocytopenic purpura; Immune thrombocytopenia IVC. See Inferior vena cava J

Jaw thrust, 58 JCAHO, 386 Jejunoileal bypass, 233 Jejunum, 235 K

Kayexalate, 15–16 Keloids, 80–81, 81f Keratin, 132 Keratinocytes, 132

30/06/22 11:32 AM

403

L

Lactated Ringer solution for burns, 68 intravenous colloid solution versus, for hemorrhagic shock, 30 for metabolic acidosis, 19 Lactic acid, 17 LAMN. See Low-grade appendiceal mucinous neoplasms Lancet Commission on Global Surgery, 375 Langerhans cell histiocytosis, 155 Langerhans cells, 133 Laparoscopic cholecystectomy acute cholecystitis treated with, 284 anesthesia for, 364 biloma after, 102 safety concerns for, 117 Laparoscopy abdominal compartment syndrome after, 103 adrenalectomy, 328 antireflux surgery, 216 appendectomy, 259–261 arrhythmia during, 115 cholecystectomy. See Laparoscopic cholecystectomy enterolithotomy, 284 gangrenous appendicitis on, 261f hand-assisted, 243 ileus after, 378 Nissen fundoplication, 116 in pregnancy, 117 robotic surgery versus, 116–117, 117f, 319 skills training in, 375–376

Brunicardi_Index_p391-420.indd 403

small bowel obstruction treatment, 237 splenectomy, 299, 305 Large bowel ischemia, 251–252 Larynx anatomy of, 146, 146f cancer of, 144 fractures of, 56 Lateral femoral cutaneous nerve injury, 320 LCGS. See Lancet Commission on Global Surgery LCH. See Langerhans cell histiocytosis LCIS. See Lobular carcinoma in situ Le Fort fractures, 144 Leadership communication in, 3–4 conflict resolution in, 3t, 4 principles of, 3, 3t styles of, 3–4 Leadership skills, 5 Learning experiential, 5 willingness to learn, 3t Left atrial pressure, 174 Left colectomy, 244 Left subclavian artery, proximal, 60–61 Left ventricle aneurysms of, 176 hypoplastic, 159 tricuspid atresia and, 168 Left ventricular assist devices, 179 Left ventricular outflow tract obstruction, 158 Left ventricular pressure-volume loops, 107 Left ventricular stroke volume, 110 Leiomyoma of colon, 249 of esophagus, 211 of uterus, 341 Leiomyosarcoma, 249 Lentigo maligna melanoma, 130 LES. See Lower esophageal sphincter Lesions gallbladder, 281 liver, 274–275, 275f Leukoplakia, vocal cord, 143 Life-threatening injuries, 49, 59 Ligament of Treitz, 42 Limb amputation, 356 Linitis plastica, 229 Lip cancer, 143 Lipid metabolism, 11 Lipodermatosclerosis, 198

Liver acute failure of, 269 adenomas of, 275, 275f anatomy of, 263, 263f–264f benign lesions of, 274–275, 275f Child-Turcotte-Pugh score of, 274, 274t cholangiocarcinoma of, 276 chronic disease of, 270 cirrhosis of, 270–271, 276, 280 colorectal metastases to, 277 drug metabolism in, 268 ducts of, 266, 267f failure of, 95 falciform ligament of, 263, 263f focal nodular hyperplasia of, 275, 275f glucose production by, 267 hemangiomas of, 274–275 hemorrhage of, angiography for, 61 hepatic artery of, 264, 265f hepatocellular carcinoma, 276–277 inflammatory bowel disease effects on, 246 lesions of, 268–269, 274–275 lobes of, 263, 264f Model for End-Stage Liver Disease, 95, 274 portal hypertension, 270–271 pyogenic abscesses of, 41, 272, 272f transplantation of, 95, 276–277, 280, 287 variceal hemorrhage of, 271 veins of, 265, 266f Liver trauma bleeding caused by, 61 description of, 50 Living donor kidney transplant, 91 Living donors, 91 Living wills, 370 LMICs. See Low- and middle-income countries LMWH. See Low molecular weight heparin Lobular carcinoma in situ, 137 Local anesthetic systemic toxicity, 364 Local tissue ischemia, 134 Long bone fractures, 351–352 Loop colostomy, 54 Low- and middle-income countries, 373–375 Low glycemic index diets, 377 Low molecular weight heparin, 203, 366 Lower esophageal sphincter, 208, 212, 214, 217f, 221 Lower extremity veins, 197

INDEX

Keyhole foraminotomy, 349 Kidney(s) magnesium homeostasis by, 16 transplantation of, 91–93, 92f Kidney grafts, 91–92, 92f Kidney transplant in adult recipients, 91–92 complications of, 92–93 graft thrombosis in, 93 living donor, 91 Kirkpatrick four-level scale, 388 Klatskin’s tumor, 276, 287 Knee dislocation, 352 KRAS mutations, 148 K-time, 28 Kulchitsky cells, 147, 147f Kupffer cells, 283

30/06/22 11:32 AM

404

INDEX

Low-grade appendiceal mucinous neoplasms, 260–261 Low-oxygen tension, 78 Luminal peptides, 223 Lumpectomy, 138, 140 Lund and Browder chart, 66 Lung abscess of, 152, 152t adenocarcinoma of, 148–149, 150f histology of, 147f Lung cancer algorithm for, 150f computed tomography of, 148 management of, 149 non-small-cell, 151 pathologic progression of, 148 predictive markers for, 149 prognostic markers for, 149 pulmonary function studies in, 152 signs and symptoms of, 149 staging of, 152 TNM staging of, 152 LVADs. See Left ventricular assist devices LVOT. See Left ventricular outflow tract obstruction LVSV. See Left ventricular stroke volume LY30, 28 Lymph nodes axillary, 137 breast cancer metastasis to, 139 colorectal cancer metastasis to, 248–249 mediastinal, 151 metastases, 139, 148 Lymphatic drainage of anorectal region, 241 of gallbladder, 280 Lymphatic massage, 205 Lymphedema, 205–206, 311 Lynch syndrome, 247–248 M

M phase, of cell cycle, 121 MACE. See Major adverse cardiac events Macrophages microbe response by, 37 questions regarding, 10, 37 in wound healing, 79 Mafenide acetate, 69 Magnesium deficiency of, 19 depletion of, 16 Magnesium citrate, 246

Brunicardi_Index_p391-420.indd 404

Magnetic resonance angiography, 184 Magnetic resonance cholangiopancreatography, 53 Magnetic resonance imaging computed tomography versus, 269 of double aortic arch, 167f glioblastoma multiforme on, 348f hernia recurrence evaluations, 319 mesenteric vein thrombosis on, 206 pheochromocytomas on, 327 Major adverse cardiac events, 172 Major histocompatibility complex, 87 Malaria, 27 Malignant hyperthermia, 363 Malignant mesothelioma, 156 Malignant wounds, 80 Mallory-Weiss tears, 211 Malnutrition/malnourished patients magnesium deficiency in, 19 total body water in, 15 wound healing, 79 Maloney dilator, 220 MALT lymphoma. See Mucosaassociated lymphoid tissue lymphoma Mammalian target of rapamycin inhibitors, 90 Mammography, 138 Marfan syndrome, 75, 175, 181, 183 Marginal artery of Drummond, 243 Marijuana smoking, 144 Marjolin’s ulcer, 80 Mass casualty incidents, 71 Massive resuscitation, 69 Mastectomy, 140–141 Maternal hypercarbia, 117 Matrix metalloproteinases, 74, 84 Mayo protocol, 276 May-Thurner syndrome, 201 MCT. See Medullary carcinoma of the thyroid McVay Cooper ligament repair, 317, 317f Mechanical bowel preparation, 246 Mechanical valve replacement, 174–175, 268 Mechanical ventilation nosocomial pneumonia caused by, 44 peak airway pressure monitoring, 111 Meckel diverticulum, 235, 335 Medial thyroid anlage, 323 Median nerve, 355, 355f Mediastinal lymphadenopathy, 151 Mediastinal mass, 155 Medical errors, 97

Medullary carcinoma of the thyroid, 325 Melanoma ABCDE initialism for, 130 acral lentiginous, 130–131 advanced, 131f anorectal, 250 cutaneous, 130, 132 description of, 130 excision of, 132 lentigo maligna, 130 metastasis of, 130 nodular, 130, 131f sentinel lymph node biopsy for, 132 superficial spreading, 130, 130f MELD. See Model for End-Stage Liver Disease MEN2A, 325 MEN2B, 325 Ménétrier disease, 231, 231f Mentees, 5 Mentoring, 5 Mentors, 5 Meralgia paresthetica, 320 6-Mercaptopurine, 88 Mesenteric circulation, 192–193 Mesenteric ischemia, 192–193 Mesenteric vein thrombosis, 206 Mesothelioma, 156 Meta-analyses, 381 Metabolic acidosis diagnosis of, 17 in hemorrhagic shock patients, 19 hyperchloremic, 18 laboratory tests for, 14 Lactated Ringer solution for, 19 metabolic alkalosis versus, 17 with normal anion gap, 13, 17 Metabolic alkalosis diagnosis of, 17 hypochloremic, hypokalemic, 17 metabolic acidosis versus, 17 Metabolic support, 7–11 Metabolism drug, 268 after injury, 11 Metastases Barrett esophagus, 208–209 colorectal cancer, 248–249 melanoma, 130 omental, 308 questions regarding, 84–85 splenic, 298 surgical resection of, 85 Methicillin-resistant Staphylococcus aureus, 129, 134 Methionine, 120

30/06/22 11:32 AM

405

Brunicardi_Index_p391-420.indd 405

MRSA. See Methicillin-resistant Staphylococcus aureus MSI. See Microsatellite instability MSLT-1, 132 mtDNA. See Mitochondrial DNA mTOR inhibitors. See Mammalian target of rapamycin inhibitors Mucosa-associated lymphoid tissue lymphoma, 230 Mucosal biopsy, in Ménétrier disease, 231, 231f Mucous cells, 147f Multidisciplinary teams, 5 Multiple organ dysfunction syndrome, 32 Murmur, holosystolic, 175 MVT. See Mesenteric vein thrombosis Mycophenolate acid, 88 Mycophenolate mofetil, 88, 89t Mycoses, 153 Mycotic aneurysms, 182–183 Myocardial infarction, cardiogenic shock and, 33 Myonecrosis, 358 Myotomy, 116, 221 Myxomas, cardiac, 179 N

N-acetylcysteine, 336 Nasojejunal feeding tubes, 42 National Nosocomial Infections Surveillance System, 47 National Surgical Quality Improvement Project, 99, 172 Nausea and vomiting, postoperative, 378 Near-total thyroidectomy, 324 NEC. See Necrotizing enterocolitis; Neuroendocrine carcinoma Neck injuries to, 52, 57 questions regarding, 143–144 Necrotizing enterocolitis, 332f, 332–333 Necrotizing fasciitis, 80, 339 Necrotizing infections, 46, 358 Necrotizing myositis, 358 Necrotizing soft-tissue infections, 42, 134 Needle thoracostomy decompression, of tension pneumothorax, 56, 58 Needlesticks, 43–44 Neisseria gonorrhoeae, 343 Nelson syndrome, 328 Neobladder, orthotopic, 337 Neostigmine, 101, 251

Nephroureterectomy, for Wilms tumor, 331 Nerve injuries, 77 NETs. See Neutrophil extracellular traps Neurapraxia, 77 Neuroendocrine carcinoma, 148 Neurogenic shock, 29–31, 35 Neuropathic pain, 320 Neurosurgery, 345–350 Neurotmesis, 77 Neutropenia in Felty syndrome, 298, 304–305 silver sulfadiazine as cause of, 68–69 Neutropenic enterocolitis, 255 Neutrophil(s) in phagocytosis, 74 polymorphonuclear, 74 Neutrophil extracellular traps, 10 New York Heart Association Classification, 171, 171f, 179 Newborn congenital diaphragmatic hernia in, 329 emesis in, 330 imperforate anus in, 336 intestinal obstruction in, 330 packed red blood cell transfusion in, 336 total body water in, 15 NHBDs. See Non–heart-beating donors NHL. See Non-Hodgkin lymphoma Nikolsky sign, 133 Nissen fundoplication gastric pressure after, 216 laparoscopic, 116 NNISS. See National Nosocomial Infections Surveillance System Nociceptive pain, 320 NOD2 gene, 237 Nodular basal cell carcinoma, 135 Nodular melanoma, 130, 131f Noncardiac surgery, 172 Non–heart-beating donors, 91 Non-Hodgkin lymphoma, 297, 299 Nonseminatous germ-cell tumors, 155 Non-small-cell lung cancer, 151 Nonsteroidal anti-inflammatory drugs, 226, 320 Nontechnical skills, 387 Norepinephrine, 31 Normal saline, 14, 18, 378 Normovolemia, 378 Northern blotting, 124 Norwood operation, 158

INDEX

Methionyl-tRNA, 120 Metoclopramide, 101 MEWS system. See Modified Early Warning Score system MH. See Malignant hyperthermia MHC. See Major histocompatibility complex Microbes, macrophage response to, 37 Microbial peritonitis, primary, 41 Microcirculation, 134 Micro-organisms, 45 MicroRNA, 119 Microsatellite instability, 247–249 Microvascular thrombosis, in thrombotic thrombocytopenic purpura, 24–25 Middle cerebral artery, 112 Midface fractures, 144 Milliosmoles, 15 Minimally invasive surgery colon resection, 243 esophagectomy, 221–222 questions regarding, 115–117 resection, 243 Mismatch repair, 247 Mithramycin, 18 Mitochondrial DNA, 8 Mitral stenosis, 174–175 Mitral valve replacement, 175 M&M conferences. See Morbidity and mortality conferences MMF. See Mycophenolate mofetil MMPs. See Matrix metalloproteinases Model for End-Stage Liver Disease, 95, 274 Modified Early Warning Score system, 108–109 Modified Norwood procedure, 160 Modified radical mastectomy, 141 MODS. See Multiple organ dysfunction syndrome Mohs micrographic surgery, 129 Molecular surgery, 119–125 Mondor disease, 140 Monocytes, 10 Monopolar coagulation, 116 Morbidity and mortality conferences, 4 Morpheaform basal cell carcinoma, 135 mOsm. See Milliosmoles Motilin, 208 Motor vehicle collisions, 59 MRCP. See Magnetic resonance cholangiopancreatography mRNA, 119–120

30/06/22 11:32 AM

406

INDEX

Nosocomial pneumonia, 44, 100 NSAIDs. See Nonsteroidal anti-inflammatory drugs NSCLC. See Non-small-cell lung cancer NSQIP. See National Surgical Quality Improvement Project Null hypothesis, 382 Nutrition for burn patients, 69 enteral. See Enteral nutrition parenteral, 11 Roux-en-Y gastric bypass-related deficiencies of, 234 in wound healing, 79 Nutritional status, 366 NYHA. See New York Heart Association Classification

Orthotopic transplants, 87 OSCE. See Objective structured clinical examination Osmolality, 15 Osmoles, 15 Osmotic pressure, 15, 16 Ostium secundum type defect, 157, 157f Outcome measures, 98, 98t Ovarian cancer, 342 Overwhelming postsplenectomy infection, 305–306 Oxygen delivery, 105 intraoperative, 41 during wound healing, 78 Oxygen saturation, 110 Oxygen utilization, 105 Oxytocin, 139

O

P

Obesity surgical management of, 233–234 total body water, 15 Objective structured clinical examination, 5 Obstetrics, 341 Obstructive pancreatography, 291f Obstructive shock, 29, 35 Obturator bypass, 194 Occult gastrointestinal bleeding, 238 Octreotide, 271 Octreotide scan, 293f, 327 Ogilvie syndrome, 101, 251 Oliguria, 112 OLT. See Orthotopic liver transplantation Omentum greater, 308 tumors of, 308 Oncogenes description of, 124 immunotherapy targeting of, 125 Online journal clubs, 389 Open splenectomy, 299, 305–306 Operative field, hair removal from, 38 Opioids, postoperative, 378 OPSI. See Overwhelming postsplenectomy infection Orchidopexy, 336 Orchiectomy, emergent, 320 Organ donation after cardiac death, 91 living donors, 91 Organ transplant, 87 Orthopedic surgery, 351–353 Orthotopic liver transplantation, 277 Orthotopic neobladder, 337

Brunicardi_Index_p391-420.indd 406

PAC. See Pulmonary artery catheters Pacesetting, 3–4 Packed red blood cell transfusion, 336 Paco2. See Partial pressure of carbon dioxide PAI-1. See Plasminogen activator inhibitor-1 Pain back, 352 multimodal strategies for, 378 neuropathic, 320 nociceptive, 320 Palliative care, surgical, 367, 370 Pampiniform plexus, 320 PAMPs. See Pathogen-associated molecular patterns Pancoast tumors, 149 Pancreas blunt trauma to, 53 contusions of, 53 cystic neoplasms of, 295f exocrine secretions, 93 exocrine tumors, 292, 293f fistula of, 294 infections of, 42 necrosis of, 42 neuroendocrine tumors of, 292 transplantation of, 93–95, 94f Pancreatectomy, distal, 54 Pancreatic cancer, 123 Pancreatic duct stent, 295 Pancreatic leak, 294–295 Pancreaticoduodenectomy, 286, 294–295 Pancreatitis acute, 42, 290 chronic, 290f–291f, 290–292, 292t

description of, 14 gallstone, 289 Ranson’s criteria for, 289, 289t Panel-reactive antibody assay, 92 PAOP. See Pulmonary artery occlusion pressure Papillary thyroid cancer, 281, 324 Paraesophageal hernia, 218, 218f–219f Parathyroid adenoma, 155 Parathyroid glands location of, 326 supernumerary, 325 Parenteral nutrition acute pancreatitis treated with, 290 enteral nutrition versus, 11 Parietal cells, 224, 231 Parkland formula, 68 Partial gastrectomy, 62 Partial pressure of carbon dioxide, 18 Partial-thickness burns, 67–68 Pass-Muir valve, 144 Patent ductus arteriosus, 162–163 Patent processus vaginalis, 314, 315f Pathogen-associated molecular patterns, 7–8, 10, 29, 37 Patient safety causes of death, 97 National Surgical Quality Improvement Project, 99 questions regarding, 97–103 Surgical Care Improvement Project, 98, 98t wrong-site surgeries, 99 Pattern recognition receptors, 7, 29 PCC. See Prothrombin complex concentrates PDGF. See Platelet-derived growth factor Peak airway pressure, 111 Pediatric patients biliary atresia in, 333 choledochal cyst in, 335 duodenal obstruction in, 333, 334f emesis in, 330 esophageal atresia in, 330, 331f gastroschisis in, bowel reduction for, 335 hemangiomas in, 362 Hirschsprung disease in, 334 imperforate anus in, 336 inguinal hernia repair in, 334 intestinal obstruction in, 330 intussusception in, 335–336 leading cause of death in, 333 Meckel diverticulum in, 335 N-acetylcysteine in, 336

30/06/22 11:32 AM

407

Brunicardi_Index_p391-420.indd 407

PGBH. See Post-gastric bypass hypoglycemia Phagocytes, 77 Phagocytosis, 74 Pheochromocytoma, 327 Pheromones, 133 Phlegmasia alba dolens, 201 Phlegmasia cerulea dolens, 201, 201f Phrenic nerve palsy, 149 Phrenosplenic ligament, 300 Physician authorship, 390 Physicians diversity of, 5 laparoscopic skills training in, 375–376 Physiologic monitoring of surgical patient, 105–113 PICO, 382 PID. See Pelvic inflammatory disease p15INK4B, 123 “Pipeline” program, 5 Pituitary fossa tumor, 328 Plasma fresh frozen, for thrombotic thrombocytopenic purpura, 25 interstitial fluid compartments and, 16 Plasminogen, 22 Plasminogen activator inhibitor-1, 22 Plastic surgery, 361–362 Plateau airway pressure, 111 Platelet(s) acquired hemostatic defect, 23 aggregation of, 21, 23, 25 circulating, decreases in, 22 drugs that inhibit function of, 23 endothelial adherence of, 21 in immune response, 10 life span of, 23 postsplenectomy increase in, 51 sequestration of, 22, 25 shortened survival of, 22 Platelet-derived growth factor, 74 Plicae circularis, 235 Plummer-Vinson syndrome, 143 PN. See Parenteral nutrition PNETs. See Primitive neuroectodermal tumors Pneumatic tourniquet, 70 Pneumatosis intestinalis, 332f, 332–333 Pneumohemothorax, 57 Pneumonia aspiration, 153 community-acquired, 44 empyema caused by, 102 nosocomial, 44, 100 ventilator-associated, 100

Pneumoperitoneum, 115–116 Pneumothorax, 53 in laparoscopic Nissen fundoplication, 116 simple, 56, 58 tension. See Tension pneumothorax Poikilothermia, 194 Point of critical oxygen delivery, 105 Poisoning, cyanide, 66 Polycythemia vera, 303 Polyethylene glycol, 246 Polymorphonuclear neutrophils, 74 Polyps, gallbladder, 286 Polytetrafluoroethylene, 195 Popliteal artery, adventitial cystic disease of, 196 Portal hypertension, 270–271, 304 Portal hypertensive gastropathy, 231 Positive end-expiratory pressure, for acute respiratory distress syndrome, 100–101 Positron emission tomography, for mediastinal lymph node staging, 151 Posterior cervical foraminotomy, 349 Posterior fossa lesions, 346, 346f Post-gastric bypass hypoglycemia, 234 Postoperative period delirium in, 365–366 empyema in, 102–103 hyperglycemia in, 40 hyponatremia in, 13 ileus in, 101, 378 nausea and vomiting in, 378 opioid requirements in, 378 urinary tract infection in, 42–43 Postsinusoidal hypertension, 270 Potassium alkalosis-related changes in, 15 elevated levels of. See Hyperkalemia serum, 15–16 Pouchitis, 256 Ppeak. See Peak airway pressure PPIs. See Proton pump inhibitors Pplateau. See Plateau airway pressure PPV. See Patent processus vaginalis; Pulse pressure variability PRA assay. See Panel-reactive antibody assay Pragmatic Randomized Optimal Platelet and Plasma Ratios trial, 26 Prasugrel, 23 pRB, 123 Predictive marker, 85

INDEX

necrotizing enterocolitis in, 332f, 332–333 packed red blood cell transfusion in, 336 pneumatosis intestinalis in, 332f, 332–333 short bowel syndrome in, 239 thyroglossal duct cyst in, 330 tracheoesophageal fistula in, 330, 331f undescended testes in, 336 Wilms tumor in, 331, 332t Pediatric surgery, 329–336 Pedicle flaps, 361 PEEP. See Positive end-expiratory pressure PEG. See Polyethylene glycol Pelvic floor dysfunction, 342 Pelvic fractures, 351–352 Pelvic hemorrhage, 50 Pelvic inflammatory disease, 343 Pemetrexed, 149 Penetrating trauma to chest, 53 to colon, 54 diaphragmatic injury from, 59 to neck, 57 to thoracoabdominal region, 59 Pentaxins, 7 Peptic ulcer disease antrectomy for, 227f–228f, 227–228 description of, 223, 226 primary hyperparathyroidism associated with, 326 Percutaneous tracheostomy, 101 Perforated appendicitis, 259 Perianal abscess, 253 Pericardial adhesions, 178 Pericardial constriction, 178 “Pericardial knock,” 178 Pericardial tamponade in left subclavian artery trauma, 61 traumatic, 55 Pericarditis, 178 Pericholangitis, 246 Perihilar cholangiocarcinoma, 287 Perihilar tumor, 281 Perioperative care, 377–379 Peripheral arterial injury, 54–55, 55t Peritonitis in elderly, 367 in pediatric patients, 335 primary microbial, 41 Permissive hypotension, 26 PET. See Positron emission tomography Petco2, 111 Peyer patches, 235–236

30/06/22 11:32 AM

408

INDEX

Pregnancy breast cancer in, 139 breast changes in, 137 injuries during, 62 laparoscopy in, 117 physiologic changes in, 62 Preload, 106–107, 110 Preoperative evaluation, 377–378 Preoperative prehabilitation programs, 367 Pressure ulcers, 134 Priapism, 338 Primary aldosteronism, 327 Primary hemostasis, 21 Primary hyperparathyroidism, 326 Primary immune thrombocytopenia, 23 Primary microbial peritonitis, 41 Primary sclerosing cholangitis, 246, 276 Primary survey, of trauma, 49, 59 Primitive neuroectodermal tumors, 154 Principalist approach, to medical ethics, 369 Process measures, 98 Proficiency-based simulation training, 388 Progesterone, 139 Prognostic marker, 85 Proinflammatory cytokines, 9 Prokaryotes, 119 Prokinetic agents, 224, 224t Prolactin, 139 Prolapse, rectal, 245, 250 Proliferative phase, of wound healing, 73f, 74 Promoter region, 119 Prophylactic antibiotics for colectomy, 246 for human bites, 133–134 for inguinal hernias, 316 for overwhelming postsplenectomy infection, 306 for surgical infections, 38–39, 45, 102 PROPPR trial. See Pragmatic Randomized Optimal Platelet and Plasma Ratios trial Prostaglandin E, 168 Prostate cancer, 338 Prostatitis, 338 Prosthetic grafts, 195 Protein C, 22 Protein synthesis, 120 Prothrombin, 21 Prothrombin complex concentrates, 24 Proton pump inhibitors, 216, 226 Proto-oncogenes, 154

Brunicardi_Index_p391-420.indd 408

Proximal left subclavian artery, 60–61 PRRs. See Pattern recognition receptors Pseudoaneurysms, 181, 183 Pseudohyponatremia, 13 Pseudomyxoma peritonei, 260 Pseudo-obstruction, colonic, 251 Pseudostratified ciliated columnar cells, 147f PTC. See Papillary thyroid cancer PTFE. See Polytetrafluoroethylene PUD. See Peptic ulcer disease Pudendal nerve, 242 Pulmonary artery banding, for truncus arteriosus, 165 Pulmonary artery catheters, 109, 109f Pulmonary artery occlusion pressure, 106–107, 109 Pulmonary artery slings, 166, 167f–168f Pulmonary blood flow, 162 Pulmonary contusions, 58, 63 Pulmonary edema, 14, 174 Pulmonary embolism, 101, 200 Pulmonary function studies, in lung cancer, 152 Pulmonary hypertension, 154 Pulmonary vein isolation, 177 Pulmonary venous obstruction, after total anomalous pulmonary venous connection repair, 165, 166f Pulmonary-to-systemic flow ratio, 162 Pulse pressure, 17 Pulse pressure variability, 110, 110f Pulse volume recording, 190 Pulsus paradoxus, 55 Push enteroscopy, 238 PVI. See Pulmonary vein isolation Pylephlebitis, 41 PVR. See Pulse volume recording Pyogenic abscesses, 41, 272, 272f Pyogenic pericarditis, 178 Pyruvate kinase deficiency, 301 Q

Quality measurements, Donebedian model of, 98 Quid chewing, 144 R

Radial nerve, 355, 355f Radiation enteritis, of small intestine, 238–239 Radiation therapy for breast cancer, 140 description of, 138

dysphagia treated with, 210 soft tissue sarcoma risks, 311 Radical lymphadenectomy, 238 Radical mastectomy, 141 Randomized controlled trials, 381 Ranson’s criteria, 289, 289t RCC. See Renal cell carcinoma RCTs. See Randomized controlled trials Reading frame, 120 Reasonable person standard, 370 Reconstructive surgery, 361–362 Rectal biopsy, for Hirschsprung disease, 334 Rectal cancer endorectal ultrasound for, 242–243 total mesorectal excision for, 249 Rectopexy, abdominal, 250 Rectovaginal fistula, 253–255, 254f Rectum adenocarcinoma of, 256 distention of, 241 foreign body entrapment in, 255 layers of, 256 prolapse of, 245, 250 questions regarding, 241–257 wall of, 242, 242f Rectus abdominis diastasis, 307 Recurrent laryngeal nerve, 212–213, 323 Refeeding syndrome, 19 Regional anesthesia, 245 Renal artery disease, 193 Renal artery revascularization, 193 Renal blood flow intra-abdominal pressure effects on, 116 pneumoperitoneum effects on, 115 Renal cell carcinoma, 337, 339 Renal failure, 91 Renal tubular acidosis, 17 Replantation, of limb or digit, 356 Resection of colon, 243–244, 378 of colorectal cancer, 248, 367 of distant metastases, 85 laparoscopic, of colon, 378 minimally invasive, 243 Restrictive cardiomyopathy, 178 Resuscitation for burns, 69 endpoints in, 35 fluids for. See Fluid resuscitation for shock, 35 Resuscitative thoracotomy, 49 RET tyrosine kinase receptor gene, 324–325

30/06/22 11:32 AM

409

S

S phase, of cell cycle, 83, 121 SAA. See Splenic artery aneurysm Saccular aneurysms, 181 Sacral fractures, 352 Sacroiliitis, 246

Brunicardi_Index_p391-420.indd 409

Saddle anesthesia, 350 Saline hypertonic, 14 normal, 14, 18 Salivary gland-type tumors, 147 Sarcoma breast, 312 Ewing, 154–155 soft tissue, 311–312 Scar hypertrophic, 80–81 remodeling of, 75 SCC. See Squamous cell carcinoma Schatzki ring, 219 Schwann cell basal lamina, 77 Scimitar sign, 196 SCIP. See Surgical Care Improvement Project Sclerosing adenosis, 137 Sclerosing cholangitis, 246, 276, 280 Sclerosing mesenteritis, 309 Scrotum, 316–317 Scurvy, 79 Seborrheic keratoses, 136 Second-degree burns, 67 Second-degree hemorrhoids, 252 Second-impact syndrome, 347 Secretin stimulation test, 225–226 “Seed and soil” theory, of metastasis, 84 SEER registry, 248 Seldinger technique, 109 Self-expanding stents, 189, 282 Sengstaken-Blakemore tube, 211 Sentinel lymph node biopsy for breast cancer, 139 for melanoma, 132 Sepsis incidence of, 43 postsplenectomy, 51, 299 survival rates for, 43 Sepsis syndrome, 46 Septic shock characteristics of, 32 hyperglycemia in, 33 insulin resistance in, 33 treatment of, 33 Sequential Organ Failure Assessment score, 38 Serine/threonine kinase receptors, 122 Serotonin, 21 Serum calcium, 15 Serum osmolality, 15 Serum sodium, 18 Sestamibi scans, 326 Sex hormones, 133 Sexually transmitted infections, 135

SGLT1. See Sodium glucose co-transporter 1 Shock blood loss percentages, 32–33 cardiogenic, 29, 34, 55–56, 56f categories of, 29 compensated, 35 cytokine elevations after, 32 hemorrhagic, 19, 30 hypotension in, 35 hypovolemic, 29, 31–32 neurogenic, 29–31, 35 obstructive, 29, 35 questions regarding, 29–36 resuscitation for, 35, 62 septic, 32–33 signs and symptoms of, 32–33 tension pneumothorax as cause of, 29 trauma-induced coagulopathy caused by, 23 traumatic, 29 treatment of, 33–34 vasodilatory, 33 vasogenic, 29 Short bowel syndrome, 239 Shoulder dislocation, 352–353 Shouldice repair, 321 SICU. See Surgical intensive care unit Side-to-side anastomoses, 244, 245f Sigmoid colectomy, 244, 250 Sigmoidectomy, 256 Silver sulfadiazine for burn wounds, 68–69 neutropenia caused by, 68–69 Simple pneumothorax, 56, 58 Simulation, 387–388 Simulation training, 388 Sinus bradycardia, 115 Sirolimus, 89t, 90 SIRS. See Systemic inflammatory response syndrome SJS. See Stevens-Johnson syndrome Skin epidermis, 133 Langerhans cells of, 133 lesions of, 135–136 partial thickness wounds of, 132 stratum basale of, 132 stratum spinosum of, 133 wound healing of, 76t Skin cancer basal cell carcinoma, 129, 135 melanoma. See Melanoma squamous cell carcinoma, 130, 136 Skin-sparing mastectomy, 141 Skull base fractures, 346–347

INDEX

Retained surgical items, 99, 99t, 103 Retrograde embolization, 108 Retrograde pyelogram, 338 Retroperitoneal fibrosis, 309 Retroperitoneal infections, 309 Retroperitoneum, 309 Retropubic urethropexy, 343 Reverse-transcriptase PCR, 124 Reynolds pentad, 284 Rhabdomyomas, 180 Rheumatic disease, 175–176 Rheumatic fever, 174–175 RI. See Rothman Index Rib(s) fractures of, 63 “onion-peel” appearance of, 154 Right colectomy, 243 Right hemicolectomy, 260 Right hepatic artery, 282, 289 Right ventricular outflow tract obstruction, 161, 170 Rituximab, 302 Rivaroxaban, 24 RNA, 120 RNA gel blot analysis, 124 RNA polymerases, 119 Robotic prostatectomy, 117 Robotic surgery description of, 243 inguinal hernia repair using, 319 laparoscopic surgery versus, 116–117, 117f, 319 transabdominal preperitoneal, 319 Rothman Index, 108 Roux-en-Y gastric bypass complications of, 233–234 gastric stasis after, 224 illustration of, 228f nutritional deficiencies after, 234 small bowel obstruction after, 234 Roux-en-Y hepaticojejunostomy, 285, 287 RT. See Resuscitative thoracotomy Rubber band ligation, for hemorrhoids, 252 “Rule of nines,” 65, 66f R-value, 28 RVOT. See Right ventricular outflow tract obstruction RYGB. See Roux-en-Y gastric bypass

30/06/22 11:32 AM

410

INDEX

SLNB. See Sentinel lymph node biopsy SMA. See Superior mesenteric artery SMAD4 protein, 123 Small bowel obstruction closed loop, 236 laparoscopic surgery for, 237 after Roux-en-Y gastric bypass, 234 treatment of, 237 water-soluble contrast agent imaging of, 236 Small intestine Crohn disease of, 237 enterocutaneous fistula of, 237 fluid in, 235 ileum, 235 ischemia of, 251–252 jejunum, 235 length of, 235 malignancies of, 237 mesentery of, 308 obstruction of. See Small bowel obstruction questions regarding, 235–239 radiation enteritis of, 238–239 short bowel syndrome, 239 transplantation of, 239 Small saphenous vein, 197 Smoke inhalation injury, 66 Smoking, 144 “Sniff Test,” 149 SOAPs. See Surgical, anesthetic, and obstetrical providers Social media, 389–390 Sodium aldosterone-mediated reabsorption of, 17 serum concentration of, 18 Sodium chloride, 14, 18 Sodium glucose co-transporter 1, 236 Sodium thiosulfate, for cyanide poisoning, 66 SOFA score. See Sequential Organ Failure Assessment score Soft tissue sarcomas, 311–312 Soft-tissue infections, necrotizing, 42, 134 Solid organ injuries, 61 Somatostatin, 293 Somatostatin receptor scintigraphy, 226 Somatostatin receptors, 292 Somatostatinoma, 293 Sorafenib, for hepatocellular carcinoma, 277 Source control, 38 Southern blotting, 124

Brunicardi_Index_p391-420.indd 410

Sphincter of Oddi, 279, 290 Spinal cord ischemia, 183 Spleen accessory, 300 aneurysms of, 304 cysts of, 303–304 functions of, 301 hereditary spherocytosis of, 297, 301 ligaments of, 300, 301f metastases to, 298 platelet sequestration in, 22 questions regarding, 297–306 traumatic injury to, 51 Splenectomy cancer risks after, 299 for chronic hemolytic anemia, 300 in chronic myelogenous leukemia, 303 complications of, 305–306 description of, 54 indications for, 297–299 infections after, 302, 305–306 laparoscopic, 299, 305 for neutropenia in Felty syndrome, 304–305 open, 299, 305–306 overwhelming postsplenectomy infection, 305–306 in polycythemia vera, 303 sepsis after, 51, 299 Splenic artery, 299 Splenic artery aneurysm, 304 Splenic vein thrombosis, 304 Splenocolic ligament, 300, 301f Splenomegaly, 299 Splenorenal ligament, 300 Splenorrhaphy, 51 Split-thickness skin grafts, 361 Spontaneous abortion, 22 Squamocolumnar junction, 209 Squamous cell carcinoma of esophagus, 209 of hand, 360 of lung, 149 precursor lesion to, 136 of skin, 130 “Square-root sign”, 178 SSIs. See Surgical site infections SSTRs. See Somatostatin receptors SSV. See Small saphenous vein Stab wounds, 59, 60t Staging of breast cancer, 137 of colorectal cancer, 248 of lung cancer, 152 of sarcoma, 312 of Wilms tumor, 332t

Staphylococcus epidermidis, 46 Staphylococcus sp. description of, 134 S. aureus, 140, 359 Steatorrhea, in Crohn’s disease, 268 Stellate cells, 236 Stents balloon-expandable, 189 pancreatic duct, 295 self-expanding, 189, 282 ureteral, 338 vascular, 189–190 Steroids. See Corticosteroids Stevens-Johnson syndrome, 133 Stewart-Hamilton equation, 108 Stomach carcinoid tumors of, 230 duodenal ulcers, 226–227 gastric antral vascular ectasia of, 230–231, 231f gastrin secretion, 223 gastrointestinal stromal tumors of, 230 Ménétrier disease of, 231, 231f parietal cells of, 224, 231 peptic ulcer disease of, 223, 226–228, 227f–228f surface epithelial cells of, 231 Zollinger-Ellison syndrome of, 225, 231 Stratum basale, 132 Stratum spinosum, 133 Streptococcus sp., 134 Stress, caloric adjustments after, 11 Stress incontinence, 342–343 Stroke, 190, 346f Structural measures, 98 Stryker device, 51 Sturge-Weber syndrome, 362 Subclavian artery malperfusion of, 187 proximal left, 60–61 Subfalcine herniation, 345 Sublay technique, for mesh repair of abdominal incisional hernia, 308 Subphrenic abscess, 51 Subtotal colectomy, 244 Subtotal parathyroidectomy, 326 Subvalvular aortic stenosis, 163 Superficial burns, 66 Superficial spreading basal cell carcinoma, 135 Superficial spreading melanoma, 130, 130f Superior laryngeal nerve injury, 100, 323

30/06/22 11:32 AM

411

Brunicardi_Index_p391-420.indd 411

source control, 38 treatment of, 46 Surgical innovation, 371 Surgical intensive care unit, 50, 61 Surgical palliative care, 367, 370 Surgical research, 371 Surgical site infections, 39t, 39–41, 47, 246 Surgical wounds, 39–40, 40t, 47 Sustentaculum lienis, 301f SvO2. See Fractional saturation of hemoglobin SVT. See Suppurative thrombophlebitis Sweat glands, 133 Syphilis ascending aortic aneurysms caused by, 183 blood transfusion transmission of, 27 Syphilitic aortitis, 183 Systemic inflammatory response syndrome, 37 T

T cells CD4+, 9 CD8+, 9 helper, 10 T regulatory cells, 9 TACO. See Transfusion-related circulatory overload Tacrolimus, 88–89, 89t Tamoxifen, 138 TAPP. See Transabdominal preperitoneal repair TAPVC. See Total anomalous pulmonary venous connection Task sharing, 373 TAVI/TAVR. See Transcatheter aortic valve implantation/ replacement TBI. See Traumatic brain injury TBSA. See Total body surface area TBW. See Total body water; Total body weight TCD. See Transcranial Doppler; Transcranial power Doppler T-cell–mediated rejection, 95 TCL. See Transverse carpal ligament TEE. See Transesophageal echocardiography TEF. See Tracheoesophageal fistula TEG. See Thromboelastography Telemanipulation, 243 Temporal bone fracture, 346

TEN. See Toxic epidermal necrolysis Tension pneumothorax cardiac failure caused by, 55 diagnosis of, 35 diagnostic criteria for, 58 needle thoracostomy decompression of, 56, 58 obstructive shock versus, 35 shock caused by, 29 signs of, 56, 58 simple pneumothorax versus, 56, 58 tube thoracostomy for, 56, 57f TEP. See Totally preperitoneal repair Testes, undescended, 336 Testicular cancer, 337, 339 Tetralogy of Fallot, 161, 170 TF. See Tissue factor TFPI. See Tissue factor pathway inhibitor TGF-ß. See Transforming growth factor-ß Th1 cells, 10 Th1 cytokines, 9 Th2 cells, 10 Th17 cells, 10 Thalassemia, 302 The Joint Commission, 99 Thermal wounds, 129 Thermodilution method, for cardiac output, 108 Third-degree burns, 67 Third-degree hemorrhoids, 252 Thoracic aorta disease, 184 Thoracic aortic disease, 183 Thoracic compartment syndrome, 68 Thoracic escharotomies, 68 Thoracoabdominal region, penetrating trauma to, 59 Thoracotomy emergency department, 49 for empyema, 102 indications for, 53 trap door, 60 Thrombin, 21–22, 205 Thrombocytopenia causes of, 22 heparin-induced, 23–24, 173, 205 primary immune, 23 Thromboelastography, 27–28 Thromboembolism, venous, 200, 202, 204, 204f, 366 Thrombolytic therapy catheter-directed, 201 contraindications for, 201 Thrombomodulin, 22 Thrombopoietin receptor antagonists, 302

INDEX

Superior mesenteric artery anatomy of, 265f injuries to, 61 in mesenteric ischemia, 192 stenosis of, 192 Superior thyroid artery, 323 Supernumerary parathyroid glands, 325 Suppurative thrombophlebitis, 203 Supracolic injuries, 61 Surface epithelial cells, gastric, 231 Surgery ambulatory, 385–386 anorectal melanoma treated with, 250 after bare coronary stent placement, 364 bowel preparation for, 246 cost-effectiveness of, 374f diversity in, 5 in elderly, 365–368 genomic, 119–125 global, 373–376 hand, 355–360 major adverse cardiac events screening, 172 minimally invasive, 115–117 molecular, 119–125 noncardiac, 172 obesity treated with, 233–234 orthopedic, 351–353 pediatric, 329–336 physiologic monitoring during, 105–113 plastic, 361–362 reconstructive, 361–362 retained surgical items, 99, 99t, 103 robotic, 116–117, 117f women in, 5 wrist, 355–360 wrong-site, 99 Surgical, anesthetic, and obstetrical providers, 375 Surgical care, globalization of, 376 Surgical Care Improvement Project, 98, 98t Surgical excision of basal cell carcinoma, 129 of burn wounds, 70 of melanoma, 132 Surgical infection definition of, 37 intravascular catheters as cause of, 46–47 prophylactic antibiotics for, 38, 45 questions regarding, 37–47 of retroperitoneum, 309

30/06/22 11:32 AM

412

INDEX

Thrombosis catheters and, 107 deep vein, 200–202, 202t, 366 graft, 93 mesenteric vein, 206 microvascular, 24–25 splenic vein, 304 Thrombotic thrombocytopenic purpura, 23–25 Thymomas, 155 Thyroglobulin, 324 Thyroglossal duct cyst, 323, 330 Thyroid cancer papillary, 281, 324 treatment of, 325 Thyroid gland arterial supply of, 323 congenital anomaly of, 323 Thyroid hormones, 324 Thyroid nodules, 324 Thyroid surgery, 325 Thyroidectomy, 323–324 Thyroxine, 324 TIC. See Trauma-induced coagulopathy Time management, 3 TIPS. See Transjugular intrahepatic portosystemic shunt Tissue factor, 21 Tissue factor pathway inhibitor, 22 Tissue injury, 23 Tissue plasminogen receptor, 22 TKIs. See Tyrosine kinase inhibitors TLRs. See Toll-like receptors TM. See Thrombomodulin TME. See Total mesorectal excision TNM staging of colorectal cancer, 248 of lung cancer, 152 To Err Is Human, 4 TOF. See Tetralogy of Fallot Toll-like receptors, 7, 9–10, 37 Tonsillar herniation, 345, 345f Total anomalous pulmonary venous connection, 159, 159f, 165–166, 166f Total body surface area, 65–66, 66f Total body water, 14–15 Total body weight, 14–15 Total colectomy, 244 Total mesorectal excision, for rectal cancer, 249 Total parathyroidectomy, 326 Total thyroidectomy, 324 Totally preperitoneal repair, 321 Tourniquet, 356 Toxic epidermal necrolysis, 133

Brunicardi_Index_p391-420.indd 412

Toxic shock syndrome, 134 tPA. See Tissue plasminogen receptor Trachea anatomy of, 146, 146f blood supply to, 145, 145f cricoid cartilage of, 146, 146f injuries to, 51 stenosis of, 146 upper, 146, 146f Tracheobronchial injury, 59 Tracheobronchial tree, 147, 147f Tracheoesophageal fistula, 330, 331f Tracheostomy description of, 51 emergent, 56 percutaneous, 101 questions regarding, 144 tracheal stenosis caused by, 146 Tracheostomy tube, 144 Traditional Chinese medicine, 379 TRALI. See Transfusion-related acute lung injury Tranexamic acid, 27 Transabdominal preperitoneal repair, 321 Transcatheter aortic valve implantation/replacement, 368 Transcranial Doppler, 112 Transcranial power Doppler, 191 Transcription in eukaryotes, 119 in prokaryotes, 119 Transesophageal echocardiography, 172–173 Transforming growth factor-ß, 123 Transfusion complications of, 26 description of, 25 diseases not transmitted by, 27 indications for, 25–26 Transfusion-related acute lung injury, 25–27 Transfusion-related circulatory overload, 27 Transhiatal esophagectomy, 209 “Transition wound,” 77 Transjugular intrahepatic portosystemic shunt, 271, 273 Translation, 120 Transmural infarction, 176 Transplantation azathioprine in, 88, 89t bleeding after, 92 cyclosporin in, 88, 89t definition of, 87 donation after cardiac death, 91 graft loss, 93

graft rejection in, 90 graft thrombosis after, 93 heart, 95 hyperacute rejection in, 88 immunosuppression in, 87, 89t, 95 infections in recipients, 90 islet cell, 94–95 kidney. See Kidney transplant liver, 95, 276–277, 280, 287 Mayo protocol for, 276 mycophenolate mofetil in, 88, 89t orthotopic liver, 277 pancreas, 93–95, 94f questions regarding, 87–95 rejection in, 88, 90, 95 screening for, 92 small intestine, 239 tacrolimus in, 88–89, 89t whole-pancreas, 95 Transposition of the great vessels, 161 Transthoracic echocardiography, 172–173 Transurethral resection of the prostate, 339 Transverse aortic arch aneurysms, 185 Transverse carpal ligament, 357 Transverse colectomy, 244 Trap door thoracotomy, 60 Trastuzumab, 125 Trauma. See also Blunt trauma; Penetrating trauma airway interventions in, 57–58 cardiogenic shock in, 55–56, 56f gastric injuries from, 62 in geriatric patients, 63 head, 53 inflammatory response after, 7 liver, 50 motor vehicle collisions, 59 neck, 52 in pregnancy, 62 primary survey of, 49, 59 questions regarding, 49–63 solid organ injuries caused by, 61 spleen, 51 tracheal, 51 transcriptional events after, 10 Trauma-induced coagulopathy, 23 Traumatic brain injury intracranial pressure monitoring in, 112 medical management of, 348–349 Traumatic shock, 29 Treponema pallidum, 183 Tricuspid atresia, 168 Tricuspid stenosis, 176 tRNA, 119

30/06/22 11:32 AM

413

U

Ulcer(s) Cushing, 349 duodenal, 226–227 malignant transformation of, 80 pressure, 134 Ulcerative colitis diagnosis of, 257 primary sclerosing cholangitis and, 246 Ulnar nerve, 355, 355f, 358 Ultrasound cardiac tamponade evaluations, 55, 56f endoscopic. See Endoscopic ultrasound

Brunicardi_Index_p391-420.indd 413

Ultraviolet radiation exposure, 130 Umbilical stump bleeding, 22 Uncal herniation, 345, 345f Uncomplicated diverticulitis, 247 Undescended testes, 336 Uremia, 23 Ureteral injuries, 338 Ureteral stents, 338 Urinary incontinence, 343 Urinary tract infection, 42–43, 338 Urolithiasis, 338 Urology, 337–339 Uterine leiomyoma, 341 UTI. See Urinary tract infection UV. See Ultraviolet radiation exposure V

Vaccinations, 135, 298, 342 VADs. See Ventricular assist devices Vaginal cancer, 343 Vagus nerve, 208 Validity, external, 383 Valsalva maneuver, 241 Valve replacement, 174–175, 268, 367 Valvulae conniventes, 235 VAP. See Ventilator-associated pneumonia Variceal hemorrhage, 271 Vascular endothelial growth factor, 84 Vascular injuries, of extremities, 54–55, 55t Vascular rings, 166, 167f Vascular stents, 189–190 Vascular-type Ehlers-Danlos syndrome, 182 Vasoactive medications, 271 Vasoactive proteins, 31 Vasodilatory shock, 33 Vasogenic shock, 29 Vasopressin, 31, 271 VEGF. See Vascular endothelial growth factor Veins anatomy of, 197–198 hepatic, 265, 266f lower extremity, 197 Vena cava filters, 202, 203f Venae comitantes, 197 Venous anastomosis, 92f Venous disease, 197–206 Venous thromboembolism, 200, 202, 204, 204f, 366 Ventilator weaning, 101 Ventilator-associated pneumonia, 100 Ventilator-induced lung injury, 112 Ventricular assist devices, 179 Ventricular compliance, 106–107

Ventricular septal defect aortic insufficiency secondary to, 175 description of, 161–162 in double-outlet right ventricle, 170 holosystolic murmur associated with, 175 tricuspid atresia without, 168 Vibrio vulnificus, 134 VILI. See Ventilator-induced lung injury VIPomas, 294 Virtual reality simulation, 387 Vision, 3, 3t Visual abstracts, 389 Vitamin(s), in wound healing, 79–80 Vitamin A, 78–80 Vitamin B12 deficiency, 224 Vitamin C, 79 Vitamin K deficiency of, 268 warfarin reversal using, 24 VO2. See Oxygen utilization Vocal cords anatomy of, 146, 146f leukoplakia of, 143 Volume receptors, 30 Volvulus cecal, 250–251 endoscopy for, 250–251 Vomiting metabolic derangements caused by, 17 postoperative, 378 von Willebrand factor, 21, 24 VSD. See Ventricular septal defect VTE. See Venous thromboembolism Vulvar carcinoma, 342 vWF. See von Willebrand factor

INDEX

Trousseau sign, 16 True aneurysms, 181 Truncus arteriosus, 165 TTE. See Transthoracic echocardiography TTP. See Thrombotic thrombocytopenic purpura T-tube, 281 Tube thoracostomy, for tension pneumothorax, 56, 57f Tuberculosis, 153 Tumor(s) appendiceal, 260 carcinoid, 260 carotid body, 191 chest wall, 154 conservative resections, 85 desmoid, 154 esophageal, 209, 213 gastrointestinal stromal, 230, 238, 249 nonseminatous germ-cell, 155 omentum, 308 pancreatic exocrine, 292, 293f pituitary fossa, 328 salivary gland-type, 147 thymic, 155 Tumor markers, 85 Tumor necrosis factor-alpha, 37 Tumor suppressor genes, 124 Tumorigenic transformation of cells, 83 TURP. See Transurethral resection of the prostate TXA. See Tranexamic acid Type A blood, 25 Type AB blood, 25 Type II error, 382 Type O-negative blood, 25 Typhlitis, 255 Tyrosine kinase inhibitors, 249 Tyrosine kinase receptors, 122

W

Wallerian degeneration, 77 Warfarin, 24, 205, 268 Warm-antibody autoimmune hemolytic anemia, 299 Wartenberg sign, 358 Water free water deficit, 15 in total body weight, 14–15 Water deficit, 15 Watermelon stomach, 230, 230f Water-soluble contrast agents, 236 Weaning, ventilator, 101 Web-based education, 389–390 West Nile virus, 27 Whipple procedure, 286 Whole-pancreas transplantation, 95

30/06/22 11:32 AM

414

INDEX

Willingness, 3, 3t Willingness to lead, 3t Willingness to learn, 3t Wilms tumor, 331, 332t Wilson disease, 269–270 Wolff-Parkinson-White syndrome, 169f, 169–170 Women fluid maintenance calculations for, 16 in surgery, 5 total body weight in, 15 Wound(s) burn. See Burn wounds classification of, 39–40, 40t closure of, in Ehler-Danlos syndrome patients, 75 collagen in, 74 in Ehler-Danlos syndrome patients, 75 gastric, 62 infection of, 102 malignant, 80 matrix deposition at site of, 74 skin, 132 stab, 59, 60t

Brunicardi_Index_p391-420.indd 414

strength of, 74 surgical, 39–40, 40t, 47 “transition wound,” 77 Wound healing amino acids in, 79 anastomoses, 75 anemia and, 78 arginine effects on, 79 callus formation in, 77 collagen in, 74, 76t diabetes mellitus effects on, 78 fetal, 77 fibroblastic phase of, 73f, 74 full-thickness wounds, 75–76 gastrointestinal tract, 75–76, 76t hypertrophic scars, 80–81 impairment of, 78 inflammation phase of, 73f, 78–79 keloid formation, 80–81, 81f low-oxygen tension effects on, 78 malnutrition effects on, 79 maturation phase of, 73f nutrition in, 79 oxygen delivery during, 78 phases of, 73f, 73–74, 361

polymorphonuclear neutrophils in, 74 proliferative phase of, 73f, 74 scar remodeling, 75 skin, 76t steroid effects on, 78 vitamins in, 79–80 WPW syndrome. See WolffParkinson-White syndrome Wrist carpal tunnel syndrome of, 357 ganglion cyst of, 360 surgery of, 355–360 Wrong-site surgeries, 99 X

Xenotransplant, 87 Z

Zenker’s diverticulum, 220 ZES. See Zollinger-Ellison syndrome Zika virus, 27 Zollinger-Ellison syndrome, 225, 231 Zona fasciculata, 8 Zone of coagulation, 67, 129 Zone of hyperemia, 67, 129 Zygoma fractures, 144

30/06/22 11:32 AM